Conflicts:
	sem/OpenVPN_vs_IPSec-Paper.tex
This commit is contained in:
2scholz 2014-12-13 23:11:40 +01:00
commit 97eafc7ac0
51 changed files with 10860 additions and 100 deletions

1
.gitignore vendored
View File

@ -7,6 +7,7 @@
*.bcf
*.blg
*.dvi
*.exe
*.fdb_latexmk
*.glg
*.glo

148
fgi1/Aufgabenblatt10.tex Normal file
View File

@ -0,0 +1,148 @@
\documentclass[10pt,a4paper,oneside,ngerman,numbers=noenddot]{scrartcl}
\usepackage[T1]{fontenc}
\usepackage[utf8]{inputenc}
\usepackage[ngerman]{babel}
\usepackage{amsmath}
\usepackage{amsfonts}
\usepackage{amssymb}
\usepackage{paralist}
\usepackage{gauss}
\usepackage{stmaryrd}
\usepackage[locale=DE,exponent-product=\cdot,detect-all]{siunitx}
\usepackage{tikz}
\usetikzlibrary{automata,matrix,fadings,calc,positioning,decorations.pathreplacing,arrows,decorations.markings}
\usepackage{polynom}
\polyset{style=C, div=:,vars=x}
\pagenumbering{arabic}
\def\thesection{10.\arabic{section})}
\def\thesubsection{\arabic{subsection}.}
\def\thesubsubsection{(\alph{subsubsection})}
\setcounter{section}{2}
\makeatletter
\renewcommand*\env@matrix[1][*\c@MaxMatrixCols c]{%
\hskip -\arraycolsep
\let\@ifnextchar\new@ifnextchar
\array{#1}}
\makeatother
\addtolength{\parskip}{\baselineskip}
\begin{document}
\author{Jim Martens}
\title{Hausaufgaben zum 11. Juni}
\maketitle
\section{} %10.3
\subsection{} %1.
\subsubsection{} %a
Es ist zu zeigen, dass F aus der angegebenen Formelmenge M folgt. Nach Definition 5.1 folgt F genau dann aus M, wenn alle Modelle von M auch Modelle von F sind. Nach Satz 5.11 sind die Modelle einer Formelmenge identisch mit den Modellen einer Konjunktion aller Mengenglieder.\\
Daraus ergibt sich, dass eine Belegung diese Formel erfüllen muss, um ein Modell von M zu sein:\\
$G = ((B \Rightarrow D) \Rightarrow (A \vee C)) \wedge ((A \vee C) \Rightarrow E) \wedge (B \Rightarrow (D \vee E)) \wedge (E \Rightarrow F)$\\
Daraus folgt wiederum, dass eine Belegung M nur erfüllt, wenn $(E \Rightarrow F)$ erfüllt ist. Diese Teilformel ist wiederum erfüllt, wenn E falsch oder F wahr ist.
M kann nur erfüllt sein, wenn E wahr ist, denn sonst müsste auch $(A \vee C)$ und damit $(B \Rightarrow D)$ falsch sein. Letzteres würde voraussetzen, dass B wahr und D falsch sind. Daraus würde folgen, dass $(B \Rightarrow (D \vee E))$ falsch ergäbe, da hier nun B wahr und sowohl D als auch E falsch wären. Damit wiederum wäre die Formel G nicht erfüllt.\\
Daraus folgt also, dass E und damit auch F wahr sein müssen, damit eine Belegung ein Modell von M sein kann. Daher ist immer wahr, wenn eine Belegung ein Modell von M ist, womit diese Belegung auch ein Modell von F ist.
\subsubsection{} %b
In diesem Fall kann E falsch sein. Damit müssten auch $(A \vee C)$ und $\neg(B \Rightarrow D)$ falsch sein. $(B \Rightarrow D)$ wiederum müsste wahr sein, was durch den Wahrheitswert falsch für B erreicht werden kann. Damit kann eine Belegung M erfüllen, wenn E falsch ist. Wenn E falsch ist, dann muss F nicht wahr sein, damit $(E \Rightarrow F)$ wahr ist, womit nicht alle Modelle von M auch Modelle von F sind.
\subsection{} %2.
F folgt genau dann aus M, wenn jede Belegung, die M wahr macht, auch F wahr macht. Damit folgt F genau dann aus M, wenn jede Belegung, die alle Formeln aus M wahr macht, $\neg F$ falsch macht. Daher kann F nicht aus $M \cup \{\neg F\}$ folgen, da jede Belegung, die diese Menge erfüllt, F falsifiziert. Wenn F nicht aus $M \cup \{\neg F\}$, dann darf F auch nicht aus M folgen. Dies ist nur möglich, wenn M unerfüllbar ist.
\section{} %10.4
\subsection{} %1.
\setcounter{subsubsection}{0}
\subsubsection{} %a
\begin{alignat*}{2}
\text{sub}_{1a}(A) &=& (A \Rightarrow \neg B) \\
\text{sub}_{1a}(B) &=& (D \wedge A) \\
\text{sub}_{1a}(C) &=& (C \vee D) \\
\intertext{Für alle anderen Aussagensymbole $A_{i}$ sei $sub_{1a}(A_{i}) = A_{i}$}
sub_{1a}(F_{a}) &=& (((A \Rightarrow \neg B) \Rightarrow \neg(D \wedge A)) \wedge (C \vee D)) = G_{a}
\end{alignat*}
\subsubsection{} %b
\begin{alignat*}{2}
\text{sub}_{1b}(A) &=& (\neg(B \vee C) \wedge E) \\
\text{sub}_{1b}(D) &=& \neg(B \vee C) \\
\intertext{Für alle anderen Aussagensymbole $A_{i}$ sei $sub_{1b}(A_{i}) = A_{i}$}
sub_{1b}(F_{b}) &=& ((\neg(B \vee C) \wedge E) \Leftrightarrow \neg(B \vee C)) \\
&=& ((\neg(B \vee C) \wedge E) \Leftrightarrow \neg(B \vee C)) = sub_{1b}(G_{b})
\end{alignat*}
\subsection{} %2.
\subsubsection{} %a
$R_{a} = \frac{\neg A}{A \Rightarrow B}$\\
\begin{tabular}{c|cc|cc}
& F & G & $\neg$F & $(F \Rightarrow G)$ \\
\hline
$\mathcal{A}_{0}$ & 0 & 0 & 1 & 1 \\
$\mathcal{A}_{1}$ & 0 & 1 & 1 & 1 \\
$\mathcal{A}_{2}$ & 1 & 0 & 0 & 0 \\
$\mathcal{A}_{3}$ & 1 & 1 & 0 & 1
\end{tabular}\\
Für alle Belegungen mit $\mathcal{A}(\neg F) = 1$ gilt auch $\mathcal{A}((F \Rightarrow G)) = 1$.\\
Falls M eine Formelmenge ist und $M \vdash_{R_{a}} G$, dann $M \models G$. \\
Daher ist die Regel korrekt.
\subsubsection{} %b
$R_{b} = \frac{A \Leftrightarrow B}{A}$\\
\begin{tabular}{c|cc|c}
& F & G & $(F \Leftrightarrow G)$ \\
\hline
$\mathcal{A}_{0}$ & 0 & 0 & 1 \\
$\mathcal{A}_{1}$ & 0 & 1 & 0 \\
$\mathcal{A}_{2}$ & 1 & 0 & 0 \\
$\mathcal{A}_{3}$ & 1 & 1 & 1
\end{tabular}\\
Für die Belegung $\mathcal{A}_{0}$ mit $\mathcal{A}((F \Leftrightarrow G)) = 1$ gilt nicht $\mathcal{A}(F) = 1$. Demzufolge ist nicht jedes Modell von $(F \Leftrightarrow G)$ auch eines von $F$. Daher ist die Regel nicht korrekt.
\subsubsection{} %c
$R_{c} = \frac{A \vee B, B \vee C}{A \vee C}$\\
\begin{tabular}{c|ccc|ccc}
& F & G & H & $(F \vee G)$ & $(G \vee H)$ & $(F \vee H)$ \\
\hline
$\mathcal{A}_{0}$ & 0 & 0 & 0 & 0 & 0 & 0\\
$\mathcal{A}_{1}$ & 0 & 0 & 1 & 0 & 1 & 1\\
$\mathcal{A}_{2}$ & 0 & 1 & 0 & 1 & 1 & 0\\
$\mathcal{A}_{3}$ & 0 & 1 & 1 & 1 & 1 & 1\\
$\mathcal{A}_{4}$ & 1 & 0 & 0 & 1 & 0 & 1\\
$\mathcal{A}_{5}$ & 1 & 0 & 1 & 1 & 1 & 1\\
$\mathcal{A}_{6}$ & 1 & 1 & 0 & 1 & 1 & 1\\
$\mathcal{A}_{7}$ & 1 & 1 & 1 & 1 & 1 & 1\\
\end{tabular}\\
Für die Belegung $\mathcal{A}_{2}$ mit $\mathcal{A}((F \vee G)) = 1$ und $\mathcal{A}((G \vee H)) = 1$ gilt nicht $\mathcal{A}((F \vee H)) = 1$.\\
Demzufolge erfüllt nicht jede Belegung, die $(F \vee G)$ und $(G \vee H)$ erfüllt, auch $(F \vee H)$. Daher ist die Regel nicht korrekt.
\subsection{} %3.
\def\thesubsubsection{(\arabic{subsubsection})}
\subsubsection{} %1
Annahme aus M
\subsubsection{} %2
Axiom $H_{6b}$ \\
$sub_{2}(A) = (G \Rightarrow H)$ \\
$sub_{2}(B) = (H \Rightarrow F)$
\subsubsection{} %3
Modus ponens\\
$sub_{3}(A) = (G \Rightarrow H) \wedge (H \Rightarrow F)$ \\
$sub_{3}(B) = (H \Rightarrow F)$
\subsubsection{} %4
Axiom $H_{1}$ \\
$sub_{4}(A) = (H \Rightarrow F)$\\
$sub_{4}(B) = G$
\subsubsection{} %5
Modus ponens\\
$sub_{5}(A) = (H \Rightarrow F)$\\
$sub_{5}(B) = (G \Rightarrow (H \Rightarrow F))$
\subsubsection{} %6
Axiom $H_{6a}$\\
$sub_{6}(A) = (G \Rightarrow H)$\\
$sub_{6}(B) = (H \Rightarrow F)$
\subsubsection{} %7
Modus ponens\\
$sub_{7}(A) = (G \Rightarrow (H \Rightarrow F))$ \\
$sub_{7}(B) = (G \Rightarrow H)$
\subsubsection{} %8
Axiom $H_{2}$\\
$sub_{8}(A) = G$\\
$sub_{8}(B) = H$\\
$sub_{8}(C) = F$
\subsubsection{} %9
Modus ponens\\
$sub_{9}(A) = (G \Rightarrow H)$\\
$sub_{9}(B) = ((G \Rightarrow (H \Rightarrow F)) \Rightarrow (G \Rightarrow F))$
\subsubsection{} %10
Modus ponens\\
$sub_{10}(A) = (G \Rightarrow (H \Rightarrow F))$\\
$sub_{10}(B) = (G \Rightarrow F)$
\end{document}

171
fgi1/Aufgabenblatt11.tex Normal file
View File

@ -0,0 +1,171 @@
\documentclass[10pt,a4paper,oneside,ngerman,numbers=noenddot]{scrartcl}
\usepackage[T1]{fontenc}
\usepackage[utf8]{inputenc}
\usepackage[ngerman]{babel}
\usepackage{amsmath}
\usepackage{amsfonts}
\usepackage{amssymb}
\usepackage{paralist}
\usepackage{gauss}
\usepackage{stmaryrd}
\usepackage[locale=DE,exponent-product=\cdot,detect-all]{siunitx}
\usepackage{tikz}
\usetikzlibrary{automata,matrix,fadings,calc,positioning,decorations.pathreplacing,arrows,decorations.markings}
\usepackage{polynom}
\polyset{style=C, div=:,vars=x}
\pagenumbering{arabic}
\def\thesection{11.\arabic{section})}
\def\thesubsection{\arabic{subsection}.}
\def\thesubsubsection{(\alph{subsubsection})}
\setcounter{section}{2}
\makeatletter
\renewcommand*\env@matrix[1][*\c@MaxMatrixCols c]{%
\hskip -\arraycolsep
\let\@ifnextchar\new@ifnextchar
\array{#1}}
\makeatother
\addtolength{\parskip}{\baselineskip}
\begin{document}
\author{Jim Martens}
\title{Hausaufgaben zum 25. Juni}
\maketitle
\section{} %11.3
\subsection{} %1.
\subsubsection{} %a
$M = \{\{A\}\}$\\
\begin{tabular}{c|cc}
& A \\
\hline
$\mathcal{A}_{0}$ & 0 \\
$\mathcal{A}_{1}$ & 1
\end{tabular}
\subsubsection{} %b
$M = \{\{A,B,C\}\}$\\
\begin{tabular}{c|ccc|ccc}
& A & B & C & $\{A,B,C\}$ & $\{\{A,B,C\}\}$ & $A \vee (B \vee C)$\\
\hline
$\mathcal{A}_{0}$ & 0 & 0 & 0 & 0 & 0 & 0 \\
$\mathcal{A}_{1}$ & 0 & 0 & 1 & 1 & 1 & 1 \\
$\mathcal{A}_{2}$ & 0 & 1 & 0 & 1 & 1 & 1\\
$\mathcal{A}_{3}$ & 0 & 1 & 1 & 1 & 1 & 1\\
$\mathcal{A}_{4}$ & 1 & 0 & 0 & 1 & 1 & 1\\
$\mathcal{A}_{5}$ & 1 & 0 & 1 & 1 & 1 & 1\\
$\mathcal{A}_{6}$ & 1 & 1 & 0 & 1 & 1 & 1\\
$\mathcal{A}_{7}$ & 1 & 1 & 1 & 1 & 1 & 1\\
\end{tabular}
\subsubsection{} %c
$M = \{\{A\}, \{B\}, \{C\}\}$\\
\begin{tabular}{c|ccc|ccc|cc}
& A & B & C & $\{A\}$ & $\{B\}$ & $\{C\}$ & $\{\{A\}, \{B\}, \{C\}\}$ & $A \wedge (B \wedge C)$\\
\hline
$\mathcal{A}_{0}$ & 0 & 0 & 0 & 0 & 0 & 0 & 0 & 0 \\
$\mathcal{A}_{1}$ & 0 & 0 & 1 & 0 & 0 & 1 & 0 & 0 \\
$\mathcal{A}_{2}$ & 0 & 1 & 0 & 0 & 1 & 0 & 0 & 0\\
$\mathcal{A}_{3}$ & 0 & 1 & 1 & 0 & 1 & 1 & 0 & 0\\
$\mathcal{A}_{4}$ & 1 & 0 & 0 & 1 & 0 & 0 & 0 & 0\\
$\mathcal{A}_{5}$ & 1 & 0 & 1 & 1 & 0 & 1 & 0 & 0\\
$\mathcal{A}_{6}$ & 1 & 1 & 0 & 1 & 1 & 0 & 0 & 0\\
$\mathcal{A}_{7}$ & 1 & 1 & 1 & 1 & 1 & 1 & 1 & 1\\
\end{tabular}
\subsection{} %2.
\begin{alignat*}{2}
M_{1} &= \{\{A\}\} \\
M_{2} &= \{\{\neg A\}\} \\
M_{3} &= \{\{A, \neg A\}\} \\
M_{4} &= \{\{A\}, \{\neg A\}\} \\
M_{5} &= \{\{A\}, \{A, \neg A\}\} \\
M_{6} &= \{\{\neg A\}, \{A, \neg A\}\} \\
M_{7} &= \{\{A\}, \{\neg A\}, \{A, \neg A\}\}
\end{alignat*}\\
$M_{3}$ ist allgemeingültig. $M_{4}$ und $M_{7}$ sind unerfüllbar. $M_{1}$ und $M_{5}$, sowie $M_{2}$ und $M_{6}$ sind äquivalent zueinander.
%\begin{tabular}{c|c|ccccccc}
%& A & $\{\{A\}\}$ & $\{\{\neg A\}\}$ & $\{\{A, \neg A\}\}$ & $\{\{A\}, \{\neg A\}\}$ & $\{\{A\}, \{A, \neg A\}\}$ & $\{\{\neg A\}, \{A, \neg A\}\}$ & $\{\{A\}, \{\neg A\}, \{A, \neg A\}\}$\\
%\hline
%$\mathcal{A}_{0}$ & 0 & 0 & 1 & 1 & 0 & 0 & 1 & 0 \\
%$\mathcal{A}_{1}$ & 1 & 1 & 0 & 1 & 0 & 1 & 0 & 0
%\end{tabular}
\subsection{} %3.
Behauptung: $M \cup \{K\}$ ist genau dann erfüllbar, wenn M erfüllbar ist.\\
Beweis:\\
Es ist zu zeigen, dass (1) $M \cup \{K\}$ erfüllbar ist, wenn M erfüllbar ist und dass (2) M erfüllbar ist, wenn $M \cup \{K\}$ erfüllbar ist.\\
\\
Teilbeweis von (1):\\
M sei erfüllbar.\\
$M \cup \{K\}$ ist erfüllbar, wenn es mindestens eine Belegung gibt, für die sowohl M als auch K erfüllt sind.\\
K ist erfüllt, wenn mindestens eines der Literale in K erfüllt ist.\\
\\
Es gilt $A, \neg A \in K$. Unabhängig von der Belegung von A ist K daher immer erfüllt, denn für $\mathcal{A}(A) = 0$ gilt $\mathcal{A}(\neg A) = 1$ und für $\mathcal{A}(A) = 1$ gilt $\mathcal{A}(A) = 1$.\\
Da K immer erfüllt ist, stellt es bei der Vereinigung mit M keine einschränkende Bedingung dar. Ob die vereinigte Menge erfüllbar ist, hängt damit von M ab. Da M aufgrund der Annahme erfüllbar ist, ist somit auch $M \cup \{K\}$ erfüllbar.\\
\\
Teilbeweis für (2):\\
$M \cup \{K\}$ sei erfüllbar.\\
$M \cup \{K\}$ ist erfüllbar, wenn es mindestens eine Belegung gibt, für die sowohl M als auch K erfüllt sind. Wie bereits gezeigt, ist K für jede Belegung erfüllt. Damit stellt K keine zusätzliche Bedingung und $M \cup \{K\}$ ist erfüllbar, wenn M erfüllbar ist.\\
Aufgrund der Annahme ist $M \cup \{K\}$ erfüllbar, womit M ebenfalls erfüllbar ist.\\
\\
Es konnten beide Richtungen gezeigt werden, womit die Behauptung bewiesen ist.
\section{} %11.4
\subsection{} %1.
\setcounter{subsubsection}{0}
\subsubsection{} %a
\begin{alignat*}{2}
F_{3} &=& C \wedge (\neg A \vee \neg E \vee D) \wedge E \wedge (\neg C \vee B) \wedge (\neg E \vee \neg B \vee A) \wedge (\neg D \vee \neg C \vee \neg A) \\
\intertext{Umformen in Implikationsschreibweise}
&=& (T \Rightarrow C) \wedge ((A \wedge E) \Rightarrow D) \wedge (T \Rightarrow E) \wedge (C \Rightarrow B) \wedge ((E \wedge B) \Rightarrow A) \wedge ((D \wedge C \wedge A) \Rightarrow \perp) \\
\intertext{Anwenden des Markierungsalgorithmus}
&=& (T \Rightarrow C^{1}) \wedge ((A^{3} \wedge E^{1}) \Rightarrow D^{4}) \wedge (T \Rightarrow E^{1}) \wedge (C^{1} \Rightarrow B^{2}) \wedge ((E^{1} \wedge B^{2}) \Rightarrow A^{3}) \wedge ((D^{4} \wedge C^{1} \wedge A^{3}) \Rightarrow \perp) \\
\intertext{Die Formel ist unerfüllbar.}
\end{alignat*}\\
\\\\\\
Anwenden der P-Resolution:\\
\begin{tabular}{cccccc}
($\neg D \vee \neg C \vee \neg A$) & C & ($\neg C \vee B$) & ($\neg E \vee \neg B \vee A$) & E & ($\neg A \vee \neg E \vee D$) \\
& ($\neg D \vee \neg A$) & B & ($\neg B \vee A$) & & ($\neg A \vee D$) \\
& & & A & & \\
& & $\neg D$ & & D & \\
& & & $\Box$ & &
\end{tabular} \\
Die Formel ist auch nach der P-Resolution unerfüllbar.
\subsubsection{} %b
\begin{alignat*}{2}
F_{4} &=& \neg A \wedge (\neg A \vee C \vee \neg E) \wedge (A \vee \neg C \vee \neg E) \wedge B \wedge (\neg B \vee D) \wedge (\neg B \vee \neg D \vee E) \\
\intertext{Umformen in Implikationsschreibweise}
&=& (A \Rightarrow \perp) \wedge ((A \wedge E) \Rightarrow C) \wedge ((C \wedge E) \Rightarrow A) \wedge (T \Rightarrow B) \wedge (B \Rightarrow D) \wedge ((B \wedge D) \Rightarrow E) \\
\intertext{Anwenden des Markierungsalgorithmus}
&=& (A \Rightarrow \perp) \wedge ((A \wedge E^{3}) \Rightarrow C) \wedge ((C \wedge E^{3}) \Rightarrow A) \wedge (T \Rightarrow B^{1}) \wedge (B^{1} \Rightarrow D^{2}) \wedge ((B^{1} \wedge D^{2}) \Rightarrow E^{3}) \\
\intertext{Die Formel ist erfüllbar.}
\end{alignat*}\\
Anwenden der P-Resolution:\\
\begin{tabular}{cccccc}
($\neg A \vee C \vee \neg E$) & ($\neg B \vee D$) & B & ($\neg B \vee \neg D \vee E$) & ($A \vee \neg C \vee \neg E$) \\
& D & & ($\neg D \vee E$) & & \\
& & E & & & \\
& ($\neg A \vee C$) & & ($A \vee \neg C$) & &
\end{tabular}\\
Es können keine weiteren Resolventen mit der P-Resolution gebildet werden. Die Formel ist daher nicht unerfüllbar und damit erfüllbar.
\subsection{} %2.
\begin{alignat*}{2}
F_{5} &=& (A \vee B) \wedge (A \vee D) \wedge (C \vee D) \wedge (\neg A \vee \neg C) \wedge (\neg B \vee \neg D) \wedge (B \vee \neg A \vee \neg D)
\end{alignat*}\\
Anwenden der P-Resolution:\\
\begin{tabular}{cccccc}
\{A,B\} & \{$\neg B, \neg D$\} & \{A,D\} & \{$\neg A, \neg C$\} & \{C,D\} & \{$B, \neg A, \neg D$\} \\
& & \{$A, \neg B$\} & & & \\
& \{A\} & & & & \\
& & \{$\neg C$\} & & \{$B, \neg D$\} & \\
& & & \{D\} & & \\
& \{$\neg B$\} & & & \{B\} & \\
& & & $\Box$ & &
\end{tabular}\\
Die Formel ist nach der P-Resolution unerfüllbar.
\subsection{} %3.
\begin{alignat*}{2}
F_{6} &=& (A \Rightarrow C) \Rightarrow ((B \Rightarrow C) \Rightarrow ((A \vee B) \Rightarrow C)) \\
\intertext{Eliminieren von Implikationen}
&=& \neg(\neg A \vee C) \vee (\neg(\neg B \vee C) \vee (\neg(A \vee B) \vee C)) \\
\intertext{Negationen nach innen ziehen}
&=& (A \wedge \neg C) \vee ((B \wedge \neg C) \vee ((\neg A \wedge \neg B) \vee C))
\end{alignat*}\\
\end{document}

106
fgi1/Aufgabenblatt12.tex Normal file
View File

@ -0,0 +1,106 @@
\documentclass[10pt,a4paper,oneside,ngerman,numbers=noenddot]{scrartcl}
\usepackage[T1]{fontenc}
\usepackage[utf8]{inputenc}
\usepackage[ngerman]{babel}
\usepackage{amsmath}
\usepackage{amsfonts}
\usepackage{amssymb}
\usepackage{paralist}
\usepackage{gauss}
\usepackage{stmaryrd}
\usepackage[locale=DE,exponent-product=\cdot,detect-all]{siunitx}
\usepackage{tikz}
\usetikzlibrary{automata,matrix,fadings,calc,positioning,decorations.pathreplacing,arrows,decorations.markings}
\usepackage{polynom}
\polyset{style=C, div=:,vars=x}
\pagenumbering{arabic}
\def\thesection{12.\arabic{section})}
\def\thesubsection{\arabic{subsection}.}
\def\thesubsubsection{(\alph{subsubsection})}
\setcounter{section}{2}
\makeatletter
\renewcommand*\env@matrix[1][*\c@MaxMatrixCols c]{%
\hskip -\arraycolsep
\let\@ifnextchar\new@ifnextchar
\array{#1}}
\makeatother
\addtolength{\parskip}{\baselineskip}
\begin{document}
\author{Jim Martens}
\title{Hausaufgaben zum 2. Juli}
\maketitle
\section{} %12.3
\subsection{} %1.
\begin{itemize}
\item Q und t müssen Variablen sein, da sie hinter Quantoren auftreten.
\item z(t), m(S, Q) und m(Q(S), z) müssen Formeln sein, da sie mit Junktoren verknüpft werden.
\item Entsprechend müssen z und m Prädikatensymbole sein und S, Q(S) und z Terme. z ist einstellig und m ist zweistellig.
\item S und z könnten eine Variable oder eine Konstante sein, denn sie sind atomare Terme.
\item Q ist ein Funktionssymbol und einstellig.
\end{itemize}
Das Problem der Formalisierung ist, dass Q eine Variable sein muss und gleichzeitig ein Funktionssymbol ist. Ebenso ist z einmal ein Prädikatensymbol und einmal ein atomarer Term.
\subsection{} %2.
\begin{tikzpicture}[shorten >=1pt,node distance=1.1cm,on grid]
\node (A_z) {$\forall z$};
\node (Impl) [below=of A_z]{$\Rightarrow$};
\node (E_x) [below left=1.0 and 2.0 of Impl] {$\exists x$};
\node (E_y) [below right=1.0 and 2.0 of Impl] {$\exists y$};
\node (and1) [below of=E_x] {$\wedge$};
\node (p1) [below left=1.0 and 1.2 of and1] {$P(g(x), y)$};
\node (p2) [below right=1.0 and 1.2 of and1] {$P(z, x)$};
\node (and2) [below=of E_y] {$\wedge$};
\node (E_z) [below left=1.0 and 1.2 of and2] {$\exists z$};
\node (z) [below=of E_z] {$z$};
\node (p3) [below right=1.0 and 1.5 of and2] {$P(g(y), f(x,z))$};
\end{tikzpicture}
\subsection{} %3.
\subsubsection{} %a
Für mindestens ($\exists x$) einen Tag (x) im Sommer gilt, dass die Sonne scheint ($P_{1}(x)$) und mindestens ($\exists y$) ein Rockkonzert ($Q_{1}(y)$) aufgeführt wird ($R_{1}(x,y)$).
\subsubsection{} %b
Für jeden Tag ($\forall x$) im Sommer gilt, dass wenn die Temperaturen über 35$^{\circ}$ steigen, ($P_{2}(x)$) es für alle ($\forall y$) Schüler, die unter 16 Jahren sind ($Q_{2}(y)$), hitzefrei ($R_{2}(x,y)$) gibt.
\subsubsection{} %c
An Tag ($\forall x$), der frei ist ($P_{3}(x)$), gehen alle ($\forall y$) Schüler, die zuhause sind ($R_{3}(x,y)$), zum Kino ($Q_{3}(y)$).
\subsubsection{} %d
\subsubsection{} %e
\section{} %11.4
\def\thesubsubsection{\arabic{subsubsection}.}
\setcounter{subsubsection}{0}
\subsubsection{} %1
$F_{41} = (P(x) \Rightarrow \exists x (R(x,x) \wedge \neg P(x)))$\\
\begin{tabular}{c|c|c|c|c|c|c|c|c}
& P & R & x & P(x) & R(x,x) & $\neg$P(x) & $\exists x (R(x,x) \wedge \neg P(x)) $ & $F_{41}$ \\
\hline
$A_{4}$ & $\{3,9\}$ & $\{(3,9),(6,6), (6,9), (9,9)\}$ & 9 & 1 & & & & 0 \\
\hline
$A_{4[x/3]}$ & $\{3,9\}$ & $\{(3,9),(6,6), (6,9), (9,9)\}$ & 3 & 1 & 0 & 0 & 0 \\
$A_{4[x/6]}$ & $\{3,9\}$ & $\{(3,9),(6,6), (6,9), (9,9)\}$ & 6 & 1 & 1 & 0 & 0 \\
$A_{4[x/9]}$ & $\{3,9\}$ & $\{(3,9),(6,6), (6,9), (9,9)\}$ & 9 & 1 & 1 & 0 & 0
\end{tabular}
\subsubsection{} %2
$F_{42} = \forall y (\exists x R(x,y) \Rightarrow \forall x R(x,y))$\\
Zur Platzersparnis wird die Spalte mit den Werten von P und R weggelassen. Die sind in allen Varianten gleich.\\
\begin{tabular}{c|c|c|c|c|c|c|c}
& x & y & R(x,y) & $\exists x R(x,y)$ & $\forall x R(x,y)$ & $ \exists x R(x,y) \wedge \forall x R(x,y)$ & $F_{42}$ \\
\hline
$A_{4}$ & 9 & 9 & & & & & 0 \\
\hline
$A_{4[y/3]}$ & 9 & 3 & & 0 & 0 & 0 & \\
$A_{4[y/6]}$ & 9 & 6 & & 1 & 0 & 0 & \\
$A_{4[y/9]}$ & 9 & 9 & & 1 & 1 & 1 & \\
\hline
$A_{4[y/3][x/3]}$ & 3 & 3 & 0 & & & & \\
$A_{4[y/3][x/6]}$ & 6 & 3 & 0 & & & & \\
$A_{4[y/3][x/9]}$ & 9 & 3 & 0 & & & & \\
\hline
$A_{4[y/6][x/3]}$ & 3 & 6 & 0 & & & & \\
$A_{4[y/6][x/6]}$ & 6 & 6 & 1 & & & & \\
$A_{4[y/6][x/9]}$ & 9 & 6 & 1 & & & & \\
\hline
$A_{4[y/9][x/3]}$ & 3 & 9 & 1 & & & & \\
$A_{4[y/9][x/6]}$ & 6 & 9 & 1 & & & & \\
$A_{4[y/9][x/9]}$ & 9 & 9 & 1 & & & & \\
\end{tabular}
\end{document}

146
fgi1/Aufgabenblatt2.tex Normal file
View File

@ -0,0 +1,146 @@
\documentclass[10pt,a4paper,oneside,ngerman,numbers=noenddot]{scrartcl}
\usepackage[T1]{fontenc}
\usepackage[utf8]{inputenc}
\usepackage[ngerman]{babel}
\usepackage{amsmath}
\usepackage{amsfonts}
\usepackage{amssymb}
\usepackage{paralist}
\usepackage{gauss}
\usepackage[locale=DE,exponent-product=\cdot,detect-all]{siunitx}
\usepackage{tikz}
\usetikzlibrary{automata,matrix,fadings,calc,positioning,decorations.pathreplacing,arrows,decorations.markings}
\usepackage{polynom}
\polyset{style=C, div=:,vars=x}
\pagenumbering{arabic}
\def\thesection{2.\arabic{section})}
\def\thesubsection{\arabic{subsection}.}
\def\thesubsubsection{(\roman{subsubsection})}
\setcounter{section}{3}
\makeatletter
\renewcommand*\env@matrix[1][*\c@MaxMatrixCols c]{%
\hskip -\arraycolsep
\let\@ifnextchar\new@ifnextchar
\array{#1}}
\makeatother
\begin{document}
\author{Jim Martens}
\title{Hausaufgaben zum 16. April}
\maketitle
\section{} %2.4
Der NFA ergibt sich folgendermaßen:\\
\begin{tikzpicture}[shorten >=1pt,node distance=1.1cm,on grid]
\node (00) {$0,0$};
\node (01) [below=of 00] {$0,1$};
\node (02) [below=of 01] {$0,2$};
\node (10) [left=of 00] {$1,0$};
\node (11) [left=of 01] {$1,1$};
\node (12) [below=of 11] {$1,2$};
\node (21) [left=of 11] {$2,1$};
\node (20) [left=of 21] {$2,0$};
\node (22) [left=of 12] {$2,2$};
\node (30) [left=of 20] {$3,0$};
\node (32) [left=of 22] {$3,2$};
\node (31) [left=of 32] {$3,1$};
\path[every node/.style={font=\scriptsize}]
(00) edge[->] node [below left=0.05 and 0.15 of 00] {1} (01)
(01) edge[->] node [below left=0.15 and 0.0005 of 01] {0} (11)
(01) edge[->] node [below right=0.05 and 0.15 of 01] {1} (02)
(02) edge[->,bend right=45] node [above right=0.15 and 0.1 of 02] {1} (00)
(11) edge[->] node [below left=0.1 and 0.1 of 11] {1} (12)
(02) edge[->] node [below right=0.1 and 0.1 of 02] {0} (12)
(11) edge[->] node [below left=0.15 and 0.0005 of 11] {0} (21)
(12) edge[->] node [below left=0.15 and 0.0005 of 12] {0} (22)
(21) edge[->] node [below left=0.1 and 0.1 of 21] {1} (22)
(12) edge[->,bend left=125] node [above left=1.0 and 0.1 of 12] {1} (10)
(00) edge[->] node [above left=0.15 and 0.0005 of 00] {0} (10)
(10) edge[->] node [below right=0.05 and 0.15 of 10] {1} (11)
(22) edge[->] node [below left=0.1 and 0.1 of 22] {1} (20)
(10) edge[->] node [above left=0.1 and 0.1 of 10] {0} (20)
(20) edge[->] node [below right=0.15 and 0.0005 of 20] {1} (21)
(22) edge[->] node [below left=0.15 and 0.0005 of 22] {0} (32)
(32) edge[->,bend right=45] node [below right=0.15 and 0.15 of 32] {0} (02)
(32) edge[->] node [below left=0.1 and 0.1 of 32] {1} (30)
(20) edge[->] node [below left=0.15 and 0.0005 of 20] {0} (30)
(30) edge[->,bend left=45] node [above left=0.15 and 0.1 of 30] {0} (00)
(30) edge[->] node [below left=0.05 and 0.15 of 30] {1} (31)
(31) edge[->] node [below right=0.15 and 0.005 of 31] {1} (32)
(31) edge[->,bend right=45] node [below right=0.15 and 0.0005 of 31] {0} (01);
\end{tikzpicture}
\section{} %2.5
\subsection{} %1.
\begin{alignat*}{2}
\hat{\Delta}(\{z_{s}, z_{0}\}, 01) &=& \hat{\Delta}(\underset{p \in \{z_{s}, z_{0}\}}{\bigcup} \Delta(p,0),1) \\
&=& \hat{\Delta}(\Delta(z_{s},0) \cup \Delta(z_{0},0),1) \\
&=& \hat{\Delta}(\{z_{s},z_{0}\} \cup \{z_{e}\},1) \\
&=& \hat{\Delta}(\{z_{s},z_{0},z_{e}\},1) \\
&=& \hat{\Delta}(\underset{p \in \{z_{s},z_{0},z_{e}\}}{\bigcup} \Delta(p,1), \lambda) \\
&=& \hat{\Delta}(\Delta(z_{s},1) \cup \Delta(z_{0},1) \cup \Delta(z_{e},1),\lambda) \\
&=& \hat{\Delta}(\{z_{s}\} \cup \emptyset \cup \{z_{e}\},\lambda) \\
&=& \hat{\Delta}(\{z_{s},z_{e}\},\lambda) \\
&=& \{z_{s},z_{e}\}
\end{alignat*}
\begin{alignat*}{2}
\hat{\Delta}(\{z_{0}, z_{1}\}, 01) &=& \hat{\Delta}(\underset{p \in \{z_{0}, z_{1}\}}{\bigcup} \Delta(p,0),1) \\
&=& \hat{\Delta}(\Delta(z_{0},0) \cup \Delta(z_{1},0),1) \\
&=& \hat{\Delta}(\{z_{e}\} \cup \emptyset ,1) \\
&=& \hat{\Delta}(\{z_{e}\},1) \\
&=& \hat{\Delta}(\underset{p \in \{z_{e}\}}{\bigcup} \Delta(p,1), \lambda) \\
&=& \hat{\Delta}(\Delta(z_{e},1),\lambda) \\
&=& \hat{\Delta}(\{z_{e}\},\lambda) \\
&=& \{z_{e}\}
\end{alignat*}
\subsection{} %2.
\begin{tikzpicture}[shorten >=1pt,node distance=2.8cm,on grid]
\node[initial,state] (zs) {$\{z_{s}\}$};
\node[state] (zsz0) [below=2.0 of zs] {$\{z_{s},z_{0}\}$};
\node[state] (zsz1) [below right=2.0 and 2.0 of zs] {$\{z_{s},z_{1}\}$};
\node[state,accepting] (zsz0ze) [below=2.0 of zsz0] {$\{z_{s},z_{0},z_{e}\}$};
\node[state,accepting] (zsze) [below=2.0 of zsz0ze] {$\{z_{s},z_{e}\}$};
\node[state,accepting] (zsz1ze) [below=2.0 of zsz1] {$\{z_{s},z_{1},z_{e}\}$};
\path[every node/.style={font=\scriptsize}]
(zs) edge[->] node [below left=0.05 and 0.15 of zs] {0} (zsz0)
(zs) edge[->] node [below right=0.1 and 0.2 of zs] {1} (zsz1)
(zsz1) edge[->,bend right=45] node [above right=0.1 and 0.2 of zsz1] {0} (zs)
(zsz0) edge[->,bend left=45] node [above left=0.1 and 0.2 of zsz0] {1} (zs)
(zsz0) edge[->] node [below left=0.05 and 0.1 of zsz0] {0} (zsz0ze)
(zsz1) edge[->] node [below right=0.05 and 0.1 of zsz1] {1} (zsz1ze)
(zsz0ze) edge[->,loop left] node [left=0.1 of zsz0ze] {0} (zsz0ze)
(zsz1ze) edge[->,loop right] node [right=0.1 of zsz1ze] {1} (zsz1ze)
(zsz0ze) edge[->] node [below left=0.05 and 0.15 of zsz0ze] {1} (zsze)
(zsz1ze) edge[->] node [below right=0.1 and 0.1 of zsz1ze] {0} (zsze)
(zsze) edge[->,bend left=45] node [above left=0.1 and 0.1 of zsze] {0} (zsz0ze)
(zsze) edge[->,bend right=45] node [above right=0.1 and 0.25 of zsze] {1} (zsz1ze);
\end{tikzpicture}
\subsection{} %3.
\begin{tikzpicture}[shorten >=1pt,node distance=1.1cm,on grid]
\node (z0) {$z_{0}$};
\node (z1) [below=of z0] {$z_{1}$};
\node (z2) [right=of z0] {$z_{2}$};
\node (z3) [below=of z2] {$z_{3}$};
\path[every node/.style={font=\scriptsize}]
(z0) edge[->] node [above right=0.15 and 0.0005 of z0] {a} (z2)
(z0) edge[->,loop left] node [left=0.15 of z0] {a} (z0)
(z1) edge[->] node [below right=0.15 and 0.0005 of z1] {a} (z3)
(z1) edge[->] node [below right=0.1 and 0.1 of z1] {a} (z2)
(z2) edge[->,bend right=45] node [above left=0.2 and 0.0005 of z2] {b} (z0)
(z2) edge[->] node [below right=0.15 and 0.15 of z2] {b} (z3)
(z3) edge[->,loop right] node [right=0.15 of z3] {b} (z3);
\end{tikzpicture}\\
Es lässt sich nicht rekonstruieren, welche Zustände Start- bzw. Endzustände sind. Ebenso lässt sich nicht rekonstruieren ob es eine Kante von $z_{1}$ aus gibt, die mit einem b benutzt wird. Außerdem fehlen Infos über mögliche a-Kanten der Zustände $z_{2}$ und $z_{3}$.
\section{} %2.6
\textbf{Behauptung:} Die Aussage $L(A_{1}) = \{ab\}\{cb\}^{*} =: M_{1}$ gilt für den Automaten $A_{1}$.\\
\textbf{Beweis:} Durch vollständige Induktion. \\
Mit $A(n)$ sei die Aussage $L(A_{1}) = \{ab(cb)^{n}|n \in \mathbb{N} \} =: M_{1}$ bezeichnet. \\
\underline{Induktionsanfang:} $A(0)$ ist wahr, da $ab$ ein vom Automaten $A_{1}$ akzeptiertes Wort ist und sich der Automat nach dem Lesen des Wortes im Endzustand befindet.\\\\
\underline{Induktionsannahme:} Für ein beliebig fest gewähltes $n \in \mathbb{N}$ gilt $A(n)$, d. h. es gelte $L(A_{1}) = \{ab(cb)^{n}|n \in \mathbb{N} \} =: M_{1}$.\\\\
\underline{Zu zeigen:} $A(n+1)$ gilt, d. h. $L(A_{1}) = \{ab(cb)^{n+1}|n \in \mathbb{N} \} =: M_{1}$ gilt.\\\\
\underline{Induktionsschluss:}\\
Sei $L(A_{1}) = \{ab(cb)^{n+1}|n \in \mathbb{N} \} =: M_{1}$. Dann gilt analog $L(A_{1}) = \{ab(cb)^{n}(cb)|n \in \mathbb{N} \} =: M_{1}$ bzw. $L(A_{1}) = \{ab(cb)^{n}|n \in \mathbb{N} \}\{cb\} =: M_{1}$. \\
Dabei gilt $L(A_{1}) = \{ab(cb)^{n}|n \in \mathbb{N} \}$ aufgrund der Induktionsannahme.\\
Demnach befindet sich der Automat nach Lesen des Wortes $ab(cb)^{n}, n \in \mathbb{N}$ im Endzustand. Liest man nun ein weiteres $cb$, so befindet sich der Automat wieder im Endzustand. Somit gilt $L(A_{1}) = \{ab(cb)^{n+1}|n \in \mathbb{N} \} =: M_{1}$.\\\\
Aus dem Induktionsanfang und dem Induktionsschritt ergibt sich nach dem Induktionsprinzip die Behauptung. \hfill $\Box$.
\end{document}

179
fgi1/Aufgabenblatt3.tex Normal file
View File

@ -0,0 +1,179 @@
\documentclass[10pt,a4paper,oneside,ngerman,numbers=noenddot]{scrartcl}
\usepackage[T1]{fontenc}
\usepackage[utf8]{inputenc}
\usepackage[ngerman]{babel}
\usepackage{amsmath}
\usepackage{amsfonts}
\usepackage{amssymb}
\usepackage{paralist}
\usepackage{gauss}
\usepackage{stmaryrd}
\usepackage[locale=DE,exponent-product=\cdot,detect-all]{siunitx}
\usepackage{tikz}
\usetikzlibrary{automata,matrix,fadings,calc,positioning,decorations.pathreplacing,arrows,decorations.markings}
\usepackage{polynom}
\polyset{style=C, div=:,vars=x}
\pagenumbering{arabic}
\def\thesection{3.\arabic{section})}
\def\thesubsection{\arabic{subsection}.}
\def\thesubsubsection{(\roman{subsubsection})}
\setcounter{section}{3}
\makeatletter
\renewcommand*\env@matrix[1][*\c@MaxMatrixCols c]{%
\hskip -\arraycolsep
\let\@ifnextchar\new@ifnextchar
\array{#1}}
\makeatother
\begin{document}
\author{Jim Martens}
\title{Hausaufgaben zum 23. April}
\maketitle
\section{} %3.4
\subsection{} %1.
\begin{alignat*}{2}
L_{1} &=& \{a^{2n}bcba^{2n} | n \geq 0 \}
\end{alignat*}
Angenommen $L_{1} \in REG$. Sei $n$ die PL-Zahl. Betrachte $z = a^{2n}bcba^{2n} \in L_{1}$. Also $|z| = 4n + 3 \geq n$. Also existiert eine Zerlegung $z = uvw$ mit (i), (ii) und (iii).\\
\begin{enumerate}[i)]
\item $|uv| \leq n$
\item $|v| \geq 1$
\item $\forall i \in \mathbb{N}: uv^{i}w \in L$
\end{enumerate}
\begin{alignat*}{2}
\overset{i)}{\Rightarrow} uv \in \{a\}^{*} &\Rightarrow & u,v \in \{a\}^{*} \\
&\overset{ii)}{\Rightarrow} & v \in \{a\}^{+} \text{ d.h. } v=a^{m} \text{ für } 1 \leq m \leq n
\end{alignat*}\\
Mit iii) folgt für $i = 0$, dass $uv^{i}w = a^{2n-m}bcba^{2n} \not\in L_{1} \lightning$.\\
Also $L_{1} \not\in REG$.
\subsection{} %2.
\begin{alignat*}{2}
L_{2} &=& \{w \in \{0,1\}^{*} | |w|_{1} = 2 \cdot |w|_{0} \}
\end{alignat*}
Angenommen $L_{2} \in REG$. Sei $n$ die PL-Zahl. Betrachte $z = 0^{n}1^{2n} \in L_{2}$. Also $|z| = 3n \geq n$. Also existiert eine Zerlegung $z = uvw$ mit (i), (ii) und (iii).\\
\begin{enumerate}[i)]
\item $|uv| \leq n$
\item $|v| \geq 1$
\item $\forall i \in \mathbb{N}: uv^{i}w \in L$
\end{enumerate}
\begin{alignat*}{2}
\overset{i)}{\Rightarrow} uv \in \{0\}^{*} &\Rightarrow & u,v \in \{0\}^{*} \\
&\overset{ii)}{\Rightarrow} & v \in \{0\}^{+} \text{ d.h. } v=a^{m} \text{ für } 1 \leq m \leq n
\end{alignat*}\\
Mit iii) folgt für $i = 0$, dass $uv^{i}w = 0^{n-m}1^{2n} \not\in L \lightning$.\\
Also $L \not\in REG$.
\section{} %3.5
\subsection{} %1.
Die Menge $\overline{\{a\}^{*}}$ lässt sich über dem Alphabet $\Sigma$ auch so schreiben: $\Sigma^{*} \setminus \{a\}^{*}$. Die regulären Ausdrücke $A = (a|b)^{*}$ und $B = a^{*}$ beschreiben jeweils $\Sigma^{*}$ und $\{a\}^{*}$.
Der Ausdruck $C = B \cdot b^{+} \cdot A$ beschreibt die gesuchte Menge $\overline{\{a\}^{*}}$.\\
Dieser Ausdruck akzeptiert beliebig viele $a$ am Anfang (auch keine), erwartet dann mindestens ein $b$ aber auch nicht mehr und gibt einem danach die freie Wahl zwischen beliebig vielen $a$ und $b$ ohne Beschränkung. Dadurch ist gewährleistet, dass in jedem Ausdruck mindestens ein $b$ vorkommt, womit weder das leere Wort noch eine Zeichenkette, die nur aus dem Buchstaben $a$ besteht, akzeptiert werden. Somit wird kein Element von $\{a\}^{*}$ akzeptiert, welches dem Komplement eben dieser Menge entspricht.
\subsection{} %2.
Berechnung eines regulären Ausdruckes für den gegebenen DFA mithilfe des Kleene-Verfahrens.
\begin{alignat*}{2}
R_{1,1}^{0} &=& \{\epsilon , b\} \\
R_{1,2}^{0} &=& \{a\} \\
R_{1,3}^{0} &=& R_{2,1}^{0} = R_{3,1}^{0} = \emptyset \\
R_{2,2}^{0} &=& \{\epsilon \} \\
R_{2,3}^{0} &=& \{a\} \\
R_{3,2}^{0} &=& \{b\} \\
R_{3,3}^{0} &=& \{\epsilon \} \end{alignat*}
\begin{alignat*}{2}
R_{1,2}^{1} &=& R_{1,2}^{0} \cup R_{1,1}^{0} \cdot (R_{1,1}^{0})^{*} \cdot R_{1,2}^{0} \\
&=& R_{1,2}^{0} \cup (R_{1,1}^{0})^{+} \cdot R_{1,2}^{0} \\
&=& (R_{1,1}^{0})^{+} \cdot R_{1,2}^{0} \cup R_{1,2}^{0} \\
&=& (R_{1,1}^{0})^{+} \cdot R_{1,2}^{0} \\
&=& \{\epsilon , b \}^{+} \cdot \{a\} \\
&=& \{b\}^{*} \cdot \{a\} \\
&\overset{\sim}{=}& ((b)^{*} \cdot a) \\
%
R_{2,3}^{1} &=& R_{2,3}^{0} \cup R_{2,1}^{0} \cdot (R_{1,1}^{0})^{*} \cdot R_{1,3}^{0} \\
&=& R_{2,3}^{0} \cup R_{2,1}^{0} \cdot (R_{1,1}^{0})^{*} \cdot R_{1,3}^{0} \\
&=& \{a\} \cup \emptyset \cdot \{\epsilon , b\}^{*} \cdot \emptyset \\
&=& \{a\} \cup \emptyset \\
&=& \{a\} \\
&\overset{\sim}{=}& a \\
%
R_{3,2}^{1} &=& R_{3,2}^{0} \cup R_{3,1}^{0} \cdot (R_{1,1}^{0})^{*} \cdot R_{1,3}^{0} \\
&=& \{b\} \cup \emptyset \cdot \{\epsilon , b\}^{*} \cdot \emptyset \\
&=& \{b\} \cup \emptyset \cdot \{b\}^{*} \cdot \emptyset \\
&=& \{b\} \cup \emptyset \\
&=& \{b\} \\
&\overset{\sim}{=}& b \\
%
R_{2,2}^{1} &=& R_{2,2}^{0} \cup R_{2,1}^{0} \cdot (R_{1,1}^{0})^{*} \cdot R_{1,2}^{0} \\
&=& \{\epsilon \} \cup (\emptyset \cdot \{\epsilon , b\}^{*} \cdot \{a\}) \\
&=& \{\epsilon \} \cup (\emptyset \cdot \{b\}^{*} \cdot \{a\}) \\
&=& \{\epsilon \} \cup \emptyset \\
&=& \{\epsilon \} \\
&\overset{\sim}{=}& \emptyset^{*} \\
%
R_{3,3}^{1} &=& R_{3,3}^{0} \cup R_{3,1}^{0} \cdot (R_{1,1}^{0})^{*} \cdot R_{1,3}^{0} \\
&=& \{\epsilon \} \cup \emptyset \cdot \{\epsilon , b\}^{*} \cdot \emptyset \\
&=& \{\epsilon \} \cup \emptyset \cdot \{b\}^{*} \cdot \emptyset \\
&=& \{\epsilon \} \cup \emptyset \\
&=& \{\epsilon \} \\
&\overset{\sim}{=}& \emptyset^{*}
\end{alignat*}\\
\begin{alignat*}{2}
R_{1,3}^{1} &=& R_{1,3}^{0} \cup R_{1,1}^{0} \cdot (R_{1,1}^{0})^{*} \cdot R_{1,3}^{0} \\
&=& R_{1,3}^{0} \cup (R_{1,1}^{0})^{+} \cdot R_{1,3}^{0} \\
&=& (R_{1,1}^{0})^{+} \cdot R_{1,3}^{0} \cup R_{1,3}^{0}\\
&=& (R_{1,1}^{0})^{+} \cdot R_{1,3}^{0}\\
&=& \{\epsilon , b \}^{+} \cdot \emptyset \\
&=& \{b \}^{*} \cdot \emptyset \\
&=& \emptyset \\
%
R_{1,2}^{2} &=& R_{1,2}^{1} \cup R_{1,2}^{1} \cdot (R_{2,2}^{1})^{*} \cdot R_{2,2}^{1}) \\
&=& R_{1,2}^{1} \cdot (\{\epsilon \} \cup (R_{2,2}^{1})^{*} \cdot R_{2,2}^{1}) \\
&=& R_{1,2}^{1} \cdot (\{\epsilon \} \cup (R_{2,2}^{1})^{+}) \\
&=& R_{1,2}^{1} \cdot (R_{2,2}^{1})^{*} \\
&=& (\{b\}^{*} \cdot \{a\}) \cdot \{\epsilon \}^{*} \\
&=& \{b\}^{*} \cdot \{a\}\\
&\overset{\sim}{=}& ((b)^{*} \cdot a) \\
%
R_{1,3}^{2} &=& R_{1,3}^{1} \cup R_{1,2}^{1} \cdot (R_{2,2}^{1})^{*} \cdot R_{2,3}^{1} \\
&=& \emptyset \cup (\{a\} \cdot \{\epsilon \}^{*} \cdot \{a\}) \\
&=& \emptyset \cup (\{a\} \cdot \{a\}) \\
&=& \emptyset \cup \{aa\} \\
&=& \{aa\} \\
&\overset{\sim}{=}& aa \\
%
R_{3,3}^{2} &=& R_{3,3}^{1} \cup R_{3,2}^{1} \cdot (R_{2,2}^{1})^{*} \cdot R_{2,3}^{1} \\
&=& \{\epsilon \} \cup (\{b\} \cdot \{\epsilon \}^{*} \cdot \{a\}) \\
&=& \{\epsilon \} \cup (\{b\} \cdot \{a\}) \\
&=& \{\epsilon \} \cup \{ba\} \\
&=& \{\epsilon , ba\} \\
&\overset{\sim}{=}& \emptyset^{*} + b \cdot a
\end{alignat*}\\
\begin{alignat*}{2}
R_{3,2}^{2} &=& R_{3,2}^{1} \cup R_{3,2}^{1} \cdot (R_{2,2}^{1})^{*} \cdot R_{2,2}^{1} \\
&=& R_{3,2}^{1} \cdot (\{\epsilon \} \cup (R_{2,2}^{1})^{*} \cdot R_{2,2}^{1}) \\
&=& R_{3,2}^{1} \cdot (\{\epsilon \} \cup (R_{2,2}^{1})^{+}) \\
&=& R_{3,2}^{1} \cdot (R_{2,2}^{1})^{*} \\
&=& \{b\} \cdot \{\epsilon \}^{*} \\
&=& \{b\} \\
&\overset{\sim}{=}& b \\
%
R_{1,3}^{3} &=& R_{1,3}^{2} \cup R_{1,3}^{2} \cdot (R_{3,3}^{2})^{*} \cdot R_{3,3}^{2} \\
&=& R_{1,3}^{2} \cdot (\{\epsilon \} \cup (R_{3,3}^{2})^{*} \cdot R_{3,3}^{2}) \\
&=& R_{1,3}^{2} \cdot (\{\epsilon \} \cup (R_{3,3}^{2})^{+}) \\
&=& R_{1,3}^{2} \cdot (R_{3,3}^{2})^{*} \\
&=& \{aa\} \cdot \{\epsilon , ba\}^{*} \\
&=& \{aa\} \cdot \{ba\}^{*} \\
&\overset{\sim}{=}& a \cdot a \cdot (b \cdot a)^{*} \\
%
R_{1,2}^{3} &=& R_{1,2}^{2} \cup R_{1,3}^{2} \cdot (R_{3,3}^{2})^{*} \cdot R_{3,2}^{2} \\
&=& (\{b\}^{*} \cdot \{a\}) \cup (\{aa\} \cdot \{\epsilon , ba\}^{*} \cdot \{b\}) \\
&=& (\{b\}^{*} \cdot \{a\}) \cup (\{aa\} \cdot \{ba\}^{*} \cdot \{b\}) \\
&\overset{\sim}{=}& (b^{*} \cdot a) + (aa \cdot (b \cdot a)^{*} \cdot b) \\
\end{alignat*}
Der reguläre Ausdruck ergibt sich aus der Vereinigung von $R_{1,3}^{3}$ und $R_{1,2}^{3}$. Es ergibt sich daher folgendes:\\
\begin{alignat*}{2}
R_{1,3}^{3} \cup R_{1,2}^{3} &=& (\{aa\} \cdot \{ba\}^{*}) \cup ((\{b\}^{*} \cdot \{a\}) \cup (\{aa\} \cdot \{ba\}^{*} \cdot \{b\})) \\
&\overset{\sim}{=}& (aa \cdot (ba)^{*}) + (b^{*} \cdot a) + (aa \cdot (ba)^{*} \cdot b)
\end{alignat*}
Der reguläre Ausdruck ist $(aa \cdot (ba)^{*}) + (b^{*} \cdot a) + (aa \cdot (ba)^{*} \cdot b)$.
\section{} %3.6
\subsection{} %1.
\subsection{} %2.
\end{document}

170
fgi1/Aufgabenblatt4.tex Normal file
View File

@ -0,0 +1,170 @@
\documentclass[10pt,a4paper,oneside,ngerman,numbers=noenddot]{scrartcl}
\usepackage[T1]{fontenc}
\usepackage[utf8]{inputenc}
\usepackage[ngerman]{babel}
\usepackage{amsmath}
\usepackage{amsfonts}
\usepackage{amssymb}
\usepackage{paralist}
\usepackage{gauss}
\usepackage{stmaryrd}
\usepackage[locale=DE,exponent-product=\cdot,detect-all]{siunitx}
\usepackage{tikz}
\usetikzlibrary{automata,matrix,fadings,calc,positioning,decorations.pathreplacing,arrows,decorations.markings}
\usepackage{polynom}
\polyset{style=C, div=:,vars=x}
\pagenumbering{arabic}
\def\thesection{4.\arabic{section})}
\def\thesubsection{\arabic{subsection}.}
\def\thesubsubsection{(\roman{subsubsection})}
\setcounter{section}{3}
\makeatletter
\renewcommand*\env@matrix[1][*\c@MaxMatrixCols c]{%
\hskip -\arraycolsep
\let\@ifnextchar\new@ifnextchar
\array{#1}}
\makeatother
\begin{document}
\author{Jim Martens}
\title{Hausaufgaben zum 30. April}
\maketitle
\section{} %4.4
\subsection{} %1.
$G_{1}$:\\
\begin{alignat*}{2}
S &\rightarrow & aS | AB \\
A &\rightarrow & aAb | \lambda \\
B &\rightarrow & bBc | \lambda
\end{alignat*}
Ich behaupte $L(G_{1}) = \{a^{i + j}b^{i + k}c^{k}|i,j,k \geq 0\} =: M$.
Zunächst zeige ich $L(G_{1}) \subseteq M$. Sei dazu $w \in L(G_{1})$, d.h. es gilt $S \overset{*}{\Longrightarrow} w$. Wie kann $w \in V_{T}^{*}$ entstanden sein? Durch $j$-malige Anwendung der Produktion $S \rightarrow aS$ erhält man zunächst $S \overset{*}{\Longrightarrow} a^{j}S$. Anschließend muss die Produktion $S \rightarrow AB$ benutzt werden. Dies führt zu $S \overset{*}{\Longrightarrow} a^{j}S \overset{*}{\Longrightarrow} a^{j}AB$. Durch $i$-malige Anwendung der Produktion $A \rightarrow aAb$ erhält man $S \overset{*}{\Longrightarrow} a^{j}S \overset{*}{\Longrightarrow} a^{j}AB \overset{*}{\Longrightarrow} a^{j}a^{i}Ab^{i}B$. Dann muss die Produktion $A \rightarrow \lambda$ gewählt werden. Es ergibt sich daher
$S \overset{*}{\Longrightarrow} a^{j}S \overset{*}{\Longrightarrow} a^{j}AB \overset{*}{\Longrightarrow} a^{j}a^{i}Ab^{i}B \Longrightarrow a^{j}a^{i}b^{i}B$. Durch $k$-malige Anwendung der Produktion $B \rightarrow bBc$ erhält man $S \overset{*}{\Longrightarrow} a^{j}S \overset{*}{\Longrightarrow} a^{j}AB \overset{*}{\Longrightarrow} a^{j}a^{i}Ab^{i}B \Longrightarrow a^{j}a^{i}b^{i}B \overset{*}{\Longrightarrow} a^{j}a^{i}b^{i}b^{k}Bc^{k}$. Abschließend muss die Produktion $B \rightarrow \lambda$ gewählt werden.\\
\\
Insgesamt muss $w$ damit die Form $a^{j}a^{i}b^{i}b^{k}c^{k} = a^{i + j}b^{i + k}c^{k}$ haben. Es gilt also $w \in M$.\\
\\
Sei umgekehrt $w \in M$. Hat $w$ die Form $a^{i + j}b^{i + k}c^{k} = a^{j}a^{i}b^{i}b^{k}c^{k}$ für $i,j,k \geq 0$, so kann $w$ durch $j$-malige Anwendung der Regel $S \rightarrow aS$, anschließende Anwendung der Regel $S \rightarrow AB$, $i$-malige Anwendung der Regel $A \rightarrow aAb$ mit anschließender Anwendung von $A \rightarrow \lambda$, $k$-maliger Anwendung von $B \rightarrow bBc$ und anschließender Anwendung von $B \rightarrow \lambda$ abgeleitet werden. Damit gilt für jedes $w \in M$ also auch $w \in L(G_{1})$ und die Behauptung ist gezeigt.
\subsection{} %2.
$G_{2}$:\\
\begin{alignat*}{2}
S &\rightarrow & 0ABD | B1CD | A1CE | AE2D \\
A &\rightarrow & 0A | \lambda \\
B &\rightarrow & 0B1 | \lambda \\
C &\rightarrow & 1C | \lambda \\
D &\rightarrow & 2D | \lambda \\
E &\rightarrow & 1E2 | \lambda
\end{alignat*}
Ich behaupte $L(G_{2}) = \{0^{i + j}1^{i}2^{n}, 0^{i}1^{i + k}2^{n}, 0^{n}1^{i + l}2^{i}, 0^{n}1^{i}2^{i + m}|i,n \geq 0 \wedge j,k,l,m \geq 1\} =: M$.\\
Zunächst zeige ich $L(G_{2}) \subseteq M$. Sei dazu $w \in L(G_{2})$, d.h. es gilt $S \overset{*}{\Longrightarrow} w$. Wie kann $w \in V_{T}^{*}$ entstanden sein? Es gibt vier Fälle für Wörter von $L_{2}$:
\begin{enumerate}
\item $|w|_{0} > |w|_{1}$ \\
\item $|w|_{0} < |w|_{1}$ \\
\item $|w|_{1} > |w|_{2}$ \\
\item $|w|_{1} < |w|_{2}$
\end{enumerate}
Es ist natürlich möglich, dass mehrere von diesen Fällen zur gleichen Zeit zutreffen, allerdings muss immer mindestens einer zutreffen. Die jeweils nicht genannten Anzahlen sind beliebig.
Für den ersten Fall ergibt sich folgende Ableitung:\\
Zunächst wird die Produktion $S \rightarrow 0ABD$ gewählt. Durch $j-1$-malige Anwendung der Produktion $A \rightarrow 0A$ erhält man zunächst $S \overset{*}{\Longrightarrow} 00^{j-1}ABD$. Als nächstes wird die Produktion $A \rightarrow \lambda$ gewählt. Durch $i$-malige Anwendung der Produktion $B \rightarrow 0B1$ erhält man $S \overset{*}{\Longrightarrow} 00^{j-1}ABD \overset{*}{\Longrightarrow} 00^{j-1}0^{i}B1^{i}D$. Es wird die Produktion $B \rightarrow \lambda$ gewählt. Durch $n$-malige Anwendung von $D \rightarrow 2D$ erhält man $S \overset{*}{\Longrightarrow} 00^{j-1}ABD \overset{*}{\Longrightarrow} 00^{j-1}0^{i}B1^{i}D \overset{*}{\Longrightarrow} 00^{j-1}0^{i}1^{i}2^{n}D$. Abschließend wird $D \rightarrow \lambda$ gewählt. Das Wort $w$ hat demnach die Form $00^{j-1}0^{i}1^{i}2^{n} = 0^{i + j}1^{i}2^{n}$ mit $i, n \geq 0 \wedge j \geq 1$.\\
\\
Für den zweiten Fall ergibt sich folgende Ableitung:\\
Zunächst wird die Produktion $S \rightarrow B1CD$ gewählt. Durch $i$-malige Anwendung der Produktion $B \rightarrow 0B1$ erhält man $S \overset{*}{\Longrightarrow}0^{i}B1^{i}1CD$. Es wird die Produktion $B \rightarrow \lambda$ gewählt. Durch $k-1$-malige Anwendung von $C \rightarrow 1C$ erhält man $S \overset{*}{\Longrightarrow}0^{i}B1^{i}1CD \overset{*}{\Longrightarrow} 0^{i}1^{i}11^{k-1}CD$. Durch $n$-malige Anwendung von $D \rightarrow 2D$ erhält man $S \overset{*}{\Longrightarrow}0^{i}B1^{i}1CD \overset{*}{\Longrightarrow} 0^{i}1^{i}11^{k-1}CD \overset{*}{\Longrightarrow} 0^{i}1^{i}11^{k-1}2^{n}D$. Abschließend wird $D \rightarrow \lambda$ gewählt. Das Wort $w$ hat demnach die Form $0^{i}1^{i}11^{k-1}2^{n} = 0^{i}1^{i + k}2^{n}$ mit $i, n \geq 0 \wedge k \geq 1$.\\
\\
Für den dritten Fall ergibt sich diese Ableitung:\\
Zunächst wird die Produktion $S \rightarrow A1CE$ gewählt. Durch $n$-malige Anwendung der Produktion $A \rightarrow 0A$ erhält man zunächst $S \overset{*}{\Longrightarrow} 0^{n}A1CE$. Direkt danach folgt die Produktion $A \rightarrow \lambda$. Durch $l-1$-malige Anwendung der Produktion $C \rightarrow 1C$ erhält man $S \overset{*}{\Longrightarrow} 0^{n}A1CE \overset{*}{\Longrightarrow} 0^{n}11^{l-1}CE$. Anschließend folgt die Produktion $C \rightarrow \lambda$. Mit $i$-maliger Anwendung der Produktion $E \rightarrow 1E2$ erhält man $S \overset{*}{\Longrightarrow} 0^{n}A1CE \overset{*}{\Longrightarrow} 0^{n}11^{l-1}CE \overset{*}{\Longrightarrow} 0^{n}11^{l-1}1^{i}E2^{i}$. Abschließend wird die Produktion $E \rightarrow \lambda$ angewendet. Das Wort $w$ hat demnach die Form $0^{n}11^{l-1}1^{i}2^{i} = 0^{n}1^{i + l}2^{i}$ mit $i,n \geq 0 \wedge l \geq 1$. \\
\\
Für den vierten Fall ergibt sich diese Ableitung:\\
Zunächst wird die Produktion $S \rightarrow AE2D$ gewählt. Durch $n$-malige Anwendung der Produktion $A \rightarrow 0A$ erhält man zunächst $S \overset{*}{\Longrightarrow} 0^{n}AE2D$. Direkt danach folgt die Produktion $A \rightarrow \lambda$. Mit $i$-maliger Anwendung der Produktion $E \rightarrow 1E2$ erhält man $S \overset{*}{\Longrightarrow} 0^{n}AE2D \overset{*}{\Longrightarrow} 0^{n}1^{i}E2^{i}2D$. Abschließend wird die Produktion $E \rightarrow \lambda$ angewendet. Durch $m-1$-malige Anwendung der Produktion $D \rightarrow 2D$ erhält man $S \overset{*}{\Longrightarrow} 0^{n}AE2D \overset{*}{\Longrightarrow} 0^{n}1^{i}E2^{i}2D \overset{*}{\Longrightarrow} 0^{n}1^{i}2^{i}22^{m-1}D$. Abschließend wird die Produktion $D \rightarrow \lambda$ gewählt. Das Wort $w$ hat demnach die Form $0^{n}1^{i}2^{i}22^{m-1} = 0^{n}1^{i}2^{i + m}$ mit $i,n \geq 0 \wedge m \geq 1$. \\
\\
Insgesamt muss $w$ damit die Form $0^{i + j}1^{i}2^{n}, 0^{i}1^{i + k}2^{n}, 0^{n}1^{i + l}2^{i}$ oder $0^{n}1^{i}2^{i + m}$ haben. Es gilt also $w \in M$.\\
\\
Sei umgekehrt $w \in M$. Hat $w$ die Form $0^{i + j}1^{i}2^{n}$, so kann $w$ durch die im ersten Fall beschriebene Ableitungsfolge abgeleitet werden. Hat $w$ die Form $0^{i}1^{i + k}2^{n}$, so kann $w$ durch die im zweiten Fall beschriebene Ableitungsfolge abgeleitet werden. Hat $w$ die Form $0^{n}1^{i + l}2^{i}$, so kann $w$ durch die im dritten Fall beschriebene Ableitungsfolge abgeleitet werden. Hat $w$ die Form $0^{n}1^{i}2^{i + m}$, so kann $w$ durch die im vierten Fall beschriebene Ableitungsfolge abgeleitet werden. Damit gilt für jedes $w \in M$ also auch $w \in L(G_{2})$ und die Behauptung ist gezeigt.
\section{} %4.5
Zunächst werden alle $\lambda$-Produktionen nach dem Verfahren von Theorem 9.12 entfernt.
\begin{alignat*}{2}
M_{0} &=& \{C\} \\
M_{1} &=& \{C,F,S\} \\
M_{2} &=& \{C,F,S\} = M_{1} \\
V_{\lambda} &=& M_{2}
\end{alignat*}
Daraus ergibt sich folgende Grammatik:\\
\begin{alignat*}{2}
S &\rightarrow & AB | AB1B | AD | DE | C \\
A &\rightarrow & EBEE \\
B &\rightarrow & A | E | SE \\
D &\rightarrow & CBG | BG | 0G | 1D \\
E &\rightarrow & 0 | 1 \\
F &\rightarrow & 0E | EF | CC | C | E
\end{alignat*}\\
Anschließend wird nach dem Verfahren von Theorem 9.11 reduziert. Zunächst werden dabei alle unproduktiven Nonterminale entfernt.\\
\begin{alignat*}{2}
M_{0} &=& \{0,1\} \\
M_{1} &=& \{0,1,E\} \\
M_{2} &=& \{0,1,E,F,B\} \\
M_{3} &=& \{0,1,E,F,B,A\} \\
M_{4} &=& \{0,1,E,F,B,A,S\} \\
M_{5} &=& \{0,1,E,F,B,A,S\} = M_{4}
\end{alignat*}
Es ergibt sich folgende Grammatik:\\
\begin{alignat*}{2}
S &\rightarrow & AB | AB1B \\
A &\rightarrow & EBEE \\
B &\rightarrow & A | E | SE \\
E &\rightarrow & 0 | 1 \\
F &\rightarrow & 0E | EF | E
\end{alignat*}
Daraufhin werden alle Produktionen entfernt, die auf der linken oder rechten Seite unerreichbare Nonterminale haben.
\begin{alignat*}{2}
M_{0} &=& \{S\} \\
M_{1} &=& \{S,A,B\} \\
M_{2} &=& \{S,A,B,E\} \\
M_{3} &=& \{S,A,B,E\} = M_{2}
\end{alignat*}
Es ergibt sich die Grammatik:\\
\begin{alignat*}{2}
S &\rightarrow & AB | AB1B \\
A &\rightarrow & EBEE \\
B &\rightarrow & A | E | SE \\
E &\rightarrow & 0 | 1
\end{alignat*}
Im dritten Schritt werden die Kettenregeln entfernt. Daraus ergibt sich diese Grammatik:\\
\begin{alignat*}{2}
S &\rightarrow & AB | AB1B \\
A &\rightarrow & EBEE \\
B &\rightarrow & EBEE | 0 | 1 | SE \\
E &\rightarrow & 0 | 1
\end{alignat*}
Im vierten Schritt werden lange Terminalregeln ersetzt. Daraus ergibt sich diese Grammatik:\\
\begin{alignat*}{2}
S &\rightarrow & AB | AB\left\langle 1 \right\rangle B \\
A &\rightarrow & EBEE \\
B &\rightarrow & EBEE | 0 | 1 | SE \\
E &\rightarrow & 0 | 1 \\
\left\langle 1 \right\rangle &\rightarrow & 1
\end{alignat*}
Im fünften Schritt werden zu lange Regeln verkürzt. Daraus ergibt sich diese Grammatik:\\
\begin{alignat*}{2}
S &\rightarrow & AB | \left\langle AB \right\rangle \left\langle \left\langle 1 \right\rangle B \right\rangle \\
\left\langle AB \right\rangle &\rightarrow & AB \\
\left\langle \left\langle 1 \right\rangle B \right\rangle &\rightarrow &\left\langle 1 \right\rangle B \\
A &\rightarrow & \left\langle EB \right\rangle \left\langle EE \right\rangle \\
\left\langle EB \right\rangle &\rightarrow & EB \\
\left\langle EE \right\rangle &\rightarrow & EE \\
B &\rightarrow & \left\langle EB \right\rangle \left\langle EE \right\rangle | 0 | 1 | SE \\
E &\rightarrow & 0 | 1 \\
\left\langle 1 \right\rangle &\rightarrow & 1
\end{alignat*}
Im sechsten Schritt wird die ursprüngliche Sprache wiederhergestellt, indem eine $\lambda$-Regel hinzugenommen wird. Daraus ergibt sich diese Grammatik:\\
\begin{alignat*}{2}
S_{neu} &\rightarrow & \lambda | AB | \left\langle AB \right\rangle \left\langle \left\langle 1 \right\rangle B \right\rangle \\
\left\langle AB \right\rangle &\rightarrow & AB \\
\left\langle \left\langle 1 \right\rangle B \right\rangle &\rightarrow &\left\langle 1 \right\rangle B \\
A &\rightarrow & \left\langle EB \right\rangle \left\langle EE \right\rangle \\
\left\langle EB \right\rangle &\rightarrow & EB \\
\left\langle EE \right\rangle &\rightarrow & EE \\
B &\rightarrow & \left\langle EB \right\rangle \left\langle EE \right\rangle | 0 | 1 | SE \\
E &\rightarrow & 0 | 1 \\
\left\langle 1 \right\rangle &\rightarrow & 1
\end{alignat*}
\section{} %4.6
\end{document}

93
fgi1/Aufgabenblatt5.tex Normal file
View File

@ -0,0 +1,93 @@
\documentclass[10pt,a4paper,oneside,ngerman,numbers=noenddot]{scrartcl}
\usepackage[T1]{fontenc}
\usepackage[utf8]{inputenc}
\usepackage[ngerman]{babel}
\usepackage{amsmath}
\usepackage{amsfonts}
\usepackage{amssymb}
\usepackage{paralist}
\usepackage{gauss}
\usepackage{stmaryrd}
\usepackage[locale=DE,exponent-product=\cdot,detect-all]{siunitx}
\usepackage{tikz}
\usetikzlibrary{automata,matrix,fadings,calc,positioning,decorations.pathreplacing,arrows,decorations.markings}
\usepackage{polynom}
\polyset{style=C, div=:,vars=x}
\pagenumbering{arabic}
\def\thesection{5.\arabic{section})}
\def\thesubsection{\arabic{subsection}.}
\def\thesubsubsection{(\roman{subsubsection})}
\setcounter{section}{3}
\makeatletter
\renewcommand*\env@matrix[1][*\c@MaxMatrixCols c]{%
\hskip -\arraycolsep
\let\@ifnextchar\new@ifnextchar
\array{#1}}
\makeatother
\begin{document}
\author{Jim Martens}
\title{Hausaufgaben zum 7. Mai}
\maketitle
\section{} %5.4
\subsection{} %1.
$G_{1}$:\\
\begin{alignat*}{2}
S &\rightarrow & aSb | A \\
A &\rightarrow & bAa | \lambda
\end{alignat*}
Behauptung: Es gilt $L(G_{1}) = L_{1}$.\\
Zunächst zeige ich $L(G_{1}) \subseteq L_{1}$. Sei dazu $w \in L(G_{1})$, d.h. es gilt $S \overset{*}{\Longrightarrow} w$. Durch $n$-malige Anwendung der Produktion $S \rightarrow aSb$ erhält man zunächst $S \overset{*}{\Longrightarrow} a^{n}Sb^{n}$. Anschließend muss die Produktion $S \rightarrow A$ gewählt werden. Durch $m$-malige Anwendung der Regel $A \rightarrow bAa$ erhält man $S \overset{*}{\Longrightarrow} a^{n}Sb^{n} \overset{*}{\Longrightarrow} a^{n}b^{m}Aa^{m}b^{n}$. Abschließend wird die Produktion $A \rightarrow \lambda$ gewählt. Insgesamt muss $w$ damit die Form $a^{n}b^{m}a^{m}b^{n}$ haben. Es gilt also $w \in L_{1}$.\\
\\
Sei umgekehrt $w \in L_{1}$. Hat $w$ die Form $a^{n}b^{m}a^{m}b^{n}$ für $n,m \geq 0$, so kann $w$ durch $n$-malige Anwendung der Produktion $S \rightarrow aSb$, Anwendung der Regel $S \rightarrow A$, $m$-malige Anwendung von $A \rightarrow bAa$ und abschließende Anwendung von $A \rightarrow \lambda$ abgeleitet werden.\\
Damit gilt für jedes $w \in L_{1}$ demnach auch $w \in L(G_{1})$ und die Behauptung ist gezeigt.\\
\\
Bei Anwendung des Pumping-Lemmas kommt es bei der Wahl des Wortes zu Problemen. Bei einer kontextfreien Sprache gilt das Pumping-Lemma für alle Wörter der Sprache. Möchte man dies nun zeigen, so müsste man unendlich viele Wörter zeigen, was nicht machbar ist.\\
Sollte man einen Widerspruch zeigen wollen, so wird man kein Wort der Sprache finden, bei dem es einen gibt, denn bei einer kontextfreien Sprache gilt das Pumping-Lemma für jedes Wort.
\subsection{} %2.
\begin{alignat*}{2}
L_{2} &=& \{a^{n}b^{m}a^{n}b^{m} | n,m \geq 0\}
\end{alignat*}\\
Angenommen $L_{2}$ wäre kontextfrei.\\
Sei $n$ die Zahl aus dem Pumping Lemma. Wähle $z = a^{n}b^{n}a^{n}b^{n}$ mit $n \in \mathbb{N}$.\\
Die ersten $a$s seien durch $a_{1}$ markiert und die ersten $b$s durch $b_{1}$. Die jeweils zweiten Blöcke durch den Index $2$.\\
Also $z \in L_{2}, |z| = 4n$.\\
Also existiert eine Zerlegung $z = uvwxy$ mit $(i)$ und $(ii)$. Ich führe nun jede dieser Zerlegungen zu einem Widerspruch mit der dritten Eigenschaft des Pumping Lemmas.\\
\begin{enumerate}[i)]
\item $|vx| \geq 1$
\item $|vwx| \leq n$
\item $\forall i \geq 0:uv^{i}wx^{i}y \in L$
\end{enumerate}
Zunächst gibt es wegen $|vwx| \leq n$ und der Form von $z$ sieben Fälle zu unterscheiden: $vwx \in \{a_{1}\}^{*}, vwx \in \{a_{1}\}^{*}\{b_{1}\}^{*}, vwx \in \{b_{1}\}^{*}, vwx \in \{b_{1}\}^{*}\{a_{2}\}^{*}, vwx \in \{a_{2}\}^{*}, vwx \in \{a_{2}\}^{*}\{b_{2}\}^{*} \text{ und } vwx \in \{b_{2}\}^{*}$. In den Fällen $vwx \in \{a_{1}\}^{*}, vwx \in \{b_{1}\}^{*}, vwx \in \{a_{2}\}^{*}$ und $vwx \in \{b_{2}\}^{*}$ führt wegen $|vx| \geq 1$ die Betrachtung von $uv^{2}wx^{2}y$ bereits zum Widerspruch. Es ist dann $uv^{2}wx^{2}y = a^{n+j}b^{n}a^{n}b^{n}$, $uv^{2}wx^{2}y = a^{n}b^{n+j}a^{n}b^{n}$, $uv^{2}wx^{2}y = a^{n}b^{n}a^{n+j}b^{n}$ bzw. $uv^{2}wx^{2}y = a^{n}b^{n}a^{n}b^{n+j}$ mit $j > 0$ und folglich gilt nicht mehr, dass der Exponent von den beiden $a$- bzw. $b$-Blöcken gleich sein muss.\\
\\
Gilt in $vwx \in \{a_{1}\}^{*}\{b_{1}\}^{*}, vwx \in \{b_{1}\}^{*}\{a_{2}\}^{*}$ bzw. $vwx \in \{a_{2}\}^{*}\{b_{2}\}^{*}$, dass $v$ oder $x$ bereits aus mehr als einem Symbol aufgebaut sind (also selbst in $\{a\}^{+}\{b\}^{+}$ bzw. $\{b\}^{+}\{a\}^{+}$ sind), so führt $uv^{2}wx^{2}y$ sofort zu einem Widerspruch, da dieses Wort dann nicht einmal mehr in $\{a\}^{*}\{b\}^{*}\{a\}^{*}\{b\}^{*}$ ist (und damit ganz bestimmt nicht in $L_{2}$).\\
\\
Ist jedoch im Fall $vwx \in \{a_{1}\}^{*}\{b_{1}\}^{*}$ das $v$ aus den $a$s und das $x$ in den $b$s, so führt ähnlich wie bei den zuerst diskutierten Fällen die Betrachtung von $uv^{2}wx^{2}y = a^{n+j}b^{n+k}a^{n}b^{n}$ zu einem Widerspruch ($n+j = n$ bzw. $n+k = n$ gilt nicht, da wegen $|vx| \geq 1$ auch $j+k \geq 1$ sein muss). Der Fall $vwx \in \{a_{2}\}^{*}\{b_{2}\}^{*}$ führt analog zu einem Widerspruch (lediglich mit den beiden hinteren Blöcken).\\
Ist im Fall von $vwx \in \{b_{1}\}^{*}\{a_{2}\}^{*}$ das $v$ in den $b$s und das $x$ in den $a$s, so führt ebenfalls ähnlich zu den zuerst diskutierten Fällen die Betrachtung von $uv^{2}wx^{2}y = a^{n}b^{n+j}a^{n+k}b^{n}$ zu einem Widerspruch (abermals gilt $n+j = n$ bzw. $n+k = n$ nicht, da $j+k \geq 1$ sein muss).\\
\\
Damit sind alle Fälle zum Widerspruch geführt und es gibt also keine Zerlegung von $z$ in $uvwxy$ derart, dass die drei Bedingungen des Pumping Lemmas erfüllt sind. Folglich ist die ursprüngliche Annahme, $L_{2}$ wäre kontextfrei nicht haltbar und $L_{2}$ ist also nicht kontextfrei.
\section{} %5.5
\subsection{} %1.
$G_{1}$:\\
\begin{alignat*}{2}
S &\rightarrow & aSc | A \\
A &\rightarrow & bc | \lambda
\end{alignat*}
Behauptung: Es gilt $L(G_{1}) = L_{1}$.\\
Zunächst zeige ich $L(G_{1}) \subseteq L_{1}$. Sei dazu $w \in L(G_{1})$, d.h. es gilt $S \overset{*}{\Longrightarrow} w$. Durch $i$-malige Anwendung der Produktion $S \rightarrow aSc$ erhält man zunächst $S \overset{*}{\Longrightarrow} a^{i}Sc^{i}$. Anschließend muss die Produktion $S \rightarrow A$ gewählt werden. Durch $j$-malige Anwendung der Regel $A \rightarrow bAc$ erhält man $S \overset{*}{\Longrightarrow} a^{i}Sc^{i} \overset{*}{\Longrightarrow} a^{i}b^{j}Ac^{i+j}$. Abschließend wird die Produktion $A \rightarrow \lambda$ gewählt. Insgesamt muss $w$ damit die Form $a^{i}b^{j}c^{i+j} = a^{i}b^{j}c^{k}$ für $k = i+j$ haben. Es gilt also $w \in L_{1}$.\\
\\
Sei umgekehrt $w \in L_{1}$. Hat $w$ die Form $a^{i}b^{j}c^{k} = a^{i}b^{j}c^{i+j}$ für $k = i+j$, so kann $w$ durch $i$-malige Anwendung der Produktion $S \rightarrow aSc$, Anwendung der Regel $S \rightarrow A$, $j$-malige Anwendung von $A \rightarrow bAc$ und abschließende Anwendung von $A \rightarrow \lambda$ abgeleitet werden.\\
Damit gilt für jedes $w \in L_{1}$ demnach auch $w \in L(G_{1})$ und die Behauptung ist gezeigt.
\subsection{} %2.
\section{} %5.6
\subsection{} %1.
$z_{0}$ ist der Start- und $z_{4}$ der einzige Endzustand. Im Zustand $z_{0}$ können beliebig viele $a$s gelesen werden (auch keine). Anschließend kann bei leerer Eingabe in den Endzustand gewechselt werden oder bei einem $b$ in der Eingabe in den Zustand $z_{1}$. Der Zustand kann nur wieder verlassen werden, wenn mindestens ein weiteres $b$ von der Eingabe gelesen wird. Im Zustand $z_{2}$ befindet man sich demnach nur, wenn zuvor jeweils 2 $b$s gelesen wurden. Entweder direkt von $z_{0}$ nach $z_{1}$ und von $z_{1}$ nach $z_{2}$ oder von $z_{2}$ nach $z_{1}$ und von $z_{1}$ nach $z_{2}$.\\
Nur beim Wechsel von Zustand $z_{1}$ nach $z_{2}$ wird ein $B$ auf den Keller gelegt. Sind alle $b$s gelesen, liegen demnach also halb so viele $B$s auf dem Keller, wie $b$s gelesen wurden.\\
In den Zustand $z_{3}$ kann nur gewechselt werden, wenn ein $a$ gelesen wird und ein $B$ auf dem Keller liegt. $z_{3}$ kann erst wieder verlassen werden, wenn alle $B$s wieder vom Keller gelöscht worden sind.\\
Zum Schluss kann in den Endzustand gewechselt werden. Befindet man sich im Endzustand wurden also beliebig viele $a$s (auch keine), gefolgt von keinen weiteren Zeichen oder $2n$ $b$s und $n$ $a$s gelesen.\\
Ist $w \in L$, so enthält das Wort zunächst beliebig viele $a$s, dann keine weiteren Zeichen oder aber $2n$ $b$s und $n$ $a$s. Dies ist gleichbedeutend mit dem Endzustand des Automaten.\\
Ist umgekehrt $w \in L(A)$, so muss das Lesen von $w$ den Automaten $A$ in den Zustand $z_{4}$ überführen, da dies der einzige Endzustand ist. Dies ist aber gleichbedeutend damit, dass nach Konstruktion von $A$ auch $w = a^{m}b^{2n}a^{n}$, also $w \in L$ gilt.
\subsection{} %2.
\end{document}

99
fgi1/Aufgabenblatt6.tex Normal file
View File

@ -0,0 +1,99 @@
\documentclass[10pt,a4paper,oneside,ngerman,numbers=noenddot]{scrartcl}
\usepackage[T1]{fontenc}
\usepackage[utf8]{inputenc}
\usepackage[ngerman]{babel}
\usepackage{amsmath}
\usepackage{amsfonts}
\usepackage{amssymb}
\usepackage{paralist}
\usepackage{gauss}
\usepackage{stmaryrd}
\usepackage[locale=DE,exponent-product=\cdot,detect-all]{siunitx}
\usepackage{tikz}
\usetikzlibrary{automata,matrix,fadings,calc,positioning,decorations.pathreplacing,arrows,decorations.markings}
\usepackage{polynom}
\polyset{style=C, div=:,vars=x}
\pagenumbering{arabic}
\def\thesection{6.\arabic{section})}
\def\thesubsection{\arabic{subsection}.}
\def\thesubsubsection{(\roman{subsubsection})}
\setcounter{section}{3}
\makeatletter
\renewcommand*\env@matrix[1][*\c@MaxMatrixCols c]{%
\hskip -\arraycolsep
\let\@ifnextchar\new@ifnextchar
\array{#1}}
\makeatother
\addtolength{\parskip}{\baselineskip}
\begin{document}
\author{Jim Martens}
\title{Hausaufgaben zum 14. Mai}
\maketitle
\section{} %6.4
\subsection{} %1.
Zunächst werden links und rechts vom Wort Marker geschrieben. Der Lesekopf wird auf das letzte $a$ bewegt. Nun wird der Lesekopf nach links bewegt, bis der linke Marker erreicht wird. Auf dem Weg wird das erste, dritte, fünfte usw. gelesene a durch ein + ersetzt. Anschließend wird das Band noch einmal komplett durchlaufen, dabei werden die mit + beschriebenen Felder ignoriert. Die TM merkt sich die Parität ($0$, $1$, $0$, etc.) in jeweils einem Zustand. Das zuerst gelesene $a$ erzeugt die Parität $1$, das zweite $0$, usw. bis der Lesekopf den rechten Marker erreicht. Die aktuelle Parität wird auf das erste unbeschriebene Feld links vom linken Marker geschrieben.
Nun bewegt sich der Lesekopf zurück auf das rechte Ende des Wortes (letztes $a$). Jetzt bewegt sich der Lesekopf wieder nach links und ersetzt das erste, dritte, fünfte usw. gelesene $a$ durch ein +. Es wird wieder das Band abgelaufen und die Parität gemerkt. Das Ergebnis wird links vom linken Marker auf das erste unbeschriebene Feld geschrieben.
Dies wiederholt sich nun so lange bis zwischen den beiden Markern keine $a$s mehr stehen. Nun werden die ganzen +-Zeichen gelöscht und abschließend auch die Marker. Der Lesekopf bewegt sich auf den Anfang der jetzt noch beschriebenen Felder.
\subsection{} %2.
\begin{tikzpicture}[shorten >=1pt,node distance=2.0cm,on grid,auto,/tikz/initial text=,>=stealth',text centered]
\node[initial,state] (z0) {$z_{0}$};
\node[state] (z1) [below=2.0 of z0] {$z_{1}$};
\node[state] (z2) [right=2.0 of z1] {$z_{2}$};
\node[state] (z3) [right=3.0 of z0] {$z_{3}$};
\node[state] (z4) [below right=4.0 and 3.0 of z0] {$z_{4}$};
\node[state] (z5) [right=4.0 of z3] {$z_{5}$};
\node[state] (z6) [right=4.0 of z4] {$z_{6}$};
\node[state] (z7) [below right=2.0 and 2.0 of z5] {$z_{7}$};
\node[state] (z8) [left=2.0 of z7] {$z_{8}$};
\node[state] (z9) [below right=2.0 and 4.0 of z8] {$z_{9}$};
\node[state,accepting] (z10) [below=2.0 of z9] {$z_{10}$};
\path[every node/.style={font=\scriptsize}]
(z0) edge[loop above,->] node[above,align=center] {0,0,L\\1,1,L} (z0)
(z0) edge[->] node[left] {\#,\$,R} (z1)
(z1) edge[loop below,->] node[below,align=center] {0,0,R\\1,1R} (z1)
(z1) edge[->] node[above] {\#,\$,L} (z2)
(z2) edge[->] node[near start] {1,\#,R} (z3)
(z2) edge[->] node[below left] {0,\#,R} (z4)
(z3) edge[loop above,->] node[above] {\#,\#,R} (z3)
(z3) edge[->] node[near start] {\$,\$,R} (z5)
(z4) edge[loop below,->] node[below] {\#,\#,R} (z4)
(z4) edge[->] node[below] {\$,\$,R} (z6)
(z5) edge[loop above,->] node[above,align=center] {0,0,R\\1,1,R} (z5)
(z6) edge[loop below,->] node[below,align=center] {1,1,R\\0,0,R} (z6)
(z5) edge[->] node[near start] {\#,1,L} (z7)
(z6) edge[->] node[above left,near start] {\#,0,L} (z7)
(z2) edge[->] node[near start] {\$,\$,-} (z8)
(z7) edge[->] node[above] {\$,\$,L} (z8)
(z7) edge[loop right,->] node[right,align=center] {1,1,L\\0,0,L} (z7)
(z8) edge[->] node[near end] {1,\#,R} (z3)
(z8) edge[->] node[above] {0,\#,R} (z4)
(z8) edge[loop above,->] node[above] {\#,\#,L} (z8)
(z8) edge[->] node[above right,near end] {\$,\#,R}
(z9)
(z9) edge[loop right,->] node[right] {\#,\#,R} (z9)
(z9) edge[->] node[right] {\$,\#,R} (z10);
\end{tikzpicture}\\
Zu Beginn wird im Zustand $z_{0}$ solange nach links gegangen, bis man an den unbeschriebenen Rand kommt. Dann wird das erste unbeschriebene Feld durch ein \$-Zeichen ersetzt und im Zustand $z_{1}$ wird einmal bis an das rechte Ende des Wortes gelaufen. Auch dort wird im Übergang zu $z_{2}$ das erste unbeschriebene Feld durch \$-Zeichen ersetzt.
Damit steht der Lesekopf nun auf dem letzten Zeichen des Wortes $w$ und das Wort $w$ ist vorne und hinten durch ein \$-Zeichen begrenzt.
Anschließend wird entweder eine $0$ oder eine $1$ gelesen. Im Falle der $1$ wird sie durch $\#$ ersetzt. Der Lesekopf bewegt sich am rechten Marker vorbei und schreibt die $1$ auf das Band ($z_{2} \rightarrow z_{3} \rightarrow z_{5} \rightarrow z_{7}$. Nun bewegt er sich wieder am Marker vorbei ($z_{7} \rightarrow z_{8}$), durchläuft die unbeschriebenen Felder bis er auf eine $0$ oder eine $1$ trifft. Im Falle der $1$ löscht er diese und durchläuft alle unbeschriebenen Felder, die rechts davon stehen bis er wieder auf den rechten Marker trifft. Er durchläuft alle hier bereits geschriebenen Felder und schreibt auf das erste unbeschriebene Feld eine $1$.
Folgend werden alle bereits beschriebenen Felder und der Marker, sowie alle unbeschriebenen Felder links vom Marker durchlaufen, bis wieder auf eine $1$ oder eine $0$ getroffen wird. Für die $0$ funktioniert es analog. nur werden statt $z_{3}$ und $z_{5}$ die Zustände $z_{4}$ und $z_{6}$ durchlaufen.
Das wiederholt sich so lange, bis das ganze Wort gelesen wurde, zwischen den beiden Markern also nur noch unbeschriebene Felder stehen, und der Lesekopf am linken Marker ankommt.
Damit wurde das Wort $w$ nun komplett gelesen und in umgekehrter Reihenfolge rechts vom rechten Marker wieder aufgeschrieben.
Abschließend wird der linke Marker gelöscht, dann werden alle unbeschriebenen Felder nach rechts durchlaufen bis der Lesekopf auf den rechten Marker trifft. Dieser wird auch noch gelöscht und der Lesekopf steht am Beginn des umgekehrten Wortes $w$. Die TM ist im Endzustand und akzeptiert damit das Wort $w$.
Ist das Eingabewort leer, so wird direkt der linke Marker geschrieben ($z_{0} \rightarrow z_{1}$) und dann der rechte ($z_{1} \rightarrow z_{2}$), wobei der Lesekopf nun auf dem linken Marker steht. Er hält auf dem Marker ($z_{2} \rightarrow z_{8}$), löscht ihn und wechselt dabei auf den rechten Marker ($z_{8} \rightarrow z_{9}$), um auch diesen sofort zu löschen ($z_{9} \rightarrow z_{10}$). Der Lesekopf befindet sich nun zwei Felder weiter rechts als am Anfang, da aber das ganze Band nun genauso leer ist wie am Anfang, ist die Position irrelevant. Die TM ist auch hier im Endzustand, womit klar ist, dass das leere Wort das leere Wort ergibt.
\pagebreak
\section{} %6.5
\end{document}

61
fgi1/Aufgabenblatt7.tex Normal file
View File

@ -0,0 +1,61 @@
\documentclass[10pt,a4paper,oneside,ngerman,numbers=noenddot]{scrartcl}
\usepackage[T1]{fontenc}
\usepackage[utf8]{inputenc}
\usepackage[ngerman]{babel}
\usepackage{amsmath}
\usepackage{amsfonts}
\usepackage{amssymb}
\usepackage{paralist}
\usepackage{gauss}
\usepackage{stmaryrd}
\usepackage[locale=DE,exponent-product=\cdot,detect-all]{siunitx}
\usepackage{tikz}
\usetikzlibrary{automata,matrix,fadings,calc,positioning,decorations.pathreplacing,arrows,decorations.markings}
\usepackage{polynom}
\polyset{style=C, div=:,vars=x}
\pagenumbering{arabic}
\def\thesection{7.\arabic{section})}
\def\thesubsection{\arabic{subsection}.}
\def\thesubsubsection{(\roman{subsubsection})}
\setcounter{section}{2}
\makeatletter
\renewcommand*\env@matrix[1][*\c@MaxMatrixCols c]{%
\hskip -\arraycolsep
\let\@ifnextchar\new@ifnextchar
\array{#1}}
\makeatother
\addtolength{\parskip}{\baselineskip}
\begin{document}
\author{Jim Martens}
\title{Hausaufgaben zum 28. Mai}
\maketitle
\section{} %7.3
\subsection{} %1.
Ich erstelle zunächst ein Array $A$ aus der Länge $|V-1|$. In dieses Array sollen die Knotenbezeichner geschrieben werden. Die Reihenfolge der Knotenbezeichner von vorne nach hinten entspricht der Reihenfolge, in denen die Knoten durchlaufen werden. \\
Wenn es eine Verbindung von $s$ nach $t$ gibt, dann verbraucht sie maximal $O(V-1)$ Platz. Der Startknoten $s$ muss nicht gespeichert werden.\\
\\
Dies folgt so: Nichtdeterministisch wird für $A[1]$ ein Bezeichner der Nachfolgeknoten von $s$ gewählt. Anschließend wird nichtdeterministisch ein Wert für $A[2]$ ausgewählt, der ein Knotenbezeichner des Knoten sein muss, der für $A[1]$ gewählt wurde. Analog funktioniert es für $A[3]$, $A[4]$ usw. Um diese nichtdeterministischen Schritte zu verdeutlichen, gehe ich beispielsweise von $|V|=4$ (vier Knoten) und einem Array mit drei Elementen aus. $s$ sei $v_{1}$ und $t$ sei $v_{4}$. Ich gehe ferner von einer TM aus. So bin ich am Anfang dann in einem Zustand $z_{0}$ und der Arrayinhalt lässt sich darstellen als $[\#,\#,\#]$, wobei die $\#$ dafür stehen, dass das Arrayelement an dieser Stelle noch nicht initialisiert wurde. Nach dem ersten nichtdeterministischen Schritt sind wir nun im Zustand $z_{1}$ und der Arrayinhalt hat die Form $[v_{2},\#,\#]$ oder $[v_{3},\#,\#]$. Notiere ich die Konfiguration der TM als ein Tupel bestehend aus dem Zustand $z$ und dem Arrayinhalt, so bin ich nun also nichtdeterministisch in den Konfigurationen $(z_{1},[v_{2},\#,\#])$ und $(z_{1},[v_{3},\#,\#])$. Nun wird in den Zustand $z_{2}$ gewechselt und der Nachfolgeknoten für die bisher gewählten Knoten gewählt bzw. geraten. Ich bin nun in den Konfigurationen $(z_{2},[v_{2},v_{3},\#,\#])$, $(z_{2},[v_{2},v_{4},\#,\#])$, $(z_{2},[v_{3},v_{1},\#,\#])$ und $(z_{2},[v_{3},v_{2},\#,\#])$. Dies setzt sich nun fort, sodass ich dann am Ende nichtdeterministisch in den Konfigurationen $(z_{4},[x,y,z,w])$ bin, wobei $x,y,z$ und $w$ alle Nachfolgeknoten des jeweils vorherigen Knoten durchlaufen. Dies ist ähnlich zu einem NFA, der nach Lesen eines Symbols nichtdeterministisch in verschiedenen Zuständen sein kann.\\
\\
Zurück zu eigentlichen Aufgabe: Nachdem nun alle möglichen Pfade im Graphen nichtdeterministisch durchlaufen worden sind, braucht nur noch überprüft zu werden, ob der Pfad korrekt ist. Dazu muss jedes Array nur mittels for-each-Schleife durchlaufen werden und auf Vorhandensein von $t$ geprüft werden. Kommt $t$ im Array vor, dann ist dies eine Erfolgsrechnung und die Überprüfung weiterer Arrays ist irrelevant. Kommt $t$ in allen Arrays nicht vor, dann existiert kein Pfad von $s$ nach $t$.\\
\\
Die Laufzeit liegt bei maximal $O(2 \cdot (V-1))$, da zunächst maximal $O(V-1)$ Schritte benötigt werden, um den Pfad zu raten (man beachte, dass hier in einer Rechnung nur maximal $O(V-1)$ Schritte gemacht werden, auch wenn die Anzahl der Schritte in allen Rechnungen viel höher ist, aber es geht eben gerade um die (maximale) Anzahl der Schritte in einer Rechnung) und dann noch weitere maximal $O(V-1)$ Schritte, um den Pfad zu überprüfen. Der Platzbedarf liegt lediglich bei maximal $O(V-1)$, da dies ausreicht, um einen geratenen Pfad zu speichern. Auch hier beachte man, dass es nur um den (maximalen) Platzbedarf in einer Rechnung geht, nicht um den Platzbedarf über alle nichtdeterministischen Rechnungen.
\subsection{} %2.
Bei maximal logarithmisch viel Platz ist das Verfahren nahezu analog. Allerdings werden nicht alle Knoten eines Pfades (außer s) gespeichert, sondern immer nur der letzte.
Nehme ich wieder das Beispiel mit den vier Knoten, dann ergibt sich für $z_{0}$ $[v_{1}]$. Für $z_{1}$ ergibt sich $[v_{2}],[v_{3}]$. Für $z_{2}$ ergibt sich $[v_{1}],[v_{2}],[v_{3}],[v_{4}]$. In diesem Fall kann die TM bereits hier abbrechen, da bei den vier geratenen Knoten jeder Knoten enthalten ist. Weitere Schritte würden also keine zusätzlichen Infos bringen. Nun können die Rechnungen überprüft werden, wobei pro Rechnung maximal $O(V-1)$ Schritte für das Raten des Pfades und $O(1)$ Schritte für das Überprüfen nötig sind. Insgesamt benötigt der Algorithmus also $O(V)$ Schritte Zeit. Er verbraucht nur $O(1)$ Platz, was einem logarithmischen Platzverbrauch entspricht.\\
Für den Fall $|V|=2$ müsste gar nichts gespeichert werden, da die TM hier nur zwei Optionen hat: $s$ hat einen Nachfolgeknoten oder keinen. Gibt es einen, dann existiert eine Verbindung, gibt es keinen, dann nicht.
\section{} %7.4
\subsection{} %1.
Es werden nichtdeterministisch alle Kanten aus $E_{2}$ folgendermaßen überprüft:\\
\begin{enumerate}
\item Bestimmen der beiden verbundenen Knoten aus $V_{2}$.
\item Bestimmen der Bilder dieser Knoten
\item Nichtdeterministisch überprüfen, ob eine entsprechende Kante in $E_{1}$ existiert.
\item Gibt es keine solche Kante, dann kann abgebrochen werden.
\end{enumerate}
Wurde nicht abgebrochen, dann kann es Teilmengen geben, die die Bedingung erfüllen.
Der Algorithmus dauert $O(3)$ lang. Damit dauert der Algorithmus nur polynomiell lang.
\subsection{} %2.
\end{document}

137
fgi1/Aufgabenblatt8.tex Normal file
View File

@ -0,0 +1,137 @@
\documentclass[10pt,a4paper,oneside,ngerman,numbers=noenddot]{scrartcl}
\usepackage[T1]{fontenc}
\usepackage[utf8]{inputenc}
\usepackage[ngerman]{babel}
\usepackage{amsmath}
\usepackage{amsfonts}
\usepackage{amssymb}
\usepackage{paralist}
\usepackage{gauss}
\usepackage{stmaryrd}
\usepackage[locale=DE,exponent-product=\cdot,detect-all]{siunitx}
\usepackage{tikz}
\usetikzlibrary{automata,matrix,fadings,calc,positioning,decorations.pathreplacing,arrows,decorations.markings}
\usepackage{polynom}
\polyset{style=C, div=:,vars=x}
\pagenumbering{arabic}
\def\thesection{8.\arabic{section})}
\def\thesubsection{\arabic{subsection}.}
\def\thesubsubsection{(\roman{subsubsection})}
\setcounter{section}{1}
\makeatletter
\renewcommand*\env@matrix[1][*\c@MaxMatrixCols c]{%
\hskip -\arraycolsep
\let\@ifnextchar\new@ifnextchar
\array{#1}}
\makeatother
\addtolength{\parskip}{\baselineskip}
\begin{document}
\author{Jim Martens}
\title{Hausaufgaben zum 4. Juni}
\maketitle
\section{} %8.2
\textit{Behauptung}\\
Für alle Formeln F$\, \in \mathcal{L}_{AL}$ gilt, $|\text{Tf(F)}| \leq |\text{F}|$.\\
\\
\textit{Induktionsanfang}\\
Teilbeweis für die auf atomare Formeln eingeschränkte Behauptung: Für jedes Aussagensymbol
A$\, \in \mathcal{A}s_{AL}$ gilt: $|\text{Tf(A)}| \leq |\text{A}|$.\\
A hat nur eine Teilformel und zwar sich selbst. Die Länge von A beträgt ebenso eins. Demnach ergibt sich $1 \leq 1$, was offensichtlich gilt.\\
\\
\textit{Induktionsannahme}\\
Es seien G$_{1}, \,$G$_{2} \in \mathcal{L}_{AL}$ Formeln, für die gilt: $|\text{Tf(G}_{1}\text{)}| \leq |\text{G}_{1}|$ und $|\text{Tf(G}_{2}\text{)}| \leq |\text{G}_{2}|$.\\
\\
\textit{Induktionsschritt}\\
Fall: $\neg \,$G$_{1}$\\
Teilbeweis für $|\text{Tf(}\neg \text{G}_{1}\text{)}| \leq |\neg \,\text{G}_{1}|$.\\
Es gilt: $|\text{Tf(}\neg \,\text{G}_{1}\text{)}| = 1 + |\text{Tf(G}_{1}\text{)}| \overset{IA}{\leq} 1 + |\text{G}_{1}| = |\neg \,\text{G}_{1}|$\\
Demnach gilt die Behauptung für $\neg \,$G$_{1}$.\\
\\
Fall: (G$_{1} \circ \,$G$_{2}$) für $\circ \in \{\vee, \wedge, \Rightarrow, \Leftrightarrow\}$\\
Teilbeweis für $|\text{Tf(G}_{1} \circ \, \text{G}_{2}\text{)}| \leq |(\text{G}_{1} \circ \,\text{G}_{2})|$.\\
Es gilt:\\
\begin{alignat*}{2}
|\text{Tf(G}_{1} \circ \, \text{G}_{2}\text{)}| &=& 1 + |\text{Tf(G}_{1}\text{)}| + |\text{Tf(G}_{2}\text{)}| \\
1 + |\text{Tf(G}_{1}\text{)}| + |\text{Tf(G}_{2}\text{)}| &\overset{IA}{\leq}& 1 + |\text{G}_{1}| + |\text{G}_{2}| \\
1 + |\text{G}_{1}| + |\text{G}_{2}| &\leq & 3 + |\text{G}_{1}| + |\text{G}_{2}| \\
3 + |\text{G}_{1}| + |\text{G}_{2}| &=& |(\text{G}_{1} \circ \,\text{G}_{2})|
\end{alignat*}
Demnach gilt die Behauptung für (G$_{1} \circ \,$G$_{2}$).\\
\\
\textit{Resumé}\\
Nach dem Prinzip der strukturellen Induktion ergibt sich damit: Für alle Formeln F$\, \in \mathcal{L}_{AL}$
gilt, $|\text{Tf(F)}| \leq |\text{F}|$.
%
%
%
\section{} %8.3
\textit{Behauptung}\\
Für alle Formeln F$\, \in \mathcal{L}_{AL}$ gilt, $|$F$| \leq 2^{|\text{Tf(F)}|+1}-3$.\\
\\
\textit{Induktionsanfang}\\
Teilbeweis für die auf atomare Formeln eingeschränkte Behauptung: Für jedes Aussagensymbol
A$\, \in \mathcal{A}s_{AL}$ gilt: $|$A$| \leq 2^{|\text{Tf(A)}|+1}-3$.\\
Die Länge von A beträgt 1. Ebenso hat A lediglich eine Teilformel und zwar sich selbst. Daraus ergibt sich:\\
\begin{alignat*}{2}
1 &\leq & 2^{1 + 1}-3 \\
1 &\leq & 2^{2} - 3 \\
1 &\leq & 4-3 = 1
\end{alignat*}
Dies gilt offensichtlich.\\
\\
\textit{Induktionsannahme}\\
Es seien G$_{1}, \,$G$_{2} \in \mathcal{L}_{AL}$ Formeln, für die gilt: $|$G$_{1}| \leq 2^{|\text{Tf(G}_{1}\text{)}|+1}-3$ und $|$G$_{2}| \leq 2^{|\text{Tf(G}_{2}\text{)}|+1}-3$.\\
\\
\textit{Induktionsschritt}\\
Fall: $\neg \,$G$_{1}$\\
Teilbeweis für $|\neg \,$G$_{1}| \leq 2^{|\text{Tf(}\neg \,\text{G}_{1}\text{)}|+1}-3$.\\
Es gilt:\\
\begin{alignat*}{2}
|\neg \,\text{G}_{1}| = 1 + |\text{G}_{1}| &\overset{IA}{\leq} & 1 + 2^{|\text{Tf(G}_{1}\text{)}|+1}-3 \\
1 + 2^{|\text{Tf(G}_{1}\text{)}|+1}-3 &\leq & \left(2^{|\text{Tf(G}_{1}\text{)}|+1}-3\right) + \left(2^{|\text{Tf(G}_{1}\text{)}|+1}-3\right) \\
\left(2^{|\text{Tf(G}_{1}\text{)}|+1}-3\right) + \left(2^{|\text{Tf(G}_{1}\text{)}|+1}-3\right) &=&
2\cdot 2^{|\text{Tf(G}_{1}\text{)}|+1}-3 \\
&=& 2^{|\text{Tf(G}_{1}\text{)}|+1+1}-3 \\
&=& 2^{|\text{Tf(}\neg \,\text{G}_{1}\text{)}|+1}-3
\end{alignat*}
Die Behauptung gilt demnach für $\neg \,$G$_{1}$.\\
\\
Fall: (G$_{1} \circ \,$G$_{2}$) für $\circ \in \{\vee, \wedge, \Rightarrow, \Leftrightarrow\}$\\
Teilbeweis für $|($G$_{1} \circ \,$G$_{2})| \leq 2^{|\text{Tf(G}_{1} \circ \,\text{G}_{2}\text{)}|+1}-3$ .\\
Es gilt:\\
\begin{alignat*}{2}
|\text{G}_{1} \circ \,\text{G}_{2}| = 3 + |\text{G}_{1}| + |\text{G}_{2}| &\overset{IA}{\leq}& 3 + 2^{|\text{Tf(G}_{1}\text{)}|+1}-3 + 2^{|\text{Tf(G}_{2}\text{)}|+1}-3 \\
3 + 2^{|\text{Tf(G}_{1}\text{)}|+1}-3 + 2^{|\text{Tf(G}_{2}\text{)}|+1}-3 &=& 2^{|\text{Tf(G}_{1}\text{)}|+1} + 2^{|\text{Tf(G}_{2}\text{)}|+1}-3 \\
2^{|\text{Tf(G}_{1}\text{)}|+1} + 2^{|\text{Tf(G}_{2}\text{)}|+1}-3 &=& 2 \cdot \left(2^{|\text{Tf(G}_{1}\text{)}|}+2^{|\text{Tf(G}_{2}\text{)}|}\right) -3 \\
2 \cdot \left(2^{|\text{Tf(G}_{1}\text{)}|}+2^{|\text{Tf(G}_{2}\text{)}|}\right) -3 &\leq & 2 \cdot \left(2^{|\text{Tf(G}_{1}\text{)}|} \cdot 2^{|\text{Tf(G}_{2}\text{)}|}\right) -3 \\
2 \cdot \left(2^{|\text{Tf(G}_{1}\text{)}|} \cdot 2^{|\text{Tf(G}_{2}\text{)}|}\right) -3 &=& 2 \cdot 2^{|\text{Tf(G}_{1}\text{)}|+|\text{Tf(G}_{2}\text{)}|} -3 \\
2 \cdot 2^{|\text{Tf(G}_{1}\text{)}|+|\text{Tf(G}_{2}\text{)}|} -3 &\leq & 2 \cdot 2^{|\text{Tf(G}_{1}\text{)}|+|\text{Tf(G}_{2}\text{)}| + 1} -3 \\
2 \cdot 2^{|\text{Tf(G}_{1}\text{)}|+|\text{Tf(G}_{2}\text{)}| + 1} -3 &=& 2 \cdot 2^{|\text{Tf(G}_{1} \circ \,\text{G}_{2}\text{)}|} -3 \\
&=& 2^{|\text{Tf(G}_{1} \circ \,\text{G}_{2}\text{)}|+1} - 3
\end{alignat*}
Demnach gilt die Behauptung für (G$_{1} \circ \,$G$_{2}$).\\
\\
\textit{Resumé}\\
Nach dem Prinzip der strukturellen Induktion ergibt sich damit: Für alle Formeln F$\, \in \mathcal{L}_{AL}$
gilt, $|$F$| \leq 2^{|\text{Tf(F)}|+1}-3$.
\section{} %8.4
\subsection{} %1.
\begin{alignat*}{2}
F^{n}_{a} &=& \begin{cases}
A & n=1\\
\neg A & n=2 \\
\neg F^{(n-1)}_{a} & n > 2
\end{cases}
\end{alignat*}
\subsection{} %2.
\begin{alignat*}{2}
F^{n}_{b} &=& \begin{cases}
A & n=1 \\
\neg A & n=2 \\
(A \wedge B) & n=3 \\
\neg F^{(n-1)}_{b} & n > 3
\end{cases}
\end{alignat*}
\end{document}

187
fgi1/Aufgabenblatt9.tex Normal file
View File

@ -0,0 +1,187 @@
\documentclass[10pt,a4paper,oneside,ngerman,numbers=noenddot]{scrartcl}
\usepackage[T1]{fontenc}
\usepackage[utf8]{inputenc}
\usepackage[ngerman]{babel}
\usepackage{amsmath}
\usepackage{amsfonts}
\usepackage{amssymb}
\usepackage{paralist}
\usepackage{gauss}
\usepackage{stmaryrd}
\usepackage[locale=DE,exponent-product=\cdot,detect-all]{siunitx}
\usepackage{tikz}
\usetikzlibrary{automata,matrix,fadings,calc,positioning,decorations.pathreplacing,arrows,decorations.markings}
\usepackage{polynom}
\polyset{style=C, div=:,vars=x}
\pagenumbering{arabic}
\def\thesection{9.\arabic{section})}
\def\thesubsection{\arabic{subsection}.}
\def\thesubsubsection{(\alph{subsubsection})}
\setcounter{section}{1}
\makeatletter
\renewcommand*\env@matrix[1][*\c@MaxMatrixCols c]{%
\hskip -\arraycolsep
\let\@ifnextchar\new@ifnextchar
\array{#1}}
\makeatother
\addtolength{\parskip}{\baselineskip}
\begin{document}
\author{Jim Martens}
\title{Hausaufgaben zum 11. Juni}
\maketitle
\section{} %9.2
\subsection{} %1.
\subsubsection{} %(a)
Anna wohnt in Aachen oder Bernd wohnt in Berlin oder Carl wohnt in Chemnitz.
\subsubsection{} %(b)
Bernd wohnt in Berlin oder Carl wohnt in Chemnitz oder Anna wohnt in Aachen.
\subsubsection{} %(c)
Wenn Anna in Aachen wohnt, dann wohnt Bernd in Berlin oder Carl wohnt in Chemnitz.
\subsubsection{} %(d)
Wenn Anna in Aachen wohnt und Bernd in Berlin wohnt, dann wohnt Carl in Chemnitz.
\subsubsection{} %(e)
Wenn Anna in Aachen wohnt, dann wohnt Carl in Chemnitz, wenn Bernd in Berlin wohnt.
\subsubsection{} %(v)
Bernd wohnt genau dann in Berlin, wenn Anna in Aachen wohnt und Carl in Chemnitz wohnt.
\subsection{} %2.
\begin{enumerate}
\item A$\, \Rightarrow \,$A
Die Formel ist allgemeingültig und erfüllbar (siehe 9.2 (a)).
\item A$\, \Rightarrow \,\neg$A
Die Formel ist erfüllbar, falsifizierbar und kontingent (siehe 9.2 (b)).
\item A$\, \Leftrightarrow \,\neg$A
Die Formel ist falsifizierbar und unerfüllbar (siehe 9.2 (c)).
\item A$\, \vee \,\neg$A
Die Formel ist allgemeingültig und erfüllbar. Egal welche Belegung A hat, eine der beiden Teilformeln (A, $\neg$A) ist immer wahr.
\item A$\, \wedge \,\neg$A
Die Formel ist unerfüllbar und falsifizierbar. Egal welche Belegung A hat, nur eine der beiden Teilformeln (A, $\neg$A) kann wahr sein.
\item A$\, \wedge \,$A
Die Formel ist erfüllbar, falsifizierbar und kontingent. Je nach Belegung ist die Formel entweder falsifiziert (falsch und falsch) oder erfüllt (wahr und wahr).
\item A$\, \vee \,$A
Die Formel ist erfüllbar, falsifizierbar und kontingent. Je nach Belegung ist die Formel entweder falsifiziert (falsch oder falsch) oder erfüllt (wahr oder wahr).
\item A$\, \Leftrightarrow \,$A
Die Formel ist allgemeingültig und erfüllbar. Je nach Belegung ergibt sich entweder (wahr biimpliziert wahr) oder (falsch biimpliziert falsch). In beiden Fällen ist die Formel erfüllt.
\item (A$\, \vee \,\neg$A)$\, \Rightarrow \,$(A$\, \wedge \neg$A)
Die Formel ist unerfüllbar und falsifizierbar. Die linke Teilformel wurde bereits als allgemeingültig gezeigt. Die rechte Teilformel wurde bereits als unerfüllbar gezeigt. Damit ist auch die ganze Formel unerfüllbar.
\item (A$\, \wedge \neg$A)$\, \Rightarrow \,$(A$\, \vee \,\neg$A)
Die Formel ist allgemeingültig und erfüllbar. Die linke Teilformel wurde bereits als unerfüllbar gezeigt und die rechte Teilformel wurde bereits als allgemeingültig gezeigt. Damit ist auch die ganze Formel allgemeingültig.
\item A$\, \Leftarrow \,\neg$A
Die Formel ist erfüllbar, falsifizierbar und kontingent. Je nach Belegung ergibt sich falsifiziert (wahr impliziert falsch) oder erfüllt (falsch impliziert wahr). Da die Formel kontingent ist, kann sie nicht allgemeingültig oder unerfüllbar sein.
\item (A$\, \Leftrightarrow \,$A)$\, \Rightarrow \,$(A$\, \Leftrightarrow \,\neg$A)
Die Formel ist unerfüllbar und falsifizierbar. Die linke Teilformel wurde bereits als allgemeingültig gezeigt. Die rechte Teilformel wurde bereits als unerfüllbar gezeigt. Damit ist auch die ganze Formel unerfüllbar.
\end{enumerate}
\subsection{} %3.
\subsubsection{} %(a)
Die Formel kann erfüllbar, allgemeingültig und eine Tautologie sein. Sie kann nicht falsifizierbar, unerfüllbar, kontingent und damit auch keine Kontradiktion sein.
Da T allgemeingültig ist, ist somit die Formel C $\, \Rightarrow \,$T immer erfüllt und demnach ebenso allgemeingültig. Da sie allgemeingültig ist, ist sie eine Tautologie, erfüllbar, nicht falsifizierbar, nicht kontingent, nicht unerfüllbar und keine Kontradiktion.
\subsubsection{} %(b)
Die Formel kann erfüllbar, falsifizierbar, allgemeingültig, eine Tautologie und kontingent sein. Sie kann nicht unerfüllbar und damit auch keine Kontradiktion sein.
Da T allgemeingültig ist, hängt der Wahrheitswert der Formel von E ab. Da E erfüllbar ist, ist die Formel auf jeden Fall nicht unerfüllbar und damit auch keine Kontradiktion. Allerdings kann E auch falsifizierbar und damit kontingent oder allgemeingültig und eine Tautologie sein.
\subsubsection{} %(c)
Die Formel kann allgemeingültig, erfüllbar und eine Tautologie sein. Sie kann nicht falsifizierbar, unerfüllbar oder kontingent sein und ist damit auch keine Kontradiktion.
K und U sind beide unerfüllbar und eine Kontradiktion, da diese beiden Eigenschaften äquivalent sind. Da beide Formeln somit unabhängig von der Belegung immer falsch ergeben, ist die Aussage K$\, \Leftrightarrow \,$ U wiederum immer richtig, womit die Formel erfüllbar, allgemeingültig und damit eine Tautologie ist. Damit ist die Formel nicht unerfüllbar, keine Kontradiktion, nicht falsifizierbar und nicht kontingent.
\subsubsection{} %(d)
Die Formel kann erfüllbar, falsifizierbar und kontingent sein. Sie kann nicht allgemeingültig, unerfüllbar und damit auch keine Tautologie oder Kontradiktion sein.
Da A allgemeingültig ist, hängt der Wahrheitswert von C ab. Da C kontingent ist, kann die Formel sowohl erfüllt als auch falsifiziert werden. Wenn C durch eine Belegung falsifiziert wird, dann auch die Formel A$\, \Rightarrow \,$C und wenn C erfüllt wird, dann auch die Formel A$\, \Rightarrow \,$C.
Somit ist die Formel weder allgemeingültig und eine Tautologie noch unerfüllbar und eine Kontradiktion.
\subsubsection{} %(e)
Die Formel kann falsifizierbar, unerfüllbar und damit eine Kontradiktion sein. Sie kann nicht erfüllbar, kontingent, allgemeingültig und damit auch keine Tautologie sein.
Da U unerfüllbar ist, wird die Formel immer falsifiziert. Daher ist die Formel falsifizierbar, unerfüllbar und eine Kontradiktion. Deswegen ist sie nicht erfüllbar, kontingent, allgemeingültig und damit auch keine Tautologie.
\subsubsection{} %(f)
Die Formel kann erfüllbar, falsifizierbar, kontingent, unerfüllbar und eine Kontradiktion sein. Sie kann nicht allgemeingültig und damit eine Tautologie sein.
Da U unerfüllbar ist, hängt der Wahrheitswert von E ab. Ist E erfüllt, ist die Formel E$\, \Rightarrow \,$U falsifiziert. Wenn E allgemeingültig ist, dann ist die Formel unerfüllbar. Ist E hingegen kontingent, dann ist die Formel auch erfüllbar und ebenso kontingent. Allgemeingültig kann die Formel jedoch auf keinen Fall sein.
\subsubsection{} %(g)
Die Formel kann erfüllbar, falsifizierbar, kontingent und allgemeingültig und damit eine Tautologie sein. Sie kann nicht unerfüllbar und eine Kontradiktion sein.
Da E und C beide erfüllbar sind, gibt es mindestens eine Belegung, bei der die Formel erfüllt ist. Wenn E allgemeingültig ist, dann ist auch die Formel allgemeingültig. Wenn E kontingent ist, dann ist die Formel falsifizierbar und damit selber kontingent.
Unerfüllbar ist die Formel nicht.
\section{} %9.3
\subsection{} %1.
\begin{tabular}{lr}
A$\, \Leftrightarrow \,$B & Elimination $\Leftrightarrow$ \\
$\equiv ($A$\, \Rightarrow \,$B$) \wedge ($B$ \, \Rightarrow \,$A$)$ & Elimination $\Rightarrow$ \\
$\equiv (\neg$A$\, \vee \,$B$) \wedge (\neg$B$\, \vee \,$A$)$ & KNF \\
A$\, \Leftrightarrow \,$B & Elimination $\Leftrightarrow$ \\
$\equiv ($A$\, \wedge \,$B$) \vee (\neg$A$\, \wedge \neg$B$)$ & DNF \\
A$\, \Leftrightarrow \,$C & Elimination $\Leftrightarrow$ \\
$\equiv ($A$\, \Rightarrow \,$C$) \wedge ($C$ \, \Rightarrow \,$A$)$ & Elimination $\Rightarrow$ \\
$\equiv (\neg$A$\, \vee \,$C$) \wedge (\neg$C$\, \vee \,$A$)$ & KNF \\
A$\, \Leftrightarrow \,$C & Elimination $\Leftrightarrow$ \\
$\equiv ($A$\, \wedge \,$C$) \vee (\neg$A$\, \wedge \neg$C$)$ & DNF \\
KNF-Erzeugung: \\
$($A$\, \Leftrightarrow \,$B$) \Rightarrow ($A$\, \Leftrightarrow \,$C$)$ & Elimination $\Rightarrow$ \\
$\equiv \neg(($A$\, \Leftrightarrow \,$B$) \wedge \neg($A$\, \Leftrightarrow \,$C$))$ \\
Ich bearbeite jetzt die beiden Teile der Konjunktion getrennt, \\
mit dem Ziel, für diese KNFen zu erzeugen:\\
$($A$\, \Leftrightarrow \,$B$)$ & Einsetzung der KNF \\
$\equiv (\neg$A$\, \vee \,$B$) \wedge (\neg$B$\, \vee \,$A$)$ & KNF\\
$\neg($A$\, \Leftrightarrow \,$C$)$ & Einsetzung der DNF \\
$\equiv \neg(($A$\, \wedge \,$C$) \vee (\neg$A$\, \wedge \neg$C$))$ & de Morgan, Doppelte Negation \\
$\equiv (\neg$A$\, \vee \neg$C$) \wedge ($A$\, \vee \,$C$)$ & KNF \\
Die Verknüpfung dieser beiden Teilresultate ergibt: \\
$\neg(($A$\, \Leftrightarrow \,$B$) \wedge \neg($A$\, \Leftrightarrow \,$C$))$ \\
$\neg((\neg$A$\, \vee \,$B$) \wedge (\neg$B$\, \vee \,$A$) \wedge (\neg$A$\, \vee \neg$C$) \wedge ($A$\, \vee \,$C$)) = \,$G & KNF \\
DNF-Erzeugung: \\
$($A$\, \Leftrightarrow \,$B$) \Rightarrow ($A$\, \Leftrightarrow \,$C$)$ & Elimination $\Rightarrow$ \\
$\equiv \neg($A$\, \Leftrightarrow \,$B$) \vee ($A$\, \Leftrightarrow \,$C$)$ \\
Ich bearbeite jetzt die beiden Teile der Disjunktion getrennt,\\
mit dem Ziel, für diese DNFen zu erzeugen:\\
$\neg($A$\, \Leftrightarrow \,$B$)$ & Einsetzung der KNF \\
$\equiv \neg((\neg$A$\, \vee \,$B$) \wedge (\neg$B$\, \vee \,$A$))$ & de Morgan, Doppelte Negation \\
$\equiv ($A$\, \wedge \neg$B$) \vee ($B$\, \wedge \, \neg$A$)$ & DNF \\
$($A$\, \Leftrightarrow \,$C$)$ & Einsetzung der DNF \\
$\equiv ($A$\, \wedge \,$C$) \vee (\neg$A$\, \wedge \neg$C$)$ & DNF \\
Die Verknüpfung dieser beiden Teilresultate ergibt: \\
$\neg($A$\, \Leftrightarrow \,$B$) \vee ($A$\, \Leftrightarrow \,$C$)$ \\
$\equiv ($A$\, \wedge \neg$B$) \vee ($B$\, \wedge \, \neg$A$) \vee ($A$\, \wedge \,$C$) \vee (\neg$A$\, \wedge \neg$C$) = \,$F & DNF
\end{tabular}
\subsection{} %2.
\begin{tabular}{c|ccc|ccc}
& A & B & C & (A$\, \Leftrightarrow \,$B) & (A$\, \Leftrightarrow \,$C) & ((A$\, \Leftrightarrow \,$B)$\, \Rightarrow \,$(A$\, \Leftrightarrow \,$C)) \\
\hline
$\mathcal{A}_{0}$ & 0 & 0 & 0 & 1 & 1 & 1 \\
$\mathcal{A}_{1}$ & 0 & 0 & 1 & 1 & 0 & 0 \\
$\mathcal{A}_{2}$ & 0 & 1 & 0 & 0 & 1 & 1 \\
$\mathcal{A}_{3}$ & 0 & 1 & 1 & 0 & 0 & 1 \\
$\mathcal{A}_{4}$ & 1 & 0 & 0 & 0 & 0 & 1 \\
$\mathcal{A}_{5}$ & 1 & 0 & 1 & 0 & 1 & 1 \\
$\mathcal{A}_{6}$ & 1 & 1 & 0 & 1 & 0 & 0 \\
$\mathcal{A}_{7}$ & 1 & 1 & 1 & 1 & 1 & 1
\end{tabular}\\
\\
\begin{tabular}{c|ccc|ccccc}
& A & B & C & $(\neg$A$\, \vee \,$B$)$ & $(\neg$B$\, \vee \,$A$)$ & $(\neg$A$\, \vee \neg$C$)$ & $($A$\, \vee \,$C$)$ & G \\
\hline
$\mathcal{A}_{0}$ & 0 & 0 & 0 & 1 & 1 & 1 & 0 & 1 \\
$\mathcal{A}_{1}$ & 0 & 0 & 1 & 1 & 1 & 1 & 1 & 0 \\
$\mathcal{A}_{2}$ & 0 & 1 & 0 & 1 & 0 & 1 & 0 & 1 \\
$\mathcal{A}_{3}$ & 0 & 1 & 1 & 1 & 0 & 1 & 1 & 1 \\
$\mathcal{A}_{4}$ & 1 & 0 & 0 & 0 & 1 & 1 & 1 & 1 \\
$\mathcal{A}_{5}$ & 1 & 0 & 1 & 0 & 1 & 0 & 1 & 1 \\
$\mathcal{A}_{6}$ & 1 & 1 & 0 & 1 & 1 & 1 & 1 & 0 \\
$\mathcal{A}_{7}$ & 1 & 1 & 1 & 1 & 1 & 0 & 1 & 1
\end{tabular}\\
\\
\begin{tabular}{c|ccc|ccccc}
& A & B & C & $($A$\, \wedge \neg$B$)$ & $($B$\, \wedge \, \neg$A$)$ & $($A$\, \wedge \,$C$)$ & $(\neg$A$\, \wedge \neg$C$)$ & F \\
\hline
$\mathcal{A}_{0}$ & 0 & 0 & 0 & 0 & 0 & 0 & 1 & 1 \\
$\mathcal{A}_{1}$ & 0 & 0 & 1 & 0 & 0 & 0 & 0 & 0 \\
$\mathcal{A}_{2}$ & 0 & 1 & 0 & 0 & 1 & 0 & 1 & 1 \\
$\mathcal{A}_{3}$ & 0 & 1 & 1 & 0 & 1 & 0 & 0 & 1 \\
$\mathcal{A}_{4}$ & 1 & 0 & 0 & 1 & 0 & 0 & 0 & 1 \\
$\mathcal{A}_{5}$ & 1 & 0 & 1 & 1 & 0 & 1 & 0 & 1 \\
$\mathcal{A}_{6}$ & 1 & 1 & 0 & 0 & 0 & 0 & 0 & 0 \\
$\mathcal{A}_{7}$ & 1 & 1 & 1 & 0 & 0 & 1 & 0 & 1
\end{tabular}\\
Der Wahrheitswert in den drei jeweils letzten Spalten ist identisch. Entsprechend sind die drei Formeln äquivalent.
\subsection{} %3.
\end{document}

25
fgi1/Template.tex Normal file
View File

@ -0,0 +1,25 @@
\textit{Behauptung}\\
Für alle Formeln F$\, \in \mathcal{L}_{AL}$ gilt, [Behauptung formuliert mit F].\\
\\
\textit{Induktionsanfang}\\
Teilbeweis für die auf atomare Formeln eingeschränkte Behauptung: Für jedes Aussagensymbol
A$\, \in \mathcal{A}s_{AL}$ gilt: [Behauptung formuliert mit A].\\
\\
\textit{Induktionsannahme}\\
Es seien G$_{1}, \,$G$_{2} \in \mathcal{L}_{AL}$ Formeln, für die gilt: [Behauptung formuliert mit G$_{1}$] und [Behauptung
formuliert mit G$_{2}$].\\
\\
\textit{Induktionsschritt}\\
Fall: $\neg \,$G$_{1}$\\
Teilbeweis für [Behauptung formuliert mit $\neg \,$G$_{1}$].\\
(Dieser Teilbeweis darf auf die Induktionsannahme zurückgreifen.)\\
\\
Fall: (G$_{1} \circ \,$G$_{2}$) für $\circ \in \{\vee, \wedge, \Rightarrow, \Leftrightarrow\}$\\
Teilbeweis für [Behauptung formuliert mit (G$_{1} \circ \,$G$_{2}$)].\\
(Dieser Teilbeweis darf auf die Induktionsannahme zurückgreifen. Dabei kann es sein, dass
man alle Operatoren gleich behandeln kann, oder man muss eine Fallunterscheidung nach
Operator machen. Dann kann es hier bis zu 4 Teilbeweise geben.)\\
\\
\textit{Resumé}\\
Nach dem Prinzip der strukturellen Induktion ergibt sich damit: Für alle Formeln F$\, \in \mathcal{L}_{AL}$
gilt, [Behauptung formuliert mit F].

View File

@ -0,0 +1,168 @@
\documentclass[10pt,a4paper,oneside,ngerman,numbers=noenddot]{scrartcl}
\usepackage[T1]{fontenc}
\usepackage[utf8x]{inputenc}
\usepackage[ngerman]{babel}
\usepackage{amsmath}
\usepackage{amsfonts}
\usepackage{amssymb}
\usepackage{paralist}
\usepackage{gauss}
\usepackage{pgfplots}
\usepackage[locale=DE,exponent-product=\cdot,detect-all]{siunitx}
\usepackage{tikz}
\usetikzlibrary{automata,matrix,fadings,calc,positioning,decorations.pathreplacing,arrows,decorations.markings,petri,shapes}
\usepackage{polynom}
\usepackage{multirow}
\usepackage[german]{fancyref}
\usepackage{morefloats}
\polyset{style=C, div=:,vars=x}
\pgfplotsset{compat=1.8}
\pagenumbering{arabic}
% ensures that paragraphs are separated by empty lines
\parskip 12pt plus 1pt minus 1pt
\parindent 0pt
% define how the sections are rendered
\def\thesection{9.\arabic{section})}
\def\thesubsection{\arabic{subsection}.}
\def\thesubsubsection{(\alph{subsubsection})}
% some matrix magic
\makeatletter
\renewcommand*\env@matrix[1][*\c@MaxMatrixCols c]{%
\hskip -\arraycolsep
\let\@ifnextchar\new@ifnextchar
\array{#1}}
\makeatother
\tikzset{
place/.style={
circle,
thick,
draw=black,
fill=white,
minimum size=6mm,
font=\bfseries
},
transitionH/.style={
rectangle,
thick,
draw=black,
fill=white,
minimum width=8mm,
inner ysep=4pt,
font=\bfseries
},
transitionV/.style={
rectangle,
thick,
fill=black,
minimum height=8mm,
inner xsep=2pt
}
}
\begin{document}
\author{Benjamin Kuffel, Jim Martens\\Gruppe 6}
\title{Hausaufgaben zum 15. Dezember}
\maketitle
\setcounter{section}{2}
\section{} %9.3
\subsection{}
Es ist zu zeigen, dass die Beschränktheit von P/T-Netzen monoton ist. Nach Satz 7.1 wissen wir bereits, dass ein Netz \(N\) unbeschränkt ist, wenn es zwei von der Anfangsmarkierung aus erreichbare Markierungen \(m_1,m_2\) gibt, für die \(m_1 \lneq m_2\) und \(\exists \sigma : m_1 \overset{\sigma}{\rightarrow} m_2\) gilt.
Setzen wir \(m_{1}\) auf die Anfangsmarkierung, dann wissen wir anhand dieses Satzes, dass das Netz genau dann unbeschränkt ist, wenn es eine weitere von \(m_0\) verschiedene Markierung gibt, die echt größer als \(m_0\) ist und von \(m_0\) aus erreichbar ist.
Im Folgenden setzen wir voraus, dass das Netz \(N\) unbeschränkt ist und es daher eine solche Schaltfolge \(\sigma\) und Markierung \(m_1\) gibt, für die Satz 7.1 gilt.
Auf diese Schaltfolge und Markierung wenden wir nun Lemma 6.17 an. Danach können wir eine beliebige Markierung \(m\) zu \(m_0\) und \(m_1\) hinzuaddieren und es gilt für das feste \(\sigma\): \(m_0 + m \overset{\sigma}{\rightarrow} m_1 + m\). Damit ist Satz 7.1 weiterhin anwendbar und das Netz weiterhin unbeschränkt.
Um die Monotonie abschließend zu zeigen, wählen wir das \(m\) so, dass \(m'_0 = m_0 + m\) gilt. Damit gilt auch \(m'_1 = m_1 + m'_0 - m_0\).
\subsection{}
\[
B := \{(0,0,0,2),(4,0,0,0),(0,6,0,0)\}
\]
Die Menge enthält alle Markierungen, mit denen entweder der Zyklus p1 - a -> p2 - b -> p1 oder p4 - c -> p4 lebendig gemacht wird. Da jede der Markierungen nur Marken auf einer der Stellen enthält, sind die Markierungen nicht vergleichbar zueinander und damit gibt es auch keine kleinere Markierung. Das Netz ist bei allen der in der Menge enthaltenen Markierungen unbeschränkt, da durch die Lebendigkeit eines der beiden Zyklen beliebig viele Marken in p3 landen können.
\section{} %9.4
\subsection{}
Wirkungsmatrix \(\Delta_{N_{9.4a}}\):
\begin{tabular}{c|cccccc}
& a & b & c & d & e & f \\
\hline
p1 & 1 & -1 & 0 & 0 & 0 & 0 \\
p2 & 0 & 0 & 1 & -1 & 0 & 0 \\
p3 & 0 & 0 & -1 & 1 & 0 & 0 \\
p4 & 0 & 0 & 0 & 0 & -1 & 1 \\
p5 & 0 & 1 & 0 & 0 & -1 & 0
\end{tabular}
\subsection{}
Die transponierte Wirkungsmatrix:
\begin{tabular}{c|ccccc}
& p1 & p2 & p3 & p4 & p5 \\
\hline
a & 1 & 0 & 0 & 0 & 0 \\
b & -1 & 0 & 0 & 0 & 1 \\
c & 0 & 1 & -1 & 0 & 0 \\
d & 0 & -1 & 1 & 0 & 0 \\
e & 0 & 0 & 0 & -1 & -1 \\
f & 0 & 0 & 0 & 1 & 0
\end{tabular}
Menge der S-Invariantenvektoren:
\[
S_{inv} = \{(0,n,n,0,0 )^{tr}| n \in \mathbb{N} \wedge n > 0 \}
\]
\subsection{}
\(N_{9.4}\) ist nicht strukturell beschränkt, da es keine positive, überdeckende S-Invariante für das Netz gibt.
\subsection{}
Wirkungsmatrix \(\Delta_{N_{9.4b}}\):
\begin{tabular}{c|cccccc}
& a & b & c & d & e & f \\
\hline
p1 & 1 & -1 & 0 & 0 & 0 & 0 \\
p2 & 0 & 0 & 1 & -1 & 0 & 0 \\
p3 & 0 & 0 & -1 & 1 & 0 & 0 \\
p4 & 0 & 0 & 0 & 0 & -1 & 1 \\
p5 & 0 & 1 & 0 & 0 & -1 & 0 \\
p6 & -1 & 1 & 0 & 0 & 0 & 0 \\
p7 & 0 & -1 & 0 & 0 & 1 & 0 \\
p8 & 0 & 0 & 0 & 0 & 1 & -1
\end{tabular}
Transponierte Wirkungsmatrix:
\begin{tabular}{c|cccccccc}
& p1 & p2 & p3 & p4 & p5 & p6 & p7 & p8 \\
\hline
a & 1 & 0 & 0 & 0 & 0 & -1 & 0 & 0 \\
b & -1 & 0 & 0 & 0 & 1 & 1 & -1 & 0\\
c & 0 & 1 & -1 & 0 & 0 & 0 & 0 & 0\\
d & 0 & -1 & 1 & 0 & 0 & 0 & 0 & 0\\
e & 0 & 0 & 0 & -1 & -1 & 0 & 1 & 1\\
f & 0 & 0 & 0 & 1 & 0 & 0 & 0 & -1
\end{tabular}
Menge der S-Invariantenvektoren:
\[
S_{inv} = \{(a,b,b,c,c,a,c,c)| a,b,c \in \mathbb{N}^{+}\}
\]
Das Netz ist strukturell beschränkt, da es eine überdeckende, positive S-Invariante gibt.
\subsection{}
Das Ursprungsnetz sah keine Beschränkung des Lagers, der Annahme oder der wartenden Kunden vor. Der Informatiker hat die Änderung vorgenommen, um der Realität Rechnung zu tragen, in der es eine begrenzte Lagerkapazität, begrenzte Annahmestellen und begrenzten Warteraum gibt. Die Diskrepanz kann ferner zustande kommen, da ein P/T-Netz keine zeitliche Abfolge darstellen kann, wenn die betreffenden Transitionen nebenläufig sind. Die dem Netz zugrundeliegende Beschreibung der Arbeitsabläufe könnte jedoch durchaus eine solche zeitliche Komponente beinhalten und voraussetzen, dass ein Kunde bedient wird, sobald einer anwesend ist. Ein solcher Schaltzwang existiert in Netzen jedoch erst einmal nicht.
\subsection{}
\begin{alignat*}{2}
&& 2 \cdot \textbf{m}(p_1) + 1 \cdot \textbf{m}(p_2) + 1 \cdot \textbf{m}(p_3) + 3 \cdot \textbf{m}(p_4) + 3 \cdot \textbf{m}(p_5) + 2 \cdot \textbf{m}(p_6) + 3 \cdot \textbf{m}(p_7) + 3 \cdot \textbf{m}(p_8) \\
&=& 2 \cdot 0 + 1 \cdot 1 + 1 \cdot 2 + 3 \cdot 0 + 3 \cdot 1 + 2 \cdot 5 + 3 \cdot 10 + 3 \cdot 8 \\
&=& 70
\end{alignat*}
\end{document}

View File

@ -0,0 +1,165 @@
\documentclass[10pt,a4paper,oneside,ngerman,numbers=noenddot]{scrartcl}
\usepackage[T1]{fontenc}
\usepackage[utf8]{inputenc}
\usepackage[ngerman]{babel}
\usepackage{amsmath}
\usepackage{amsfonts}
\usepackage{amssymb}
\usepackage{paralist}
\usepackage{gauss}
\usepackage[locale=DE,exponent-product=\cdot,detect-all]{siunitx}
\usepackage{tikz}
\usetikzlibrary{matrix,fadings,calc,positioning,decorations.pathreplacing,arrows,decorations.markings}
\usepackage{polynom}
\polyset{style=C, div=:,vars=x}
\pagenumbering{arabic}
\def\thesection{\arabic{section})}
\def\thesubsection{\alph{subsection})}
\def\thesubsubsection{(\roman{subsubsection})}
\makeatletter
\renewcommand*\env@matrix[1][*\c@MaxMatrixCols c]{%
\hskip -\arraycolsep
\let\@ifnextchar\new@ifnextchar
\array{#1}}
\makeatother
\begin{document}
\author{Jim Martens}
\title{Hausaufgaben zum 11. April}
\maketitle
\section{} %1
\begin{equation*}
\frac{3}{x+5} \geq 3
\end{equation*}
1. Fall $x > -5$:\\
\begin{alignat*}{3}
&& \frac{3}{x+5} &\geq & 3 \\
\Leftrightarrow && 3 &\geq & 3(x+5) \\
\Leftrightarrow && 3 &\geq & 3x + 15 \\
\Leftrightarrow && -12 &\geq & 3x \\
\Leftrightarrow && -4 &\geq & x
\end{alignat*}
\\
2. Fall $x < -5$:\\
\begin{alignat*}{3}
&& \frac{3}{x+5} &\geq & 3 \\
\Leftrightarrow && 3 &\leq & 3(x+5) \\
\Leftrightarrow && 3 &\leq & 3x + 15 \\
\Leftrightarrow && -12 &\leq & 3x \\
\Leftrightarrow && -4 &\leq & x
\end{alignat*}
\\
$L = [-4]$
\section{} %2
\begin{equation*}
|3x-4| \geq 2
\end{equation*}
1. Fall $x \geq \frac{4}{3}$:\\
\begin{alignat*}{3}
&& |3x-4| &\geq & 2 \\
\Leftrightarrow && 3x-4 &\geq & 2 \\
\Leftrightarrow && 3x &\geq & 6 \\
\Leftrightarrow && x &\geq & 2
\end{alignat*}
\\
2. Fall $x < \frac{4}{3}$:\\
\begin{alignat*}{3}
&& |3x-4| &\geq & 2 \\
\Leftrightarrow && -(3x-4) &\geq & 2 \\
\Leftrightarrow && -3x + 4 &\geq & 2 \\
\Leftrightarrow && -3x &\geq & -2 \\
\Leftrightarrow && x &\leq & \frac{2}{3}
\end{alignat*}
\\
$L = (-\infty,\frac{2}{3}] \cup [2,\infty)$
\section{} %3
\subsection{} %a
\begin{alignat*}{3}
&& |a_{n} - a| &=& |\frac{2n-1}{n+3} - 2| \\
\Leftrightarrow && &=& |\frac{2n-1}{n+3} - \frac{2(n+3)}{n+3}| \\
\Leftrightarrow && &=& |\frac{2n-1 - 2n - 6}{n+3}| \\
\Leftrightarrow && &=& |\frac{-7}{n+3}| \\
\Leftrightarrow && &=& \frac{7}{n+3}
\end{alignat*}
\subsection{} %b
Es sei $\varepsilon > 0$. Aufgrund von a) gilt:\\
\begin{alignat*}{3}
&& |a_{n} - a| &<& \varepsilon \label{eq:1}\tag{1}\\
\Leftrightarrow && |a_{n} - a| = |\frac{-7}{n+3}| = \frac{7}{n+3} &<& \varepsilon \\
\Leftrightarrow && -7 &<& \varepsilon (n+3) \\
\Leftrightarrow && \frac{-7}{\varepsilon} &<& n+3 \\
\Leftrightarrow && \frac{-7}{\varepsilon} - 3 &<& n
\end{alignat*}
\\
Wählt man $N > \frac{-7}{\varepsilon} - 3$, so ergibt sich aus \eqref{eq:1}, dass $|a_{n} - a| < \varepsilon$ für alle $n \geq N$ gilt. Das zeigt $(a_{n}) \rightarrow a = 2$.
\subsection{} %c
Es sei $\varepsilon = \frac{1}{10}$:\\
\begin{alignat*}{3}
&& \frac{-7}{\frac{1}{10}} - 3 &<& n \\
\Leftrightarrow && -70 - 3 &<& n \\
\Leftrightarrow && -73 &<& n
\end{alignat*}
Wählt man $N = -72$, so ergibt sich aus \eqref{eq:1}, dass $|a_{n} - a| < \varepsilon$ für alle $n \geq N$ gilt.\\
\\
Es sei $\varepsilon = \frac{1}{100}$:\\
\begin{alignat*}{3}
&& \frac{-7}{\frac{1}{100}} - 3 &<& n \\
\Leftrightarrow && -700 - 3 &<& n \\
\Leftrightarrow && -703 &<& n
\end{alignat*}
Wählt man $N = -702$, so ergibt sich aus \eqref{eq:1}, dass $|a_{n} - a| < \varepsilon$ für alle $n \geq N$ gilt.\\
\\
Es sei $\varepsilon = \frac{1}{100000}$:\\
\begin{alignat*}{3}
&& \frac{-7}{\frac{1}{100000}} - 3 &<& n \\
\Leftrightarrow && -700000 - 3 &<& n \\
\Leftrightarrow && -700003 &<& n
\end{alignat*}
Wählt man $N = -700002$, so ergibt sich aus \eqref{eq:1}, dass $|a_{n} - a| < \varepsilon$ für alle $n \geq N$ gilt.
\section{} %4
\textbf{Behauptung:} Die folgende Aussage gilt für alle $n \in \mathbb{N}$:\\
\begin{equation*}
0 \leq a_{n} < \frac{1}{2} \label{eq:2}\tag{2}
\end{equation*}\\
Die Folge ($a_{n}$) sei rekursiv definiert durch \\
\begin{alignat*}{2}
a_{1} &=& \frac{2}{5} \label{eq:3}\tag{3}\\
a_{n+1} &=& a_{n}^{2} + \frac{1}{4} \label{eq:4}\tag{4}
\end{alignat*}
\textbf{Beweis:} Durch vollständige Induktion.\\
Mit $A(n)$ sei die Aussage \eqref{eq:2} bezeichnet.\\\\
\underline{Induktionsanfang:} \\
$A(1)$ ist richtig, da die Aussage \eqref{eq:2} für \eqref{eq:3} wie folgt gilt:
\begin{alignat*}{2}
0 \leq \frac{2}{5} = \frac{4}{10} < \frac{1}{2} = \frac{5}{10}
\end{alignat*}\\
\underline{Induktionsannahme:}\\
Die Aussage \eqref{eq:2} gilt für ein beliebig fest gewähltes $n \in \mathbb{N}$.\\\\
\underline{Zu zeigen:}\\
$A(n+1)$ gilt, d. h. Folgendes gilt für die Aussage \eqref{eq:4}:\\
\begin{equation*}
0 \leq a_{n+1} < \frac{1}{2} \label{eq:5}\tag{5}
\end{equation*}
\underline{Induktionsschluss:}\\
Aus \eqref{eq:5} folgt für $0 \leq a_{n+1}$ Folgendes:\\
\begin{alignat*}{3}
&& 0 &\leq & a_{n}^{2} + \frac{1}{4}
\end{alignat*}
Diese Aussage gilt, da $\frac{1}{4}$ auf triviale Weise die Aussage erfüllt und $a_{n}^{2}$ immer positiv oder gleich Null sein muss, da eine beliebige Zahl zum Quadrat immer größer gleich Null ist.\\
Für $a_{n+1} < \frac{1}{2}$ ergibt sich Folgendes:\\
\begin{alignat*}{5}
&& a_{n}^{2} + \frac{1}{4} &<& \frac{1}{2} && \;&|& -\frac{1}{4} \\
\Leftrightarrow && a_{n}^{2} &<& \frac{1}{4} && && \\
\Leftrightarrow && a_{n} \cdot a_{n} &<& \frac{1}{4} && &&
\end{alignat*}
Aufgrund der Induktionsannahme gilt $a_{n} < \frac{1}{2}$. Daher ist das Quadrat von $a_{n}$ auf jeden Fall kleiner als $\frac{1}{4}$.\\
Nach dem Induktionsprinzip folgt aus dem Induktionsanfang und dem Induktionsschluss die Behauptung. \hfill $\Box$\\
Es ist zu zeigen, dass $a_{n+1} \geq a_{n}$ für alle $n \in \mathbb{N}$ gilt. Es ergibt sich Folgendes:\\
\begin{alignat*}{5}
&& a_{n+1} &\geq & a_{n} && && \\
\Leftrightarrow && a_{n}^{2} + \frac{1}{4} &\geq & a_{n} && \;&|& -a_{n} \\
\Leftrightarrow && a_{n}^{2} - a_{n} + \frac{1}{4} &\geq & 0 && \;&|& \text{Binomische Formel erzeugen} \\
\Leftrightarrow && (a_{n} - \frac{1}{2})^{2} &\geq & 0 && &&
\end{alignat*}
Diese Aussage gilt, da ein Quadrat einer beliebigen Zahl immer größer gleich Null ist.
\end{document}

View File

@ -0,0 +1,253 @@
\documentclass[10pt,a4paper,oneside,ngerman,numbers=noenddot]{scrartcl}
\usepackage[T1]{fontenc}
\usepackage[utf8]{inputenc}
\usepackage[ngerman]{babel}
\usepackage{amsmath}
\usepackage{amsfonts}
\usepackage{amssymb}
\usepackage{paralist}
\usepackage{gauss}
\usepackage{pgfplots}
\usepackage[locale=DE,exponent-product=\cdot,detect-all]{siunitx}
\usepackage{tikz}
\usetikzlibrary{matrix,fadings,calc,positioning,decorations.pathreplacing,arrows,decorations.markings}
\usepackage{polynom}
\polyset{style=C, div=:,vars=x}
\pgfplotsset{compat=1.8}
\pagenumbering{arabic}
\def\thesection{\arabic{section})}
\def\thesubsection{\alph{subsection})}
\def\thesubsubsection{(\roman{subsubsection})}
\makeatletter
\renewcommand*\env@matrix[1][*\c@MaxMatrixCols c]{%
\hskip -\arraycolsep
\let\@ifnextchar\new@ifnextchar
\array{#1}}
\makeatother
\begin{document}
\author{Jim Martens (6420323)}
\title{Hausaufgaben zum 27. Juni}
\maketitle
\section{} %1
\subsubsection{} %i
\begin{alignat*}{2}
f(x,y) &=& 2x^{2}y^{2} - 3xy + 4x + 2 \\
f_{x} &=& 4xy^{2} - 3y + 4 \\
f_{y} &=& 4x^{2}y - 3x
\end{alignat*}
\subsubsection{} %ii
\begin{alignat*}{2}
f(x,y) &=& \cos(x^{2}y) \cdot e^{xy} \\
f_{x} &=& -\sin(x^{2}y) \cdot 2xy + \cos(x^{2}y) \cdot e^{xy} \cdot y \\
f_{y} &=& -\sin(x^{2}y) \cdot x^{2} + \cos(x^{2}y) \cdot e^{xy} \cdot x \\
\end{alignat*}
\subsubsection{} %iii
\begin{alignat*}{2}
f(x,y) &=& \frac{\sin x + \cos y}{x^{2} + y^{2}} \\
f_{x} &=& \frac{\cos x \cdot (x^{2} + y^{2}) - (\sin x + \cos y) \cdot 2x}{(x^{2} + y^{2})^{2}} \\
f_{y} &=& \frac{-\sin y \cdot (x^{2} + y^{2}) - (\sin x + \cos y) \cdot 2y}{(x^{2} + y^{2})^{2}}
\end{alignat*}
\subsubsection{} %iv
\begin{alignat*}{2}
f(x,y) &=& \sqrt{1 - x^{2} - y^{2}} = (1 - x^{2} - y^{2})^{\frac{1}{2}} \\
f_{x} &=& \frac{1}{2}(1 - x^{2} - y^{2})^{-\frac{1}{2}} \cdot (-2x) \\
f_{y} &=& \frac{1}{2}(1 - x^{2} - y^{2})^{-\frac{1}{2}} \cdot (-2y)
\end{alignat*}
\section{} %2
\begin{alignat*}{2}
f(x,y) &=& x^{2}y^{3} + y \cdot e^{x^{2}y} \\
f_{x} &=& 2xy^{3} + y \cdot e^{x^{2}y} \cdot 2xy\\
&=& 2xy^{3} + e^{x^{2}y} \cdot 2xy^{2} \\
f_{y} &=& 3x^{2}y^{2} + e^{x^{2}y} + y \cdot e^{x^{2}y} \cdot x^{2} \\
f_{xx} &=& 2y^{3} + e^{x^{2}y} \cdot 2xy \cdot 2xy^{2} + e^{x^{2}y} \cdot 2y^{2}\\
&=& 2y^{3} + e^{x^{2}y} \cdot 4x^{2}y^{3} + e^{x^{2}y} \cdot 2y^{2} \\
f_{yx} &=& 6xy^{2} + e^{x^{2}y} \cdot x^{2} \cdot 2xy^{2} + e^{x^{2}y} \cdot 4xy\\
&=& 6xy^{2} + e^{x^{2}y} \cdot 2x^{3}y^{2} + e^{x^{2}y} \cdot 4xy \\
f_{xy} &=& 6xy^{2} + e^{x^{2}y} \cdot 2xy + y \cdot (e^{x^{2}y} \cdot 2xy \cdot x^{2} + e^{x^{2}y} \cdot 2x)\\
&=& 6xy^{2} + e^{x^{2}y} \cdot 2x^{3}y^{2} + e^{x^{2}y} \cdot 4xy \\
f_{yy} &=& 6x^{2}y + e^{x^{2}y} \cdot x^{2} + e^{x^{2}y} \cdot x^{2} + y \cdot e^{x^{2}y} \cdot x^{2} \cdot x^{2} \\
&=& 6x^{2}y + e^{x^{2}y} \cdot 2x^{2} + e^{x^{2}y} \cdot x^{4}y
\end{alignat*}
\section{} %3
\setcounter{subsubsection}{0}
\subsubsection{} %i
\begin{alignat*}{2}
f(x,y) &=& 2x^{2} + y^{2} - 2xy -2x -4y + 5 \\
I f_{x} &=& 4x - 2y - 2 = 0 \\
II f_{y} &=& 2y - 2x - 4 = 0 \\
f_{xx} &=& 4 \\
f_{xy} &=& - 2 \\
f_{yx} &=& - 2 \\
f_{yy} &=& 2 \\
I + II &=& 2x -6 = 0 \\
&\Leftrightarrow & 2x = 6 \\
&\Leftrightarrow & x = 3 \\
\intertext{in II einsetzen}
&\Rightarrow & 2y - 2 \cdot 3 -4 = 0 \\
&\Leftrightarrow & 2y - 6 - 4 = 0 \\
&\Leftrightarrow & 2y - 10 = 0 \\
&\Leftrightarrow & 2y = 10 \\
&\Leftrightarrow & y = 5 \\
\intertext{Einsetzen von beiden Werten in I}
&\Rightarrow & 4 \cdot 3 - 2 \cdot 5 - 2 = 0 \\
&\Leftrightarrow & 12 - 10 - 2 = 0 \\
&\Leftrightarrow & 0 = 0 \\
\intertext{Die einzige kritische Stelle befindet sich an (3,5). Aufstellen der Hesse-Matrix}
H &=& \begin{pmatrix} 4 & -2 \\
-2 & 2 \end{pmatrix} \\
\bigtriangleup_{1} &=& 4 > 0 \\
\bigtriangleup_{2} &=& 4 > 0 \\
\intertext{Die Hesse-Matrix ist positiv definit und damit befindet sich an der kritischen Stelle ein lokales Minimum.}
\end{alignat*}
\subsubsection{} %ii
\begin{alignat*}{2}
f(x,y) &=& x^{2} + 2y^{2} - 3xy -x -y +7 \\
I f_{x} &=& 2x - 3y - 1 = 0 \\
II f_{y} &=& 4y - 3x - 1 = 0 \\
f_{xx} &=& 2 \\
f_{xy} &=& -3 \\
f_{yx} &=& -3 \\
f_{yy} &=& 4 \\
II + I &=& -x + y -2 = 0 \\
&\Leftrightarrow & y = x + 2 \\
\intertext{Einsetzen in I}
&\Rightarrow & 2x - 3(x+2) - 1 = 0 \\
&\Leftrightarrow & 2x - 3x - 6 - 1 = 0 \\
&\Leftrightarrow & -x - 7 = 0 \\
&\Leftrightarrow & x = -7 \\
\intertext{Einsetzen in I}
&\Rightarrow & 2 \cdot (-7) - 3y - 1 = 0 \\
&\Leftrightarrow & -14 - 3y - 1 = 0 \\
&\Leftrightarrow & 3y = -15 \\
&\Leftrightarrow & y = -5 \\
\intertext{Einsetzen beider Werte in II}
&\Rightarrow & 4 \cdot (-5) - 3 \cdot (-7) - 1 = 0 \\
&\Leftrightarrow & -20 + 21 - 1 = 0 \\
&\Leftrightarrow & 0 = 0 \\
\intertext{Die einzige kritische Stelle befindet sich an (-7, -5). Aufstellen der Hesse-Matrix}
H &=& \begin{pmatrix} 2 & -3 \\
-3 & 4 \end{pmatrix} \\
\bigtriangleup_{1} &=& 2 > 0 \\
\bigtriangleup_{2} &=& -1 < 0 \\
\intertext{Die Hesse-Matrix ist damit weder positiv noch negativ definit und daher infinit. Daher liegt an der kritischen Stelle kein lokales Extremum vor.}
\end{alignat*}
\subsubsection{} %iii
\begin{alignat*}{2}
f(x,y) &=& 2x^{3} + y^{3} - 12x -27y +2 \\
I f_{x} &=& 6x^{2} -12 = 0\\
II f_{y} &=& 3y^{2} - 27 = 0\\
f_{xx} &=& 12x \\
f_{xy} &=& 0 \\
f_{yx} &=& 0 \\
f_{yy} &=& 6y \\
I &\Rightarrow & 6x^{2} - 12 = 0 \\
&\Leftrightarrow & 6x^{2} = 12 \\
&\Leftrightarrow & x^{2} = 2 \\
&\Rightarrow & x_{1} = \sqrt{2} \\
&\Rightarrow & x_{2} = -\sqrt{2} \\
I + II &=& 6x^{2} + 3y^{2} - 39 = 0 \\
\intertext{Einsetzen von den x-Werten und berechnen von y}
x_{1} &\Rightarrow & 6 \cdot 2 + 3y^{2} -39 = 0 \\
&\Leftrightarrow & 3y^{2} - 27 = 0 \\
&\Leftrightarrow & 3y^{2} = 27 \\
&\Leftrightarrow & y^{2} = 9 \\
&\Leftrightarrow & y \pm 3 \\
x_{2} &\Rightarrow & 6 \cdot 2 + 3y^{2} - 39 = 0 \\
&\Leftrightarrow & 3y^{2} = 27 \\
&\Leftrightarrow & y \pm 3 \\
\intertext{Es gibt also vier kritische Stellen: $(\sqrt{2}, 3), (\sqrt{2}, -3), (-\sqrt{2}, 3)$ und $(-\sqrt{2},-3)$. Aufstellen der Hesse-Matrix}
H &=& \begin{pmatrix}12x & 0 \\
0 & 6y\end{pmatrix} \\
\intertext{Berechnen der Definitheit für erste kritische Stelle:}
\bigtriangleup_{1} &=& 12 \cdot \sqrt{2} > 0 \\
\bigtriangleup_{2} &=& 12 \cdot \sqrt{2} \cdot 18 \\
&=& 216 \cdot \sqrt{2} > 0 \\
\intertext{Die Hesse-Matrix ist für die erste kritische Stelle positiv definit. An der ersten kritischen Stelle liegt also ein lokales Minimum vor. Berechnen der Definitheit für die zweite kritische Stelle:}
\bigtriangleup_{1} &=& 12 \cdot \sqrt{2} > 0 \\
\bigtriangleup_{2} &=& 12 \cdot \sqrt{2} \cdot (-18) \\
&=& -216 \cdot \sqrt{2} < 0 \\
\intertext{Die Hesse-Matrix ist für die zweite kritische Stelle infinit. An der zweiten kritischen Stelle liegt also kein lokales Extremum vor. Berechnen der Definitheit für die dritte kritische Stelle:}
\bigtriangleup_{1} &=& -12 \cdot \sqrt{2} < 0 \\
\bigtriangleup_{2} &=& -12 \cdot \sqrt{2} \cdot 18 \\
&=& -216 \cdot \sqrt{2} < 0 \\
\intertext{Die Hesse-Matrix ist für die dritte kritische Stelle infinit. An der dritten kritischen Stelle liegt also kein lokales Extremum vor. Berechnen der Definitheit für die vierte kritische Stelle:}
\bigtriangleup_{1} &=& -12 \cdot \sqrt{2} < 0 \\
\bigtriangleup_{2} &=& -12 \cdot \sqrt{2} \cdot (-18) \\
&=& 216 \cdot \sqrt{2} > 0 \\
\intertext{Die Hesse-Matrix ist für die vierte kritische Stelle negativ definit. An der vierten kritischen Stelle liegt also ein lokales Maximum vor.}
\end{alignat*}
\section{} %4
\subsection{} %a
\begin{alignat*}{2}
C(x,y) &=& 0.01x^{2} + 0.02xy + 0.16y^{2} + 5x + 6y + 120 \\
&=& \frac{1}{100}x^{2} + \frac{1}{50}xy + \frac{4}{25}y^{2} + 5x + 6y + 120 \\
\intertext{Aufstellen der Gewinnfunktion}
G(x,y) &=& 12x + 28y - C(x,y) \\
&=& 12x + 28y - \frac{1}{100}x^{2} - \frac{1}{50}xy - \frac{4}{25}y^{2} - 5x - 6y - 120 \\
&=& -\frac{1}{100}x^{2} - \frac{4}{25}y^{2} - \frac{1}{50}xy + 7x + 22y - 120 \\
I\, G_{x} &=& -\frac{1}{50}x - \frac{1}{50}y + 7 = 0 \\
II\, G_{y} &=& -\frac{8}{25}y - \frac{1}{50}x + 22 = 0 \\
G_{xx} &=& -\frac{1}{50} \\
G_{xy} &=& 0 \\
G_{yx} &=& 0 \\
G_{yy} &=& -\frac{8}{25} \\
I &\Rightarrow & -\frac{1}{50}x - \frac{1}{50}y + 7 = 0 \\
&\Leftrightarrow & \frac{1}{50}x = 7 - \frac{1}{50}y \\
&\Leftrightarrow & x = 350 - y \\
\intertext{Einsetzen in II}
II &\Rightarrow & -\frac{8}{25}y - \frac{1}{50} (350-y) + 22 = 0 \\
&\Leftrightarrow & \frac{8}{25}y = 22 - \frac{1}{50} (350 - y) \\
&\Leftrightarrow & \frac{8}{25}y = 22 - 7 + \frac{1}{50}y \\
&\Leftrightarrow & \frac{15}{50}y = 15 \\
&\Leftrightarrow & \frac{3}{10}y = 15 \\
&\Leftrightarrow & y = 50 \\
\intertext{Einsetzen in I}
&\Rightarrow & x = 350 - 50 = 300
\intertext{Die einige kritische Stelle von G(x,y) befindet sich an $\left(300, 50\right)$. Aufstellen der Hesse-Matrix:}
H &=& \begin{pmatrix}-\frac{1}{50} & 0 \\
0 & -\frac{8}{25} \end{pmatrix} \\
\bigtriangleup_{1} &=& -\frac{1}{50} < 0 \\
\bigtriangleup_{2} &=& \frac{4}{625} > 0 \\
\intertext{Die Hesse-Matrix ist an der kritischen Stelle negativ definit. An der kritischen Stelle befindet sich daher ein Maximum. Der höchste Gewinn ist demnach mit 300 Einheiten des Gutes A und 50 Einheiten des Gutes B zu erreichen.}
\end{alignat*}
\subsection{} %b
\begin{alignat*}{2}
G(x,y) &=& -\frac{1}{100}x^{2} - \frac{4}{25}y^{2} - \frac{1}{50}xy + 7x + 22y - 120 \\
n(x,y) &=& x + 2y = 320 \\
&\Leftrightarrow & x= 320 - 2y \\
\intertext{Einsetzen in G(x,y)}
G(y) &=& -\frac{1}{100} \cdot (320-2y)^{2} - \frac{4}{25}y^{2} - \frac{1}{50} \cdot (320 - 2y)y + 7(320-2y) + 22y - 120 \\
&=& -\frac{1}{100} \cdot (102400 -1280y + 4y^{2}) - \frac{4}{25}y^{2} - \frac{1}{50} \cdot (320y -2y^{2}) + 2240 - 14y + 22y - 120 \\
&=& -1024 + \frac{128}{10}y - \frac{1}{25}y^{2} - \frac{4}{25}y^{2} - \frac{32}{5}y + \frac{1}{25}y^{2} + 8y + 2120 \\
&=& \frac{64}{5}y - \frac{5}{25}y^{2} - \frac{32}{5} + \frac{1}{25}y^{2} + 1096 + 8y \\
&=& -\frac{4}{25}y^{2} + \frac{72}{5}y + 1096 \\
G'(y) &=& -\frac{8}{25}y + \frac{72}{5} = 0 \\
&\Leftrightarrow & \frac{8}{25}y = \frac{72}{5} \\
&\Leftrightarrow & y = 45 \\
G''(y) &=& -\frac{8}{25} < 0 \\
\intertext{Unter der Nebenbedingung n(x,y) gibt es ein lokales Maximum für 45 Einheiten von Gut B.
Einsetzen von y in die Nebenbedingung:}
n(x) &=& x + 2 \cdot 45 = 320 \\
&\Leftrightarrow & x + 90 = 320 \\
&\Leftrightarrow & x = 230
\intertext{Die optimalen Mengen des Outputs liegen bei 230 Einheiten von Gut A und 45 Einheiten von Gut B.}
\end{alignat*}
\subsection{} %c
Berechnen des maximalen Gewinns für Fall a)\\
\begin{alignat*}{2}
G(300,50) &=& -\frac{1}{100} \cdot 300^{2} - \frac{4}{25} \cdot 50^{2} - \frac{1}{50} \cdot 300 \cdot 50 + 7 \cdot 300 + 22 \cdot 50 - 120 \\
&=& -900 - \frac{4}{25} \cdot 2500 - 300 + 2100 + 1100 - 120 \\
&=& 1880 - 400 \\
&=& 1480 \\
\intertext{Der maximale Gewinn im Fall a) beträgt 1480 Geldeinheiten.}
\end{alignat*}\\
Berechnen des maximalen Gewinns für Fall b)\\
\begin{alignat*}{2}
G(230,45) &=& -\frac{1}{100} \cdot 230^{2} - \frac{4}{25} \cdot 45^{2} - \frac{1}{50} \cdot 230 \cdot 45 + 7 \cdot 230 + 22 \cdot 45 - 120 \\
&=& -529 - \frac{4}{25} \cdot 2025 - \frac{1}{50} \cdot 10350 + 1610 + 990 - 120 \\
&=& 1951 - 324 - 207 \\
&=& 1420
\intertext{Der maximale Gewinn im Fall b) beträgt 1420 Geldeinheiten.}
\end{alignat*}
\end{document}

View File

@ -0,0 +1,222 @@
\documentclass[10pt,a4paper,oneside,ngerman,numbers=noenddot]{scrartcl}
\usepackage[T1]{fontenc}
\usepackage[utf8]{inputenc}
\usepackage[ngerman]{babel}
\usepackage{amsmath}
\usepackage{amsfonts}
\usepackage{amssymb}
\usepackage{paralist}
\usepackage{gauss}
\usepackage{pgfplots}
\usepackage[locale=DE,exponent-product=\cdot,detect-all]{siunitx}
\usepackage{tikz}
\usetikzlibrary{matrix,fadings,calc,positioning,decorations.pathreplacing,arrows,decorations.markings}
\usepackage{polynom}
\polyset{style=C, div=:,vars=x}
\pgfplotsset{compat=1.8}
\pagenumbering{arabic}
\def\thesection{\arabic{section})}
\def\thesubsection{\alph{subsection})}
\def\thesubsubsection{(\roman{subsubsection})}
\makeatletter
\renewcommand*\env@matrix[1][*\c@MaxMatrixCols c]{%
\hskip -\arraycolsep
\let\@ifnextchar\new@ifnextchar
\array{#1}}
\makeatother
\begin{document}
\author{Jim Martens (6420323)}
\title{Hausaufgaben zum 4. Juli}
\maketitle
\section{} %1
\subsection{} %a
\begin{alignat*}{2}
f(x,y,z) &=& 2x^{2} + y^{2} + 4z^{2} - 2yz - 2x - 6y + 8 \\
I f_{x} &=& 4x - 2 = 0\\
II f_{y} &=& 2y - 7z - 6 = 0\\
III f_{z} &=& 8z - 2y = 0 \\
I &\Rightarrow & 4x - 2 = 0 \\
&\Leftrightarrow & 4x = 2 \\
&\Leftrightarrow & x = \frac{1}{2} \\
III &\Rightarrow & 8z - 2y = 0 \\
&\Leftrightarrow & 8z = 2y \\
&\Leftrightarrow & 4z = y \\
II &\Rightarrow & 2 \cdot 4z - 7z - 6 = 0 \\
&\Leftrightarrow & 8z - 7z = 6 \\
&\Leftrightarrow & z = 6 \\
III &\Rightarrow & 4 \cdot 6 = y \\
&\Leftrightarrow & 24 = y \\
\intertext{Die einzige kritische Stelle befindet sich an ($\frac{1}{2}, 24, 6$).}
f_{xx} &=& 4 \\
f_{yx} &=& 0 \\
f_{zx} &=& 0 \\
f_{xy} &=& 0 \\
f_{yy} &=& 2 \\
f_{zy} &=& -7 \\
f_{xz} &=& 0 \\
f_{yz} &=& -2 \\
f_{zz} &=& 8 \\
\intertext{Aufstellen der Hesse-Matrix}
H &=& \begin{pmatrix}4 & 0 & 0 \\
0 & 2 & -7 \\
0 & -2 & 8\end{pmatrix} \\
\bigtriangleup_{1} &=& 4 > 0\\
\bigtriangleup_{2} &=& 8 > 0\\
\bigtriangleup_{3} &=& 64 - 56 = 8 > 0\\
\intertext{Die Hesse-Matrix ist positiv definit. Daher liegt an der kritischen Stelle ein Minimum vor.}
\end{alignat*}
\subsection{} %b
\begin{alignat*}{2}
grad\,f(1,1,1) &=& (4 - 2, 2 - 7 - 6, 8 - 2) \\
&=& (2, -11, 6) \\
||grad\,f(1,1,1)|| &=& \sqrt{2^{2} + 11^{2} + 6^{2}} \\
&=& \sqrt{4 + 121 + 36} \\
&=& \sqrt{161} \\
\intertext{In Richtung von (2, -11, 6) steigt die Temperatur von (1,1,1) aus am stärksten an. In Richtung (-2, 11, -6) sinkt die Temperatur am stärksten. Die Größe des stärksten Anstiegs beträgt $\sqrt{161}$.}
\end{alignat*}
\section{} %2
\subsection{} %a
\begin{alignat*}{2}
A &=& \begin{pmatrix}-i & -1 \\
3 & i \end{pmatrix} \\
B &=& \begin{pmatrix} i \\
1 + i\end{pmatrix} \\
C &=& \begin{pmatrix}-i & i \end{pmatrix} \\
AB &=& \begin{pmatrix}-i \cdot i + (1+i)\cdot (-1) \\
3i + i(1+i) \end{pmatrix} \\
&=& \begin{pmatrix}-i^{2} - 1 - i \\
3i + i + i^{2} \end{pmatrix} \\
&=& \begin{pmatrix}- 1 - i \\
4i - 1 \end{pmatrix} \\
\intertext{AC existiert nicht, da A mehr Spalten hat, als C Zeilen hat.}
BC &=& \begin{pmatrix}i \cdot (-i) & i \cdot i \\
(1+i) \cdot (-i) & (1+i)i \end{pmatrix} \\
&=& \begin{pmatrix}-i^{2} & i^{2} \\
-i - i^{2} & i + i^{2} \end{pmatrix} \\
&=& \begin{pmatrix}1 & -1 \\
-i +1 & i -1 \end{pmatrix} \\
CB &=& \begin{pmatrix}-i \cdot i + i(1+i) \end{pmatrix} \\
&=& \begin{pmatrix}-i^{2} + i + i^{2} \end{pmatrix} \\
&=& \begin{pmatrix}1 + i -1 \end{pmatrix} \\
&=& \begin{pmatrix}i \end{pmatrix}
\end{alignat*}
\subsection{} %b
\begin{alignat*}{2}
\overline{z} &=& \frac{3 + 2i}{4 - 3i} \\
&=& \frac{(3 + 2i)(4+3i)}{(4 - 3i)(4+3i)} \\
&=& \frac{12 + 9i + 8i + 6i^{2}}{16 + 12i - 12i - 9i^{2}} \\
&=& \frac{12 - 6 + 17i}{16 + 9} \\
&=& \frac{6 + 17i}{25} \\
&=& \frac{6}{25} + \frac{17}{25}i \\
z &=& \frac{6}{25} - \frac{17}{25}i \\
a &=& \frac{6}{25} \\
b &=& \frac{17}{25}
\end{alignat*}
\subsection{} %c
\begin{alignat*}{2}
z_{1} &=& -1 - i = (-1, -1) \\
z_{2} &=& \sqrt{2} \cdot \cos \frac{\pi}{4} + \sqrt{2} \cdot i \cdot \sin \frac{\pi}{4} \\
&=& \sqrt{2} \cdot \cos \left(\frac{1}{2} \cdot \frac{\pi}{2} \right) + \sqrt{2} \cdot \sin \left(\frac{1}{2} \cdot \frac{\pi}{2} \right) \\
&=& 1 + i = (1, 1) \\
z_{3} &=& (-1 -i)(1+ i) \\
&=& -1 - i - i - i^{2} \\
&=& -1 + 1 - 2i = 0 - 2i = (0, -2) \\
z_{4} &=& -1 + i = (-1, 1)
\end{alignat*}
\begin{tikzpicture}[>=stealth]
\begin{axis}[
ymin=-5,ymax=5,
x=1cm,
y=1cm,
axis x line=middle,
axis y line=middle,
axis line style=->,
xlabel={$\Re$},
ylabel={$\Im$},
xmin=-5,xmax=5
]
\node at (axis cs: -1,-1) {$z_{1}$};
\node at (axis cs: 1,1) {$z_{2}$};
\node at (axis cs: 0,-2) {$z_{3}$};
\node at (axis cs: -1,1) {$z_{4}$};
\end{axis}
\end{tikzpicture}
\subsection{} %d
\subsubsection{} %i
In dieser Teilmenge sind alle komplexen Zahlen enthalten, die sich auf der Geraden befinden, die durch die Punkte (1,1) und (2,0) geht.
\subsubsection{} %ii
In der Teilmenge sind alle komplexen Zahlen enthalten, die sich auf der Kreislinie eines Kreises mit dem Radius 1 um den Punkt (1,1) befinden.
\section{} %3
\begin{alignat*}{2}
f(x,y) &=& -\frac{1}{5}x^{2} - xy - \frac{25}{10}y^{2} + 48x + 235y - 88 \\
g(x,y) &=& \frac{1}{5}x + y = 40
\end{alignat*}
\subsection{} %a
\begin{alignat*}{2}
g_{x} &=& \frac{1}{5} \\
g_{y} &=& 1 \\
\begin{pmatrix}\frac{\partial g}{\partial x} (x,y) & \frac{\partial g}{\partial y} (x,y)\end{pmatrix} &=& \begin{pmatrix}\frac{1}{5} & 1 \end{pmatrix} \\
\intertext{Der Rang dieser Matrix ist für alle $x,y$ gleich 2. Damit ist die Regularitätsbedingung erfüllt.}
L(x,y,\lambda) &=& -\frac{1}{5}x^{2} - xy - \frac{25}{10}y^{2} + 48x + 235y - 88 + \lambda(\frac{1}{5}x + y - 40) \\
I L_{x} &=& - \frac{2}{5}x - y + 48 + \lambda \cdot \frac{1}{5} = 0 \\
II L_{y} &=& - x - 5y + 235 + \lambda = 0 \\
III L_{\lambda} &=& \frac{1}{5}x + y - 40 = 0 \\
II &\Rightarrow & \lambda = x + 5y - 235 \\
I &\Rightarrow & - \frac{2}{5}x - y + 48 + (x + 5y - 235) \cdot \frac{1}{5} = 0\\
&\Leftrightarrow & - \frac{2}{5}x - y + 48 + \frac{1}{5}x + y - 47 = 0 \\
\intertext{Beachten der dritten Gleichung}
&\Leftrightarrow & - \frac{1}{5}x - \frac{1}{5}x - y + 48 - 7 = 0\\
&\Leftrightarrow & - \frac{1}{5}x + 1=\frac{1}{5}x + y - 40 \\
\intertext{Beachten der dritten Gleichung}
&\Leftrightarrow & - \frac{1}{5}x + 1 = 0 \\
&\Leftrightarrow & \frac{1}{5}x = 1 \\
&\Leftrightarrow & x = 5 \\
\intertext{Einsetzen in I}
&\Rightarrow & - \frac{1}{5} \cdot 5 - \frac{1}{5} \cdot 5 - y + 48 - 7 = 0 \\
&\Leftrightarrow & - 1 - 1 - y + 41 = 0 \\
&\Leftrightarrow & -y + 39 = 0 \\
&\Leftrightarrow & y = 39 \\
\intertext{Einsetzen in III}
\lambda &=& \frac{1}{5} \cdot 5 + 39 - 40 \\
&=& 1 + 39 - 40 \\
&=& 0
\intertext{Die einzige kritische Stelle befindet sich an (5, 39).}
\end{alignat*}
\subsection{} %b
\begin{alignat*}{2}
L_{xx} &=& - \frac{2}{5} \\
L_{yx} &=& - 1 \\
L_{xy} &=& - 1 \\
L_{yy} &=& - 5 \\
\overline{H} &=& \begin{pmatrix}0 & \frac{1}{5} & 1 \\
\frac{1}{5} & -\frac{2}{5} & -1\\
1 & -1 & -5 \end{pmatrix} \\
det \, \overline{H} &=& -\frac{1}{5} - \frac{1}{5} + \frac{2}{5} + \frac{1}{5} \\
&=& \frac{1}{5} > 0 \\
\intertext{An der kritischen Stelle liegt ein Maximum vor.}
\end{alignat*}
\section{} %4
\begin{alignat*}{2}
f(x,y) &=& -\frac{1}{5}x^{2} - xy - \frac{25}{10}y^{2} + 48x + 235y - 88 \\
g(x,y) &=& \frac{1}{5}x + y -40 = 0 \\
&\Leftrightarrow & y = -\frac{1}{5}x + 40 \\
\intertext{Einsetzen in f(x,y)}
f(x) &=& -\frac{1}{5}x^{2} - x \cdot (-\frac{1}{5}x + 40) - \frac{25}{10} \cdot (-\frac{1}{5}x + 40)^{2} + 48x + 235 \cdot (-\frac{1}{5}x + 40) - 88 \\
&=& -\frac{1}{5}x^{2} + \frac{1}{5}x^{2} - 40x - \frac{25}{10} \cdot (\frac{1}{25}x^{2} -16x + 1600) + 48x -47x + 9400 - 88 \\
&=& - 40x - \frac{1}{10}x^{2} + 40x - 4000 + 48x -47x + 9400 - 88 \\
&=& - \frac{1}{10}x^{2} + x + 5312 \\
f'(x) &=& -\frac{1}{5}x + 1 = 0 \\
&\Leftrightarrow & \frac{1}{5}x = 1 \\
&\Leftrightarrow & x = 5 \\
f''(x) &=& -\frac{1}{5} < 0 \\
\intertext{Unter der Nebenbedingung g(x,y) gibt es ein lokales Maximum für x = 5.
Einsetzen von x in die Nebenbedingung:}
g(y) &=& \frac{1}{5} \cdot 5 + y -40 = 0 \\
&\Leftrightarrow & 1 + y - 40 = 0 \\
&\Leftrightarrow & y - 39 = 0 \\
&\Leftrightarrow & y = 39 \\
\intertext{An der Stelle (5, 39) ist der Gewinn maximal.}
\end{alignat*}
\end{document}

View File

@ -0,0 +1,102 @@
\documentclass[10pt,a4paper,oneside,ngerman,numbers=noenddot]{scrartcl}
\usepackage[T1]{fontenc}
\usepackage[utf8]{inputenc}
\usepackage[ngerman]{babel}
\usepackage{amsmath}
\usepackage{amsfonts}
\usepackage{amssymb}
\usepackage{paralist}
\usepackage{gauss}
\usepackage{pgfplots}
\usepackage[locale=DE,exponent-product=\cdot,detect-all]{siunitx}
\usepackage{tikz}
\usetikzlibrary{matrix,fadings,calc,positioning,decorations.pathreplacing,arrows,decorations.markings}
\usepackage{polynom}
\polyset{style=C, div=:,vars=x}
\pgfplotsset{compat=1.8}
\pagenumbering{arabic}
\def\thesection{\arabic{section})}
\def\thesubsection{\alph{subsection})}
\def\thesubsubsection{(\roman{subsubsection})}
\makeatletter
\renewcommand*\env@matrix[1][*\c@MaxMatrixCols c]{%
\hskip -\arraycolsep
\let\@ifnextchar\new@ifnextchar
\array{#1}}
\makeatother
\begin{document}
\author{Jim Martens (6420323)}
\title{Hausaufgaben zum 11. Juli}
\maketitle
\section{} %1
\begin{alignat*}{2}
\iint\limits_{I} f(x,y)\, d(x,y) &=& \iint\limits_{I} (2x^{2}y)\, d(x,y) \\
\intertext{Version 1}
&=& \int\limits_{1}^{2} \left( \int\limits_{-1}^{3} (2x^{2}y) \, dy \right) \, dx \\
&=& \int\limits_{1}^{2} \left( 2x^{2} \int\limits_{-1}^{3} y \, dy \right)\, dx \\
&=& \int\limits_{1}^{2} \left( 2x^{2} \left[\frac{1}{2}y^{2}\right]_{-1}^{3} \right)\, dx \\
&=& \int\limits_{1}^{2} \left( 2x^{2} \left[\frac{9}{2} - \frac{1}{2}\right] \right)\, dx \\
&=& \int\limits_{1}^{2} \left( 2x^{2} \cdot 4 \right)\, dx \\
&=& 8 \int\limits_{1}^{2} x^{2}\, dx \\
&=& 8 \left[\frac{1}{3}x^{3} \right]_{1}^{2} \\
&=& 8 \left[\frac{8}{3} - \frac{1}{3} \right] \\
&=& 8 \cdot \frac{7}{3} = \frac{56}{3} \\
\intertext{Version 2}
&=& \int\limits_{-1}^{3} \left( \int\limits_{1}^{2} (2x^{2}y) \, dx \right) \, dy \\
&=& \int\limits_{-1}^{3} \left( 2y\int\limits_{1}^{2} x^{2} \, dx \right) \, dy \\
&=& \int\limits_{-1}^{3} \left( 2y\left[\frac{1}{3}x^{3} \right]_{1}^{2} \right) \, dy \\
&=& \int\limits_{-1}^{3} \left( 2y\left[\frac{8}{3} - \frac{1}{3} \right] \right) \, dy \\
&=& \int\limits_{-1}^{3} \left( 2y \cdot \frac{7}{3} \right) \, dy \\
&=& \frac{14}{3}\int\limits_{-1}^{3} y \, dy \\
&=& \frac{14}{3}\left[\frac{1}{2}y^{2}\right]_{-1}^{3} \\
&=& \frac{14}{3}\left[\frac{9}{2} - \frac{1}{2}\right] \\
&=& \frac{14}{3} \cdot 4 = \frac{56}{3}
\end{alignat*}
\section{} %2
\subsubsection{} %i
\begin{alignat*}{2}
\iint\limits_{G} f(x,y) \, d(x,y) &=& \iint\limits_{G} (xy^{2}) \, d(x,y) \\
&=& \int\limits_{0}^{1} \left(\int\limits_{0}^{3x} xy^{2}\, dy \right) \, dx \\
&=& \int\limits_{0}^{1} \left(x\int\limits_{0}^{3x} y^{2}\, dy \right) \, dx \\
&=& \int\limits_{0}^{1} \left(x \left[\frac{1}{3}y^{3} \right]_{0}^{3x} \right) \, dx \\
&=& \int\limits_{0}^{1} \left(x \left[\frac{1}{3} \cdot 27x^{3} \right] \right) \, dx \\
&=& \int\limits_{0}^{1} \left(x \cdot 9x^{3} \right) \, dx \\
&=& \int\limits_{0}^{1} \left(9x^{4} \right) \, dx \\
&=& 9\int\limits_{0}^{1} x^{4} \, dx \\
&=& 9\left[\frac{1}{5}x^{5} \right]_{0}^{1} \\
&=& 9 \cdot \frac{1}{5} = \frac{9}{5}
\end{alignat*}
\subsubsection{} %ii
\begin{alignat*}{2}
\iint\limits_{G} f(x,y) \, d(x,y) &=& \iint\limits_{G} (xy^{2}) \, d(x,y) \\
&=& \int\limits_{0}^{1} \left(\int\limits_{3x}^{3} xy^{2} \, dy \right)\, dx \\
&=& \int\limits_{0}^{1} \left(x\int\limits_{3x}^{3} y^{2} \, dy \right)\, dx \\
&=& \int\limits_{0}^{1} \left(x \left[\frac{1}{3}y^{3} \right]_{3x}^{3} \right)\, dx \\
&=& \int\limits_{0}^{1} \left(x \left[\frac{27}{3} - \frac{27}{3}x^{3} \right] \right)\, dx \\
&=& \int\limits_{0}^{1} \left(9x - 9x^{4} \right)\, dx \\
&=& \int\limits_{0}^{1} \left(9(x - x^{4}) \right)\, dx \\
&=& 9\int\limits_{0}^{1} x - x^{4}\, dx \\
&=& 9\int\limits_{0}^{1} x \, dx - 9\int\limits_{0}^{1} x^{4}\, dx \\
&=& 9 \left[\frac{1}{2}x^{2} \right]_{0}^{1} - 9\left[\frac{1}{5}x^{5} \right]_{0}^{1} \\
&=& 9 \cdot \frac{1}{2} - 9 \cdot \frac{1}{5} \\
&=& \frac{9}{2} - \frac{9}{5} \\
&=& \frac{45}{10} - \frac{18}{10} = \frac{27}{10}
\end{alignat*}
\section{} %3
\subsection{} %a
Klarerweise gilt $f_{4}(n) = O(f_{5}(n))$. Ebenfalls gilt $f_{3}(n) = O(f_{1}(n))$. Außerdem ist klar, dass $f_{1}(n) = O(f_{4}(n))$ gilt.
Damit ergibt sich die Reihenfolge $f_{3}, f_{1}, f_{4}, f_{5}$.
Es müssen noch $f_{2}$ und $f_{6}$ eingeordnet werden. $f_{2}(n) = O(f_{1}(n))$ gilt ebenso wie $f_{3}(n) = O(f_{2}(n))$. $f_{2}$ kann demnach zwischen $f_{3}$ und $f_{1}$ eingeordnet werden, womit sich die Reihenfolge $f_{3}, f_{2}, f_{1}, f_{4}, f_{5}$ ergibt.
Abschließend muss noch $f_{6}(n)$ eingeordnet werden. $n^{2}$ kommt als Faktor auch in $f_{1}$ vor. Es bleibt daher die Frage, ob $\sqrt{n}$ schneller wächst als $\log_{2}(n)$. Dem ist so, da Wurzelfunktionen allgemein schneller wachsen als Logarithmusfunktionen. Daher gilt $f_{6}(n) = O(f_{1}(n))$. Gleichzeitig gilt, dass $\sqrt{2n}$ langsamer wächst als $f_{6}(n)$, womit auch $f_{2}(n) = O(f_{6}(n))$ gilt. Die fertige Reihenfolge ist daher $f_{3}, f_{2}, f_{6}, f_{1}, f_{4}, f_{5}$.
\subsection{} %b
\section{} %4
\subsection{} %a
\subsection{} %b
\begin{alignat*}{2}
\intertext{Es gilt $n = \lfloor x \rfloor, x \geq 1$ für beide Funktionen:}
f(n) &=& n \\
g(n) &=& n^{1+ \lceil \sin (x) \rceil}
\end{alignat*}
\end{document}

View File

@ -0,0 +1,216 @@
\documentclass[10pt,a4paper,oneside,ngerman,numbers=noenddot]{scrartcl}
\usepackage[T1]{fontenc}
\usepackage[utf8]{inputenc}
\usepackage[ngerman]{babel}
\usepackage{amsmath}
\usepackage{amsfonts}
\usepackage{amssymb}
\usepackage{paralist}
\usepackage{gauss}
\usepackage[locale=DE,exponent-product=\cdot,detect-all]{siunitx}
\usepackage{tikz}
\usetikzlibrary{matrix,fadings,calc,positioning,decorations.pathreplacing,arrows,decorations.markings}
\usepackage{polynom}
\polyset{style=C, div=:,vars=x}
\pagenumbering{arabic}
\def\thesection{\arabic{section})}
\def\thesubsection{\alph{subsection})}
\def\thesubsubsection{(\roman{subsubsection})}
\makeatletter
\renewcommand*\env@matrix[1][*\c@MaxMatrixCols c]{%
\hskip -\arraycolsep
\let\@ifnextchar\new@ifnextchar
\array{#1}}
\makeatother
\begin{document}
\author{Jim Martens (6420323)}
\title{Hausaufgaben zum 18. April}
\maketitle
\section{} %1
\subsubsection{} %(i)
\begin{alignat*}{3}
\underset{n \rightarrow \infty}{\text{lim}} \left(\frac{-3n^{4}+2n^{2}+n+1}{-7n^{4}+25} \right) &\Rightarrow & \frac{-3n^{4}+2n^{2}+n+1}{-7n^{4}+25} && \\
\intertext{Ausklammern von $n^{4}$}
&\Leftrightarrow & \frac{-3 + \frac{2}{n^{2}} + \frac{1}{n^{3}} + \frac{1}{n^{4}}}{-7 + \frac{25}{n^{4}}} &\rightarrow & \frac{3}{7}
\end{alignat*}
\subsubsection{} %(ii)
\begin{alignat*}{3}
\underset{n \rightarrow \infty}{\text{lim}} \left(\frac{-3n^{4}+2n^{2}+n+1}{-7n^{5}+25} \right) &\Rightarrow & \frac{-3n^{4}+2n^{2}+n+1}{-7n^{5}+25} && \\
\intertext{Ausklammern von $n^{4}$ im Zähler und $n^{5}$ im Nenner}
&\Leftrightarrow & \frac{1}{n} \cdot \frac{-3 + \frac{2}{n^{2}} + \frac{1}{n^{3}} + \frac{1}{n^{4}}}{-7 + \frac{25}{n^{5}}} &\rightarrow & 0
\end{alignat*}
\subsubsection{} %(iii)
\begin{alignat*}{3}
\underset{n \rightarrow \infty}{\text{lim}} \left(\frac{-3n^{5}+2n^{2}+n+1}{-7n^{4}+25} \right) &\Rightarrow & \frac{-3n^{5}+2n^{2}+n+1}{-7n^{4}+25} && \\
\intertext{Ausklammern von $n^{5}$ im Zähler und $n^{4}$ im Nenner}
&\Leftrightarrow & n \cdot \frac{-3 + \frac{2}{n^{3}} + \frac{1}{n^{4}} + \frac{1}{n^{5}}}{-7 + \frac{25}{n^{4}}} &\rightarrow & \infty
\end{alignat*}
\subsubsection{} %(iv)
\begin{alignat*}{3}
\hspace{-2.5cm}
\underset{n \rightarrow \infty}{\text{lim}} \left( \frac{6n^{3}+2n-3}{9n^{2}+2} - \frac{2n^{3}+5n^{2}+7}{3n^{2}+3} \right) &\Rightarrow & \frac{6n^{3}+2n-3}{9n^{2}+2} - \frac{2n^{3}+5n^{2}+7}{3n^{2}+3} && \\
\intertext{Auf gleichen Nenner bringen}
\hspace{-2.5cm}
&\Leftrightarrow & \frac{(6n^{3}+2n-3)(3n^{2}+3) - (2n^{3}+5n^{2}+7)(9n^{2}+2)}{(9n^{2}+2)(3n^{2}+3)} && \\
\intertext{Klammern auflösen und zusammenfassen}
\hspace{-2.5cm}
&\Leftrightarrow & \frac{-45n^{4} + 20n^{3}-82n^{2}+6n-23}{27n^{4}+33n^{2}+6} && \\
\intertext{Ausklammern von $n^{4}$}
\hspace{-2.5cm}
&\Leftrightarrow & \frac{-45 + \frac{20}{n}-\frac{82}{n^{2}}+\frac{6}{n^{3}}-\frac{23}{n^{4}}}{27+\frac{33}{n^{2}}+\frac{6}{n^{4}}} &\rightarrow & \frac{-45}{27} = \frac{-5}{3}
\end{alignat*}
\subsubsection{} %(v)
\begin{alignat*}{3}
\underset{n \rightarrow \infty}{\text{lim}} \left( \frac{\sqrt{9n^{4}+n^{2}+1}-2n^{2}+3}{\sqrt{2n^{2}+1} \cdot \sqrt{2n^{2}+n+1}} \right) &\Rightarrow & \frac{\sqrt{9n^{4}+n^{2}+1}-2n^{2}+3}{\sqrt{2n^{2}+1} \cdot \sqrt{2n^{2}+n+1}} && \\
\intertext{Anwendung der Wurzelgesetze}
&\Leftrightarrow &\frac{\sqrt{9n^{4}+n^{2}+1}-2n^{2}+3}{\sqrt{(2n^{2}+1) \cdot (2n^{2}+n+1)}} && \\
\intertext{Zusammenfassen}
&\Leftrightarrow &\frac{\sqrt{9n^{4}+n^{2}+1}-2n^{2}+3}{\sqrt{4n^{4}+2n^{3}+4n^{2} + 1}} && \\
\intertext{$n^{2}$ ausklammern}
&\Leftrightarrow &\frac{\sqrt{9+\frac{1}{n^{2}}+\frac{1}{n^{4}}}-2+\frac{3}{n^{2}}}{\sqrt{4+\frac{2}{n}+\frac{4}{n^{2}} + \frac{1}{n^{4}}}} &\rightarrow & \frac{7}{2}
\end{alignat*}
\section{} %2
\subsection{} %a
\subsubsection{} %i
\begin{alignat*}{3}
a_{0} &=& 1 &&&\\
a_{1} &=& \frac{2}{5} &&& \\
a_{2} &=& \left(\frac{2}{5}\right)^{2} = \frac{4}{25} &&& \\
a_{3} &=& \left(\frac{2}{5}\right)^{3} = \frac{8}{125} &&& \\
a_{4} &=& \left(\frac{2}{5}\right)^{4} = \frac{16}{625} &&& \\
s_{0} &=& a_{0} &=&& 1 \\
s_{1} &=& a_{0} + a_{1} = 1 + \frac{2}{5} = \frac{7}{5} &=&& 1.4 \\
s_{2} &=& a_{0} + a_{1} + a_{2} = \frac{7}{5} + \frac{4}{25} = \frac{39}{25} &=&& 1.56 \\
s_{3} &=& a_{0} + a_{1} + a_{2} + a_{3} = \frac{39}{25} + \frac{8}{125} = \frac{203}{125} &=&& 1.624 \\
s_{4}&=& a_{0} + a_{1} + a_{2} + a_{3} + a_{4} = \frac{203}{125} + \frac{16}{625} = \frac{1031}{625} &=&& 1.6496
\end{alignat*}\\
Bestimmung des Grenzwertes mithilfe der Geometrischen Summenformel:\\
\begin{alignat*}{2}
\underset{n \rightarrow \infty}{\text{lim}} \left( \sum\limits_{i=0}^{n} \left(\frac{2}{5}\right)^{i} \right) &=& \underset{n \rightarrow \infty}{\text{lim}} \left(\frac{1-\left(\frac{2}{5}\right)^{n+1}}{1-\left(\frac{2}{5}\right)} \right) \\
&=& \frac{1}{\frac{3}{5}} = \frac{5}{3}
\end{alignat*}
\subsubsection{} %ii
\begin{alignat*}{3}
a_{0} &=& 1 &&& \\
a_{1} &=& \frac{5}{2}&&& \\
a_{2} &=& \left(\frac{5}{2}\right)^{2} = \frac{25}{4} &&&\\
a_{3} &=& \left(\frac{5}{2}\right)^{3} = \frac{125}{8} &&& \\
a_{4} &=& \left(\frac{5}{2}\right)^{4} = \frac{625}{16} &&& \\
s_{0} &=& a_{0} &=& 1 \\
s_{1} &=& a_{0} + a_{1} = 1 + \frac{5}{2} = \frac{7}{2} &=&& 3.5 \\
s_{2} &=& a_{0} + a_{1} + a_{2} = \frac{7}{2} + \frac{25}{4} = \frac{39}{4} &=&& 9.75 \\
s_{3} &=& a_{0} + a_{1} + a_{2} + a_{3} = \frac{39}{4} + \frac{125}{8} = \frac{203}{8} &=&& 25.375 \\
s_{4}&=& a_{0} + a_{1} + a_{2} + a_{3} + a_{4} = \frac{203}{8} + \frac{625}{16} = \frac{1031}{16} &=&& 64,4375
\end{alignat*}\\
Die Reihe divergiert, da geometrische Reihen immer divergieren, wenn der Betrag von q größer als $1$ ist. Dies ist mit $\frac{5}{2}$ der Fall.
\subsubsection{} %iii
\begin{alignat*}{3}
a_{0} &=& 1 &&&\\
a_{1} &=& -\frac{2}{5} &&& \\
a_{2} &=& \left(-\frac{2}{5}\right)^{2} = \frac{4}{25} &&& \\
a_{3} &=& \left(-\frac{2}{5}\right)^{3} = -\frac{8}{125} &&& \\
a_{4} &=& \left(-\frac{2}{5}\right)^{4} = \frac{16}{625} &&& \\
s_{0} &=& a_{0} &=&& 1 \\
s_{1} &=& a_{0} + a_{1} = 1 - \frac{2}{5} = \frac{3}{5} &=&& 0.6 \\
s_{2} &=& a_{0} + a_{1} + a_{2} = \frac{3}{5} + \frac{4}{25} = \frac{19}{25} &=&& 0.76 \\
s_{3} &=& a_{0} + a_{1} + a_{2} + a_{3} = \frac{19}{25} - \frac{8}{125} = \frac{87}{125} &=&& 0.696 \\
s_{4}&=& a_{0} + a_{1} + a_{2} + a_{3} + a_{4} = \frac{87}{125} + \frac{16}{625} = \frac{451}{625} &=&& 0.7216
\end{alignat*} \\
Bestimmung des Grenzwertes mithilfe der Geometrischen Summenformel:\\
\begin{alignat*}{2}
\underset{n \rightarrow \infty}{\text{lim}} \left( \sum\limits_{i=0}^{n} \left(-\frac{2}{5}\right)^{i} \right) &=& \underset{n \rightarrow \infty}{\text{lim}} \left(\frac{1-\left(-\frac{2}{5}\right)^{n+1}}{1+\left(\frac{2}{5}\right)} \right) \\
&=& \frac{1}{\frac{7}{5}} = \frac{5}{7}
\end{alignat*}
\subsection{} %b
\subsubsection{} %i
Bestimmung des Grenzwertes mithilfe der Geometrischen Summenformel:\\
\begin{alignat*}{2}
\underset{n \rightarrow \infty}{\text{lim}} \left( \sum\limits_{i=0}^{n} \left(-\frac{3}{10}\right)^{i} \right) &=& \underset{n \rightarrow \infty}{\text{lim}} \left(\frac{1-\left(-\frac{3}{10}\right)^{n+1}}{1+\left(\frac{3}{10}\right)} \right) \\
&=& \frac{1}{\frac{13}{10}} = \frac{10}{13}
\end{alignat*}
Die Reihe konvergiert gegen den Wert $\frac{10}{13} \approx 0.769$.
\subsubsection{} %ii
\begin{alignat*}{3}
&& \underset{n \rightarrow \infty}{\text{lim}} \left( \sum\limits_{i=0}^{n} x^{i} \right) &=& \frac{5}{8} \\
\Rightarrow && \frac{1}{1-x} &=& \frac{5}{8} \\
\overset{\cdot (1-x)}{\Leftrightarrow} && 1 &=& \frac{5}{8} - \frac{5}{8}x \\
\overset{-\frac{5}{8}}{\Leftrightarrow} && \frac{3}{8} &=& -\frac{5}{8}x \\
\overset{\cdot -\frac{8}{5}}{\Leftrightarrow} && -\frac{3}{5} &=& x
\end{alignat*}
$x$ ist gleich $-\frac{3}{5}$.
\section{} %3
\subsection{}
\subsubsection{} %(i)
Die Reihe konvergiert, da der Betrag von $q = \frac{7}{9}$ kleiner als $1$ ist.\\
Berechnung des Grenzwertes mithilfe der Geometrischen Summenformel:\\
\begin{alignat*}{2}
\underset{n \rightarrow \infty}{\text{lim}} \left( \sum\limits_{i=0}^{n} \left(\frac{7}{9}\right)^{i} \right) &=& \underset{n \rightarrow \infty}{\text{lim}} \left(\frac{1-\left(\frac{7}{9}\right)^{n+1}}{1-\left(\frac{7}{9}\right)} \right) \\
&=& \frac{1}{\frac{2}{9}} = \frac{9}{2}
\end{alignat*}
\subsubsection{} %(ii)
Die Reihe konvergiert, da der Betrag von $q = -\frac{7}{9}$ kleiner als $1$ ist.\\
Berechnung des Grenzwertes mithilfe der Geometrischen Summenformel:\\
\begin{alignat*}{2}
\sum\limits_{i=1}^{\infty} \left(-\frac{7}{9}\right)^{i} &=& \sum\limits_{i=0}^{\infty} \left(-\frac{7}{9}\right)^{i} - \left(-\frac{7}{9}\right)^{0} \\
&=& \underset{n \rightarrow \infty}{\text{lim}} \left( \sum\limits_{i=0}^{n} \left(-\frac{7}{9}\right)^{i} - \left(-\frac{7}{9}\right)^{0} \right) \\
&=& \underset{n \rightarrow \infty}{\text{lim}} \left( \sum\limits_{i=0}^{n} \left(-\frac{7}{9}\right)^{i} \right) - 1 \\
&=& \frac{1}{1 + \frac{7}{9}} - 1 = \frac{1}{\frac{16}{9}} \\
&=& \frac{9}{16} - 1 = -\frac{7}{16}
\end{alignat*}
Der Grenzwert ist $-\frac{7}{16}$.
\subsubsection{} %(iii)
\begin{alignat*}{2}
\sum\limits_{i=2}^{\infty} (-1)^{i} \cdot \left( \frac{7}{9} \right)^{i+1} &=& \sum\limits_{i=2}^{\infty} (-1)^{i} \cdot \left( \frac{7}{9} \right)^{i} \cdot \left( \frac{7}{9} \right) \\
&=& \sum\limits_{i=2}^{\infty} \left( -\frac{7}{9} \right)^{i} \cdot \left( \frac{7}{9} \right) \\
&=& \left( \frac{7}{9} \right) \cdot \sum\limits_{i=2}^{\infty} \left( -\frac{7}{9} \right)^{i} \\
&=& \underset{n \rightarrow \infty}{\text{lim}} \left( \frac{7}{9} \cdot \sum\limits_{i=2}^{n} \left( -\frac{7}{9} \right)^{i} \right) \\
&=& \underset{n \rightarrow \infty}{\text{lim}} \left( \frac{7}{9} \cdot \left( \sum\limits_{i=0}^{n} \left( -\frac{7}{9} \right)^{i} - \left( -\frac{7}{9} \right)^{0} - \left( -\frac{7}{9} \right)^{1} \right) \right) \\
&=& \underset{n \rightarrow \infty}{\text{lim}} \left( \frac{7}{9} \cdot \left( \sum\limits_{i=0}^{n} \left( -\frac{7}{9} \right)^{i} - 1 + \frac{7}{9} \right) \right) \\
&=& \underset{n \rightarrow \infty}{\text{lim}} \left( \frac{7}{9} \cdot \left( \frac{1- \left(-\frac{7}{9} \right)^{n+1}}{1 - \left(-\frac{7}{9} \right)} - 1 + \frac{7}{9} \right) \right) \\
&=& \underset{n \rightarrow \infty}{\text{lim}} \left( \frac{7}{9} \cdot \left( \frac{1- \left(-\frac{7}{9} \right)^{n+1}}{1 + \frac{7}{9}} - 1 + \frac{7}{9} \right) \right)\\
&\rightarrow & \frac{7}{9} \cdot \left( \frac{1}{1 + \frac{7}{9}} - 1 + \frac{7}{9} \right) \\
&=& \frac{7}{9} \cdot \left( \frac{9}{16} - 1 + \frac{7}{9} \right) \\
&=& \frac{7}{9} \cdot \left( -\frac{7}{16} + \frac{7}{9} \right) \\
&=& \frac{7}{9} \cdot \frac{49}{144} \\
&=& \frac{343}{1296} \approx 0.26
\end{alignat*}\\
Der Grenzwert beträgt $\frac{343}{1296}$.
\subsubsection{} %(iv)
\begin{alignat*}{3}
\sum\limits_{i=2}^{\infty} \frac{1}{(i+1)i} &=& \sum\limits_{i=2}^{\infty} \left( \frac{1}{i} - \frac{1}{i+1} \right) \\
&=& \sum\limits_{i=2}^{\infty} \frac{1}{i} - \sum\limits_{i=2}^{\infty} \frac{1}{i+1} \\
&=& \frac{1}{2} + \frac{1}{3} + \frac{1}{4} + ... + \frac{1}{n} - \frac{1}{3} - \frac{1}{4} - ... - \frac{1}{n+1} \\
&\Rightarrow & \underset{n \rightarrow \infty}{\text{lim}} \left(\frac{1}{2} - \frac{1}{n+1} \right) = \frac{1}{2}
\end{alignat*}\\
Der Grenzwert ist $\frac{1}{2}$.
\section{} %4
\subsection{}
\subsubsection{} %(i)
\begin{alignat*}{2}
\underset{n \rightarrow \infty}{\text{lim}} \left(1 + \frac{1}{n} \right)^{n+3} &=& \underset{n \rightarrow \infty}{\text{lim}} \left(1 + \frac{1}{n} \right)^{n} \cdot \underset{n \rightarrow \infty}{\text{lim}} \left(1 + \frac{1}{n} \right)^{3} \\
&=& e \cdot 1 = e
\end{alignat*}\\
\subsubsection{} %(ii)
\begin{alignat*}{2}
\underset{n \rightarrow \infty}{\text{lim}} \left(1 + \frac{1}{n} \right)^{3n} &=& \underset{n \rightarrow \infty}{\text{lim}} \left(1 + \frac{1}{n} \right)^{n} \cdot \underset{n \rightarrow \infty}{\text{lim}} \left(1 + \frac{1}{n} \right)^{n} \cdot \underset{n \rightarrow \infty}{\text{lim}} \left(1 + \frac{1}{n} \right)^{n} \\
&=& e \cdot e \cdot e \\
&=& e^{3}
\end{alignat*}\\
\subsubsection{} %(iii)
\begin{alignat*}{2}
\underset{n \rightarrow \infty}{\text{lim}} \left(1 + \frac{1}{n} \right)^{3} &=& \underset{n \rightarrow \infty}{\text{lim}} \left(1 + \frac{1}{n} \right) \cdot \underset{n \rightarrow \infty}{\text{lim}} \left(1 + \frac{1}{n} \right) \cdot \underset{n \rightarrow \infty}{\text{lim}} \left(1 + \frac{1}{n} \right) \\
&=& 1 \cdot 1 \cdot 1 = 1
\end{alignat*}
\subsubsection{} %(iv)
\begin{alignat*}{2}
\underset{n \rightarrow \infty}{\text{lim}} \left(1 + \frac{1}{3n} \right)^{3n} &=& \underset{n \rightarrow \infty}{\text{lim}} \left( \left(1 + \frac{1}{3n} \right)^{3} \right)^{n} \\
&=& \underset{n \rightarrow \infty}{\text{lim}} \left( \left(1 + \frac{1}{3n} \right)^{2} \cdot \left(1 + \frac{1}{3n} \right) \right)^{n} \\
&=& \underset{n \rightarrow \infty}{\text{lim}} \left( \left(1 + \frac{2}{3n} + \frac{1}{9n^{2}} \right) \cdot \left(1 + \frac{1}{3n} \right) \right)^{n} \\
&=& \underset{n \rightarrow \infty}{\text{lim}} \left( 1 + \frac{1}{3n} + \frac{2}{3n} + \frac{2}{9n^{2}} + \frac{1}{9n^{2}} + \frac{1}{27n^{3}} \right)^{n} \\
&=& \underset{n \rightarrow \infty}{\text{lim}} \left( 1 + \frac{3}{3n} + \frac{3}{9n^{2}} + \frac{1}{27n^{3}} \right)^{n} \\
&=& \underset{n \rightarrow \infty}{\text{lim}} \left( 1 + \frac{1}{n} + \frac{1}{3n^{2}} + \frac{1}{27n^{3}} \right)^{n} \\
&\rightarrow & 1
\end{alignat*}
\end{document}

View File

@ -0,0 +1,105 @@
\documentclass[10pt,a4paper,oneside,ngerman,numbers=noenddot]{scrartcl}
\usepackage[T1]{fontenc}
\usepackage[utf8]{inputenc}
\usepackage[ngerman]{babel}
\usepackage{amsmath}
\usepackage{amsfonts}
\usepackage{amssymb}
\usepackage{paralist}
\usepackage{gauss}
\usepackage[locale=DE,exponent-product=\cdot,detect-all]{siunitx}
\usepackage{tikz}
\usetikzlibrary{matrix,fadings,calc,positioning,decorations.pathreplacing,arrows,decorations.markings}
\usepackage{polynom}
\polyset{style=C, div=:,vars=x}
\pagenumbering{arabic}
\def\thesection{\arabic{section})}
\def\thesubsection{\alph{subsection})}
\def\thesubsubsection{(\roman{subsubsection})}
\makeatletter
\renewcommand*\env@matrix[1][*\c@MaxMatrixCols c]{%
\hskip -\arraycolsep
\let\@ifnextchar\new@ifnextchar
\array{#1}}
\makeatother
\begin{document}
\author{Jim Martens (6420323)}
\title{Hausaufgaben zum 25. April}
\maketitle
\section{} %1
\subsection{} %a
\vspace{11cm}
Die Unstetigkeitsstellen sind $x=2$ und $x=6$.
\subsection{} %b
\vspace{4cm}
Die Funktion $g(x)$ ist periodisch. Betrachtet man die Periode für $x=0$ bis $x=1$, so ist ersichtlich, dass sowohl $x=0$ als auch $x=1$ Unstetigkeitsstellen sind. Nähert man sich an die beiden Stellen von links an, so stimmt der Grenzwert nicht mit dem Funktionswert überein. Nähert man sich von rechts an, stimmt er überein.\\
Stetigkeit erfordert jedoch, dass der Grenzwert existiert und gleich dem Funktionswert ist, unabhängig von der Folge mit der man sich annähert.
Daher ist $g(x)$ in diesen beiden Stellen unstetig. Aufgrund der Periodizität der Funktion ist $g(x)$ an allen Stellen $x \in \mathbb{Z}$ unstetig.\\
\\
Betrachtet man hingegen eine andere Stelle in der Periode, so stimmen Grenzwert und Funktionswert überein, unabhängig davon ob man sich von rechts oder links annähert. Daher ist $g(x)$ in allen Stellen der Periode mit $x \neq 0$ und $x \neq 1$ stetig. Aufgrund der Periodizität der Funktion ist $g(x)$ an allen Stellen $x \not\in \mathbb{Z}$ stetig.
\section{} %2
\subsection{} %a
\begin{alignat*}{2}
\underset{n \rightarrow \infty}{\text{lim}} a_{n} &=& \underset{n \rightarrow \infty}{\text{lim}} \frac{\sqrt{3n^{2}-2n+5}-\sqrt{n}}{\sqrt{n^{2}-n+1}+4n} \\
\intertext{Ausklammern von $n^{2}$ unterhalb der Wurzeln}
&=& \underset{n \rightarrow \infty}{\text{lim}} \frac{\sqrt{n^{2} (3 - \frac{2}{n} + \frac{5}{n^{2}})} - \sqrt{n^{2} \cdot \frac{1}{n}}}{\sqrt{n^{2} (1 -\frac{1}{n} + \frac{1}{n^{2}})} + 4n} \\
\intertext{Wurzelgesetze anwenden}
&=& \underset{n \rightarrow \infty}{\text{lim}} \frac{\sqrt{n^{2}} \cdot \sqrt{3 - \frac{2}{n} + \frac{5}{n^{2}}} - \sqrt{n^{2}} \cdot \sqrt{\frac{1}{n}}}{\sqrt{n^{2}} \cdot \sqrt{1 -\frac{1}{n} + \frac{1}{n^{2}}} + 4n} \\
\intertext{Wurzel auflösen}
&=& \underset{n \rightarrow \infty}{\text{lim}} \frac{n \cdot \sqrt{3 - \frac{2}{n} + \frac{5}{n^{2}}} - n \cdot \sqrt{\frac{1}{n}}}{n \cdot \sqrt{1 -\frac{1}{n} + \frac{1}{n^{2}}} + 4n} \\
\intertext{$n$ ausklammern}
&=& \underset{n \rightarrow \infty}{\text{lim}} \frac{n \cdot (\sqrt{3 - \frac{2}{n} + \frac{5}{n^{2}}} - \sqrt{\frac{1}{n}})}{n \cdot (\sqrt{1 -\frac{1}{n} + \frac{1}{n^{2}}} + 4)} \\
\intertext{$n$ kürzen}
&=& \underset{n \rightarrow \infty}{\text{lim}} \frac{\sqrt{3 - \frac{2}{n} + \frac{5}{n^{2}}} - \sqrt{\frac{1}{n}}}{\sqrt{1 -\frac{1}{n} + \frac{1}{n^{2}}} + 4} \\
\intertext{lim mit Wurzelfunktion vertauschen, da Wurzelfunktion stetig}
&=& \frac{\sqrt{\underset{n \rightarrow \infty}{\text{lim}} (3 - \frac{2}{n} + \frac{5}{n^{2}})} - \sqrt{\underset{n \rightarrow \infty}{\text{lim}}(\frac{1}{n})}}{\sqrt{\underset{n \rightarrow \infty}{\text{lim}} (1 -\frac{1}{n} + \frac{1}{n^{2}})} + 4} \\
\intertext{limes anwenden und Nullfolgen entfernen}
&=& \frac{\sqrt{3} - \sqrt{0}}{\sqrt{1} + 4} \\
\intertext{Zusammenfassen}
&=& \frac{\sqrt{3}}{5}
\end{alignat*}
\subsection{} %b
\begin{alignat*}{2}
&& \underset{n \rightarrow \infty}{\text{lim}} \left( \text{cos} \left( \frac{\sqrt{10n^{2}-n}-n}{2n+3} \right) \right) \\
\intertext{Ausklammern von $n^{2}$ unter der Wurzel}
&=& \underset{n \rightarrow \infty}{\text{lim}} \left( \text{cos} \left( \frac{\sqrt{n^{2} (10-\frac{1}{n})}-n}{2n+3} \right) \right) \\
\intertext{Wurzelgesetze anwenden}
&=& \underset{n \rightarrow \infty}{\text{lim}} \left( \text{cos} \left( \frac{\sqrt{n^{2}} \cdot \sqrt{10-\frac{1}{n}}-n}{2n+3} \right) \right) \\
\intertext{Wurzel auflösen}
&=& \underset{n \rightarrow \infty}{\text{lim}} \left( \text{cos} \left( \frac{n \cdot \sqrt{10-\frac{1}{n}}-n}{2n+3} \right) \right) \\
\intertext{Ausklammern von $n$ in Zähler und Nenner}
&=& \underset{n \rightarrow \infty}{\text{lim}} \left( \text{cos} \left( \frac{n \cdot (\sqrt{10 - \frac{1}{n}} - 1)}{n \cdot (2 + \frac{3}{n})} \right) \right) \\
\intertext{Kürzen von $n$}
&=& \underset{n \rightarrow \infty}{\text{lim}} \left( \text{cos} \left( \frac{\sqrt{10 - \frac{1}{n}} - 1}{2 + \frac{3}{n}} \right) \right) \\
\intertext{cos mit lim vertauschen, da Cosinusfunktion stetig}
&=& \text{cos} \left( \underset{n \rightarrow \infty}{\text{lim}} \left( \frac{\sqrt{10 - \frac{1}{n}} - 1}{2 + \frac{3}{n}} \right) \right) \\
\intertext{lim in Wurzel ziehen, da Wurzelfunktion stetig}
&=& \text{cos} \left( \frac{\sqrt{\underset{n \rightarrow \infty}{\text{lim}} (10 - \frac{1}{n})} - \underset{n \rightarrow \infty}{\text{lim}} (1) }{\underset{n \rightarrow \infty}{\text{lim}} (2 + \frac{3}{n})} \right) \\
&=& \text{cos} \left( \frac{\sqrt{10} - 1}{2} \right)\\
&\approx & 0.47
\end{alignat*}
\section{} %3
$g \circ f$ kann auch so geschrieben werden $g(f(x))$. Vereinfacht gesagt, liefert $g$ den Funktionswert an der Stelle, die dem Funktionswert von $f$ an der Stelle $x$ entspricht.\\
Das berücksichtigend wissen wir, dass $f$ an der Stelle $x_{0}$ stetig ist. Der Funktionswert für diese Stelle ist $f(x_{0}) = y_{0}$. Wir wissen ferner, dass $g$ an der Stelle $y_{0}$ stetig ist.\\
\\
Da der Funktionswert von $f$ an der Stelle $x_{0}$ der Stelle entspricht, an der $g$ bekanntermaßen stetig ist, werden hier zwei stetige Funktionen nacheinander ausgeführt. Und die Nacheinanderausführung von zwei stetigen Funktionen ist selbst wiederum stetig.\\
Es ist somit ersichtlich, dass $g(f(x_{0}))$ den soeben beschrieben Fall darstellt und damit klarstellt, dass $g \circ f$ ebenfalls an der Stelle $x_{0}$ stetig ist.
\section{} %4
$g(x)$:\\
\begin{alignat*}{2}
\underset{x \rightarrow 0}{\text{lim}} g(x) &\Rightarrow & \underset{x \rightarrow 0}{\text{lim}} (x^{2} \cdot 1 ) \geq \underset{x \rightarrow 0}{\text{lim}} \left(x^{2} \cdot \text{sin} \left(\frac{1}{x}\right) \right) \geq \underset{x \rightarrow 0}{\text{lim}} (x^{2} \cdot -1) \\
&\Rightarrow & 0 \geq \underset{x \rightarrow 0}{\text{lim}} \left(x^{2} \cdot \text{sin} \left(\frac{1}{x} \right) \right) \geq 0
\end{alignat*}
Daraus folgt, dass $g(x)$ für alle $x \in \mathbb{R}$ stetig ist. \\
$h(x)$:\\
\begin{alignat*}{2}
\underset{n \rightarrow \infty}{\text{lim}} h(x_{n}) &=& \underset{n \rightarrow \infty}{\text{lim}} \text{sin} \left( \frac{1}{x_{n}} \right) \\
\intertext{$x_{n}$ sei $\frac{1}{2\pi n}, n \in \mathbb{N}$}
&=& \underset{n \rightarrow \infty}{\text{lim}} \text{sin} \left(\frac{1}{\frac{1}{2 \pi n}}\right) \\
&=& \underset{n \rightarrow \infty}{\text{lim}} \text{sin} (2 \pi n) \\
&=& \text{sin} \left( \underset{n \rightarrow \infty}{\text{lim}} (2 \pi n) \right) = 0
\end{alignat*}
$h(x)$ ist stetig für alle $x = x_{n} = \frac{1}{2\pi n}$. Die Funktion ist nicht stetig für andere $x$.
\end{document}

View File

@ -0,0 +1,284 @@
\documentclass[10pt,a4paper,oneside,ngerman,numbers=noenddot]{scrartcl}
\usepackage[T1]{fontenc}
\usepackage[utf8]{inputenc}
\usepackage[ngerman]{babel}
\usepackage{amsmath}
\usepackage{amsfonts}
\usepackage{amssymb}
\usepackage{paralist}
\usepackage{gauss}
\usepackage{pgfplots}
\usepackage[locale=DE,exponent-product=\cdot,detect-all]{siunitx}
\usepackage{tikz}
\usetikzlibrary{matrix,fadings,calc,positioning,decorations.pathreplacing,arrows,decorations.markings}
\usepackage{polynom}
\polyset{style=C, div=:,vars=x}
\pagenumbering{arabic}
\def\thesection{\arabic{section})}
\def\thesubsection{\alph{subsection})}
\def\thesubsubsection{(\roman{subsubsection})}
\makeatletter
\renewcommand*\env@matrix[1][*\c@MaxMatrixCols c]{%
\hskip -\arraycolsep
\let\@ifnextchar\new@ifnextchar
\array{#1}}
\makeatother
\begin{document}
\author{Jim Martens (6420323)}
\title{Hausaufgaben zum 2. Mai}
\maketitle
\section{} %1
\subsection{} %a
\subsubsection{} %i
\begin{alignat*}{2}
f(x) &=& 7x^{5} + 3x^{3} + x + 1 \\
f'(x) &=& 35x^{4} + 9x^{2} + 1
\end{alignat*}
\subsubsection{} %ii
\begin{alignat*}{2}
f(x) &=& (3x^{7} - 4x^{3} + x^{2} - 3x + 1)^{8} \\
f'(x) &=& 8 \cdot (3x^{7} - 4x^{3} + x^{2} - 3x + 1)^{7} \cdot (21x^{6} - 12x^{2} + 2x - 3)
\end{alignat*}
\subsubsection{} %iii
\begin{alignat*}{2}
f(x) &=& (3x^{4} + 2x) \cdot \sqrt{x^{2} + 1} \\
f'(x) &=& (12x^{3} + 2) \cdot \sqrt{x^{2} + 1} + (3x^{4} + 2x) \cdot \frac{2x}{2 \cdot \sqrt{x^{2} + 1}}
\end{alignat*}
\subsubsection{} %iv
\begin{alignat*}{2}
f(x) &=& (x^{3} + 1) \cdot \ln (x^{4} + 3x^{2} + 1) \\
f'(x) &=& 3x \cdot \ln (x^{4} + 3x^{2} + 1) + (x^{3} + 1) \cdot \frac{4x^{3} + 6x}{x^{4} + 3x^{2} + 1}
\end{alignat*}
\subsubsection{} %v
\begin{alignat*}{2}
f(x) &=& e^{x^{3} + x^{2} + 1} \cdot \sqrt{x} \\
f'(x) &=& e^{x^{3} + x^{2} + 1} \cdot (3x^{2} + 2x) \cdot \sqrt{x} + e^{x^{3} + x^{2} + 1} \cdot \frac{1}{2 \cdot \sqrt{x}}
\end{alignat*}
\subsubsection{} %vi
\begin{alignat*}{2}
f(x) &=& \sqrt{x^{4} + 1} \cdot \ln x \\
f'(x) &=& \frac{4x^{3}}{2 \cdot \sqrt{x^{4} + 1}} \cdot \ln x + \sqrt{x^{4} + 1} \cdot \frac{1}{x}
\end{alignat*}
\subsection{} %b
\begin{alignat*}{2}
q(x) &=& \frac{5x^{2} + 1}{x - 3} \\
q'(x) &=& \frac{10x \cdot (x-3) - (5x^{2} + 1) \cdot 1}{(x-3)^{2}} \\
\intertext{Ausmultiplizieren}
&=& \frac{10x^{2} - 30x - (5x^{2} + 1)}{x^{2} - 6x + 9}\\
\intertext{Zusammenfassen}
&=& \frac{5x^{2} - 30x - 1}{x^{2} - 6x + 9}\\
q''(x) &=& \frac{(10x-30) \cdot (x^{2} - 6x + 9) - (5x^{2} - 30x - 1) \cdot (2x - 6)}{(x^{2} - 6x + 9)^{2}} \\
\intertext{Ausmultiplizieren}
&=& \frac{10x^{3} - 60x^{2} + 90x - 30x^{2} + 180x - 270 - (10x^{3} - 30x^{2} - 60x^{2} + 180x -x^{2} - 9)}{x^{4} - 6x^{3} + 9x^{2} - 6x^{3} + 36x^{2} - 54x + 9x^{2} - 54x + 81} \\
\intertext{Zusammenfassen}
&=& \frac{10x^{3} - 90x^{2} + 270x - 270 -10x^{3} + 91x^{2} - 180x + 9}{x^{4} - 12x^{3} + 54x^{2} - 108x + 81} \\
\intertext{Zusammenfassen}
&=& \frac{x^{2} + 90x - 261}{x^{4} - 12x^{3} + 54x^{2} - 108x + 81} \\
q'''(x) &=& \frac{(2x + 90) \cdot (x^{4} - 12x^{3} + 54x^{2} - 108x + 81) - (x^{2} + 90x - 261) \cdot (4x^{3} - 36x^{2} + 108x - 108)}{(x^{4} - 12x^{3} + 54x^{2} - 108x + 81)^{2}} \\
\intertext{Ausmultiplizieren und Zusammenfassen}
&=& \frac{-2x^{5} - 258x^{4} + 3204x^{3} - 14364x^{2} +28350x - 20898}{x^{8} - 24x^{7} + 252x^{6} -1404x^{5} +5670x^{4} - 13716x^{3} + 20412x^{2} -8748x - 2187}
\end{alignat*}
\section{} %2
\begin{alignat*}{2}
f(x) &=& \left| 3 - \frac{1}{2}x \right| \\
\intertext{Sei $x_{n} = 6 + \frac{1}{n} : n \in \mathbb{N}$}
\underset{n \rightarrow \infty}{\text{lim}} \frac{f(x_{n}) - f(x_{0})}{x_{n} - x_{0}} &=& \underset{n \rightarrow \infty}{\text{lim}} \frac{\left| 3 - \frac{1}{2} \cdot (6 + \frac{1}{n}) \right| - \left| 3 - \frac{1}{2} \cdot 6 \right|}{6 + \frac{1}{n} - 6} \\
&=& \underset{n \rightarrow \infty}{\text{lim}} \frac{\left| 3 - 3 - \frac{1}{2n} \right| - \left| 3 - 3 \right|}{\frac{1}{n}} \\
&=& \underset{n \rightarrow \infty}{\text{lim}} \frac{\left|-\frac{1}{2n} \right|}{\frac{1}{n}} \\
&=& \underset{n \rightarrow \infty}{\text{lim}} \frac{\frac{1}{2n}}{\frac{1}{n}} \\
&=& \underset{n \rightarrow \infty}{\text{lim}} \frac{n}{2n} = \frac{1}{2} \\
\intertext{Sei $x_{n} = 6 - \frac{1}{n} : n \in \mathbb{N}$}
\underset{n \rightarrow \infty}{\text{lim}} \frac{f(x_{n}) - f(x_{0})}{x_{n} - x_{0}} &=& \underset{n \rightarrow \infty}{\text{lim}} \frac{\left| 3 - \frac{1}{2} \cdot (6 - \frac{1}{n}) \right| - \left| 3 - \frac{1}{2} \cdot 6 \right|}{6 - \frac{1}{n} - 6} \\
&=& \underset{n \rightarrow \infty}{\text{lim}} \frac{\left| 3 - 3 + \frac{1}{2n} \right| - \left| 3 - 3 \right|}{-\frac{1}{n}} \\
&=& \underset{n \rightarrow \infty}{\text{lim}} \frac{\left|\frac{1}{2n} \right|}{-\frac{1}{n}} \\
&=& \underset{n \rightarrow \infty}{\text{lim}} \frac{\frac{1}{2n}}{-\frac{1}{n}} \\
&=& \underset{n \rightarrow \infty}{\text{lim}} -\frac{n}{2n} = -\frac{1}{2}
\end{alignat*}
Der Grenzwert existiert an der Stelle $x _{0} = 6$ nicht. Daher ist die Funktion $f$ an der Stelle $x_{0} = 6$ nicht differenzierbar.
\begin{tikzpicture}[>=stealth]
\begin{axis}[
ymin=-10,ymax=10,
x=1em,
y=1em,
axis x line=middle,
axis y line=middle,
axis line style=->,
xlabel={$x$},
ylabel={$y$},
]
\addplot[no marks, black, -] expression[domain=-10:6,samples=100]{3 - (1/2)*x} node[pos=0.65,anchor=north]{};
\addplot[no marks, black, -] expression[domain=6:10,samples=100]{-1*(3 - (1/2)*x)} node[pos=0.65,anchor=north]{};
\draw (16em, 10em) circle (2pt);
\end{axis}
\end{tikzpicture}
\section{} %3
\subsection{} %a
\begin{alignat*}{2}
f(x) &=& (x^{4}+1)^{x+2} \\
&=& e^{ln\left((x^{4}+1)^{x+2}\right)} \\
&=& e^{(x+2) \cdot \ln(x^{4}+1)} \\
f'(x) &=& e^{(x+2) \cdot \ln(x^{4}+1)} \cdot \left((x+2) \cdot \ln(x^{4}+1)\right)' \\
&=& e^{(x+2) \cdot \ln(x^{4}+1)} \cdot \left((x+2)' \cdot \ln(x^{4}+1) + (x+2) \cdot \ln(x^{4}+1)'\right) \\
&=& e^{(x+2) \cdot \ln(x^{4}+1)} \cdot \left(1 \cdot \ln(x^{4}+1) + (x+2) \cdot \frac{1}{x^{4}+1} \cdot (x^{4}+1)'\right) \\
&=& (x^{4}+1)^{x+2} \cdot \left(\ln(x^{4}+1) + (x+2) \cdot \frac{4x^{3}}{x^{4}+1}\right)
\end{alignat*}
\subsection{} %b
\begin{alignat*}{2}
f(x) &=& x^{\frac{1}{2}} \\
&=& e^{\ln \left(x^{\frac{1}{2}} \right)} \\
&=& e^{\frac{1}{2} \cdot \ln (x)} \\
f'(x) &=& e^{\frac{1}{2} \cdot \ln (x)} \cdot \left(\frac{1}{2} \cdot \ln (x) \right)' \\
&=& e^{\frac{1}{2} \cdot \ln (x)} \cdot \frac{1}{2} \cdot \frac{1}{x} \cdot (x)' \\
&=& e^{\frac{1}{2} \cdot \ln (x)} \cdot \frac{1}{2x} \\
&=& x^{\frac{1}{2}} \cdot \frac{1}{2x} \\
&=& \frac{1}{2} \cdot \frac{x^{\frac{1}{2}}}{x} \\
&=& \frac{1}{2} \cdot x^{-\frac{1}{2}} \\
g(x) &=& \left( \frac{1}{2}\right)^{x} \\
&=& e^{\ln \left( \left( \frac{1}{2}\right)^{x} \right)} \\
&=& e^{x \cdot \ln \left( \frac{1}{2}\right)} \\
g'(x) &=& e^{x \cdot \ln \left( \frac{1}{2}\right)} \cdot \left(x \cdot \ln \left( \frac{1}{2}\right) \right)' \\
&=& e^{x \cdot \ln \left( \frac{1}{2}\right)} \cdot (x)' \cdot \ln \left( \frac{1}{2}\right) \\
&=& e^{x \cdot \ln \left( \frac{1}{2}\right)} \cdot 1 \cdot \ln \left( \frac{1}{2}\right) \\
&=& \left( \frac{1}{2}\right)^{x} \cdot \ln \left( \frac{1}{2}\right)
\end{alignat*}
\subsection{} %c
\subsubsection{} %i
\begin{alignat*}{2}
g(x) &=& (x^{2}+1)^{4x+1} \\
&=& e^{\ln \left( (x^{2}+1)^{4x+1}\right)} \\
&=& e^{(4x+1) \cdot \ln (x^{2}+1)} \\
g'(x) &=& e^{(4x+1) \cdot \ln (x^{2}+1)} \cdot \left((4x+1) \cdot \ln (x^{2}+1) \right)' \\
&=& e^{(4x+1) \cdot \ln (x^{2}+1)} \cdot \left((4x+1)' \cdot \ln (x^{2}+1) + (4x+1) \cdot \ln (x^{2}+1)' \right) \\
&=& e^{(4x+1) \cdot \ln (x^{2}+1)} \cdot \left(4 \cdot \ln (x^{2}+1) + (4x+1) \cdot \frac{1}{x^{2}+1} \cdot (x^{2}+1)' \right) \\
&=& e^{(4x+1) \cdot \ln (x^{2}+1)} \cdot \left(4 \cdot \ln (x^{2}+1) + (4x+1) \cdot \frac{2x}{x^{2}+1} \right) \\
&=& (x^{2}+1)^{4x+1} \cdot \left(4 \cdot \ln (x^{2}+1) + \frac{8x^{2} + 2x}{x^{2}+1} \right)
\end{alignat*}
\subsubsection{} %ii
\begin{alignat*}{2}
h(x) = (x-3)^{3x^{4}+5} \\
&=& e^{\ln \left((x-3)^{3x^{4}+5} \right)} \\
&=& e^{(3x^{4}+5) \cdot \ln (x-3)} \\
h'(x) &=& e^{(3x^{4}+5) \cdot \ln (x-3)} \cdot \left((3x^{4}+5) \cdot \ln (x-3) \right)' \\
&=& e^{(3x^{4}+5) \cdot \ln (x-3)} \cdot \left((3x^{4}+5)' \cdot \ln (x-3) + (3x^{4}+5) \cdot \ln (x-3)' \right) \\
&=& e^{(3x^{4}+5) \cdot \ln (x-3)} \cdot \left(12x^{3} \cdot \ln (x-3) + (3x^{4}+5) \cdot \frac{1}{x-3} \cdot (x-3)' \right) \\
&=& e^{(3x^{4}+5) \cdot \ln (x-3)} \cdot \left(12x^{3} \cdot \ln (x-3) + (3x^{4}+5) \cdot \frac{1}{x-3} \right) \\
&=& (x-3)^{3x^{4}+5} \cdot \left(12x^{3} \cdot \ln (x-3) + \frac{3x^{4}+5}{x-3} \right)
\end{alignat*}
\section{} %4
\subsection{} %a
\begin{alignat*}{2}
g(p) &=& 10^{5} \cdot \left( \frac{1}{p} - \frac{3}{p^{2}} \right) \\
&=& 10^{5} \cdot \left( p^{-1} - 3p^{-2} \right) \\
g'(p) &=& 10^{5} \cdot \left( p^{-1} - 3p^{-2} \right)' \\
&=& 10^{5} \cdot \left((p^{-1})' - (3p^{-2})' \right) \\
&=& 10^{5} \cdot \left(-p^{-2} - (-6p^{-3}) \right) \\
&=& 10^{5} \cdot \left(-p^{-2} + 6p^{-3} \right) \\
&=& 10^{5} \cdot \left(-\frac{1}{p^{2}} + \frac{6}{p^{3}} \right) \\
g''(p) &=& 10^{5} \cdot \left(-p^{-2} + 6p^{-3} \right)' \\
&=& 10^{5} \cdot \left((-p^{-2})' + (6p^{-3})' \right) \\
&=& 10^{5} \cdot \left(2p^{-3} - 18p^{-4} \right) \\
&=& 10^{5} \cdot \left(\frac{2}{p^{3}} - \frac{18}{p^{4}} \right)
\end{alignat*}
\begin{enumerate}
\item Bestimmung der Nullstellen der 1. Ableitung:
\begin{alignat*}{2}
-\frac{1}{p^{2}} + \frac{6}{p^{3}} &=& 0 \\
-\frac{1}{p^{2}} &=& - \frac{6}{p^{3}} \\
\frac{1}{p^{2}} &=& \frac{6}{p^{3}} \\
\frac{p^{3}}{p^{2}} &=& 6 \\
p &=& 6 \\
\intertext{Einsetzen in $g'(p)$}
0 &=& 10^{5} \cdot \left(-\frac{1}{6^{2}} + \frac{6}{6^{3}} \right) \\
&=& 10^{5} \cdot \left(-\frac{1}{36} + \frac{6}{216} \right) \\
&=& 10^{5} \cdot \left(-\frac{1}{36} + \frac{1}{36} \right) \\
&=& 10^{5} \cdot 0 = 0
\end{alignat*} \\
\item Einsetzen von $p=6$ in $g''(p)$:
\begin{alignat*}{2}
g''(6) &=& 10^{5} \cdot \left(\frac{2}{6^{3}} - \frac{18}{6^{4}} \right) \\
&=& 10^{5} \cdot \left(\frac{2}{216} - \frac{18}{1296} \right) \\
&=& 10^{5} \cdot \left(\frac{2}{216} - \frac{3}{216} \right) \\
&=& 10^{5} \cdot \left(- \frac{1}{216} \right) = -\frac{10^{5}}{216}
\end{alignat*}
Daraus ergibt sich, dass $p=6$ ein lokales Maximum der Funktion $g(p)$ ist. Es bleibt noch festzustellen, dass es auch das globale Maximum ist. \\
\item Feststellung des globalen Maximums:\\
Da es nur eine Nullstelle für die erste Ableitung gibt, kann es insofern nur einen Extrempunkt geben. Da der Funktionswert der 2. Ableitung für diese Stelle negativ ist, liegt an der Stelle ein Maximum. Der Funktionswert von $g(p)$ für die Stelle wird wie folgt berechnet:
\begin{alignat*}{2}
g(p) &=& 10^{5} \cdot \left(\frac{1}{p} - \frac{3}{p^{2}} \right) \\
g(6) &=& 10^{5} \cdot \left( \frac{1}{6} - \frac{3}{3^{2}} \right) \\
&=& 10^{5} \cdot \left( \frac{1}{6} - \frac{3}{9} \right) \\
&=& 10^{5} \cdot \left( \frac{1}{6} - \frac{2}{6} \right) \\
&=& 10^{5} \cdot \left(- \frac{1}{6} \right) \\
&=& -\frac{10^{5}}{6} = \frac{5000}{3} \\
&=& \frac{4998}{3} + \frac{2}{3} \\
&=& 1666 + \frac{2}{3}
\end{alignat*} \\\\\\
Zuletzt muss noch geprüft werden, ob die Grenzen des Definitionsbereiches einen höheren Wert aufweisen.
\begin{alignat*}{2}
g(p) &=& 10^{5} \cdot \left(\frac{1}{p} - \frac{3}{p^{2}} \right) \\
g(3) &=& 10^{5} \cdot \left(\frac{1}{3} - \frac{3}{3^{2}} \right) \\
&=& 10^{5} \cdot \left(\frac{1}{3} - \frac{3}{9} \right) \\
&=& 10^{5} \cdot \left(\frac{1}{3} - \frac{1}{3} \right) \\
&=& 10^{5} \cdot 0 = 0 \\
g(100) &=& 10^{5} \cdot \left(\frac{1}{100} - \frac{3}{100^{2}} \right) \\
&=& 10^{5} \cdot \left(\frac{1}{100} - \frac{3}{10000} \right) \\
&=& 10^{5} \cdot \left(\frac{100}{10000} - \frac{3}{10000} \right) \\
&=& 10^{5} \cdot \frac{97}{10000} \\
&=& 970
\end{alignat*}
Die Funktion hat nur einen Extrempunkt und die beiden Definitionsgrenzen weisen einen niedrigeren Funktionswert auf, als der zuvor bestimmte lokale Extrempunkt mit $p = 6$. Somit ist der einzige lokale Extrempunkt auch der globale Extrempunkt. Da es sich bei dem Extrempunkt um ein Maximum handelt, ist er somit das globale Maximum.
\end{enumerate}
\subsection{} %b
\subsubsection{} %i
Bestimmung der Nullstellen von $f, f'$ und $f''$:\\
\begin{alignat*}{2}
f(x) &=& -2x^{3} - x + 25 \\
f'(x) &=& -6x^{2} - 1 \\
f''(x) &=& -18x \\
f'''(x) &=& -18 \\
f(2) &=& -2 \cdot 2^{3} - 2 + 25 \\
&=& -2 \cdot 8 +23 \\
&=& -16 + 23 = 7 \\
f(3) &=& -2 \cdot 3^{3} - 3 + 25 \\
&=& -2 \cdot 27 + 22 \\
&=& -54 + 22 \\
&=& -32 \\
f'(0) &=& -6 \cdot 0^{2} - 1 \\
&=& 0 -1 = -1 \\
f''(0) &=& -18 \cdot 0 = 0
\end{alignat*}
Die Nullstelle von $f(x)$ liegt zwischen $x=2$ und $x=3$. $f'(x)$ hat keine Nullstelle. Der höchste erreichbare Wert ist $-1$.
$f''(x)$ hat eine Nullstelle, welche nachweislich bei $x=0$ liegt.
$f'''(x)$ hat offensichtlich keine Nullstelle.\\
\\
$f(x) > 0$ für $x \leq 2$ und $f(x) < 0$ für $x \geq 3$. $f'(x) < 0$ für alle $x \in \mathbb{R} \wedge x \geq -5 \wedge x \leq 5$. $f''(x) < 0$ für $x < 0$ und $f''(x) > 0$ für $x > 0$. $f'''(x) > 0$ für alle $x \in \mathbb{R} \wedge x \geq -5 \wedge x \leq 5$. \\
Es gibt keine Maxima oder Minima. Allerdings gibt es einen Wendepunkt bei $x=0$, da $f''(0) = 0$ und $f'''(0) \neq 0$. \\
Da es keine Nullstellen der ersten Ableitung gibt und $f(2) > f(3)$ gilt, ist $f(x)$ auf dem gesamten Intervall streng monoton fallend. Der höchste Punkt ist demnach am Beginn des Definitionsbereiches, der niedrigste am Ende.
Daraus ergibt sich:\\
\begin{alignat*}{2}
f(-5) &=& -2 \cdot (-5)^{3} - (-5) + 25 \\
&=& -2 \cdot (-125) + 30 \\
&=& 250 + 30 = 280 \\
f(5) &=& -2 \cdot 5^{3} - 5 + 25 \\
&=& -2 \cdot 125 + 20 \\
&=& -250 + 20 = -230
\end{alignat*}
$f(x)$ nimmt das globale Maximum bei $x=-5$ und das globale Minimum bei $x=5$ an.
\subsubsection{} %ii
\begin{alignat*}{2}
g(x) &=& x^{3} - 6x^{2} + 3x + 8 \\
g'(x) &=& 3x^{2} - 12x + 3 \\
g''(x) &=& 6x - 12 \\
g'''(x) &=& 6
\end{alignat*}
\subsubsection{} %iii
\begin{alignat*}{2}
h(x) &=& e^{2x-3} - e^{x+2} \\
h'(x) &=& e^{2x-3} \cdot 2 - e^{x+2} \\
h''(x) &=& e^{2x-3} \cdot 4 - e^{x+2} \\
h'''(x) &=& e^{2x-3} \cdot 8 - e^{x+2}
\end{alignat*}
\end{document}

View File

@ -0,0 +1,349 @@
\documentclass[10pt,a4paper,oneside,ngerman,numbers=noenddot]{scrartcl}
\usepackage[T1]{fontenc}
\usepackage[utf8]{inputenc}
\usepackage[ngerman]{babel}
\usepackage{amsmath}
\usepackage{amsfonts}
\usepackage{amssymb}
\usepackage{paralist}
\usepackage{gauss}
\usepackage{pgfplots}
\usepackage[locale=DE,exponent-product=\cdot,detect-all]{siunitx}
\usepackage{tikz}
\usetikzlibrary{matrix,fadings,calc,positioning,decorations.pathreplacing,arrows,decorations.markings}
\usepackage{polynom}
\polyset{style=C, div=:,vars=x}
\pagenumbering{arabic}
\def\thesection{\arabic{section})}
\def\thesubsection{\alph{subsection})}
\def\thesubsubsection{(\roman{subsubsection})}
\makeatletter
\renewcommand*\env@matrix[1][*\c@MaxMatrixCols c]{%
\hskip -\arraycolsep
\let\@ifnextchar\new@ifnextchar
\array{#1}}
\makeatother
\begin{document}
\author{Jim Martens (6420323)}
\title{Hausaufgaben zum 16. Mai}
\maketitle
\section{} %1
\subsubsection{} %i
\begin{alignat*}{2}
f(x) &=& \frac{1}{\sqrt[4]{x^{5}}} \cdot \sqrt[3]{\sqrt{x^{7}}} \\
\intertext{Wurzeln umformen}
&=& \frac{1}{(x^{5})^{\frac{1}{4}}} \cdot (\sqrt{x^{7}})^{\frac{1}{3}} \\
\intertext{Wurzeln umformen}
&=& \frac{1}{(x^{5})^{\frac{1}{4}}} \cdot ((x^{7})^{\frac{1}{2}})^{\frac{1}{3}} \\
\intertext{Zusammenfassen}
&=& \frac{1}{x^{\frac{5}{4}}} \cdot x^{\frac{7}{6}} \\
\intertext{Umformen}
&=& x^{-\frac{5}{4}} \cdot x^{\frac{7}{6}} \\
\intertext{Zusammenfassen}
&=& x^{\frac{7}{6} - \frac{5}{4}} \\
\intertext{Auf selben Nenner bringen}
\intertext{Zusammenfassen}
&=& x^{\frac{14 - 15}{12}} \\
&=& x^{-\frac{1}{12}} \\
f'(x) &=& -\frac{1}{12}x^{-\frac{13}{12}}
\end{alignat*}
\subsubsection{} %ii
\begin{alignat*}{2}
f(x) &=& \sin (x^{2}) \\
f'(x) &=& \cos (x^{2}) \cdot 2x
\end{alignat*}
\subsubsection{} %iii
\begin{alignat*}{2}
f(x) &=& \sin ^{2} x \\
&=& \sin x \cdot \sin x \\
f'(x) &=& \cos x \cdot \sin x + \sin x \cdot \cos x
\end{alignat*}
\subsubsection{} %iv
\begin{alignat*}{2}
f(x) &=& \sin x \cdot \cos x \\
f'(x) &=& \cos x \cdot \cos x + \sin x \cdot (- \sin x) \\
&=& \cos x \cdot \cos x - \sin x \cdot + \sin x)
\end{alignat*}
\subsubsection{} %v
\begin{alignat*}{2}
f(x) &=& \arcsin (\sqrt{x}) \\
f'(x) &=& \frac{1}{\sqrt{1 - \sqrt{x}}} \cdot (\sqrt{x})' \\
&=& \frac{1}{\sqrt{1 - \sqrt{x}}} \cdot \frac{1}{2 \sqrt{x}}
\end{alignat*}
\subsubsection{} %vi
\begin{alignat*}{2}
f(x) &=& (x^{3}-1)^{\arctan x} \\
&=& e^{\ln \left((x^{3}-1)^{\arctan x} \right)} \\
&=& e^{\left(\arctan x \right) \cdot \ln \left(x^{3}-1 \right)} \\
f'(x) &=& e^{\left(\arctan x \right) \cdot \ln \left(x^{3}-1 \right)} \cdot \left( \left(\arctan x \right) \cdot \ln \left(x^{3}-1 \right)\right)' \\
&=& e^{\left(\arctan x \right) \cdot \ln \left(x^{3}-1 \right)} \cdot \left( \left(\arctan x \right)' \cdot \ln \left(x^{3}-1 \right) + \left(\arctan x \right) \cdot \ln \left(x^{3}-1 \right)' \right) \\
&=& e^{\left(\arctan x \right) \cdot \ln \left(x^{3}-1 \right)} \cdot \left( \frac{1}{\sqrt{1 - x}} \cdot \ln \left(x^{3}-1 \right) + \left(\arctan x \right) \cdot \frac{1}{x^{3}-1} \cdot (x^{3}-1)' \right) \\
&=& e^{\left(\arctan x \right) \cdot \ln \left(x^{3}-1 \right)} \cdot \left( \frac{1}{\sqrt{1 - x}} \cdot \ln \left(x^{3}-1 \right) + \left(\arctan x \right) \cdot \frac{3x^{2}}{x^{3}-1}\right) \\
&=& (x^{3}-1)^{\arctan x} \cdot \left( \frac{1}{\sqrt{1 - x}} \cdot \ln \left(x^{3}-1 \right) + \left(\arctan x \right) \cdot \frac{3x^{2}}{x^{3}-1}\right)
\end{alignat*}
\section{} %2
\begin{alignat*}{2}
f(x) &=& \frac{2x}{1+x^{2}} \\
f'(x) &=& \frac{2 \cdot (1+x^{2}) - 2x \cdot 2x}{\left(1+x^{2} \right)^{2}} \\
&=& \frac{2 \cdot (1+x^{2}) - 4x^{2}}{\left(1+x^{2} \right)^{2}} \\
f''(x) &=& \frac{\left(2 \cdot (1+x^{2}) - 4x^{2} \right)' \cdot \left(1+x^{2} \right)^{2} - ((2 \cdot (1+x^{2}) - 4x^{2}) \cdot \left(\left(1+x^{2} \right)^{2} \right)'}{\left(1+x^{2} \right)^{4}} \\
&=& \frac{(4x-8x) \cdot \left(1+x^{2} \right)^{2} - (2 \cdot (1+x^{2}) - 4x^{2}) \cdot 2 \cdot (1+x^{2}) \cdot 2x}{\left(1+x^{2} \right)^{4}} \\
\intertext{$(1+x^{2})$ ausklammern und kürzen}
&=& \frac{-4x \cdot \left(1+x^{2} \right) - (2 \cdot (1+x^{2}) - 4x^{2}) \cdot 2 \cdot 2x}{\left(1+x^{2} \right)^{3}} \\
&=& \frac{4x \cdot \left( -\left(1+x^{2} \right) - (2 \cdot (1+x^{2}) - 4x^{2})\right)}{\left(1+x^{2} \right)^{3}} \\
&=& -\frac{4x \cdot \left(1+x^{2} + (2 \cdot (1+x^{2}) - 4x^{2}\right)}{\left(1+x^{2} \right)^{3}} \\
\intertext{Zusammenfassen}
&=& -\frac{4x \cdot \left(3-x^{2}\right)}{\left(1+x^{2} \right)^{3}}
\end{alignat*}
\begin{enumerate}
\item $x \in \mathbb{R}$
\item
\begin{alignat*}{2}
f(x) &=& 0 \\
\intertext{$f(x)$ mit Zähler ersetzen, da der über Nullstelle bestimmt}
2x &=& 0 \\
\intertext{geteilt durch $2$}
x &=& 0 \\
f'(x) &=& 0 \\
\intertext{$f'(x)$ mit Zähler ersetzen, da der über Nullstelle bestimmt}
2 - 2x^{2} &=& 0 \\
\intertext{$2$ subtrahieren}
-2x^{2} &=& -2 \\
\intertext{geteilt durch $-2$}
x^{2} &=& 1 \\
\intertext{Wurzel ziehen}
x_{1} &=& 1 \\
x_{2} &=& -1 \\
f''(x) &=& 0 \\
\intertext{$f''(x)$ mit Zähler ersetzen, da der über Nullstelle bestimmt}
-4x \cdot \left(3-x^{2}\right) &=& 0 \\
&\Rightarrow & x_{1} = 0, x_{2} = \sqrt{3}, x_{3} = - \sqrt{3}
\end{alignat*}
\item Es gibt keine Randpunkte des Definitionsbereiches.
\item
\begin{alignat*}{2}
f(-1) &=& \frac{2\cdot (-1)}{1 + (-1)^{2}} \\
&=& \frac{-2}{2} = -1 \\
f(1) &=& \frac{2 \cdot 1}{1 + 1^{2}} \\
&=& \frac{2}{2} = 1 \\
f'(-2) &=& \frac{2 - 2 \cdot (-2)^{2}}{\left(1+ (-2)^{2} \right)^{2}} \\
&=& \frac{2 - 2 \cdot 4}{\left(1+ 4 \right)^{2}} \\
&=& \frac{-6}{\left(5 \right)^{2}} \\
&=& \frac{-6}{25} \\
f'(0) &=& \frac{2 - 2 \cdot 0^{2}}{\left(1+ 0^{2} \right)^{2}} \\
&=& \frac{2}{1} = 2 \\
f'(2) &=& \frac{2 - 2 \cdot 2^{2}}{\left(1+ 2^{2} \right)^{2}} \\
&=& \frac{2 - 8}{\left(5 \right)^{2}} \\
&=& \frac{-6}{25}
\end{alignat*}\\
\begin{alignat*}{2}
f''(-2) &=& -\frac{4 \cdot (-2) \cdot \left(3- (-2)^{2}\right)}{\left(1+ (-2)^{2} \right)^{3}} \\
&=& -\frac{-8 \cdot \left(3- 4 \right)}{\left(1+ 4 \right)^{3}} \\
&=& -\frac{-8 \cdot -1}{5^{3}} \\
&=& -\frac{8}{125} \\
f''(-1) &=& -\frac{4 \cdot (-1) \cdot \left(3- (-1)^{2}\right)}{\left(1+ (-1)^{2} \right)^{3}} \\
&=& -\frac{-4 \cdot \left(3- 1\right)}{\left(1+ 1 \right)^{3}} \\
&=& -\frac{-4 \cdot 2}{2^{3}} \\
&=& -\frac{-8}{8} \\
&=& 1 \\
f''(1) &=& -\frac{4 \cdot 1 \cdot \left(3- 1^{2}\right)}{\left(1+ 1^{2} \right)^{3}} \\
&=& -\frac{4 \cdot 2}{2^{3}} \\
&=& -\frac{8}{8} \\
&=& -1 \\
f''(2) &=& -\frac{4 \cdot 2 \cdot \left(3- 2^{2}\right)}{\left(1+ 2^{2} \right)^{3}} \\
&=& -\frac{8 \cdot \left(3- 4\right)}{\left(1+ 4 \right)^{3}} \\
&=& -\frac{8 \cdot -1}{5^{3}} \\
&=& -\frac{-8}{125} \\
&=& \frac{8}{125}
\end{alignat*}\\
$f(x)$ ist negativ für $x < 0$ und positiv für $x > 0$. $f$ ist streng monoton fallend für $x < -1$, streng monoton steigend für $-1 < x < 1$ und streng monoton fallend für $x > 1$. $f$ ist streng konkav für $x < -\sqrt{3}$, streng konvex für $- \sqrt{3} < x < 0$, streng konkav für $0 < x < \sqrt{3}$ und streng konvex für $x > \sqrt{3}$.
\label{enum:1}
\item Aus \ref{enum:1}\text{.} ergibt sich: $f$ hat ein Minimum an $x=-1$, ein Maximum an $x=1$, Wendepunkte an $x=-\sqrt{3}, x=0$ und $x=\sqrt{3}$.
\item Mithilfe von Satz 22 zeige ich, dass $f$ für $x \rightarrow \infty$ eine Asymptote $g(x) =ax +b$ besitzt und berechne diese:\\
\begin{alignat*}{2}
a = \lim_{x \rightarrow \infty} \left( \frac{f(x)}{x} \right) &=& \lim_{x \rightarrow \infty} \left(\frac{2x}{x+x^{3}} \right) \\
&=& \lim_{x \rightarrow \infty} \left(\frac{(2) \cdot x}{(\frac{1}{x^{2}}+1) \cdot x^{3}} \right) \\
&=& \lim_{x \rightarrow \infty} \left(\frac{2}{(\frac{1}{x^{2}}+1) \cdot x^{2}} \right) \\
\intertext{Der Nenner geht gegen unendlich, der Zähler ist konstant, daher geht der Ausdruck gegen Null.}
&=& 0\\
\\
b = \lim_{x \rightarrow \infty} \left( f(x) - ax \right) &=& \lim_{x \rightarrow \infty} \left( \frac{2x}{1+x^{2}} - 0 \right) \\
&=& \lim_{x \rightarrow \infty} \left( \frac{(2) \cdot x}{(\frac{1}{x^{2}}+1) \cdot x^{2}} \right) \\
\lim_{x \rightarrow \infty} \left( \frac{2}{(\frac{1}{x^{2}}+1) \cdot x} \right) \\
\intertext{Der Nenner geht gegen unendlich, der Zähler ist konstant, daher geht der Ausdruck gegen Null.}
&=& 0
\end{alignat*}\\
Also besitzt $f$ für $x \rightarrow \infty$ die Asymptote $g(x)=0$. Eine ähnliche Überlegung ergibt, dass $f$ für $x \rightarrow -\infty$ ebenfalls die Asymptote $g(x)=0$ besitzt.\\
Es gibt einen Schnittpunkt von $f$ mit der Asymptote bei $x=0$, da\\
\begin{alignat*}{2}
\frac{2x}{1+x^{2}} &=& 0 \\
\intertext{Mit $1+x^{2}$ multiplizieren geht, da der Term nie gleich 0 sein kann.}
2x &=& 0 \\
x &=& 0
\end{alignat*}\\
gilt.
\item
\begin{alignat*}{2}
f(0) &=& \frac{2 \cdot 0}{1 + 0^{2}} \\
&=& \frac{0}{1} = 0\\
f(-1) &=& \frac{2 \cdot -1}{1 + (-1)^{2}} \\
&=& \frac{-2}{2} = -1 \\
f(1) &=& \frac{2 \cdot 1}{1 + 1^{2}} \\
&=& \frac{2}{2} = 1 \\
f(-\sqrt{3}) &=& \frac{2 \cdot (-\sqrt{3})}{1 + (-\sqrt{3})^{2}} \\
&=& \frac{-2\sqrt{3}}{1 + 3} \\
&=& \frac{-2\sqrt{3}}{4} \\
&=& -\frac{\sqrt{3}}{2} \\
f(\sqrt{3}) &=& \frac{2 \cdot \sqrt{3}}{1 + \sqrt{3}^{2}} \\
&=& \frac{2\sqrt{3}}{1 + 3} \\
&=& \frac{2\sqrt{3}}{4} \\
&=& \frac{\sqrt{3}}{2}
\end{alignat*}
\begin{tikzpicture}[>=stealth]
\begin{axis}[
ymin=-5,ymax=5,
x=1em,
y=1em,
axis x line=middle,
axis y line=middle,
axis line style=->,
xlabel={$x$},
ylabel={$y$},
]
\addplot[no marks, black, -] expression[domain=-10:10,samples=100]{(2*x)/(1+(x*x))} node[pos=0.65,anchor=north]{};
\end{axis}
\end{tikzpicture}
\end{enumerate}
\section{} %3
\begin{alignat*}{2}
f(x) &=& 4x^{3} - 10x +5 \\
f'(x) &=& 12x^{2} - 10 \\
\intertext{Die Berechnung der Näherungswerte erfolgt durch folgende Formel:}
x_{n+1} &=& x_{n} - \frac{f(x_{n})}{f'(x_{n})} \\
\intertext{Exemplarische Berechnung von $f(x)$ für $x=2$:}
f(2) &=& 4 \cdot 2^{3} - 10 \cdot 2 +5 \\
&=& 4 \cdot 8 - 20 + 5 \\
&=& 32 -15 \\
&=& 17 \\
\intertext{Exemplarische Berechnung von $f'(x)$ für $x=2$:}
f'(2) &=& 12 \cdot 2^{2} -10 \\
&=& 48 - 10 \\
&=& 38 \\
\intertext{Berechnung der Näherungswerte:}
x_{1} &=& 2 - \frac{17}{38} \\
&=& \frac{76 -17}{38} \\
&=& \frac{59}{38} \\
x_{2} &=& \frac{59}{38} - \frac{f(\frac{59}{38})}{f'(\frac{59}{38})} \\
&\approx & 1.317784436 \\
x_{3} &\approx & 1.317784436 - \frac{f(1.317784436)}{f'(1.317784436)} \\
&\approx & 1.227756731 \\
x_{4} &\approx & 1.227756731 - \frac{f(1.227756731)}{f'(1.227756731)} \\
&\approx & 1.212272195 \\
x_{5} &\approx & 1.212272195 - \frac{f(1.212272195)}{f'(1.212272195)} \\
&\approx & 1.211811475 \\
x_{6} &\approx & 1.211811475 - \frac{f(1.211811475)}{f'(1.211811475)} \\
&\approx & 1.21181107 \\
x_{7} &=& 1.21181107 - \frac{f(1.21181107)}{f'(1.21181107)} \\
&=& 1.21181107
\end{alignat*}
\section{} %4
Die Länge der Schnur berechnet sie wie folgt:\\
\begin{alignat*}{2}
L &=& 2x + y \\
\intertext{Nach y umgestellt ergibt sich}
y &=& L - 2x \\
\intertext{Eingesetzt in die Formel für den Flächeninhalt von Rechtecken ergibt sich:}
A(x) &=& x \cdot (L-2x) \\
&=& -2x^{2} + Lx \\
A'(x) &=& -4x + L \\
A''(x) &=& -4 \\
\intertext{Gleichsetzen von $A'(x)$ mit Null}
0 &=& -4x + L \\
4x &=& L \\
x &=& \frac{L}{4} \\
\intertext{Einsetzen in zweite Ableitung}
A''(\frac{L}{4}) &=& -4 \Rightarrow \text{$< 0$, daher Maximum}
\end{alignat*}
Daraus folgt, dass die beiden gleichlangen Seiten des Rechtecks, die von der Schnur begrenzt werden, jeweils ein Viertel der Schnur ausmachen. Die dritte von der Schnur begrenzte Seite ist folglich halb so lang wie die Schnur.
\section{} %5
\subsection{} %a
Die Oberfläche ($A$) und das Volumen ($V$) der Dose sei durch folgende Formeln gegeben:\\
\begin{alignat*}{2}
A &=& 2\pi r^{2} + 2\pi rh \\
V &=& 1000 cm^{3} = \pi r^{2}h \\
\intertext{Umstellen von $V$ nach $h$}
h &=& \frac{1000 cm^{3}}{\pi r^{2}} \\
\intertext{Einsetzen in $A$}
A(r) &=& 2 \pi r^{2} + 2 \pi r \cdot \frac{1000 cm^{3}}{\pi r^{2}} \\
\intertext{Kürzen}
&=& 2 \pi r^{2} + 2 \cdot \frac{1000 cm^{3}}{r} \\
\intertext{Zusammenfassen}
&=& 2 \pi r^{2} + 2000 cm^{3} \cdot r^{-1} \\
A'(r) &=& 4 \pi r - 2000 cm^{3} \cdot r^{-2} \\
A''(r) &=& 4 \pi + 4000 cm^{3} \cdot r^{-3} \\
\intertext{Erste Ableitung mit Null gleichsetzen}
0 &=& 4 \pi r - 2000 cm^{3} \cdot r^{-2} \\
2000 cm^{3} \cdot r^{-2} &=& 4 \pi r \\
2000 cm^{3} &=& 4 \pi r^{3} \\
\frac{2000 cm^{3}}{4 \pi} &=& r^{3} \\
\frac{500 cm^{3}}{\pi} &=& r^{3} \\
\intertext{Dritte Wurzel ziehen}
\sqrt[3]{\frac{500 cm^{3}}{\pi}} &=& r \\
\intertext{Einsetzen in zweite Ableitung}
A''\left(\sqrt[3]{\frac{500 cm^{3}}{\pi}} \right) &=& 4 \pi + 4000 cm^{3} \cdot \left(\sqrt[3]{\frac{500 cm^{3}}{\pi}}\right)^{-3} \\
\intertext{Umformen}
&=& 4 \pi + 4000 cm^{3} \cdot \frac{1}{\left(\sqrt[3]{\frac{500 cm^{3}}{\pi}} \right)^{3}} \\
\intertext{Auflösen der Wurzel}
&=& 4 \pi + 4000 cm^{3} \cdot \frac{1}{\frac{500 cm^{3}}{\pi}} \\
\intertext{Auflösen Doppelbruch}
&=& 4 \pi + 4000 cm^{3} \cdot \frac{\pi}{500 cm^{3}} \\
\intertext{Kürzen}
&=& 4 \pi + 8 \cdot \pi \\
&=& 12 \pi \Rightarrow \text{$>0$, daher Minimum} \\
\intertext{Einsetzen des Radius in Formel für $h$}
h &=& \frac{1000 cm^{3}}{\pi \cdot \left(\sqrt[3]{\frac{500 cm^{3}}{\pi}} \right)^{2}} \\
\intertext{Umformen}
&=& \frac{1000 cm^{3}}{\pi \cdot \left(\sqrt[3]{500 cm^{3}\pi^{-1}} \right)^{2}} \\
\intertext{Umformen}
&=& \frac{1000 cm^{3}}{\pi \cdot \left(\left(500 cm^{3}\pi^{-1}\right)^{\frac{1}{3}} \right)^{2}} \\
\intertext{innere Klammer auflösen}
&=& \frac{1000 cm^{3}}{\pi \cdot \left((500 cm^{3})^{\frac{1}{3}} \cdot \pi^{-\frac{1}{3}} \right)^{2}} \\
\intertext{Klammer auflösen}
&=& \frac{1000 cm^{3}}{\pi \cdot (500 cm^{3})^{\frac{2}{3}} \cdot \pi^{-\frac{2}{3}}} \\
\intertext{Zusammenfassen und Bruch auseinanderziehen}
&=& \frac{1000 cm^{3}}{(500 cm^{3})^{\frac{2}{3}} } \cdot \frac{1}{\pi^{\frac{1}{3}}}
\end{alignat*}
\subsection{} %b
Die Oberfläche $A$ wird in diesem Fall ähnlich berechnet. Allerdings muss die Grundfläche nur einmal eingerechnet werden:\\
\begin{alignat*}{2}
A &=& \pi r^{2} + 2 \pi r h \\
\intertext{Die Berechnung von r ist analog. Lediglich der Faktor vor dem $\pi r^{2}$ ist anders. Daraus folgt für $A(r)$:}
A(r) &=& \pi r^{2} + 2000 cm^{3} \cdot r^{-1} \\
A'(r) &=& 2 \pi r - 2000 cm^{3} \cdot r^{-2} \\
\intertext{Aufgrund der Änderung ergibt sich nach umstellen nach r folgendes:}
\frac{2000 cm^{3}}{2 \pi} &=& r^{3} \\
\intertext{Durch Kürzen ergibt sich:}
\frac{1000 cm^{3}}{\pi} &=& r^{3} \\
\intertext{Ziehen der dritten Wurzel:}
\sqrt[3]{\frac{1000 cm^{3}}{\pi}} &=& r \\
\intertext{Beim Einsetzen in die zweite Ableitung verändert sich nicht viel:}
A''\left(\sqrt[3]{\frac{1000 cm^{3}}{\pi}} \right) = 2 \pi + 4000 cm^{3} \cdot \left(\sqrt[3]{\frac{1000 cm^{3}}{\pi}} \right)^{-3} \\
&=& 2 \pi + 4000 cm^{3} \cdot \frac{1}{ \left(\sqrt[3]{\frac{1000 cm^{3}}{\pi}} \right)^{3}} \\
&=& 2 \pi + 4000 cm^{3} \cdot \frac{1}{ \frac{1000 cm^{3}}{\pi}} \\
&=& 2 \pi + 4000 cm^{3} \cdot \frac{\pi}{ 1000 cm^{3}} \\
&=& 2 \pi + 4 \pi \\
&=& 6 \pi \Rightarrow \text{$>0$, daher Minimum}\\
\intertext{Durch Einsetzen in Gleichung von $h$ ergibt sich nun:}
h &=& \frac{1000 cm^{3}}{\pi \cdot \left(\sqrt[3]{\frac{1000 cm^{3}}{\pi}} \right)^{2}} \\
&=& \frac{1000 cm^{3}}{\pi \cdot \left(\left(\frac{1000 cm^{3}}{\pi}\right)^{\frac{1}{3}} \right)^{2}} \\
&=& \frac{1000 cm^{3}}{\pi \cdot \left(\left(1000 cm^{3} \cdot \pi^{-1} \right)^{\frac{1}{3}} \right)^{2}} \\
&=& \frac{1000 cm^{3}}{\pi \cdot \left((1000 cm^{3})^{\frac{1}{3}} \cdot \pi^{-\frac{1}{3}}\right)^{2}} \\
&=& \frac{1000 cm^{3}}{\pi \cdot (1000 cm^{3})^{\frac{2}{3}} \cdot \pi^{-\frac{2}{3}}} \\
&=& \frac{1000 cm^{3}}{(1000 cm^{3})^{\frac{2}{3}} \cdot \pi^{\frac{1}{3}}} \\
&=& \frac{1000 cm^{3}}{(1000 cm^{3})^{\frac{2}{3}}} \cdot \frac{1}{\pi^{\frac{1}{3}}} \\
\intertext{Kürzen}
&=& (1000 cm^{3})^{-\frac{1}{3}} \cdot \frac{1}{\pi^{\frac{1}{3}}}
\end{alignat*}
\end{document}

View File

@ -0,0 +1,392 @@
\documentclass[10pt,a4paper,oneside,ngerman,numbers=noenddot]{scrartcl}
\usepackage[T1]{fontenc}
\usepackage[utf8]{inputenc}
\usepackage[ngerman]{babel}
\usepackage{amsmath}
\usepackage{amsfonts}
\usepackage{amssymb}
\usepackage{paralist}
\usepackage{gauss}
\usepackage{pgfplots}
\usepackage[locale=DE,exponent-product=\cdot,detect-all]{siunitx}
\usepackage{tikz}
\usetikzlibrary{matrix,fadings,calc,positioning,decorations.pathreplacing,arrows,decorations.markings}
\usepackage{polynom}
\polyset{style=C, div=:,vars=x}
\pagenumbering{arabic}
\def\thesection{\arabic{section})}
\def\thesubsection{\alph{subsection})}
\def\thesubsubsection{(\roman{subsubsection})}
\makeatletter
\renewcommand*\env@matrix[1][*\c@MaxMatrixCols c]{%
\hskip -\arraycolsep
\let\@ifnextchar\new@ifnextchar
\array{#1}}
\makeatother
\begin{document}
\author{Jim Martens (6420323)}
\title{Hausaufgaben zum 30. Mai}
\maketitle
\section{} %1
\subsection{} %a
Berechnen von $\lim\limits_{n \rightarrow \infty} O_{n}$. Es gilt:\\
\begin{alignat*}{2}
O_{n} &=&& \frac{b-a}{n} \sum\limits_{i=1}^{n} f(x_{i}) = \frac{3}{n} \sum\limits_{i=1}^{n} f\left( \frac{3i}{n}\right) = \frac{3}{n} \sum\limits_{i=1}^{n} \left( \frac{3i}{n}\right)^{3} = \frac{3}{n^{4}} \sum\limits_{i=1}^{n} 27 \cdot i^{3}\\
&=&& \frac{81}{n^{4}} \sum\limits_{i=1}^{n} i^{3} = \frac{81}{n^{4}} \cdot \frac{n^{2}(n+1)^{2}}{4} = \frac{81 \cdot n^{2}(n+1)^{2}}{4 \cdot n^{4}} = \frac{81 \cdot (n+1)^{2}}{4 \cdot n^{2}} \\
&=&& \frac{81 \cdot (n^{2} + 2n + 1)}{4 \cdot n^{2}} = \frac{81n^{2} + 162n + 81}{4n^{2}}
\end{alignat*}\\
Also gilt $\lim\limits_{n \rightarrow \infty} O_{n} = \lim\limits_{n \rightarrow \infty} \left(\frac{81n^{2} + 162n + 81}{4n^{2}} \right) = \frac{81}{4}$. Der gesuchte Flächeninhalt hat den Wert $\frac{81}{4}$.
\subsection{} %b
\begin{alignat*}{2}
\int\limits_{a}^{b} f(x) \,dx = \int\limits_{0}^{3} x^{3} \,dx &=& \left[\frac{1}{4}x^{4}\right]_{0}^{3} \\
&=& \frac{1}{4} \cdot 3^{4} - \frac{1}{4} \cdot 0^{4} = \frac{1}{4} \cdot 81 - 0 = \frac{81}{4}
\end{alignat*}
\section{} %2
\subsubsection{} %i
\begin{alignat*}{2}
\int\limits_{1}^{3} (x^{2} -x-6) \,dx &=& \left[\frac{1}{3}x^{3} - \frac{1}{2}x^{2} - 6x \right]_{1}^{3} \\
&=& \left(\frac{1}{3} \cdot 3^{3} - \frac{1}{2} \cdot 3^{2} - 6 \cdot 3\right) - \left(\frac{1}{3} \cdot 1^{3} - \frac{1}{2} \cdot 1^{2} - 6 \cdot 1\right) \\
&=& \left(9 - \frac{9}{2} - 18\right) - \left(\frac{1}{3} - \frac{1}{2} - 6\right) = \left(\frac{18 - 9 - 36}{2}\right) - \left(\frac{2 - 3 - 36}{6}\right) \\
&=& \left(\frac{-27}{2}\right) - \left(\frac{-37}{6}\right) = \frac{-27}{2} + \frac{37}{6} = \frac{-81 + 37}{6} \\
&=& -\frac{44}{6} = -\frac{22}{3}
\end{alignat*}
\begin{tikzpicture}[>=stealth]
\begin{axis}[
ymin=-7,ymax=1,
x=1em,
y=1em,
axis x line=middle,
axis y line=middle,
axis line style=->,
xlabel={$x$},
ylabel={$y$},
xmin=0,xmax=4
]
\addplot[no marks, black, -] expression[domain=1:3,samples=100]{x*x -x -6} node[pos=0.65,anchor=north]{};
\node at (axis cs: 2.75,-3) {f};
\node at (axis cs: 1.75,-2.5) {A};
\draw[>=stealth] (axis cs:1,0) -- (axis cs:1,-6) node [pos=0.65,anchor=north]{};
\end{axis}
\end{tikzpicture}
\subsubsection{} %ii
\begin{alignat*}{2}
\int\limits_{1}^{3} x^{\frac{1}{3}} \,dx &=& \left[\frac{3}{4}x^{\frac{4}{3}} \right]_{1}^{3} \\
&=& \left(\frac{3}{4} \cdot 3^{\frac{4}{3}}\right) - \left(\frac{3}{4} \cdot 1^{\frac{4}{3}}\right) = 3^{\frac{4}{3}} \cdot \frac{3}{4} - \frac{3}{4} \\
&=& \left(3^{\frac{4}{3}} - 1\right) \cdot \frac{3}{4}
\end{alignat*}
\begin{tikzpicture}[>=stealth]
\begin{axis}[
ymin=0,ymax=3,
x=1cm,
y=1cm,
axis x line=middle,
axis y line=middle,
axis line style=->,
xlabel={$x$},
ylabel={$y$},
xmin=0,xmax=4
]
\addplot[no marks, black, -] expression[domain=1:3,samples=100]{x^(1/3)} node[pos=0.65,anchor=north]{};
\node at (axis cs: 2,1.5) {f};
\node at (axis cs: 2,0.5) {A};
\draw[>=stealth] (axis cs:1,0) -- (axis cs:1,1) node [pos=0.65,anchor=north]{};
\draw[>=stealth] (axis cs:3,0) -- (axis cs:3,1.44224957) node [pos=0.65,anchor=north]{};
\end{axis}
\end{tikzpicture}
\subsubsection{} %iii
\begin{alignat*}{2}
\int\limits_{1}^{3} \frac{1}{1+x^{2}} \,dx &=& \left[\arctan x \right]_{1}^{3} \\
&=& \arctan 3 - \arctan 1
\end{alignat*}
\begin{tikzpicture}[>=stealth]
\begin{axis}[
ymin=0,ymax=2,
x=1cm,
y=1cm,
axis x line=middle,
axis y line=middle,
axis line style=->,
xlabel={$x$},
ylabel={$y$},
xmin=0,xmax=4
]
\addplot[no marks, black, -] expression[domain=1:3,samples=100]{1/(1+x*x)} node[pos=0.65,anchor=north]{};
\node at (axis cs: 2,0.5) {f};
\node at (axis cs: 1.2,0.2) {A};
\draw[>=stealth] (axis cs:1,0) -- (axis cs:1,0.5) node [pos=0.65,anchor=north]{};
\draw[>=stealth] (axis cs:3,0) -- (axis cs:3,0.1) node [pos=0.65,anchor=north]{};
\end{axis}
\end{tikzpicture}
\subsubsection{} %iv
\begin{alignat*}{2}
\int\limits_{1}^{3} \ln x \,dx &=& \left[x \cdot \ln x - x \right]_{1}^{3} \\
&=& \left(3 \cdot \ln 3 - 3 \right) - \left(1 \cdot \ln 1 - 1 \right) = 3 \cdot \ln 3 - 3 + 1 \\
&=& 3 \cdot \ln 3 - 2
\end{alignat*}
\begin{tikzpicture}[>=stealth]
\begin{axis}[
ymin=0,ymax=2,
x=1cm,
y=1cm,
axis x line=middle,
axis y line=middle,
axis line style=->,
xlabel={$x$},
ylabel={$y$},
xmin=0,xmax=4
]
\addplot[no marks, black, -] expression[domain=1:3,samples=100]{ln x} node[pos=0.65,anchor=north]{};
\node at (axis cs: 2,1) {f};
\node at (axis cs: 2.2,0.4) {A};
%\draw[>=stealth] (axis cs:1,0) -- (axis cs:1,0.5) node [pos=0.65,anchor=north]{};
\draw[>=stealth] (axis cs:3,0) -- (axis cs:3,1.098612289) node [pos=0.65,anchor=north]{};
\end{axis}
\end{tikzpicture}
\subsubsection{} %v
\begin{alignat*}{2}
\int\limits_{1}^{3} e^{-x} \,dx &=& \left[-e^{-x} \right]_{1}^{3} \\
&=& \left(-e^{-3} \right) - \left( -e^{-1}\right) = -e^{-3} + e^{-1}
\end{alignat*}
\begin{tikzpicture}[>=stealth]
\begin{axis}[
ymin=0,ymax=2,
x=1cm,
y=1cm,
axis x line=middle,
axis y line=middle,
axis line style=->,
xlabel={$x$},
ylabel={$y$},
xmin=0,xmax=4
]
\addplot[no marks, black, -] expression[domain=1:3,samples=100]{e^(-x)} node[pos=0.65,anchor=north]{};
\node at (axis cs: 2,0.5) {f};
\node at (axis cs: 1.15,0.12) {A};
\draw[>=stealth] (axis cs:1,0) -- (axis cs:1,0.367879441) node [pos=0.65,anchor=north]{};
\draw[>=stealth] (axis cs:3,0) -- (axis cs:3,0.049787068) node [pos=0.65,anchor=north]{};
\end{axis}
\end{tikzpicture}
\section{} %3
\setcounter{subsubsection}{0}
\subsubsection{} %i
\begin{alignat*}{2}
\int (x^{4} + 2x^{3} -x +5) \,dx &=& \frac{1}{5}x^{5} + \frac{1}{4}x^{4} - \frac{1}{2}x^{2} + 5x
\end{alignat*}
\subsubsection{} %ii
\begin{alignat*}{2}
\int \frac{1}{\sqrt{x^{3}}} \,dx &=& \int \frac{1}{x^{\frac{3}{2}}} \,dx = \int x^{\frac{2}{3}} \,dx \\
&=& \frac{3}{5}x^{\frac{5}{3}}
\end{alignat*}
\subsubsection{} %iii
\begin{alignat*}{2}
\int x \cdot \sin(3x) \,dx &=& \frac{-\cos(3x)}{3} \cdot x + \frac{\sin(3x)}{9} \\
\intertext{Probe:}
\left(\frac{-\cos(3x)}{3} \cdot x + \frac{\sin(3x)}{9}\right)' &=& \left(\frac{-\cos(3x)}{3}\right)' \cdot x + \frac{-\cos(3x)}{3} \cdot 1 + \left(\frac{\sin(3x)}{9}\right)' \\
&=& \frac{\sin(3x) \cdot 3}{3} \cdot x + \frac{-\cos(3x)}{3} + \frac{\cos(3x) \cdot 3}{9} \\
&=& \sin(3x) \cdot x + \frac{-\cos(3x)}{3} + \frac{\cos(3x)}{3} \\
&=& \sin(3x) \cdot x
\end{alignat*}
\subsubsection{} %iv
\begin{alignat*}{2}
\int x^{3} \cdot \ln x \,dx &=& \frac{1}{4}x^{4} \cdot \ln x - \int \frac{1}{4}x^{4} \cdot \frac{1}{x}\,dx \\
&=& \frac{1}{4}x^{4} \cdot \ln x - \int \frac{1}{4}x^{3}\,dx \\
&=& \frac{1}{4}x^{4} \cdot \ln x - \frac{1}{16}x^{4} \\
\intertext{Probe:}
\left(\frac{1}{4}x^{4} \cdot \ln x - \frac{1}{4}x^{4}\right)' &=& (\frac{1}{4}x^{4} \cdot \ln x)' - \left(\frac{1}{16}x^{4}\right)' \\
&=& x^{3} \cdot \ln x + \frac{1}{4}x^{4} \cdot \frac{1}{x} - \frac{1}{4}x^{3} \\
&=& x^{3} \cdot \ln x + \frac{1}{4}x^{3} - \frac{1}{4}x^{3}\\
&=& x^{3} \cdot \ln x
\end{alignat*}
\subsubsection{} %v
\begin{alignat*}{2}
\int x^{2}e^{x} \,dx &=& x^{2}e^{x} - \int 2x \cdot e^{x}\,dx \\
&=& x^{2}e^{x} - 2 \cdot \int x \cdot e^{x}\,dx \\
&=& x^{2}e^{x} - 2 \cdot \left(x \cdot e^{x} - \int 1 \cdot e^{x}\,dx \right) \\
&=& x^{2}e^{x} - 2 \cdot (x \cdot e^{x} - e^{x}) \\
&=& x^{2}e^{x} - 2x \cdot e^{x} +2e^{x} \\
\intertext{Probe:}
(x^{2}e^{x} - 2x \cdot e^{x} +2e^{x})' &=& (x^{2}e^{x})' - (2x \cdot e^{x})' + 2e^{x} \\
&=& (2x \cdot e^{x} + x^{2}e^{x}) - (2e^{x} + 2x \cdot e^{x}) + 2e^{x} \\
&=& 2x \cdot e^{x} + x^{2}e^{x} - 2e^{x} - 2x \cdot e^{x} + 2e^{x} \\
&=& x^{2}e^{x}
\end{alignat*}
\section{} %4
\setcounter{subsubsection}{0}
\subsubsection{} %i
\begin{alignat*}{2}
t &=& \sqrt{2x+5} \\
\frac{dt}{dx} &=& \frac{2}{2 \cdot \sqrt{2x+5}} \\
\sqrt{2x+5}\,dt &=& dx \\
dx &=& t\,dt\\
\int \cos(\sqrt{2x+5}) \,dx &=& \int \cos(t) \cdot t\,dt \\
&=& \int \cos(t) \cdot t\,dt \\
&=& \sin(t) \cdot t - \int \sin(t) \cdot 1 \,dt \\
&=& \sin(t) \cdot t + \cos(t) \\
\int \cos(\sqrt{2x+5}) \,dx &=& \sqrt{2x+5} \cdot \sin(\sqrt{2x+5}) + \cos(\sqrt{2x+5})
\end{alignat*}
\begin{alignat*}{3}
\intertext{Probe:}
\left(\sqrt{2x+5} \cdot \sin(\sqrt{2x+5}) + \cos(\sqrt{2x+5})\right)' &=& (\sqrt{2x+5} \cdot \sin(\sqrt{2x+5}))' + (\cos(\sqrt{2x+5}))' \\
\intertext{Ableiten}
&=\begin{split}\frac{2 \cdot \sin(\sqrt{2x+5})}{2 \cdot \sqrt{2x+5}}\\ + \sqrt{2x+5} \cdot \frac{\cos(\sqrt{2x+5}) \cdot 2}{2 \cdot \sqrt{2x+5}} \\ + (-\sin(\sqrt{2x+5}) \cdot (\sqrt{2x+5})')\end{split}\\
\intertext{Zusammenfassen und Ableiten}
&= \begin{split}\frac{\sin(\sqrt{2x+5})}{\sqrt{2x+5}} + \cos(\sqrt{2x+5})\\ - \frac{\sin(\sqrt{2x+5}) \cdot 2}{2 \cdot \sqrt{2x+5}}\end{split} \\
\intertext{Zusammenfassen}
&= \begin{split}\frac{\sin(\sqrt{2x+5})}{\sqrt{2x+5}} + \cos(\sqrt{2x+5})\\ - \frac{\sin(\sqrt{2x+5})}{\sqrt{2x+5}}\end{split} \\
\intertext{Zusammenfassen}
&=& \cos(\sqrt{2x+5})
\end{alignat*}
\subsubsection{} %ii
\begin{alignat*}{2}
t &=& \sqrt[3]{x} \\
\frac{dt}{dx} &=& \frac{1}{3 \cdot \left(\sqrt[3]{x}\right)^{2}} \\
dx &=& 3 \cdot \left(\sqrt[3]{x}\right)^{2}\,dt \\
dx &=& 3 \cdot t^{2}\,dt\\
\int \sin(\sqrt[3]{x})\,dx &=& \int \sin(t) \cdot 3t^{2}\,dt\\
&=& 3 \int \sin(t) \cdot t^{2}\,dt \\
&=& 3\left(-\cos(t) \cdot t^{2} - \int -\cos(t) \cdot 2t \right) \\
3\left(-\cos(t) \cdot t^{2} + 2\int \cos(t) \cdot t \right) \\
&=& 3\left(-\cos(t) \cdot t^{2} + 2\left( \sin(t) \cdot t - \int \sin(t) \cdot 1 \right)\right) \\
&=& 3\left(-\cos(t) \cdot t^{2} + 2\left( \sin(t) \cdot t + \cos(t) \right)\right) \\
&=& 3\left(-\cos(t) \cdot t^{2} + \sin(t) \cdot 2t + 2 \cdot \cos(t)\right) \\
&=& -3t^{2} \cdot \cos(t) + 6t \cdot \sin(t) + 6 \cdot \cos(t) \\
\int \sin(\sqrt[3]{x})\,dx &=& -3 \cdot \sqrt[3]{x}^{2} \cdot \cos(\sqrt[3]{x}) + 6 \cdot \sqrt[3]{x} \cdot \sin(\sqrt[3]{x}) + 6 \cdot \cos(\sqrt[3]{x}) \\
&=& -3 \cdot x^{\frac{2}{3}} \cdot \cos(x^{\frac{1}{3}}) + 6 \cdot x^{\frac{1}{3}} \cdot \sin(x^{\frac{1}{3}}) + 6 \cdot \cos(x^{\frac{1}{3}})
\end{alignat*}
\begin{alignat*}{2}
\intertext{Probe:}
&& \left(-3 \cdot x^{\frac{2}{3}} \cdot \cos(x^{\frac{1}{3}}) + 6 \cdot x^{\frac{1}{3}} \cdot \sin(x^{\frac{1}{3}}) + 6 \cdot \cos(x^{\frac{1}{3}})\right)' \\
&=& \left(-3 \cdot x^{\frac{2}{3}} \cdot \cos(x^{\frac{1}{3}})\right)' + 6\left(\left(x^{\frac{1}{3}} \cdot \sin(x^{\frac{1}{3}})\right)' + \left(\cos(x^{\frac{1}{3}})\right)'\right) \\
&=& -3\left(\frac{2}{3}x^{-\frac{1}{3}} \cdot \cos(x^{\frac{1}{3}}) + x^{\frac{2}{3}} \cdot (-\sin(x^{\frac{1}{3}})) \cdot \frac{1}{3}x^{-\frac{2}{3}}\right)+ 6\left(\left(x^{\frac{1}{3}} \cdot \sin(x^{\frac{1}{3}})\right)' + \left(\cos(x^{\frac{1}{3}})\right)'\right) \\
&=& -3\left(\frac{2}{3}x^{-\frac{1}{3}} \cdot \cos(x^{\frac{1}{3}}) - \frac{1}{3} \cdot \sin(x^{\frac{1}{3}})\right)+ 6\left(\left(x^{\frac{1}{3}} \cdot \sin(x^{\frac{1}{3}})\right)' + \left(\cos(x^{\frac{1}{3}})\right)'\right) \\
\intertext{Ableiten}
&= \begin{split}-3\left(\frac{2}{3}x^{-\frac{1}{3}} \cdot \cos(x^{\frac{1}{3}}) - \frac{1}{3} \cdot \sin(x^{\frac{1}{3}})\right)\\ + 6\left(\left(\frac{1}{3}x^{-\frac{2}{3}} \cdot \sin(x^{\frac{1}{3}}) + x^{\frac{1}{3}} \cdot \cos(x^{\frac{1}{3}}) \cdot \frac{1}{3}x^{-\frac{2}{3}} \right) + \left(\cos(x^{\frac{1}{3}})\right)'\right) \end{split} \\
\intertext{Zusammenfassen}
&= \begin{split}-3\left(\frac{2}{3}x^{-\frac{1}{3}} \cdot \cos(x^{\frac{1}{3}}) - \frac{1}{3} \cdot \sin(x^{\frac{1}{3}})\right)\\ + 6\left(\frac{1}{3}x^{-\frac{2}{3}} \cdot \sin(x^{\frac{1}{3}}) + \frac{1}{3}x^{-\frac{1}{3}} \cdot \cos(x^{\frac{1}{3}}) + \left(\cos(x^{\frac{1}{3}})\right)'\right) \end{split} \\
\intertext{Ableiten}
&= \begin{split}-3\left(\frac{2}{3}x^{-\frac{1}{3}} \cdot \cos(x^{\frac{1}{3}}) - \frac{1}{3} \cdot \sin(x^{\frac{1}{3}})\right)\\ + 6\left(\frac{1}{3}x^{-\frac{2}{3}} \cdot \sin(x^{\frac{1}{3}}) + \frac{1}{3}x^{-\frac{1}{3}} \cdot \cos(x^{\frac{1}{3}}) - \sin(x^{\frac{1}{3}}) \cdot \frac{1}{3}x^{-\frac{2}{3}} \right) \end{split} \\
\intertext{Zusammenfassen}
&= \begin{split}-3\left(\frac{2}{3}x^{-\frac{1}{3}} \cdot \cos(x^{\frac{1}{3}}) - \frac{1}{3} \cdot \sin(x^{\frac{1}{3}})\right)\\ + 6\left(\frac{1}{3}x^{-\frac{1}{3}} \cdot \cos(x^{\frac{1}{3}})\right) \end{split} \\
\intertext{Zusammenfassen}
&=& -2x^{-\frac{1}{3}} \cdot \cos(x^{\frac{1}{3}}) + \sin(x^{\frac{1}{3}}) + 2x^{-\frac{1}{3}} \cdot \cos(x^{\frac{1}{3}}) \\
\intertext{Zusammenfassen}
&=& \sin(x^{\frac{1}{3}})
\end{alignat*}
\subsubsection{} %iii
\begin{alignat*}{2}
t &=& \sqrt{\frac{2}{7}x+3} \\
\frac{dt}{dx} &=& \frac{2}{14 \cdot \sqrt{\frac{2}{7}x+3}} \\
\frac{dt}{dx} &=& \frac{1}{7 \cdot \sqrt{\frac{2}{7}x+3}} \\
dx &=& 7 \cdot \sqrt{\frac{2}{7}x+3}\, dt \\
&=& 7t\\
\int e^{\sqrt{\frac{2}{7}x+3}}\,dx &=& \int e^{t} \cdot 7t\,dt \\
&=& e^{t} \cdot 7t - \int e^{t} \cdot 7\,dt \\
&=& e^{t} \cdot 7t - 7\int e^{t}\,dt \\
&=& e^{t} \cdot 7t - 7 \cdot e^{t} \\
&=& e^{t}(7t - 7) \\
&=& e^{\sqrt{\frac{2}{7}x+3}}\left(7 \cdot \sqrt{\frac{2}{7}x+3} - 7\right)
\end{alignat*}
\begin{alignat*}{2}
\intertext{Probe:}
&& \left(e^{\sqrt{\frac{2}{7}x+3}}\left(7 \cdot \sqrt{\frac{2}{7}x+3} - 7\right) \right)' \\
&=& \left(e^{\sqrt{\frac{2}{7}x+3}}\right)' \cdot \left(7 \cdot \sqrt{\frac{2}{7}x+3} - 7\right) + \left(e^{\sqrt{\frac{2}{7}x+3}}\right) \cdot \left(7 \cdot \sqrt{\frac{2}{7}x+3} - 7\right)' \\
&=& e^{\sqrt{\frac{2}{7}x+3}} \cdot \frac{2}{14 \cdot \sqrt{\frac{2}{7}x+3}} \cdot \left(7 \cdot \sqrt{\frac{2}{7}x+3} - 7\right) + \left(e^{\sqrt{\frac{2}{7}x+3}}\right) \cdot \left(7 \cdot \left(\frac{2}{7}x+3\right)^{\frac{1}{2}} - 7\right)' \\
&=& e^{\sqrt{\frac{2}{7}x+3}} \cdot \frac{1}{7 \cdot \sqrt{\frac{2}{7}x+3}} \cdot \left(7 \cdot \sqrt{\frac{2}{7}x+3} - 7\right) + \left(e^{\sqrt{\frac{2}{7}x+3}}\right) \cdot \left(7 \cdot \left(\frac{2}{7}x+3\right)^{\frac{1}{2}}\right)'\\
&=& e^{\sqrt{\frac{2}{7}x+3}} \cdot \frac{1}{7 \cdot \sqrt{\frac{2}{7}x+3}} \cdot \left(7 \cdot \sqrt{\frac{2}{7}x+3} - 7\right) + \left(e^{\sqrt{\frac{2}{7}x+3}}\right) \cdot \left(\frac{7}{2} \cdot \left(\frac{2}{7}x+3\right)^{-\frac{1}{2}} \cdot \frac{2}{7}\right) \\
\intertext{Zusammenfassen}
&=& e^{\sqrt{\frac{2}{7}x+3}} \cdot \left(1 - \frac{1}{\sqrt{\frac{2}{7}x+3}}\right) + \left(e^{\sqrt{\frac{2}{7}x+3}}\right) \cdot \left(\frac{2}{7}x+3\right)^{-\frac{1}{2}} \\
\intertext{Ausklammern}
&=& e^{\sqrt{\frac{2}{7}x+3}} \left(\left(1 - \frac{1}{\sqrt{\frac{2}{7}x+3}}\right) + \frac{1}{\sqrt{\frac{2}{7}x+3}} \right) \\
\intertext{Klammern auflösen}
&=& e^{\sqrt{\frac{2}{7}x+3}} \left(1 - \frac{1}{\sqrt{\frac{2}{7}x+3}} + \frac{1}{\sqrt{\frac{2}{7}x+3}}\right) \\
\intertext{Zusammenfassen}
&=& e^{\sqrt{\frac{2}{7}x+3}}
\end{alignat*}
\subsubsection{} %iv
\begin{alignat*}{2}
t &=& \ln(2x+1) \\
e^{t} &=& 2x+1 \\
x &=& \frac{e^{t}-1}{2} \\
\frac{dx}{dt} &=& \frac{1}{2}\left(e^{t}\right) \\
dx &=& \frac{1}{2} \cdot e^{t}\,dt \\
\int \left(\ln (2x+1)\right)^{2}\,dx &=& \int t^{2} \cdot \frac{1}{2} \cdot e^{t}\,dt \\
&=& \frac{1}{2} \int t^{2} \cdot e^{t} \\
&=& \frac{1}{2}\left(t^{2} \cdot e^{t} - \int 2t \cdot e^{t} \right) \\
&=& \frac{1}{2}\left(t^{2} \cdot e^{t} - 2\int t \cdot e^{t} \right) \\
&=& \frac{1}{2}\left(t^{2} \cdot e^{t} - 2\left(t \cdot e^{t} - \int 1 \cdot e^{t} \right)\right) \\
&=& \frac{1}{2}\left(t^{2} \cdot e^{t} - 2t \cdot e^{t} + 2 \cdot e^{t} \right) \\
&=& \frac{1}{2}t^{2} \cdot e^{t} - t \cdot e^{t} + e^{t} \\
&=& \frac{1}{2} \left(\ln(2x+1)\right)^{2} \cdot e^{\ln(2x+1)} - \ln(2x+1) \cdot e^{\ln(2x+1)} + e^{\ln(2x+1)} \\
&=& \left(\frac{1}{2} \left(\ln(2x+1)\right)^{2} - \ln(2x+1) + 1\right)e^{\ln(2x+1)} \\
&=& \left(\frac{1}{2} \left(\ln(2x+1)\right)^{2} - \ln(2x+1) + 1\right) \cdot (2x+1)
\end{alignat*}
\begin{alignat*}{2}
\intertext{Probe:}
&& \left(\left(\frac{1}{2} \left(\ln(2x+1)\right)^{2} - \ln(2x+1) + 1\right)\cdot (2x+1)\right)' \\
&=& \left(\frac{1}{2} \left(\ln(2x+1)\right)^{2} - \ln(2x+1) + 1\right)' \cdot (2x+1) + \left(\frac{1}{2}\left(\ln(2x+1)\right)^{2} - \ln(2x+1) + 1\right) \cdot (2x+1)' \\
&=& \left(\frac{2 \cdot \ln(2x+1)}{2x+1} - \frac{2}{2x+1}\right) \cdot (2x+1) + \left(\frac{1}{2}\left(\ln(2x+1)\right)^{2} - \ln(2x+1) + 1\right) \cdot 2 \\
&=& \left(\frac{2 \cdot \ln(2x+1) - 2}{2x+1}\right) \cdot (2x+1) + \left(\frac{1}{2}\left(\ln(2x+1)\right)^{2} - \ln(2x+1) + 1\right) \cdot 2 \\
\intertext{Zusammenfassen}
&=& \frac{2 \cdot \ln(2x+1) \cdot (2x+1) - 2 \cdot (2x+1)}{2x+1} + \left(\frac{1}{2}\left(\ln(2x+1)\right)^{2} - \ln(2x+1) + 1\right) \cdot 2 \\
\intertext{Zusammenfassen}
&=& 2 \cdot \ln(2x+1) - 2 + \left(\frac{1}{2}\left(\ln(2x+1)\right)^{2} - \ln(2x+1) + 1\right) \cdot 2 \\
\intertext{Zusammenfassen}
&=& 2 \cdot \ln(2x+1) - 2 + \left(\ln(2x+1)\right)^{2} - 2 \cdot \ln(2x+1) + 2 \\
\intertext{Zusammenfassen}
&=& \left(\ln(2x+1)\right)^{2}
\end{alignat*}
\section{} %5
\setcounter{subsubsection}{0}
\subsubsection{} %i
\begin{alignat*}{2}
f(x) &=& x^{3} - 12x^{2} + 36x + 1\\
f'(x) &=& 3x^{2} -24x + 36 \\
f''(x) &=& 6x - 24 \\
f'''(x) &=& 6 \\
\intertext{Berechnung Nullstellen erste Ableitung}
f'(x) &=& 0 \\
0 &=& 3x^{2} - 24x + 36 \\
0 &=& x^{2} - 8x + 12 \\
\intertext{pq-Formel}
x_{1,2} &=& 4 \pm \sqrt{4^{2} -12} \\
&=& 4 \pm \sqrt{16 - 12} \\
&=& 4 \pm \sqrt{4} \\
x_{1} &=& 4 + 2 = 6 \\
x_{2} &=& 4-2 = 2 \\
\intertext{Einsetzen in zweite Ableitung}
f''(2) &=& 6 \cdot 2 - 24 \\
&=& -12 \Rightarrow \text{ Maximum} \\
f''(6) &=& 6 \cdot 6 - 24 \\
&=& 12 \Rightarrow \text{ Minimum} \\
\intertext{Einsetzen in Funktion}
f(2) &=& 2^{3} - 12 \cdot 2^{2} + 36 \cdot 2 + 1 \\
&=& 8 - 48 + 72 + 1 \\
&=& 33 \\
f(6) &=& 6^{3} - 12 \cdot 6^{2} + 36 \cdot 6 + 1 \\
&=& 216 - 12 \cdot 36 + 216 + 1 \\
&=& 216 - 432 + 216 + 1 \\
&=& 1 \\
\intertext{Überprüfen der Intervallgrenzen - nur $f(0)$ noch nötig, da $f(6)$ bereits berechnet}
f(0) &=& 0^{3} - 12 \cdot 0^{2} + 36 \cdot 0 + 1 \\
&=& 1
\end{alignat*}
Die Tageshöchsttemperatur ist demnach $33\,^{\circ} \mathrm{C}$.
\subsubsection{} %ii
Wie in i) berechnet, ist die Tagestiefsttemperatur $1\,^{\circ} \mathrm{C}$.
\subsubsection{} %iii
Der Mittelwert wird klassisch so berechnet: Addieren aller Einzelwerte und Teilen der Summe durch deren Anzahl. In diesem Fall entspricht der Flächeninhalt zwischen dem Integral und der x-Achse der Summe aller einzelnen Temperaturen. Nun muss das Ergebnis noch mal $\frac{1}{6}$ genommen werden.\\
\begin{alignat*}{2}
\frac{1}{6} \cdot \int\limits_{0}^{6} (x^{3} - 12x^{2} + 36x +1) \,dx &=& \frac{1}{6} \cdot \left[\frac{1}{4}x^{4} - 4x^{3} + 18x^{2} + x \right]_{0}^{6} \\
&=& \frac{1}{6} \cdot \left(\left(\frac{1}{4} \cdot 6^{4} - 4 \cdot 6^{3} + 18 \cdot 6^{2} + 6 \right) - \left(\frac{1}{4} \cdot 0^{4} - 4 \cdot 0^{3} + 18 \cdot 0^{2} + 0 \right)\right) \\
&=& \frac{1}{6} \cdot \left(\frac{1}{4} \cdot 1296 - 4 \cdot 216 + 18 \cdot 36 + 6 \right)\\
&=& \frac{1}{6} \cdot \left(324 - 864 + 648 + 6 \right) \\
&=& \frac{1}{6} \cdot 114 = 19
\end{alignat*}\\
Die Durchschnittstemperatur des Tages ist demnach $19\,^{\circ} \mathrm{C}$.
\end{document}

View File

@ -0,0 +1,525 @@
\documentclass[10pt,a4paper,oneside,ngerman,numbers=noenddot]{scrartcl}
\usepackage[T1]{fontenc}
\usepackage[utf8]{inputenc}
\usepackage[ngerman]{babel}
\usepackage{amsmath}
\usepackage{amsfonts}
\usepackage{amssymb}
\usepackage{paralist}
\usepackage{gauss}
\usepackage{pgfplots}
\usepackage[locale=DE,exponent-product=\cdot,detect-all]{siunitx}
\usepackage{tikz}
\usetikzlibrary{matrix,fadings,calc,positioning,decorations.pathreplacing,arrows,decorations.markings}
\usepackage{polynom}
\polyset{style=C, div=:,vars=x}
\pgfplotsset{compat=1.8}
\pagenumbering{arabic}
\def\thesection{\arabic{section})}
\def\thesubsection{\alph{subsection})}
\def\thesubsubsection{(\roman{subsubsection})}
\makeatletter
\renewcommand*\env@matrix[1][*\c@MaxMatrixCols c]{%
\hskip -\arraycolsep
\let\@ifnextchar\new@ifnextchar
\array{#1}}
\makeatother
\begin{document}
\author{Jim Martens (6420323)}
\title{Hausaufgaben zum 6. Juni}
\maketitle
\section{} %1
\subsubsection{} %i
\begin{alignat*}{2}
\int \frac{x + 1}{x^{2} - x +6}\,dx &=& \int \frac{x}{x^{2} - x + 6}\,dx + \int \frac{1}{x^{2} -x + 6}\,dx \\
&=& \frac{1}{2}\int \frac{2x - 1 + 1}{x^{2} - x + 6}\,dx + \int \frac{1}{x^{2} -x + 6}\,dx \\
&=& \frac{1}{2}\int \frac{2x - 1}{x^{2} - x + 6}\,dx + \frac{1}{2}\int \frac{1}{x^{2} - x + 6}\,dx + \int \frac{1}{x^{2} -x + 6}\,dx \\
&=& \frac{1}{2} \cdot \ln |x^{2} - x + 6| + \frac{1}{2}\int \frac{1}{x^{2} - x + 6}\,dx + \int \frac{1}{x^{2} -x + 6}\,dx
\end{alignat*}
\begin{alignat*}{2}
x^{2} - x + 6 &=& x^{2} - x + \frac{1}{4} + 6 - \frac{1}{4} \\
&=& \left( x - \frac{1}{2}\right)^{2} + 6 - \frac{1}{4}
\end{alignat*}
\begin{alignat*}{2}
\intertext{Setze $c = 6 - \frac{1}{4}$. Substitution: $t = x - \frac{1}{2}$, $\frac{dt}{dx} = 1$ und $dx = dt$.}
&& \frac{1}{2} \cdot \ln |x^{2} - x + 6| + \frac{1}{2}\int \frac{1}{x^{2} - x + 6}\,dx + \int \frac{1}{x^{2} -x + 6}\,dx\\
&=& \frac{1}{2} \cdot \ln |x^{2} - x + 6| + \frac{1}{2}\int \frac{1}{\left( x - \frac{1}{2}\right)^{2} + c}\,dx + \int \frac{1}{\left( x - \frac{1}{2}\right)^{2} + c}\,dx \\
&=& \frac{1}{2} \cdot \ln |x^{2} - x + 6| + \frac{1}{2}\int \frac{1}{t^{2} + c}\,dt + \int \frac{1}{t^{2} + c}\,dt
\end{alignat*}
\begin{alignat*}{2}
\intertext{Bestimmung Nullstellen des Nennerpolynoms}
0 &=& x^{2} - x + 6 \\
x_{1,2} &=& \frac{1}{2} \pm \sqrt{\left(\frac{1}{2}\right)^{2} - 6} \\
&=& \frac{1}{2} \pm \sqrt{- \frac{23}{4}}
\end{alignat*}
\begin{alignat*}{2}
\intertext{Eine negative Wurzel kann nicht gelöst werden. Demnach gibt es keine Nullstellen. Setzen von $r = \sqrt{c}$. Dann ist $c = r^{2}$.}
&& \frac{1}{2} \cdot \ln |x^{2} - x + 6| + \frac{1}{2}\int \frac{1}{x^{2} - x + 6}\,dx + \int \frac{1}{x^{2} -x + 6}\,dx\\
&=& \frac{1}{2} \cdot \ln |x^{2} - x + 6| + \frac{1}{2}\int \frac{1}{t^{2} + r^{2}}\,dt + \int \frac{1}{t^{2} + r^{2}}\,dt \\
&=& \frac{1}{2} \cdot \ln |x^{2} - x + 6| + \frac{1}{2} \cdot \frac{1}{r} \cdot \arctan \left(\frac{t}{r} \right) + \frac{1}{r} \cdot \arctan \left(\frac{t}{r} \right) \\
&=& \frac{1}{2} \cdot \ln |x^{2} - x + 6| + \frac{1}{2} \cdot \frac{1}{\sqrt{c}} \cdot \arctan \left(\frac{t}{\sqrt{c}} \right) + \frac{1}{\sqrt{c}} \cdot \arctan \left(\frac{t}{\sqrt{c}} \right) \\
\intertext{Resubsitution}
&& \frac{1}{2} \cdot \ln |x^{2} - x + 6| + \frac{1}{2}\int \frac{1}{x^{2} - x + 6}\,dx + \int \frac{1}{x^{2} -x + 6}\,dx \\
&=& \frac{1}{2} \cdot \ln |x^{2} - x + 6| + \frac{1}{2} \cdot \frac{1}{\sqrt{c}} \cdot \arctan \left(\frac{x - \frac{1}{2}}{\sqrt{c}} \right) + \frac{1}{\sqrt{c}} \cdot \arctan \left(\frac{x - \frac{1}{2}}{\sqrt{c}} \right)
\end{alignat*}
\begin{alignat*}{2}
\intertext{Probe:}
&& \left(\frac{1}{2} \cdot \ln |x^{2} - x + 6| + \frac{1}{2} \cdot \frac{1}{\sqrt{c}} \cdot \arctan \left(\frac{x - \frac{1}{2}}{\sqrt{c}} \right) + \frac{1}{\sqrt{c}} \cdot \arctan \left(\frac{x - \frac{1}{2}}{\sqrt{c}} \right)\right)' \\
&=& \left(\frac{1}{2} \cdot \ln |x^{2} - x + 6|\right)' + \left(\frac{1}{2} \cdot \frac{1}{\sqrt{c}} \cdot \arctan \left(\frac{x - \frac{1}{2}}{\sqrt{c}} \right)\right)' + \left(\frac{1}{\sqrt{c}} \cdot \arctan \left(\frac{x - \frac{1}{2}}{\sqrt{c}} \right)\right)' \\
&=& \frac{1}{2} \cdot \frac{2x - 1}{x^{2} - x + 6} + \frac{1}{2} \cdot \frac{1}{\sqrt{c}} \cdot \frac{1}{\left(\frac{x - \frac{1}{2}}{\sqrt{c}} \right)^{2} + 1} \cdot \left(\frac{x - \frac{1}{2}}{\sqrt{c}} \right)' + \frac{1}{\sqrt{c}} \cdot \frac{1}{\left(\frac{x - \frac{1}{2}}{\sqrt{c}} \right)^{2} + 1} \cdot \left(\frac{x - \frac{1}{2}}{\sqrt{c}} \right)' \\
&=& \frac{1}{2} \cdot \frac{2x - 1}{x^{2} - x + 6} + \frac{1}{2} \cdot \frac{1}{\sqrt{c}} \cdot \frac{1}{\sqrt{c}} \cdot \frac{1}{\frac{\left(x - \frac{1}{2}\right)^{2}}{c} + 1} \cdot \left(x - \frac{1}{2}\right)' + \frac{1}{\sqrt{c}} \cdot \frac{1}{\sqrt{c}} \cdot \frac{1}{\frac{\left(x - \frac{1}{2}\right)^{2}}{c} + 1} \cdot \left(x - \frac{1}{2}\right)' \\
&=& \frac{1}{2} \cdot \frac{2x - 1}{x^{2} - x + 6} + \frac{1}{2} \cdot \frac{1}{c} \cdot \frac{1}{\frac{\left(x - \frac{1}{2}\right)^{2}}{c} + 1} + \frac{1}{c} \cdot \frac{1}{\frac{\left(x - \frac{1}{2}\right)^{2}}{c} + 1} \\
&=& \frac{1}{2} \cdot \frac{2x - 1}{x^{2} - x + 6} + \frac{1}{2} \cdot \frac{1}{c\left(\frac{\left(x - \frac{1}{2}\right)^{2}}{c} + 1\right)} + \frac{1}{c\left(\frac{\left(x - \frac{1}{2}\right)^{2}}{c} + 1\right)} \\
&=& \frac{1}{2} \cdot \frac{2x - 1}{x^{2} - x + 6} + \frac{1}{2} \cdot \frac{1}{\left(x - \frac{1}{2}\right)^{2} + c} + \frac{1}{\left(x - \frac{1}{2}\right)^{2} + c} \\
&=& \frac{1}{2} \cdot \frac{2x - 1}{x^{2} - x + 6} + \frac{1}{2} \cdot \frac{1}{x^{2} - x + \frac{1}{4} + 6 - \frac{1}{4}} + \frac{1}{x^{2} - x + \frac{1}{4} + 6 - \frac{1}{4}} \\
&=& \frac{1}{2} \cdot \frac{2x - 1}{x^{2} - x + 6} + \frac{1}{2} \cdot \frac{1}{x^{2} - x + 6} + \frac{1}{x^{2} - x + 6} \\
&=& \frac{1}{2} \cdot \frac{2x - 1 + 1}{x^{2} - x + 6} + \frac{1}{x^{2} - x + 6} \\
&=& \frac{1}{2} \cdot \frac{2x}{x^{2} - x + 6} + \frac{1}{x^{2} - x + 6} \\
&=& \frac{x}{x^{2} - x + 6} + \frac{1}{x^{2} - x + 6} \\
&=& \frac{x + 1}{x^{2} - x + 6}
\end{alignat*}
\subsubsection{} %ii
\begin{alignat*}{2}
0 &=& x^{2} - 4x + 4 \\
x_{1,2} &=& 2 \pm \sqrt{2^{2} - 4} \\
&=& 2 \pm \sqrt{0} \\
x_{1} = x_{2} &=& 2 \\
x^{2} - 4x + 4 &=& (x-2)(x-2) = (x-2)^{2}
\end{alignat*}
\begin{alignat*}{2}
\frac{2x + 1}{x^{2} -4x +4} = \frac{2x + 1}{(x-2)^{2}} &=& \frac{A}{(x-2)^{2}} + \frac{B}{x-2} \\
2x + 1 &=& \frac{A(x-2)^{2}}{(x-2)^{2}} + \frac{B(x-2)^{2}}{x-2} \\
&=& A + B(x-2) \\
&=& A + Bx -2B \\
&=& Bx - 2B + A
\end{alignat*}
\begin{alignat*}{2}
\intertext{Vergleichen der Koeffizienten}
B &=& 2 \\
-2B + A &=& 1 \\
-2 \cdot 2 + A &=& 1 \\
-4 + A &=& 1 \\
A &=& 5
\end{alignat*}
\begin{alignat*}{2}
\intertext{Partialbruchzerlegung:}
\frac{2x+1}{x^{2}-4x+4} &=& \frac{5}{(x-2)^{2}} + \frac{2}{x-2}
\end{alignat*}
\begin{alignat*}{2}
\intertext{Integrieren}
&& \int \frac{2x+1}{x^{2}-4x+4}\,dx \\
&=& \int \left(\frac{5}{(x-2)^{2}} + \frac{2}{x-2}\right)\,dx \\
&=& \int \frac{5}{(x-2)^{2}}\,dx + \int \frac{2}{x-2}\,dx \\
&=& 5 \int (x-2)^{-2}\,dx + 2 \int \frac{1}{x-2}\,dx \\
&=& -5 (x-2)^{-1} + 2 \cdot \ln |x-2|
\end{alignat*}
\begin{alignat*}{2}
\intertext{Probe}
&& \left(-5 (x-2)^{-1} + 2 \cdot \ln |x-2|\right)' \\
&=& \left(-5 (x-2)^{-1}\right)' + \left(2 \cdot \ln |x-2|\right)' \\
&=& 5(x-2)^{-2} + \frac{2}{x-2} \\
&=& \frac{5}{(x-2)^{2}} + \frac{2}{x-2} \\
&=& \frac{5 + 2(x-2)}{(x-2)^{2}} \\
&=& \frac{5 + 2x -4)}{(x-2)^{2}} \\
&=& \frac{2x + 1}{x^{2} -4x +4}
\end{alignat*}
\subsubsection{} %iii
\begin{alignat*}{2}
\int \frac{4x + 1}{x^{2} + 4x +8}\,dx &=& \int \frac{4x}{x^{2} + 4x + 8}\,dx + \int \frac{1}{x^{2} + 4x + 8}\,dx \\
&=& 2\int \frac{2x +4 - 4}{x^{2} + 4x + 8}\,dx + \int \frac{1}{x^{2} + 4x + 8}\,dx \\
&=& 2\int \frac{2x + 4}{x^{2} + 4x + 8}\,dx - 8\int \frac{1}{x^{2} + 4x + 8}\,dx + \int \frac{1}{x^{2} + 4x + 8}\,dx \\
&=& 2 \cdot \ln |x^{2} + 4x + 8| - 8\int \frac{1}{x^{2} + 4x + 8}\,dx + \int \frac{1}{x^{2} + 4x + 8}\,dx
\end{alignat*}
\begin{alignat*}{2}
x^{2} + 4x + 8 &=& x^{2} + 4x + \frac{16}{4} + 8 - \frac{16}{4} \\
&=& (x +2)^{2} + 4
\end{alignat*}
\begin{alignat*}{2}
\intertext{Setze $c = 4$. Substitution: $t = x + 2$, $\frac{dt}{dx} = 1$ und $dx = dt$.}
&& 2 \cdot \ln |x^{2} + 4x + 8| - 8\int \frac{1}{x^{2} + 4x + 8}\,dx + \int \frac{1}{x^{2} + 4x + 8}\,dx\\
&=& 2 \cdot \ln |x^{2} + 4x + 8| -8\int \frac{1}{(x+2)^{2} + c}\,dx + \int \frac{1}{(x+2)^{2} + c}\,dx \\
&=& 2 \cdot \ln |x^{2} + 4x + 8| -8\int \frac{1}{t^{2} + c}\,dt + \int \frac{1}{t^{2} + c}\,dt
\end{alignat*}
\begin{alignat*}{2}
\intertext{Bestimmung Nullstellen des Nennerpolynoms}
0 &=& x^{2} + 4x + 8 \\
x_{1,2} &=& -2 \pm \sqrt{(2)^{2} - 8} \\
&=& -2 \pm \sqrt{-4}
\end{alignat*}
\begin{alignat*}{2}
\intertext{Eine negative Wurzel kann nicht gelöst werden. Demnach gibt es keine Nullstellen. Setzen von $r = \sqrt{c}$. Dann ist $c = r^{2}$.}
&& 2 \cdot \ln |x^{2} + 4x + 8| - 8\int \frac{1}{x^{2} + 4x + 8}\,dx + \int \frac{1}{x^{2} + 4x + 8}\,dx\\
&=& 2 \cdot \ln |x^{2} + 4x + 8| -8\int \frac{1}{t^{2} + r^{2}}\,dt + \int \frac{1}{t^{2} + r^{2}}\,dt \\
&=& 2 \cdot \ln |x^{2} + 4x + 8| -8 \cdot \frac{1}{r} \cdot \arctan \left(\frac{t}{r} \right) + \frac{1}{r} \cdot \arctan \left(\frac{t}{r} \right) \\
&=& 2 \cdot \ln |x^{2} + 4x + 8| -8 \cdot \frac{1}{\sqrt{c}} \cdot \arctan \left(\frac{t}{\sqrt{c}} \right) + \frac{1}{\sqrt{c}} \cdot \arctan \left(\frac{t}{\sqrt{c}} \right) \\
\intertext{Resubsitution}
&& 2 \cdot \ln |x^{2} + 4x + 8| - 8\int \frac{1}{x^{2} + 4x + 8}\,dx + \int \frac{1}{x^{2} + 4x + 8}\,dx \\
&=& 2 \cdot \ln |x^{2} + 4x + 8| -8 \cdot \frac{1}{\sqrt{c}} \cdot \arctan \left(\frac{x+2}{\sqrt{c}} \right) + \frac{1}{\sqrt{c}} \cdot \arctan \left(\frac{x+2}{\sqrt{c}} \right)
\end{alignat*}
\begin{alignat*}{2}
\intertext{Probe:}
&& \left(2 \cdot \ln |x^{2} + 4x + 8| -8 \cdot \frac{1}{\sqrt{c}} \cdot \arctan \left(\frac{x+2}{\sqrt{c}} \right) + \frac{1}{\sqrt{c}} \cdot \arctan \left(\frac{x+2}{\sqrt{c}} \right)\right)' \\
&=& \left(2 \cdot \ln |x^{2} + 4x + 8|\right)' + \left(-6 \cdot \frac{1}{\sqrt{c}} \cdot \arctan \left(\frac{x+2}{\sqrt{c}} \right)\right)' + \left(\frac{1}{\sqrt{c}} \cdot \arctan \left(\frac{x+2}{\sqrt{c}} \right)\right)' \\
&=& 2 \cdot \frac{2x + 4}{x^{2} + 4x + 8} - 8 \cdot \frac{1}{\sqrt{c}} \cdot \frac{1}{\left(\frac{x + 2}{\sqrt{c}} \right)^{2} + 1} \cdot \left(\frac{x + 2}{\sqrt{c}} \right)' + \frac{1}{\sqrt{c}} \cdot \frac{1}{\left(\frac{x + 2}{\sqrt{c}} \right)^{2} + 1} \cdot \left(\frac{x + 2}{\sqrt{c}} \right)' \\
&=& 2 \cdot \frac{2x + 4}{x^{2} + 4x + 8} - 8 \cdot \frac{1}{\sqrt{c}} \cdot \frac{1}{\sqrt{c}} \cdot \frac{1}{\frac{(x + 2)^{2}}{c} + 1} \cdot (x + 2)' + \frac{1}{\sqrt{c}} \cdot \frac{1}{\sqrt{c}} \cdot \frac{1}{\frac{(x + 2)^{2}}{c} + 1} \cdot (x + 2)' \\
&=& 2 \cdot \frac{2x + 4}{x^{2} + 4x + 8} - 8 \cdot \frac{1}{c} \cdot \frac{1}{\frac{(x + 2)^{2}}{c} + 1} + \frac{1}{c} \cdot \frac{1}{\frac{(x + 2)^{2}}{c} + 1} \\
&=& 2 \cdot \frac{2x + 4}{x^{2} + 4x + 8} - 8 \cdot \frac{1}{c\left(\frac{(x + 2)^{2}}{c} + 1\right)} + \frac{1}{c\left(\frac{(x + 2)^{2}}{c} + 1\right)} \\
&=& 2 \cdot \frac{2x + 4}{x^{2} + 4x + 8} - 8 \cdot \frac{1}{(x + 2)^{2} + c} + \frac{1}{(x + 2)^{2} + c} \\
&=& 2 \cdot \frac{2x + 4}{x^{2} + 4x + 8} - 8 \cdot \frac{1}{x^{2} + 4x + 4 + 4} + \frac{1}{x^{2} + 4x + 4 + 4} \\
&=& 2 \cdot \frac{2x + 4}{x^{2} + 4x + 8} - 2\cdot \frac{4}{x^{2} + 4x + 8} + \frac{1}{x^{2} + 4x + 8} \\
&=& 2 \cdot \frac{2x + 4 - 4}{x^{2} + 4x + 8} + \frac{1}{x^{2} + 4x + 8} \\
&=& 2 \cdot \frac{2x}{x^{2} + 4x + 8} + \frac{1}{x^{2} + 4x + 8} \\
&=& \frac{4x + 1}{x^{2} + 4x + 8}
\end{alignat*}
\section{} %2
\subsection{} %a
\begin{alignat*}{2}
f(x) &=& e^{-x} \\
f'(x) &=& -e^{-x} \\
f''(x) &=& e^{-x} \\
f'''(x) &=& -e^{-x} \\
\intertext{Wendepunkte bestimmen}
f''(x) &=& 0 \\
e^{-x} &=& 0 \\
-x &=& \ln 0 \Rightarrow \text{$\ln 0$ ist nicht definiert, daher kann es keine Wendepunkte für $e^{-x}$ geben.}
\end{alignat*}
\begin{alignat*}{2}
g(x) &=& \frac{1}{1+x} \\
g'(x) &=& -(x+1)^{-2} \\
g''(x) &=& 2(x+1)^{-3} \\
g'''(x) &=& -6(x+1)^{-4} \\
\intertext{Wendepunkte bestimmen}
g''(x) &=& 0 \\
2(x+1)^{-3} &=& 0 \\
\frac{2}{(x+1)^{3}} &=& 0 \Rightarrow \text{Die Funktion wird niemals $0$. Daher kann es keine Wendepunkte geben.}
\end{alignat*}
\begin{alignat*}{2}
h(x) &=& \frac{1}{1+x^{2}} \\
h'(x) &=& -(1+x^{2})^{-2} \cdot 2x \\
h''(x) &=& 2(1+x^{2})^{-3} \cdot 2x \cdot 2x -(1+x^{2})^{-2} \cdot 2 \\
&=& 2(1+x^{2})^{-3} \cdot 4x^{2} - 2(1+x^{2})^{-2} \\
h'''(x) &=& -6(1+x^{2})^{-4} \cdot 2x \cdot 4x^{2} + 2(1+x^{2})^{-3} \cdot 8x + 4(1+x^{2})^{-3} \cdot 2x \\
&=& -6(1+x^{2})^{-4} \cdot 8x^{3} + 2(1+x^{2})^{-3} \cdot 8x + 4(1+x^{2})^{-3} \cdot 2x \\
\intertext{Wendepunkte bestimmen}
h''(x) &=& 0 \\
2(1+x^{2})^{-3} \cdot 4x^{2} - 2(1+x^{2})^{-2} &=& 0 \\
\intertext{Es ergibt sich das Ergebnis:}
x = \frac{1}{\sqrt{3}}
\end{alignat*}
\begin{alignat*}{2}
\intertext{Probe}
2(1+\left(\frac{1}{\sqrt{3}}\right)^{2})^{-3} \cdot 4\left(\frac{1}{\sqrt{3}}\right)^{2} - 2(1+\left(\frac{1}{\sqrt{3}}\right)^{2})^{-2} &=& 0 \\
2(1+\frac{1}{3})^{-3} \cdot 4 \cdot \frac{1}{3} - 2(1+\frac{1}{3})^{-2} &=& 0 \\
\frac{2 \cdot \frac{4}{3}}{\left(\frac{4}{3}\right)^{3}} - \frac{2}{\left(\frac{4}{3}\right)^{2}} &=& 0 \\
\frac{2 \cdot \frac{4}{3}}{\frac{64}{27}} - \frac{2}{\frac{16}{9}} &=& 0 \\
2 \cdot \frac{4}{3} \cdot \frac{27}{64} - 2 \cdot \frac{9}{16} &=& 0 \\
\frac{4}{3} \cdot \frac{54}{64} - \frac{18}{16} &=& 0 \\
\frac{4}{3} \cdot \frac{27}{32} - \frac{9}{8} &=& 0 \\
\intertext{27 mit 3 kürzen und 32 mit 4 kürzen}
1 \cdot \frac{9}{8} - \frac{9}{8} &=& 0 \\
\frac{9 - 9}{8} &=& 0 \\
0 &=& 0
\end{alignat*}
\begin{tikzpicture}[>=stealth]
\begin{axis}[
ymin=0,ymax=5,
x=1cm,
y=1cm,
axis x line=middle,
axis y line=middle,
axis line style=->,
xlabel={$x$},
ylabel={$y$},
xmin=0,xmax=5
]
\addplot[no marks, black, -] expression[domain=0:5,samples=100]{e^(-x)} node[pos=0.65,anchor=north]{};
\node at (axis cs: 1,0.8) {f};
\end{axis}
\end{tikzpicture}
\begin{tikzpicture}[>=stealth]
\begin{axis}[
ymin=0,ymax=5,
x=1cm,
y=1cm,
axis x line=middle,
axis y line=middle,
axis line style=->,
xlabel={$x$},
ylabel={$y$},
xmin=0,xmax=5
]
\addplot[no marks, black, -] expression[domain=0:5,samples=100]{1/(1+x)} node[pos=0.65,anchor=north]{};
\node at (axis cs: 1,0.8) {g};
\end{axis}
\end{tikzpicture}\\
\begin{tikzpicture}[>=stealth]
\begin{axis}[
ymin=0,ymax=5,
x=1cm,
y=1cm,
axis x line=middle,
axis y line=middle,
axis line style=->,
xlabel={$x$},
ylabel={$y$},
xmin=0,xmax=5
]
\addplot[no marks, black, -] expression[domain=0:5,samples=100]{1/(1+x^2)} node[pos=0.65,anchor=north]{};
\node at (axis cs: 2,0.8) {h};
\draw (axis cs:0.577350269,0.75) circle (2pt);
\end{axis}
\end{tikzpicture}
\subsection{} %b
\begin{alignat*}{2}
\intertext{Integrieren von f}
\int\limits_{0}^{b} e^{-x}\,dx &=& \left[ -e^{-x}\right]_{0}^{b} \\
&=& -e^{-b} + e^{0} \\
&=& -\frac{1}{e^{b}} + 1 \rightarrow 1 \text{ für } b \rightarrow \infty \\
&\Longrightarrow & \lim\limits_{b \rightarrow \infty} \int\limits_{0}^{b} e^{-x}\,dx = 1
\end{alignat*}
\begin{alignat*}{2}
\intertext{Integrieren von g}
\int\limits_{0}^{b} \frac{1}{1+x}\,dx &=& \left[\ln |1+x|\right]_{0}^{b} \\
&=& \ln |1 + b| - \ln |1 + 0| \\
&=& \ln |1 + b| \rightarrow \infty \text{ für } b \rightarrow \infty \\
&\Longrightarrow & \lim\limits_{b \rightarrow \infty} \int\limits_{0}^{b} \frac{1}{1+x}\,dx = \infty
\end{alignat*}
\begin{alignat*}{2}
\intertext{Integrieren von h}
\int\limits_{0}^{b} \frac{1}{1+x^{2}} \,dx &=& \left[ \arctan x \right]_{0}^{b} \\
&=& \arctan b - \arctan 0 \\
&=& \arctan b \rightarrow \frac{\pi}{2} \text{ für } b \rightarrow \infty \\
&\Longrightarrow & \lim\limits_{b \rightarrow \infty} \int\limits_{0}^{b} \frac{1}{1+x^{2}}\,dx = \frac{\pi}{2}
\end{alignat*}
\subsection{} %c
\begin{tikzpicture}[>=stealth]
\begin{axis}[
ymin=0,ymax=2,
x=2cm,
y=2cm,
axis x line=middle,
axis y line=middle,
axis line style=->,
xlabel={$x$},
ylabel={$y$},
xmin=-1,xmax=1
]
\addplot[no marks, black, -] expression[domain=-1:1,samples=100]{1/sqrt(1-x^2)} node[pos=0.65,anchor=north]{};
\node at (axis cs: 2,0.8) {f};
\end{axis}
\end{tikzpicture}
\begin{alignat*}{2}
\intertext{Integrieren von f}
\int\limits_{-1}^{1} \frac{1}{\sqrt{1-x^{2}}}\,dx &=& \left[\arcsin x\right]_{-1}^{1} \\
&=& \arcsin 1 - \arcsin (-1) \\
&=& \frac{\pi}{2} + \frac{\pi}{2}\\
&=& \pi
\end{alignat*}
\section{} %3
\begin{alignat*}{2}
\intertext{$n=4$}
\int\limits_{0}^{1} \sin x \,dx &\approx & \frac{1}{8}\left(\sin(0) + 2\sin\left(\frac{1}{4}\right) + 2\sin\left(\frac{1}{2}\right) + 2\sin\left(\frac{3}{4}\right) + \sin(1)\right) \\
&\approx & 0.4573009376
\end{alignat*}
\begin{alignat*}{2}
\intertext{$n=5$}
\int\limits_{0}^{1} \sin x \,dx &\approx & \frac{1}{10}\left(\sin(0) + 2\sin\left(\frac{1}{5}\right) +
2\sin\left(\frac{2}{5}\right) + 2\sin\left(\frac{3}{5}\right) + 2\sin\left(\frac{4}{5}\right) + \sin(1)\right) \\
&\approx & 0.4581643460
\end{alignat*}
\begin{alignat*}{2}
\intertext{$n=10$}
\int\limits_{0}^{1} \sin x \,dx &\approx \begin{split} \frac{1}{20}\left(\sin(0) + 2\sin\left(\frac{1}{10}\right) +
2\sin\left(\frac{1}{5}\right) + 2\sin\left(\frac{3}{10}\right) + 2\sin\left(\frac{2}{5}\right) +
2\sin\left(\frac{1}{2}\right)\right.\\ +
\left. 2\sin\left(\frac{3}{5}\right) + 2\sin\left(\frac{7}{10}\right) + 2\sin\left(\frac{4}{5}\right) +
2\sin\left(\frac{9}{10}\right) + \sin(1)\right)\end{split} \\
&\approx & 0.4593145489
\end{alignat*}
\section{} %4
\subsection{} %a
\begin{alignat*}{2}
f(1) &=& 10 \cdot e^{-\frac{2}{5}} \\
&\approx & 6.7032 \\
f(2) &=& 10 \cdot 2 \cdot e^{-frac{2}{5}\cdot 2} \\
&=& 20 \cdot e^{-\frac{4}{5}} \\
&\approx & 8.9866 \\
f(6) &=& 10 \cdot 6 \cdot e^{-\frac{2}{5}\cdot 6}\\
&=& 60 \cdot e^{-\frac{12}{5}} \\
&\approx & 5.4431 \\
f(12) &=& 10 \cdot 12 \cdot e^{-\frac{2}{5}\cdot 12} \\
&=& 120 \cdot e^{-\frac{24}{5}} \\
&\approx & 0.9876 \\
f(24) &=& 10 \cdot 24 \cdot e^{-\frac{2}{5}\cdot 24} \\
&=& 240 \cdot e^{-\frac{48}{5}} \\
&=& 0.0163
\end{alignat*}
\subsection{} %b
\begin{alignat*}{2}
f'(t) &=& 10 \cdot e^{-\frac{2}{5}t} - 10t \cdot e^{-\frac{2}{5}t} \cdot \frac{2}{5} \\
&=& 10 \cdot e^{-\frac{2}{5}t} - \frac{20}{5}t \cdot e^{-\frac{2}{5}t} \\
&=& 10 \cdot e^{-\frac{2}{5}t} - 4t \cdot e^{-\frac{2}{5}t} \\
&=& (10 - 4t) \cdot e^{-\frac{2}{5}t} \\
&=& \frac{10-4t}{e^{\frac{2}{5}t}} \\
f''(x) &=& \left((10 - 4t) \cdot e^{-\frac{2}{5}t}\right)' \\
&=& (10 - 4t)' \cdot e^{-\frac{2}{5}t} + (10 - 4t) \cdot \left(e^{-\frac{2}{5}t}\right)' \\
&=& -4 \cdot e^{-\frac{2}{5}t} + (10 - 4t) \cdot e^{-\frac{2}{5}t} \cdot \left(-\frac{2}{5}\right) \\
\intertext{Berechnung der Nullstelle(n) von $f'(x)$}
f'(x) &=& 0 \\
\intertext{$f'(x)$ mit Zähler ersetzen, da der über Nullstelle bestimmt}
10-4t &=& 0 \\
10 &=& 4t \\
\frac{5}{2} &=& t \\
\intertext{Einsetzen in $f(x)$}
f\left(\frac{5}{2}\right) &=& 10 \cdot \frac{5}{2} \cdot e^{-\frac{2}{5} \cdot \frac{5}{2}} \\
&=& 25 \cdot e^{-1} \\
&\approx & 9.1970
\end{alignat*}
\subsection{} %c
\begin{alignat*}{2}
\int 10t \cdot e^{-\frac{2}{5}t}\,dx &=& -\frac{5}{2}\cdot e^{-\frac{2}{5}t} \cdot 10t - \int -\frac{5}{2} \cdot e^{-\frac{2}{5}t} \cdot 10\,dx \\
&=& -\frac{5}{2}\cdot e^{-\frac{2}{5}t} \cdot 10t + 25 \int e^{-\frac{2}{5}t}\,dx \\
&=& -\frac{5}{2}\cdot e^{-\frac{2}{5}t} \cdot 10t - \frac{5}{2} \cdot e^{-\frac{2}{5}t} \cdot 25 \\
&=& -\frac{5}{2}\cdot e^{-\frac{2}{5}t} \cdot \left( 10t + 25\right) \\
\frac{1}{6} \int\limits_{0}^{6} 10t \cdot e^{-\frac{2}{5}t}\,dx &=& \frac{1}{6} \cdot \left[-\frac{5}{2}\cdot e^{-\frac{2}{5}t} \cdot (10t + 25)\right]_{0}^{6} \\
&=& \frac{1}{6}\left(-\frac{5}{2}\cdot e^{-\frac{2}{5} \cdot 6} \cdot ( 10 \cdot 6 + 25) - \left(-\frac{5}{2}\cdot e^{-\frac{2}{5} \cdot 0} \cdot ( 10 \cdot 0 + 25)\right)\right) \\
&=& \frac{1}{6}\left(-\frac{5}{2}\cdot e^{-\frac{12}{5}} \cdot 85 + \frac{5}{2}\cdot e^{0} \cdot 25\right) \\
&=& \frac{1}{6}\left(-\frac{425}{2}\cdot e^{-\frac{12}{5}} + \frac{125}{2}\right) \\
&\approx & 7.2037
\end{alignat*}
\subsection{} %d
\begin{alignat*}{2}
\intertext{Die Stammfunktion wurde bereits in c) berechnet. Daher setze ich direkt die Werte entsprechend ein.}
\frac{1}{6} \int\limits_{6}^{12} 10t \cdot e^{-\frac{2}{5}t}\,dx &=& \frac{1}{6} \cdot \left[-\frac{5}{2}\cdot e^{-\frac{2}{5}t} \cdot (10t + 25)\right]_{6}^{12} \\
&=& \frac{1}{6}\left(-\frac{5}{2}\cdot e^{-\frac{2}{5} \cdot 12} \cdot ( 10 \cdot 12 + 25) - \left(-\frac{5}{2}\cdot e^{-\frac{2}{5} \cdot 6} \cdot ( 10 \cdot 6 + 25)\right)\right) \\
&=& \frac{1}{6}\left(-\frac{5}{2}\cdot e^{-\frac{24}{5}} \cdot 145 + \frac{5}{2}\cdot e^{-\frac{12}{5}} \cdot 85\right) \\
&=& \frac{1}{6}\left(-\frac{725}{2}\cdot e^{-\frac{24}{5}} + \frac{425}{2}\cdot e^{-\frac{12}{5}}\right) \\
&\approx & 2.7157
\end{alignat*}
\subsection{} %e
\begin{alignat*}{2}
f'(t) &=& \frac{10-4t}{e^{\frac{2}{5}t}} \\
f''(t) &=& -4 \cdot e^{-\frac{2}{5}t} + (10 - 4t) \cdot e^{-\frac{2}{5}t} \cdot \left(-\frac{2}{5}\right) \\
&=& e^{-\frac{2}{5}t} \cdot \left(-4 + (10 - 4t) \cdot \left(-\frac{2}{5}\right) \right) \\
&=& e^{-\frac{2}{5}t} \cdot \left(-4 - 4 + \frac{8}{5}t \right) \\
&=& e^{-\frac{2}{5}t} \cdot \left(-8 + \frac{8}{5}t \right) \\
&=& \frac{-8 + \frac{8}{5}t}{ e^{-\frac{2}{5}t}} \\
f'''(t) &=& \left(e^{-\frac{2}{5}t} \cdot \left(-8 + \frac{8}{5}t \right)\right)' \\
&=& e^{-\frac{2}{5}t} \cdot \left(-\frac{2}{5} \right) \cdot \left(-8 + \frac{8}{5}t \right) + e^{-\frac{2}{5}t} \cdot \left(-8 + \frac{8}{5}t \right)' \\
&=& e^{-\frac{2}{5}t} \cdot \left(\frac{16}{5} - \frac{16}{25}t \right) + e^{-\frac{2}{5}t} \cdot \left(\frac{8}{5}\right) \\
&=& e^{-\frac{2}{5}t} \cdot \left(\frac{16}{5} - \frac{16}{25}t + \frac{8}{5}\right) \\
&=& e^{-\frac{2}{5}t} \cdot \left(\frac{24}{5} - \frac{16}{25}t\right) \\
\intertext{Nullstelle(n) von $f''(x)$ berechnen}
f''(x) &=& 0 \\
\intertext{$f''(x)$ mit dem Zähler ersetzen, da der für die Nullstelle zuständig ist}
-8 + \frac{8}{5}t &=& 0 \\
\frac{8}{5} &=& 8 \\
t &=& 5 \\
f'''(5) &=& e^{-\frac{2}{5} \cdot 5} \cdot \left(\frac{24}{5} - \frac{16}{25} \cdot 5\right) \\
&=& e^{-2} \cdot \left(\frac{24}{5} - \frac{16}{5}\right) \\
&=& e^{-2} \cdot \frac{8}{5} > 0 \Rightarrow \text{Minimum} \\
f(5) &=& 10 \cdot 5 \cdot e^{-\frac{2}{5} \cdot 5} \\
&=& 50 \cdot e^{-2} \\
&\approx & 6.7668
\end{alignat*}
\begin{tikzpicture}[>=stealth]
\begin{axis}[
ymin=0,ymax=10,
x=1em,
y=1em,
axis x line=middle,
axis y line=middle,
axis line style=->,
xlabel={$t$},
ylabel={$f(t)$},
xmin=0,xmax=24
]
\addplot[no marks, black, -] expression[domain=0:24,samples=100]{10*x*e^(-(2/5)*x)} node[pos=0.65,anchor=north]{};
\draw (axis cs:5,6.766764162) circle (2pt);
\end{axis}
\end{tikzpicture}
\section{} %5
\subsection{} %a
\begin{alignat*}{2}
h(x) &=& (x^{2}+1)^{\cos(x)} \\
&=& e^{\ln \left((x^{2}+1)^{\cos(x)}\right)}\\
&=& e^{\cos(x) \cdot \ln (x^{2}+1)} \\
h'(x) &=& e^{\cos(x) \cdot \ln (x^{2}+1)} \cdot \left(\cos(x) \cdot \ln (x^{2}+1)\right)' \\
&=& e^{\cos(x) \cdot \ln (x^{2}+1)} \cdot \left((\cos(x))' \cdot \ln (x^{2}+1) + \cos(x) \cdot (\ln (x^{2}+1))'\right) \\
&=& e^{\cos(x) \cdot \ln (x^{2}+1)} \cdot \left(-\sin(x) \cdot \ln (x^{2}+1) + \cos(x) \cdot \frac{(x^{2}+1)'}{x^{2}+1}\right) \\
&=& e^{\cos(x) \cdot \ln (x^{2}+1)} \cdot \left(-\sin(x) \cdot \ln (x^{2}+1) + \cos(x) \cdot \frac{2x}{x^{2}+1}\right) \\
&=& (x^{2}+1)^{\cos(x)} \cdot \left(-\sin(x) \cdot \ln (x^{2}+1) + \cos(x) \cdot \frac{2x}{x^{2}+1}\right)
\end{alignat*}
\subsection{} %b
\begin{alignat*}{2}
t &=& \sqrt{\frac{x}{4} + 3} \\
t^{2} &=& \frac{x}{4} + 3 \\
t^{2} - 3 &=& \frac{x}{4} \\
4t^{2} - 12 &=& x \\
8t &=& \frac{dx}{dt} \\
8t\,dt &=& dx
\end{alignat*}
\begin{alignat*}{2}
\int \sin \left(\sqrt{\frac{x}{4} + 3}\right) &\Rightarrow & \int \sin (t) \cdot 8t\,dt \\
\int \sin (t) \cdot 8t\,dt &=& -\cos(t) \cdot 8t - \int (-\cos (t) \cdot 8)\,dt \\
&=& -\cos(t) \cdot 8t + 8\int \cos(t)\,dt \\
&=& -\cos(t) \cdot 8t + 8 \cdot \sin(t) \\
\int \sin \left(\sqrt{\frac{x}{4} + 3}\right) &=& -\cos \left(\sqrt{\frac{x}{4} + 3} \right) \cdot 8 \cdot \left(\sqrt{\frac{x}{4} + 3} \right) + 8 \cdot \sin \left(\sqrt{\frac{x}{4} + 3}\right)
\end{alignat*}
\subsection{} %c
\begin{alignat*}{2}
g(x) = x^{2}-x-6 &=& 0 \\
x_{1,2} &=& \frac{1}{2} \pm \sqrt{\frac{1}{4} + 6 } \\
&=& \frac{1}{2} \pm \sqrt{\frac{25}{4}} \\
&=& \frac{1}{2} \pm \frac{5}{2} \\
x_{1} &=& \frac{6}{2} = 3 \\
x_{2} &=& -\frac{4}{2} = -2 \\
g(x) = x^{2}-x-6 &=& (x-3)(x+2)
\end{alignat*}
\begin{alignat*}{2}
\frac{3x+2}{x^{2}-x-6} = \frac{1}{(x-3)(x+2)} &=& \frac{A}{x-3} + \frac{B}{x+2} \\
&=& \frac{A(x+2) + B(x-3)}{(x-3)(x+2)} \\
&=& \frac{Ax + Bx + 2A - 3B}{(x-3)(x+2)} \\
&=& \frac{(A+B)x + 2A - 3B}{(x-3)(x+2)} \\
A + B &=& 3 \\
2A - 3B &=& 2 \\
A &=& 3 - B \\
2(3-B) - 3B &=& 2 \\
6 - 2B - 3B &=& 2 \\
-5B &=& -4 \\
5B &=& 4 \\
B &=& \frac{4}{5}\\
A &=& 3 - \frac{4}{5} \\
&=& \frac{11}{5} \\
\frac{3x+2}{x^{2}-x-6} &=& \frac{11}{5} \cdot \frac{1}{x-3} + \frac{4}{5} \cdot \frac{1}{x+2}
\end{alignat*}
\begin{alignat*}{2}
\int \frac{3x+2}{x^{2}-x-6}\,dx &=& \int \frac{11}{5} \cdot \frac{1}{x-3}\,dx + \int \frac{4}{5} \cdot \frac{1}{x+2}\,dx \\
&=& \frac{11}{5} \int \frac{1}{x-3}\,dx + \frac{4}{5} \int \frac{1}{x+2}\,dx \\
&=& \frac{11}{5} \cdot \ln |x-3| + \frac{4}{5} \cdot \ln |x+2|
\end{alignat*}
\subsection{} %d
\begin{alignat*}{2}
g(x) = x^{2} + 8x + 16 &=& (x+4)^{2} \\
\frac{x+1}{(x+4)^{2}} &=& \frac{A}{(x+4)^{2}} + \frac{B}{x+4} \\
&=& \frac{A + B(x+4)}{(x+4)^{2}} \\
&=& \frac{A + Bx + 4B}{(x+4)^{2}} \\
&=& \frac{Bx + A + 4B}{(x+4)^{2}} \\
\Rightarrow B &=&1 \\
A + 4B &=& 1 \\
A &=& -4B + 1 \\
A &=& -4 + 1 \\
A &=& -3 \\
\frac{x+1}{x^{2}+8x+16} &=& -\frac{3}{(x+4)^{2}} + \frac{1}{x+4}
\end{alignat*}
\begin{alignat*}{2}
\int \frac{x+1}{x^{2}+8x+16} \,dx &=& \int -\frac{3}{(x+4)^{2}}\,dx + \int \frac{1}{x+4}\,dx \\
&=& -3\int (x+4)^{-2}\,dx + \int \frac{1}{x+4}\,dx \\
&=& -3\left(-(x+4)^{-1}\right)+ \ln |x+4| \\
&=& 3(x+4)^{-1} + \ln |x+4|
\end{alignat*}
\end{document}

View File

@ -0,0 +1,297 @@
\documentclass[10pt,a4paper,oneside,ngerman,numbers=noenddot]{scrartcl}
\usepackage[T1]{fontenc}
\usepackage[utf8]{inputenc}
\usepackage[ngerman]{babel}
\usepackage{amsmath}
\usepackage{amsfonts}
\usepackage{amssymb}
\usepackage{paralist}
\usepackage{gauss}
\usepackage{pgfplots}
\usepackage[locale=DE,exponent-product=\cdot,detect-all]{siunitx}
\usepackage{tikz}
\usetikzlibrary{matrix,fadings,calc,positioning,decorations.pathreplacing,arrows,decorations.markings}
\usepackage{polynom}
\polyset{style=C, div=:,vars=x}
\pgfplotsset{compat=1.8}
\pagenumbering{arabic}
\def\thesection{\arabic{section})}
\def\thesubsection{\alph{subsection})}
\def\thesubsubsection{(\roman{subsubsection})}
\makeatletter
\renewcommand*\env@matrix[1][*\c@MaxMatrixCols c]{%
\hskip -\arraycolsep
\let\@ifnextchar\new@ifnextchar
\array{#1}}
\makeatother
\begin{document}
\author{Jim Martens (6420323)}
\title{Hausaufgaben zum 13. Juni}
\maketitle
\section{} %1
\subsection{} %a
\begin{alignat*}{2}
\intertext{Anwenden der Limes-Version des Wurzelkriteriums}
\lim\limits_{i \rightarrow \infty} \sqrt[i]{\left| \frac{i}{2^{i}}\right|} &=& \lim\limits_{i \rightarrow \infty} \frac{\sqrt[i]{\left| i \right|}}{\sqrt[i]{\left| 2^{i}\right|}} \\
&=& \lim\limits_{i \rightarrow \infty} \frac{\sqrt[i]{|i|}}{2} \\
\intertext{Anwenden, dass $\sqrt[i]{|i|} \rightarrow 1$ für $i \rightarrow \infty$ gilt}
&=& \frac{1}{2} < 1 \Rightarrow \text{ Konvergenz}
\end{alignat*}
\subsection{} %b
\begin{alignat*}{2}
\intertext{Anwenden der Limes-Version des Quotientenkriteriums}
\lim\limits_{i \rightarrow \infty} \left|\frac{\frac{(-1)^{i+1} \cdot (i+1)!}{(i+1)^{i+1}}}{\frac{(-1)^{i} \cdot i!}{i^{i}}} \right| &=& \lim\limits_{i \rightarrow \infty} \left| \frac{(-1)^{i+1} \cdot (i+1)! \cdot i^{i}}{(i+1)^{i+1} \cdot (-1)^{i} \cdot i!}\right| \\
&=& \lim\limits_{i \rightarrow \infty} \left| \frac{(-1)^{i+1} \cdot (i+1)! \cdot i^{i}}{(-1)^{i} \cdot i! \cdot (i+1)^{i+1}}\right| \\
\intertext{Kürzen}
&=& \lim\limits_{i \rightarrow \infty} \left| \frac{(-1) \cdot (i+1) \cdot i^{i}}{(i+1)^{i+1}}\right| \\
\intertext{Kürzen}
&=& \lim\limits_{i \rightarrow \infty} \left| \frac{(-1) \cdot i^{i}}{(i+1)^{i}}\right| \\
\intertext{Ausklammern von $i^{i}$ im Nenner, Nullfolgen werden durch $...$ symbolisiert}
&=& \lim\limits_{i \rightarrow \infty} \left| \frac{(-1) \cdot i^{i}}{(1 + ...) \cdot i^{i}}\right| \\
&=& \lim\limits_{i \rightarrow \infty} \left| \frac{-1}{(1 + ...)}\right| \\
&=& \left|\frac{-1}{1}\right| = 1 \Rightarrow \text{ Keine Aussage möglich}
\end{alignat*}
\subsection{} %c
\subsubsection{} %i
\begin{alignat*}{2}
\intertext{Es sei $x \in \mathbb{R}$ eine beliebig fest gewählte Zahl.}
\lim\limits_{i \rightarrow \infty} \left|\frac{(i+1)^{2}2^{i+1}x^{i+1}}{i^{2}2^{i}x^{i}} \right| &=& \lim\limits_{i \rightarrow \infty} \left|\frac{(i+1)^{2} \cdot 2x}{i^{2}} \right| \\
&=& \lim\limits_{i \rightarrow \infty} \left|\frac{(i+1)^{2}}{i^{2}} \cdot 2x \right| \\
&=& \lim\limits_{i \rightarrow \infty} \left(\left|\frac{(i+1)^{2}}{i^{2}}\right| \cdot |2x| \right) \\
\intertext{Anwenden, dass $x$ fest gewählt ist}
&=& 2|x| \cdot \lim\limits_{i \rightarrow \infty} \left|\frac{(i+1)^{2}}{i^{2}}\right| \\
&=& 2|x| \cdot \lim\limits_{i \rightarrow \infty} \left|\frac{i^{2} + 2i + 1}{i^{2}}\right| \\
\intertext{Ausklammern von $i^{2}$ im Zähler und Nenner, anschließend kürzen}
&=& 2|x| \cdot \lim\limits_{i \rightarrow \infty} \left|\frac{1 + \frac{2}{i} + \frac{1}{i^{2}}}{1}\right| \\
&=& 2|x| \cdot 1 = 2|x|
\end{alignat*}
\begin{alignat*}{2}
2|x| < 1 &\Longleftrightarrow & |x| < \frac{1}{2} \\
2|x| > 1 &\Longleftrightarrow & |x| > \frac{1}{2}
\end{alignat*}
Demzufolge liegt Konvergenz vor, falls $|x| < \frac{1}{2}$ gilt; Divergenz liegt vor, falls $|x| > \frac{1}{2}$ gilt. Also gilt $R = \frac{1}{2}$.
\subsubsection{} %ii
\begin{alignat*}{2}
\intertext{Es sei $x \in \mathbb{R}$ eine beliebig fest gewählte Zahl.}
\lim\limits_{i \rightarrow \infty} \sqrt[i]{\left| i^{2} \cdot 2^{i} \cdot x^{i} \right|} &=& \lim\limits_{i \rightarrow \infty} \left( \sqrt[i]{\left| i^{2} \right|} \cdot \sqrt[i]{\left| 2^{i} \right|} \cdot \sqrt[i]{\left| x^{i} \right|} \right) \\
&=& \lim\limits_{i \rightarrow \infty} \left( \sqrt[i]{\left| i^{2} \right|} \cdot |2| \cdot |x| \right) \\
\intertext{Anwenden, dass $x$ fest gewählt ist}
&=& 2 \cdot |x| \cdot \lim\limits_{i \rightarrow \infty} \sqrt[i]{\left| i^{2} \right|} \\
&=& 2 \cdot |x| \cdot \lim\limits_{i \rightarrow \infty} \sqrt[i]{|i|} \cdot \lim\limits_{i \rightarrow \infty} \sqrt[i]{|i|} \\
\intertext{Anwenden, dass $\sqrt[i]{|i|} \rightarrow 1$ für $i \rightarrow \infty$ gilt}
&=& 2 \cdot |x| \cdot 1 \cdot 1 = 2|x|
\end{alignat*}
\begin{alignat*}{2}
2|x| < 1 &\Longleftrightarrow & |x| < \frac{1}{2} \\
2|x| > 1 &\Longleftrightarrow & |x| > \frac{1}{2}
\end{alignat*}
Demzufolge liegt Konvergenz vor, falls $|x| < \frac{1}{2}$ gilt; Divergenz liegt vor, falls $|x| > \frac{1}{2}$ gilt. Also gilt $R = \frac{1}{2}$.
\section{} %2
\setcounter{subsubsection}{0}
\subsubsection{} %i
\begin{alignat*}{2}
\lim\limits_{i \rightarrow \infty} \left|\frac{\frac{-1}{2^{i+2}}}{\frac{-1}{2^{i+1}}} \right| &=& \lim\limits_{i \rightarrow \infty} \left|\frac{(-1) \cdot 2^{i+1}}{2^{i+2} \cdot (-1)} \right| \\
\intertext{Kürzen}
&=& \lim\limits_{i \rightarrow \infty} \left|\frac{1}{2} \right| \\
&=& \frac{1}{2} < 1 \Rightarrow \text{ Konvergenz}
\end{alignat*}
\begin{alignat*}{2}
\intertext{Berechnen des Grenzwertes}
\sum\limits_{i=1}^{\infty} \frac{-1}{2^{i+1}} &=& - \sum\limits_{i=1}^{\infty} \frac{1}{2^{i+1}} \\
&=& - \sum\limits_{i=1}^{\infty} \frac{1^{i+1}}{2^{i+1}} \\
&=& - \sum\limits_{i=1}^{\infty} \left(\frac{1}{2}\right)^{i+1} \\
&=& - \sum\limits_{i=2}^{\infty} \left(\frac{1}{2}\right)^{i} \\
&=& -\left( \sum\limits_{i=0}^{\infty} \left(\frac{1}{2}\right)^{i} - \left(\frac{1}{2}\right)^{0} - \left(\frac{1}{2}\right)^{1} \right) \\
&=& -\left( \frac{1}{1 - \frac{1}{2}} - 1 - \frac{1}{2} \right) \\
&=& -\left( 2 - 1 - \frac{1}{2} \right) \\
&=& -\left( \frac{1}{2} \right)
\end{alignat*}
\subsubsection{} %ii
\begin{alignat*}{2}
\sum\limits_{i=1}^{\infty} \frac{(-1)^{i} \cdot i}{2(i+1)} &=& \sum\limits_{i=0}^{\infty} \frac{(-1)^{i} \cdot i}{2(i+1)} - \frac{(-1)^{0} \cdot 0}{2(0+1)} \\
&=& \sum\limits_{i=0}^{\infty} \frac{(-1)^{i} \cdot i}{2(i+1)} - 0 \\
&=& \sum\limits_{i=0}^{\infty} (-1)^{i} \cdot \frac{i}{2(i+1)} \\
\intertext{Die Glieder der Reihe werden betragsmäßig immer größer und bilden daher keine Nullfolge. Deswegen divergiert die Reihe.}
a_{0} &=& 0 \\
a_{1} &=& \frac{1}{4} \\
a_{2} &=& \frac{1}{3} = \frac{8}{24} \\
a_{3} &=& \frac{3}{8} = \frac{9}{24} = \frac{15}{40}\\
a_{4} &=& \frac{2}{5} = \frac{16}{40}
\end{alignat*}
\subsubsection{} %iii
\begin{alignat*}{2}
\sum\limits_{i=1}^{\infty} \frac{1}{2(i+1)} &=& \frac{1}{2} \cdot \sum\limits_{i=2}^{\infty} \frac{1}{i} \\
&=& \frac{1}{2} \cdot \left( \sum\limits_{i=1}^{\infty} \frac{1}{i} - 1\right)
\intertext{Die Harmonische Reihe divergiert. Damit divergiert auch jede Reihe, die durch Hinzufügen, Weglassen oder Abändern endlich vieler Glieder entsteht.
Daher divergiert diese Reihe.}
\end{alignat*}
\subsubsection{} %iv
\begin{alignat*}{2}
\sum\limits_{i=1}^{\infty} \frac{(-1)^{i+1}}{2^{i}} &=& \sum\limits_{i=0}^{\infty} \frac{(-1)^{i+1}}{2^{i}} - \frac{(-1)^{0+1}}{2^{0}} \\
&=& \sum\limits_{i=0}^{\infty} \frac{(-1)^{i+1}}{2^{i}} - \frac{-1}{1} \\
&=& \sum\limits_{i=0}^{\infty} \frac{(-1)^{i+1}}{2^{i}} + 1
\end{alignat*}
\begin{alignat*}{2}
\lim\limits_{i \rightarrow \infty} \sqrt[i]{\left|\frac{(-1)^{i+1}}{2^{i}} \right|} &=& \lim\limits_{i \rightarrow \infty} \frac{\sqrt[i]{\left|(-1)^{i+1}\right|}}{\sqrt[i]{|2^{i}|}} \\
&=& \lim\limits_{i \rightarrow \infty} \frac{\sqrt[i]{|(-1)^{i}| \cdot |(-1)|}}{2} \\
&=& \lim\limits_{i \rightarrow \infty} \frac{\sqrt[i]{|(-1)^{i}|} \cdot \sqrt[i]{|-1|}}{2} \\
&=& \frac{1}{2} < 1 \Rightarrow \text{ Konvergenz}
\end{alignat*}
\begin{alignat*}{2}
\sum\limits_{i=0}^{\infty} \frac{(-1)^{i+1}}{2^{i}} + 1 &=& \sum\limits_{i=0}^{\infty} (-1) \cdot \frac{(-1)^{i}}{2^{i}} + 1 \\
&=& \sum\limits_{i=0}^{\infty} (-1) \cdot \left(-\frac{1}{2}\right)^{i} + 1 \\
&=& -\sum\limits_{i=0}^{\infty} \left(-\frac{1}{2}\right)^{i} + 1 \\
&=& -\left(\frac{1}{1 + \frac{1}{2}} \right) + 1 \\
&=& -\left(\frac{1}{\frac{3}{2}} \right) + 1 \\
&=& -\frac{2}{3} + 1 \\
&=& \frac{1}{3}
\end{alignat*}
\subsubsection{} %v
\begin{alignat*}{2}
\sum\limits_{i=0}^{\infty} \frac{(-1)^{i}}{2i+1} &=& \sum\limits_{i=0}^{\infty} (-1)^{i} \cdot \frac{1}{2i+1} \\
\intertext{Nach dem Leibnitz-Kriterium konvergiert diese Reihe, da die Reihenglieder $a_{i} = \frac{1}{2i+1}$ eine monotone Nullfolge bilden.}
s_{0} &=& 1 \\
s_{1} &=& 1 - \frac{1}{3} = \frac{2}{3}\\
s_{2} &=& 1 - \frac{1}{3} + \frac{1}{5} = \frac{13}{15} \approx 0.867 \\
s_{3} &=& \frac{13}{15} - \frac{1}{7} = \frac{91}{105} - \frac{15}{105} = \frac{76}{105} \approx 0.724
\intertext{Der Grenzwert könnte $\approx 0.7853$ sein.}
\end{alignat*}
\subsubsection{} %vi
\begin{alignat*}{2}
\sum\limits_{i=1}^{\infty} \frac{(-1)^{i}}{2i} &=& \sum\limits_{i=1}^{\infty} (-1)^{i} \cdot \frac{1}{2i} \\
\intertext{Nach dem Leibnitz-Kriterium ist diese Reihe konvergent.}
s_{0} &=& -\frac{1}{2} \\
s_{1} &=& -\frac{1}{2} + \frac{1}{4} = -\frac{1}{4} \\
s_{2} &=& -\frac{1}{4} - \frac{1}{6} = -\frac{10}{24} = -\frac{5}{12} \approx -0.4166667 \\
s_{3} &=& -\frac{5}{12} + \frac{1}{8} = -\frac{40}{96} + \frac{12}{96} = -\frac{28}{96} \approx -0.2916667
\intertext{Der Grenzwert könnte $\approx -0.3541667$ sein.}
\end{alignat*}
\section{} %3
\subsection{} %a
\begin{tikzpicture}[>=stealth]
\begin{axis}[
ymin=0,ymax=10,
x=1cm,
y=1cm,
axis x line=middle,
axis y line=middle,
axis line style=->,
xlabel={$x$},
ylabel={$f(x)$},
xmin=0,xmax=10
]
\addplot[no marks, black, -] expression[domain=0:10,samples=100]{1/x} node[pos=0.65,anchor=north]{};
\addplot[no marks, black, -] expression[domain=1:2,samples]{1} node[pos=0.65,anchor=north]{};
\draw[>=stealth] (axis cs:1,1) -- (axis cs:1,0) node [pos=0.65,anchor=north]{};
\draw[>=stealth] (axis cs:2,1) -- (axis cs:2,0) node [pos=0.65,anchor=north]{};
\addplot[no marks, black, -] expression[domain=2:3,samples]{1/2} node[pos=0.65,anchor=north]{};
\draw[>=stealth] (axis cs:3,0.5) -- (axis cs:3,0) node [pos=0.65,anchor=north]{};
\addplot[no marks, black, -] expression[domain=3:4,samples]{1/3} node[pos=0.65,anchor=north]{};
\draw[>=stealth] (axis cs:4,0.3333333333333) -- (axis cs:4,0) node [pos=0.65,anchor=north]{};
\addplot[no marks, black, -] expression[domain=4:5,samples]{1/4} node[pos=0.65,anchor=north]{};
\draw[>=stealth] (axis cs:5,0.25) -- (axis cs:5,0) node [pos=0.65,anchor=north]{};
\addplot[no marks, black, -] expression[domain=5:6,samples]{1/5} node[pos=0.65,anchor=north]{};
\draw[>=stealth] (axis cs:6,0.2) -- (axis cs:6,0) node [pos=0.65,anchor=north]{};
\addplot[no marks, black, -] expression[domain=6:7,samples]{1/6} node[pos=0.65,anchor=north]{};
\draw[>=stealth] (axis cs:7,0.1666666667) -- (axis cs:7,0) node [pos=0.65,anchor=north]{};
\addplot[no marks, black, -] expression[domain=7:8,samples]{1/7} node[pos=0.65,anchor=north]{};
\draw[>=stealth] (axis cs:8,0.142857142857) -- (axis cs:8,0) node [pos=0.65,anchor=north]{};
\addplot[no marks, black, -] expression[domain=8:9,samples]{1/8} node[pos=0.65,anchor=north]{};
\draw[>=stealth] (axis cs:9,0.125) -- (axis cs:9,0) node [pos=0.65,anchor=north]{};
\addplot[no marks, black, -] expression[domain=9:10,samples]{1/9} node[pos=0.65,anchor=north]{};
\draw[>=stealth] (axis cs:10,0.111111111) -- (axis cs:10,0) node [pos=0.65,anchor=north]{};
\node at (axis cs: 1.5,1.25) {1};
\node at (axis cs: 2.5,0.75) {$\frac{1}{2}$};
\node at (axis cs: 3.5,0.5833333) {$\frac{1}{3}$};
\node at (axis cs: 4.5,0.5) {$\frac{1}{4}$};
\node at (axis cs: 5.5,0.45) {$\frac{1}{5}$};
\node at (axis cs: 6.5,0.4166667) {$\frac{1}{6}$};
\node at (axis cs: 7.5,0.392857142857) {$\frac{1}{7}$};
\node at (axis cs: 8.5,0.375) {$\frac{1}{8}$};
\node at (axis cs: 9.5,0.36111111) {$\frac{1}{9}$};
\end{axis}
\end{tikzpicture}\\
Anhand der Skizze erkennt man:\\
\begin{alignat*}{3}
H_{n} &\geq & \int\limits_{1}^{n+1} \frac{1}{x}\,dx &=& [\ln x]_{1}^{n+1} \\
&\geq & &=& \ln (n+1) - \ln 1 \\
&\geq & &=& \ln (n+1) \\
\end{alignat*}
\subsection{} %b
Der Logarithmus $\ln$ ist divergent. Da die harmonische Reihe nach entsprechender Formel größer oder gleich dem natürlichen Logarithmus ist, wächst somit die harmonische Reihe ebenfalls über alle Schranken.
\section{} %4
\subsection{} %a
Aus Aufgabe 3 ist bekannt, dass Folgendes gilt:\\
\begin{alignat*}{2}
\ln(n+1) &\leq & H_{n} \; (n = 1,2,...)\\
\intertext{Da der natürliche Logarithmus monoton wachsend ist, gilt somit auch dies:}
\ln(n) &\leq & H_{n} \; (n = 1,2,...)
\end{alignat*}
\begin{tikzpicture}[>=stealth]
\begin{axis}[
ymin=0,ymax=10,
x=1cm,
y=1cm,
axis x line=middle,
axis y line=middle,
axis line style=->,
xlabel={$x$},
ylabel={$f(x)$},
xmin=0,xmax=9
]
\addplot[no marks, black, -] expression[domain=0:9,samples=100]{1/x} node[pos=0.65,anchor=north]{};
\addplot[no marks, black, -] expression[domain=1:2,samples=100]{1/2} node[pos=0.65,anchor=north]{};
\draw[>=stealth] (axis cs:1,0.5) -- (axis cs:1,0) node [pos=0.65,anchor=north]{};
\draw[>=stealth] (axis cs:2,0.5) -- (axis cs:2,0) node [pos=0.65,anchor=north]{};
\addplot[no marks, black, -] expression[domain=2:3,samples=100]{1/3} node[pos=0.65,anchor=north]{};
\draw[>=stealth] (axis cs:3,0.3333333333333) -- (axis cs:3,0) node [pos=0.65,anchor=north]{};
\addplot[no marks, black, -] expression[domain=3:4,samples=100]{1/4} node[pos=0.65,anchor=north]{};
\draw[>=stealth] (axis cs:4,0.25) -- (axis cs:4,0) node [pos=0.65,anchor=north]{};
\addplot[no marks, black, -] expression[domain=4:5,samples=100]{1/5} node[pos=0.65,anchor=north]{};
\draw[>=stealth] (axis cs:5,0.2) -- (axis cs:5,0) node [pos=0.65,anchor=north]{};
\addplot[no marks, black, -] expression[domain=5:6,samples=100]{1/6} node[pos=0.65,anchor=north]{};
\draw[>=stealth] (axis cs:6,0.1666666667) -- (axis cs:6,0) node [pos=0.65,anchor=north]{};
\addplot[no marks, black, -] expression[domain=6:7,samples=100]{1/7} node[pos=0.65,anchor=north]{};
\draw[>=stealth] (axis cs:7,0.142857142857) -- (axis cs:7,0) node [pos=0.65,anchor=north]{};
\addplot[no marks, black, -] expression[domain=7:8,samples=100]{1/8} node[pos=0.65,anchor=north]{};
\draw[>=stealth] (axis cs:8,0.125) -- (axis cs:8,0) node [pos=0.65,anchor=north]{};
\addplot[no marks, black, -] expression[domain=8:9,samples=100]{1/9} node[pos=0.65,anchor=north]{};
\draw[>=stealth] (axis cs:9,0.111111111) -- (axis cs:9,0) node [pos=0.65,anchor=north]{};
%\addplot[no marks, black, -] expression[domain=9:10,samples=100]{1/10} node[pos=0.65,anchor=north]{};
%\draw[>=stealth] (axis cs:10,0.1) -- (axis cs:10,0) node [pos=0.65,anchor=north]{};
\node at (axis cs: 1.5,0.75) {$\frac{1}{2}$};
\node at (axis cs: 2.5,0.5833333) {$\frac{1}{3}$};
\node at (axis cs: 3.5,0.5) {$\frac{1}{4}$};
\node at (axis cs: 4.5,0.45) {$\frac{1}{5}$};
\node at (axis cs: 5.5,0.4166667) {$\frac{1}{6}$};
\node at (axis cs: 6.5,0.392857142857) {$\frac{1}{7}$};
\node at (axis cs: 7.5,0.375) {$\frac{1}{8}$};
\node at (axis cs: 8.5,0.36111111) {$\frac{1}{9}$};
%\node at (axis cs: 9.5,0.35) {$\frac{1}{10}$};
\end{axis}
\end{tikzpicture}\\
Anhand der Skizze erkennt man:\\
\begin{alignat*}{3}
H_{n} - 1 &\leq & \int\limits_{1}^{n} \frac{1}{x}\,dx &=& [\ln x]_{1}^{n} \\
&\leq & &=& \ln (n) - \ln 1 \\
&\leq & &=& \ln (n)
\end{alignat*}
\begin{alignat*}{3}
\intertext{Daraus ergibt sich:}
H_{n} - 1 &\leq & \ln(n) &\leq & H_{n}
\end{alignat*}
\subsection{} %b
\begin{alignat*}{3}
\intertext{Aus a) ergibt sich:}
\lim\limits_{n \rightarrow \infty} \frac{H_{n}-1}{H_{n}} &\leq & \lim\limits_{n \rightarrow \infty} \frac{\ln(n)}{H_{n}} &\leq & \lim\limits_{n \rightarrow \infty} \frac{H_{n}}{H_{n}} \\
\lim\limits_{n \rightarrow \infty} \frac{H_{n}}{H_{n}} - \frac{1}{H_{n}} &\leq & \lim\limits_{n \rightarrow \infty} \frac{\ln(n)}{H_{n}} &\leq & 1 \\
1 - 0 &\leq & \lim\limits_{n \rightarrow \infty} \frac{\ln(n)}{H_{n}} &\leq & 1 \\
1 &\leq & \lim\limits_{n \rightarrow \infty} \frac{\ln(n)}{H_{n}} &\leq & 1
\intertext{Nach dem Einschließungssatz muss der Grenzwert von $\lim\limits_{n \rightarrow \infty} \frac{\ln(n)}{H_{n}}$ $1$ sein.}
\end{alignat*}
\end{document}

View File

@ -0,0 +1,185 @@
\documentclass[10pt,a4paper,oneside,ngerman,numbers=noenddot]{scrartcl}
\usepackage[T1]{fontenc}
\usepackage[utf8]{inputenc}
\usepackage[ngerman]{babel}
\usepackage{amsmath}
\usepackage{amsfonts}
\usepackage{amssymb}
\usepackage{paralist}
\usepackage{gauss}
\usepackage{pgfplots}
\usepackage[locale=DE,exponent-product=\cdot,detect-all]{siunitx}
\usepackage{tikz}
\usetikzlibrary{matrix,fadings,calc,positioning,decorations.pathreplacing,arrows,decorations.markings}
\usepackage{polynom}
\polyset{style=C, div=:,vars=x}
\pgfplotsset{compat=1.8}
\pagenumbering{arabic}
\def\thesection{\arabic{section})}
\def\thesubsection{\alph{subsection})}
\def\thesubsubsection{(\roman{subsubsection})}
\makeatletter
\renewcommand*\env@matrix[1][*\c@MaxMatrixCols c]{%
\hskip -\arraycolsep
\let\@ifnextchar\new@ifnextchar
\array{#1}}
\makeatother
\begin{document}
\author{Jim Martens (6420323)}
\title{Hausaufgaben zum 20. Juni}
\maketitle
\section{} %1
\subsection{} %a
\begin{alignat*}{2}
T_{8}(x) &=&& 1 - \frac{1}{2}x^{2} + \frac{1}{24}x^{4} - \frac{1}{720}x^{6} + \frac{1}{40320}x^{8} \\
T_{9}(x) &=&& 1 - \frac{1}{2}x^{2} + \frac{1}{24}x^{4} - \frac{1}{720}x^{6} + \frac{1}{40320}x^{8} \\
T_{10}(x) &=&& 1 - \frac{1}{2}x^{2} + \frac{1}{24}x^{4} - \frac{1}{720}x^{6} + \frac{1}{40320}x^{8} - \frac{1}{10!}x^{10}\\
T_{11}(x) &=&& 1 - \frac{1}{2}x^{2} + \frac{1}{24}x^{4} - \frac{1}{720}x^{6} + \frac{1}{40320}x^{8} - \frac{1}{10!}x^{10} \\
T_{12}(x) &=&& 1 - \frac{1}{2}x^{2} + \frac{1}{24}x^{4} - \frac{1}{720}x^{6} + \frac{1}{40320}x^{8} - \frac{1}{10!}x^{10} + \frac{1}{12!}x^{12}\\
T_{13}(x) &=&& 1 - \frac{1}{2}x^{2} + \frac{1}{24}x^{4} - \frac{1}{720}x^{6} + \frac{1}{40320}x^{8} - \frac{1}{10!}x^{10} + \frac{1}{12!}x^{12}
\end{alignat*}
\begin{alignat*}{2}
T_{9}(1) &=& 1 - \frac{1}{2} \cdot 1^{2} + \frac{1}{24} \cdot 1^{4} - \frac{1}{720} \cdot 1^{6} + \frac{1}{40320} \cdot 1^{8} \\
&\approx & 0.5403026 \\
T_{11}(1) &=& 1 - \frac{1}{2} \cdot 1^{2} + \frac{1}{24} \cdot 1^{4} - \frac{1}{720} \cdot 1^{6} + \frac{1}{40320} \cdot 1^{8} - \frac{1}{10!} \cdot 1^{10} \\
&\approx & 0.5403023 \\
T_{13}(1) &=& 1 - \frac{1}{2} \cdot 1^{2} + \frac{1}{24} \cdot 1^{4} - \frac{1}{720} \cdot 1^{6} + \frac{1}{40320} \cdot 1^{8} - \frac{1}{10!} \cdot 1^{10} + \frac{1}{12!} \cdot 1^{12} \\
&\approx & 0.5403023
\end{alignat*}
\subsection{} %b
\begin{alignat*}{2}
f(x) &=& \sqrt{1+x} = (1+x)^{\frac{1}{2}} \\
f'(x) &=& \frac{1}{2} \cdot (1+x)^{-\frac{1}{2}} \\
f''(x) &=& -\frac{1}{4} \cdot (1+x)^{-\frac{3}{2}} \\
f'''(x) &=& \frac{3}{8} \cdot (1+x)^{-\frac{5}{2}} \\
f^{(4)}(x) &=& -\frac{15}{16} \cdot (1+x)^{-\frac{7}{2}}
\end{alignat*}
\begin{alignat*}{2}
T_{0}(x) &=&& 1 \\
T_{1}(x) &=&& 1 + \frac{1}{2}x \\
T_{2}(x) &=&& 1 + \frac{1}{2}x - \frac{1}{8}x^{2}\\
T_{3}(x) &=&& 1 + \frac{1}{2}x - \frac{1}{8}x^{2} + \frac{3}{48}x^{3} \\
T_{4}(x) &=&& 1 + \frac{1}{2}x - \frac{1}{8}x^{2} + \frac{3}{48}x^{3} - \frac{15}{384}x^{4}
\end{alignat*}
\begin{alignat*}{2}
g(x) &=& \sqrt[3]{1+x} = (1+x)^{\frac{1}{3}} \\
g'(x) &=& \frac{1}{3} \cdot (1+x)^{-\frac{2}{3}} \\
g''(x) &=& -\frac{2}{9} \cdot (1+x)^{-\frac{5}{3}} \\
g'''(x) &=& \frac{10}{27} \cdot (1+x)^{-\frac{8}{3}} \\
g^{(4)}(x) &=& -\frac{80}{81} \cdot (1+x)^{-\frac{11}{3}}
\end{alignat*}
\begin{alignat*}{2}
T_{0}(x) &=&& 1 \\
T_{1}(x) &=&& 1 + \frac{1}{3}x \\
T_{2}(x) &=&& 1 + \frac{1}{3}x - \frac{1}{9}x^{2} \\
T_{3}(x) &=&& 1 + \frac{1}{3}x - \frac{1}{9}x^{2} + \frac{5}{81}x^{3} \\
T_{4}(x) &=&& 1 + \frac{1}{3}x - \frac{1}{9}x^{2} + \frac{5}{81}x^{3} - \frac{10}{243}x^{4}
\end{alignat*}
\subsection{} %c
\begin{alignat*}{2}
f(x) &=& e^{x} \cdot \sin x \\
f'(x) &=& e^{x} \cdot \cos x \\
f''(x) &=& -e^{x} \cdot \sin x \\
f'''(x) &=& -e^{x} \cdot \cos x \\
f^{(4)}(x) &=& e^{x} \cdot \sin x \\
f^{(5)}(x) &=& e^{x} \cdot \cos x
\end{alignat*}
\begin{alignat*}{2}
T_{5}(x) &=& 0 + x - 0x^{2} - \frac{1}{6}x^{3} + 0x^{4} + \frac{1}{120}x^{5} \\
&=& x - \frac{1}{6}x^{3} + \frac{1}{120}x^{5}
\end{alignat*}
\section{} %2
\subsubsection{} %i
\begin{alignat*}{2}
\lim\limits_{x \rightarrow 1} \left(\frac{x^{3}-3x^{2}+x+2}{x^{2}-5x+6} \right) &=& \frac{1-3+1+2}{1-5+6} \\
&=& \frac{1}{2}
\end{alignat*}
\subsubsection{} %ii
\begin{alignat*}{3}
\lim\limits_{x \rightarrow 2} \left(\frac{x^{3}-3x^{2}+x+2}{x^{2}-5x+6} \right) &=& \lim\limits_{x \rightarrow 2} \left(\frac{3x^{2} - 6x + 1}{2x - 5} \right) &=& \frac{3 \cdot 2^{2} - 6 \cdot 2 + 1}{2 \cdot 2 - 5} \\
&& &=& \frac{12 - 12 + 1}{4 - 5} \\
&& &=& \frac{1}{-1} \\
&& &=& -1
\end{alignat*}
\subsubsection{} %iii
\begin{alignat*}{2}
\lim\limits_{x \rightarrow 0} (1+3x)^{\frac{1}{2x}}
\end{alignat*}
\subsubsection{} %iv
\begin{alignat*}{2}
\lim\limits_{x \rightarrow 0} \left(\frac{1}{e^{x} - 1} - \frac{1}{\sin x} \right)
\end{alignat*}
\section{} %3
\subsection{} %a
\begin{alignat*}{2}
f(x) &=& 3^{x} \\
f'(x) &=& 3^{x} \cdot \ln 3 \\
t'(x) = f'(2) &=& 9 \cdot \ln 3 \\
&\approx & 9.88751 \\
\intertext{Die Steigung der Tangente beträgt $9 \cdot \ln 3$ oder rund $9.88751$.}
t(x) &=& \ln (3) \cdot 9x + b \\
\intertext{Bestimmen des Schnittpunkts mit der y-Achse}
b &=& t(x) - \ln (3) \cdot 9x \\
&=& t(2) - \ln (3) \cdot 18 \\
&=& 9 - \ln (3) \cdot 18 \\
&\approx & -10.77502 \\
t(x) &=& \ln 3 \cdot 9x + 9 - \ln (3) \cdot 18 \\
\intertext{Bestimmen des Schnittpunkts mit der x-Achse}
0 &=& \ln (3) \cdot 9x + 9 - \ln (3) \cdot 18 \\
\ln (3) \cdot 18 - 9 &=& \ln (3) \cdot 9x \\
\frac{\ln (3) \cdot 18 - 9}{\ln 3} &=& 9x \\
\frac{\ln (3) \cdot 18 - 9}{\ln (3) \cdot 9} &=& x \\
x &\approx & 1.08976
\end{alignat*}
\subsection{} %b
\begin{alignat*}{2}
f(x) &=& \sqrt[7]{x+2} \\
&=& (x+2)^{\frac{1}{7}} \\
f'(x) &=& \frac{1}{7} \cdot (x+2)^{-\frac{6}{7}} \\
f''(x) &=& -\frac{6}{49} \cdot (x+2)^{-\frac{13}{7}}
\end{alignat*}
\begin{alignat*}{2}
T_{0} &=&& 2^{\frac{1}{7}} \\
T_{1} &=&& 2^{\frac{1}{7}} + \frac{1}{7} \cdot 2^{-\frac{6}{7}}x \\
T_{2} &=&& 2^{\frac{1}{7}} + \frac{1}{7} \cdot 2^{-\frac{6}{7}}x - \frac{3}{49} \cdot 2^{-\frac{13}{7}}
\end{alignat*}
\subsection{} %c
\begin{alignat*}{2}
x_{n} &=& \frac{1}{2\pi n} \\
\lim\limits_{n\rightarrow \infty} f(x_{n}) = \lim\limits_{n\rightarrow \infty} \cos \left(\frac{1}{x_{n}} \right) &=& \lim\limits_{n\rightarrow \infty} \cos \left(\frac{1}{\frac{1}{2 \pi n}} \right) \\
&=& \lim\limits_{n\rightarrow \infty} \cos (2 \pi n) = 1
\end{alignat*} \\
Der mögliche Grenzwert 1 stimmt nicht mit dem Funktionswert überein. Daher ist $h(x)$ im Punkt $x_{0}=0$ nicht stetig.
\subsection{} %d
\setcounter{subsubsection}{0}
\subsubsection{} %i
$\mathcal{B}$ enthält alle Folgen, die gegen eine reelle Zahl konvergieren, als auch solche, die zwischen zwei Werten oszillieren.
Lediglich uneigentlich konvergente Folgen sind nicht enthalten.
\subsubsection{} %ii
Aus $i)$ ergibt sich, dass nicht jede Folge in $\mathcal{B}$ konvergiert. Oszillierende Folgen konvergieren nicht, allerdings erfüllen sie die Bedingungen von $\mathcal{B}$.
\section{} %4
\subsection{} %a
\begin{alignat*}{2}
\lim\limits_{x \rightarrow \infty} \left(\frac{a^{x}}{x^{n}}\right) &=& \lim\limits_{x \rightarrow \infty} \left(\frac{a^{x} \cdot \ln a}{n \cdot x^{n-1}}\right) \\
\intertext{Nach n Ableitungen}
&=& \lim\limits_{x \rightarrow \infty} \left(\frac{a^{x} \cdot (\ln a)^{n}}{n! \cdot x^{0}}\right) \\
\intertext{Zähler geht gegen unendlich, Nenner gegen konstante Zahl $n!$. Daher existiert der Grenzwert nicht.}
&=& \lim\limits_{x \rightarrow \infty} \left(\frac{a^{x} \cdot (\ln a)^{n}}{n!}\right) = \infty
\end{alignat*}
\subsection{} %b
\begin{alignat*}{2}
\lim\limits_{x \rightarrow \infty} \left(\frac{x^{r}}{\ln^{k} x} \right) &=& \lim\limits_{x \rightarrow \infty} \left(\frac{rx^{r-1}}{k \cdot \ln^{k-1} x \cdot \frac{1}{x}} \right) \\
&=& \lim\limits_{x \rightarrow \infty} \left(\frac{rx^{r}}{k \cdot \ln^{k-1} x} \right) \\
\intertext{Nach der k-ten Ableitung sieht es so aus}
&=& \lim\limits_{x \rightarrow \infty} \left(\frac{r^{k} \cdot x^{r}}{k! \cdot \ln x} \right) \\
\intertext{Herausziehen der Konstanten}
&=& \frac{r^{k}}{k!} \cdot \lim\limits_{x \rightarrow \infty} \left(\frac{x^{r}}{\ln x} \right) \\
\intertext{Nach Satz 27 im Skript, existiert der Grenzwert $\lim\limits_{x \rightarrow \infty} \left(\frac{x^{r}}{\ln x} \right)$ nicht bzw. ist unendlich. Unendlich mal eine Konstante ist immer noch unendlich.}
&=& \infty
\end{alignat*}
\subsection{} %c
\setcounter{subsubsection}{0}
\subsubsection{} %i
\subsubsection{} %ii
\end{document}

View File

@ -0,0 +1,107 @@
\documentclass[10pt,a4paper,oneside,ngerman,numbers=noenddot]{scrartcl}
\usepackage[T1]{fontenc}
\usepackage[utf8]{inputenc}
\usepackage[ngerman]{babel}
\usepackage{amsmath}
\usepackage{amsfonts}
\usepackage{amssymb}
\usepackage{paralist}
\usepackage[locale=DE,exponent-product=\cdot,detect-all]{siunitx}
\usepackage{tikz}
\usetikzlibrary{matrix,fadings,calc,positioning,decorations.pathreplacing,arrows,decorations.markings}
\usepackage{polynom}
\polyset{style=C, div=:,vars=x}
\pagenumbering{arabic}
\def\thesection{\arabic{section})}
\def\thesubsection{\alph{subsection})}
\def\thesubsubsection{(\roman{subsubsection})}
\begin{document}
\author{Jim Martens}
\title{Hausaufgaben zum 10./11. Januar}
\maketitle
\section{} %1
\subsection{} %a
$H_{1}$ ist keine Untergruppe, da das neutrale Element der Multiplikation fehlt.
$H_{2}$ ist keine Untergruppe, da sie nicht abgeschlossen ist. $4 \cdot 4 = 16 = 3$ (mod $13$) ist nicht in $H_{2}$ enthalten.
$H_{3}$ ist eine Untergruppe von $G$, da das neutrale Element $1$ vorhanden ist, die Menge abgeschlossen ist ($12 \cdot 12 = 144 = 1$ (mod $13$) und jedes Element ein Inverses hat ($1$ ist zu sich selbst invers und $12$ ist zu sich selbst invers).
\subsection{} %b
$H = <3> = \{1,3,9\}$
$a=1 \rightarrow 1H = H = 3H = 9H$ \\
$a=2 \rightarrow 2H = \{2,6,5\} = 6H$ \\
$a=4 \rightarrow 4H = \{4,12,10\} = 10H = 12H$ \\
$a=5 \rightarrow 5H = \{5,2,6\}$ \\
$a=7 \rightarrow 7H = \{7,8,11\} = 8H = 11H$
\section{} %2
\subsection{} %a
$H = \{id, (12)\}$\\
\\
Linksnebenklassen:\\
$id \circ H = \{id \circ id, id \circ (1,2)\} = \{id, (1,2)\}$\\
$(1,2) \circ H = \{(1,2) \circ id, (1,2) \circ (1,2)\} = \{(1,2), id\}$\\
$(1,3) \circ H = (1,2,3) \circ H = \{(1,3) \circ id, (1,3) \circ (1,2)\} = \{(1,3), (1,2,3)\}$\\
$(2,3) \circ H = (1,3,2) \circ H = \{(2,3) \circ id, (2,3) \circ (1,2)\} = \{(2,3), (1,3,2)\}$\\
\\
Rechtsnebenklassen:\\
$H \circ id = \{id \circ id, (1,2) \circ id\} = \{id, (1,2)\}$\\
$H \circ (1,2) = \{id \circ (1,2), (1,2) \circ (1,2)\} = \{(1,2), id\}$\\
$H \circ (1,3) = H \circ (1,3,2) = \{id \circ (1,3), (1,2) \circ (1,3)\} = \{(1,3), (1,3,2)\}$\\
$H \circ (2,3) = H \circ (1,2,3) = \{id \circ (2,3), (1,2) \circ (2,3)\} = \{(2,3), (1,2,3)\}$
\subsection{} %b
$S_{6}$ enthält $6! = 720$ Elemente. $H$ ist eine mögliche Untergruppe von $G$, da $H$ $360$ Elemente hat und dies ein Teiler von $720$ ist. Nach dem Satz von Lagrange kommen nur Untergruppen in Frage, deren Mächtigkeit ein Teiler der Mächtigkeit der "Obergruppe" ist.
%todo
\subsection{} %c
$G = \{1,5,11,13,17,19,23,25,29,31,35,37,41\}$
Die Linksnebenklasse $gH$ ist gleich der Rechtsnebenklasse $Hg$, weil sowohl Addition als auch Multiplikation assoziativ sind.
\section{} %3
\subsection{} %a
\begin{alignat*}{2}
x + y &=& (4x^{2} - x + 2) + (2x^{3} + x^{2} - 3x + 2) \\
&=& 2x^{3} + 5x^{2} - 4x + 4 \\
x \cdot y &=& (4x^{2} - x + 2) \cdot (2x^{3} + x^{2} - 3x + 2) \\
&=& 8x^{5} + 4x^{4} - 12x^{3} + 8x^{2} - 2x^{3} -x^{2} + 3x^{2} -2x + 4x^{3} + 2x^{2} -6x + 4 \\
&=& 8x^{5} + 4x^{4} - 10x^{3} + 12x^{2} - 8x +4
\end{alignat*}
\subsection{} %b
\begin{alignat*}{2}
Koeffizient(a(x) \cdot b(x),x^{7}) &=& -2x^{7} + 6x^{7} - 18x^{7} + 9x^{7} - 7x^{7} + 40x^{7} + 6x^{7} + 2x^{7} \\
&=& 36x^{7}
\end{alignat*}
Der Koeffizient des Produktes von $a(x) \cdot b(x)$ für $x^{7}$ lautet $36$.
\subsection{} %c
\begin{alignat*}{2}
a(x) + b(x) &=& 3x^{4} + 4x^{3} + x^{2} + 3x + 5 \\
a(x) \cdot b(x) &=& 2x^{7} + 4x^{5} + 2x^{3} + x^{6} + 2x^{5} + x^{2} + 4x^{5} + 3x^{3} + 4x + x^{4} + 2x^{2} + 1 \\
&=& 2x^{7} + x^{6} + 0x^{5} + x^{4} + 0x^{3} + 3x^{2} + 1 \\
&=& 2x^{7} + x^{6} + x^{4} + 3x^{2} + 1
\end{alignat*}
\section{} %4
\subsection{} %a
\polylongdiv{x^5 + 2x^4 + 3x^3 + x^2 + 4x + 2}{x^2 + 4x +3}
\begin{equation*}
x^{5} + 2x^{4} + 3x^{3} + x^{2} + 4x + 2 = (x^{3} -2x + 9) \cdot (x^{2} + 4x + 3) + (-26x - 25)
\end{equation*}
\subsection{} %b
Der normierte größte gemeinsame Teiler der beiden Polynome ergibt sich wie folgt:\\
\hspace{-2.5cm}
\polylongdiv{6x^5 + 7x^4 - 7x^3 - 22x^2 - 25x - 15}{3x^4 + 2x^3 - 6x^2 - 6x -9}\\
\\
\polylongdiv{3x^4 + 2x^3 - 6x^2 - 6x -9}{3x^3 - 4x^2 - x - 6}\\
\\
\polylongdiv{3x^3 - 4x^2 - x - 6}{3x^2 + 2x + 3}\\
\\
\begin{alignat*}{4}
6x^{5} + 7x^{4} - 7x^{3} - 22x^{2} - 25x - 15 &=& (2x + 1) &\cdot & (3x^{4} + 2x^{3} - 6x^{2} - 6x - 9) &+& (3x^{3} - 4x^{2} - x - 6) \\
3x^{4} + 2x^{3} - 6x^{2} - 6x - 9 &=& (x + 2) &\cdot & (3x^{3} - 4x^{2} - x - 6) &+& (3x^{2} - 2x + 3) \\
3x^{3} - 4x^{2} - x - 6 &=& (x - 2) &\cdot & (3x^{2} - 2x + 3) &+& 0
\end{alignat*}
\\
Der größte normierte Teiler ist demzufolge $x^2 + \frac{2}{3}x + 1$.
\end{document}

View File

@ -0,0 +1,490 @@
\documentclass[10pt,a4paper,oneside,ngerman,numbers=noenddot]{scrartcl}
\usepackage[T1]{fontenc}
\usepackage[utf8]{inputenc}
\usepackage[ngerman]{babel}
\usepackage{amsmath}
\usepackage{amsfonts}
\usepackage{amssymb}
\usepackage{paralist}
\usepackage{gauss}
\usepackage[locale=DE,exponent-product=\cdot,detect-all]{siunitx}
\usepackage{tikz}
\usetikzlibrary{matrix,fadings,calc,positioning,decorations.pathreplacing,arrows,decorations.markings}
\usepackage{polynom}
\polyset{style=C, div=:,vars=x}
\pagenumbering{arabic}
\def\thesection{\arabic{section})}
\def\thesubsection{\alph{subsection})}
\def\thesubsubsection{(\roman{subsubsection})}
\makeatletter
\renewcommand*\env@matrix[1][*\c@MaxMatrixCols c]{%
\hskip -\arraycolsep
\let\@ifnextchar\new@ifnextchar
\array{#1}}
\makeatother
\begin{document}
\author{Jim Martens}
\title{Hausaufgaben zum 17./18. Januar}
\maketitle
\section{} %1
\subsection{} %a
\[
\begin{bmatrix}[ccc|c]
2 & 1 & 1 & 1 \\
1 & -1 & 1 & 4 \\
3 & 1 & 2 & 4
\end{bmatrix}
\]
$\overset{II \curvearrowright I, I \curvearrowright II}{\leadsto}$
\[
\begin{bmatrix}[ccc|c]
1 & -1 & 1 & 4 \\
2 & 1 & 1 & 1 \\
3 & 1 & 2 & 4
\end{bmatrix}
\]
$\overset{II = II - 2I, III = III - 3I}{\leadsto}$
\[
\begin{bmatrix}[ccc|c]
1 & -1 & 1 & 4 \\
0 & 3 & -1 & -7 \\
0 & 4 & -1 & -8
\end{bmatrix}
\]
$\overset{II = \frac{1}{3}II}{\leadsto}$
\[
\begin{bmatrix}[ccc|c]
1 & -1 & 1 & 4 \\
0 & 1 & -\frac{1}{3} & -\frac{7}{3} \\
0 & 4 & -1 & -8
\end{bmatrix}
\]
$\overset{III = III - 4II}{\leadsto}$
\[
\begin{bmatrix}[ccc|c]
1 & -1 & 1 & 4 \\
0 & 1 & -\frac{1}{3} & -\frac{7}{3} \\
0 & 0 & \frac{1}{3} & \frac{4}{3}
\end{bmatrix}
\]
$\overset{III = 3III}{\leadsto}$
\[
\begin{bmatrix}[ccc|c]
1 & -1 & 1 & 4 \\
0 & 1 & -\frac{1}{3} & -\frac{7}{3} \\
0 & 0 & 1 & 4
\end{bmatrix}
\]
\begin{alignat*}{3}
\overset{III}{\Rightarrow} & x_{3} &\,=\,& 4 && \\
\overset{II}{\Rightarrow} & x_{2} - \frac{4}{3} &\,=\,& - \frac{7}{3} && \;| +\frac{4}{3} \\
\Leftrightarrow & x_{2} &\,=\,& -1 & \\
\overset{I}{\Rightarrow} & x_{1} - (-1) + 4 &\,=\,& 4 && \;|\text{Zus.} \\
\Leftrightarrow & x_{1} + 5 &\,=\,& 4 && \;| -5 \\
\Leftrightarrow & x_{1} &\,=\,& -1 &&
\end{alignat*}
Es gibt genau eine Lösung.
\subsection{} %b
\[
\begin{bmatrix}[ccc|c]
2 & 1 & 1 & 2 \\
1 & -1 & 1 & 3 \\
3 & 0 & 2 & 5
\end{bmatrix}
\]
$\overset{II \curvearrowright I, I \curvearrowright II}{\leadsto}$
\[
\begin{bmatrix}[ccc|c]
1 & -1 & 1 & 3 \\
2 & 1 & 1 & 2 \\
3 & 0 & 2 & 5
\end{bmatrix}
\]
$\overset{II = II - 2I, III = III - 3I}{\leadsto}$
\[
\begin{bmatrix}[ccc|c]
1 & -1 & 1 & 3 \\
0 & -1 & -1 & -4 \\
0 & -3 & -1 & -4
\end{bmatrix}
\]
$\overset{II = -1II}{\leadsto}$
\[
\begin{bmatrix}[ccc|c]
1 & -1 & 1 & 3 \\
0 & 1 & 1 & 4 \\
0 & -3 & -1 & -4
\end{bmatrix}
\]
$\overset{III = III + 3II}{\leadsto}$
\[
\begin{bmatrix}[ccc|c]
1 & -1 & 1 & 3 \\
0 & 1 & 1 & 4 \\
0 & 0 & 2 & 8
\end{bmatrix}
\]
$\overset{III = \frac{1}{2}III}{\leadsto}$
\[
\begin{bmatrix}[ccc|c]
1 & -1 & 1 & 3 \\
0 & 1 & 1 & 4 \\
0 & 0 & 1 & 4
\end{bmatrix}
\]
\begin{alignat*}{3}
\overset{III}{\Rightarrow} & x_{3} &\,=\,& 4 && \\
\overset{II}{\Rightarrow} & x_{2} + x_{3} &\,=\,& 4 && \\
\Rightarrow & x_{2} + 4 &\,=\,& 4 && \;| -4 \\
\Leftrightarrow & x_{2} &\,=\,& 0 & \\
\overset{I}{\Rightarrow} & x_{1} - x_{2} + x_{3} &\,=\,& 3 && \\
\Rightarrow & x_{2} - 0 + 4 &\,=\,& 3 && \;| -4 \\
\Leftrightarrow & x_{1} &\,=\,& -1 &&
\end{alignat*}
Es gibt genau eine Lösung.
\subsection{} %c
\[
\begin{bmatrix}[ccc|c]
2 & 1 & 1 & -1 \\
1 & -1 & 1 & 3 \\
3 & 0 & 2 & 0
\end{bmatrix}
\]
$\overset{II \curvearrowright I, I \curvearrowright II}{\leadsto}$
\[
\begin{bmatrix}[ccc|c]
1 & -1 & 1 & 3 \\
2 & 1 & 1 & -1 \\
3 & 0 & 2 & 0
\end{bmatrix}
\]
$\overset{II = II - 2I, III = III - 3I}{\leadsto}$
\[
\begin{bmatrix}[ccc|c]
1 & -1 & 1 & 3 \\
0 & 3 & -1 & -7 \\
0 & 3 & -1 & -9
\end{bmatrix}
\]
$\overset{II = \frac{1}{3}II}{\leadsto}$
\[
\begin{bmatrix}[ccc|c]
1 & -1 & 1 & 3 \\
0 & 1 & -\frac{1}{3} & -\frac{7}{3} \\
0 & 3 & -1 & -9
\end{bmatrix}
\]
$\overset{III = III - 3II}{\leadsto}$
\[
\begin{bmatrix}[ccc|c]
1 & -1 & 1 & 3 \\
0 & 1 & -\frac{1}{3} & -\frac{7}{3} \\
0 & 0 & 0 & -2
\end{bmatrix}
\]
\begin{alignat*}{3}
\overset{III}{\Rightarrow} & 0x_{3} &\,=\,& -2 &&
\end{alignat*}
Es gibt keine Lösung, da die Gleichung $0x_{3} = - 2$ durch kein $x_{3}$ erfüllt wird.
\subsection{} %d
\[
\begin{bmatrix}[ccc|c]
3 & \frac{3}{2} & \frac{3}{2} & 6 \\
2 & 1 & 1 & 4 \\
4 & 2 & 2 & 8
\end{bmatrix}
\]
$\overset{II \curvearrowright I, I \curvearrowright II}{\leadsto}$
\[
\begin{bmatrix}[ccc|c]
2 & 1 & 1 & 4 \\
3 & \frac{3}{2} & \frac{3}{2} & 6 \\
4 & 2 & 2 & 8
\end{bmatrix}
\]
$\overset{I = \frac{1}{2}I}{\leadsto}$
\[
\begin{bmatrix}[ccc|c]
1 & \frac{1}{2} & \frac{1}{2} & 2 \\
3 & \frac{3}{2} & \frac{3}{2} & 6 \\
4 & 2 & 2 & 8
\end{bmatrix}
\]
$\overset{II = II - 3I, III = III - 4I}{\leadsto}$
\[
\begin{bmatrix}[ccc|c]
1 & \frac{1}{2} & \frac{1}{2} & 2 \\
0 & 0 & 0 & 0 \\
0 & 0 & 0 & 0
\end{bmatrix}
\]
\begin{alignat*}{3}
\overset{III}{\Rightarrow} & 0x_{3} &\,=\,& 0 && \\
\Leftrightarrow & x_{3} &\,=\,& t, t \in \mathbb{R} && \\
\overset{II}{\Rightarrow} & 0x_{2} &\,=\,& 0 && \\
\Leftrightarrow & x_{2} &\,=\,& s, s \in \mathbb{R} && \\
\overset{I}{\Rightarrow} & x_{1} + \frac{1}{2}s + \frac{1}{2}t &\,=\,& 2 && \;|-\frac{1}{2}s, -\frac{1}{2}t \\
\Leftrightarrow & x_{1} &\,=\,& 2 - \frac{1}{2}s - \frac{1}{2}t &&
\end{alignat*}
Es gibt unendlich viele Lösungen.
\section{} %2
$x_{1}$, $x_{2}$ und $x_{5}$ sind die führenden Variablen. Die restlichen Variablen sind die freien Variablen.
\\
\\
\begin{alignat*}{3}
\overset{IV}{\Rightarrow} & 0x_{6} &\,=\,& 0 && \\
\Leftrightarrow & x_{6} &\,=\,& t, t \in \mathbb{R} \\
\overset{III}{\Rightarrow} & x_{5} - 3t &\,=\,& -2 && | + 3t \\
\Leftrightarrow & x_{5} &\,=\,& 3t - 2 & \\
\overset{II}{\Rightarrow} & x_{2} + 2x_{3} + 3x_{3} &\,=\,& 1 && \\
\Leftrightarrow & x_{3} &\,=\,& r, r \in \mathbb{R} & \\
& x_{4} &\,=\,& s, s \in \mathbb{R} && \\
\Rightarrow & x_{2} + 2r + 3s &\,=\,& 1 && | -2r, -3s \\
\Leftrightarrow & x_{2} &\,=\,& -2r - 3s + 1 && \\
\overset{I}{\Rightarrow} & x_{1} + 2(-2r - 3s + 1) - r + 3s - (3t - 2) + 2t &\,=\,& 1 && | \text{Kl. aufl.} \\
\Leftrightarrow & x_{1} - 4r - 6s + 2 - r + 3s -3t + 2 + 2t &\,=\,& 1 && | \text{Zus.} \\
\Leftrightarrow & x_{1} - t - 5r - 3s + 4 &\,=\,& 1 && | +t, +5r, +3s, -4 \\
\Leftrightarrow & x_{1} &\,=\,& 5r + 3s + t - 3 &&
\end{alignat*}
\section{} %3
Prüfe, ob $u$ und $w$ Linearkombinationen von $v_{1}$, $v_{2}$ und $v_{3}$ sind:\\
\[
\begin{bmatrix}[ccc|cc]
1 & 0 & -1 & 1 & -2 \\
0 & -1 & 4 & 3 & 2 \\
0 & 1 & 2 & 6 & 4 \\
3 & 2 & 1 & 15 & 1
\end{bmatrix}
\]
$\overset{IV = IV - 3I}{\leadsto}$
\[
\begin{bmatrix}[ccc|cc]
1 & 0 & -1 & 1 & -2 \\
0 & -1 & 4 & 3 & 2 \\
0 & 1 & 2 & 6 & 4\\
0 & 2 & 4 & 12 & 7
\end{bmatrix}
\]
$\overset{II \curvearrowright III, III \curvearrowright II}{\leadsto}$
\[
\begin{bmatrix}[ccc|cc]
1 & 0 & -1 & 1 & -2 \\
0 & 1 & 2 & 6 & 4\\
0 & -1 & 4 & 3 & 2 \\
0 & 2 & 4 & 12 & 7
\end{bmatrix}
\]
$\overset{III = III + II, IV = IV - 2II}{\leadsto}$
\[
\begin{bmatrix}[ccc|cc]
1 & 0 & -1 & 1 & -2 \\
0 & 1 & 2 & 6 & 4 \\
0 & 0 & 6 & 9 & 6 \\
0 & 0 & 0 & 0 & -1
\end{bmatrix}
\]
$\overset{III = \frac{1}{6}III}{\leadsto}$
\[
\begin{bmatrix}[ccc|cc]
1 & 0 & -1 & 1 & -2 \\
0 & 1 & 2 & 6 & 4 \\
0 & 0 & 1 & \frac{3}{2} & 1 \\
0 & 0 & 0 & 0 & -1
\end{bmatrix}
\]
$u$:\\
\begin{alignat*}{3}
\overset{IV}{\Rightarrow} & 0v_{3} &\,=\,& 0 && \\
\Leftrightarrow & v_{3} &\,=\,& t, t \in \mathbb{R} && \\
\overset{III}{\Rightarrow} & v_{3} &\,=\,& \frac{3}{2} && \\
\overset{II}{\Rightarrow} & v_{2} + 2(\frac{3}{2}) &\,=\,& 6 && \;| \text{Kl. aufl.} \\
\Leftrightarrow & v_{2} + 3 &\,=\,& 6 && \;| -3 \\
\Leftrightarrow & v_{2} &\,=\,& 3 && \\
\overset{I}{\Rightarrow} & v_{1} + 0 \cdot 3 - \frac{3}{2} &\,=\,& 1 && \;| \text{Zus.}\\
\Leftrightarrow & v_{1} - \frac{3}{2} &\,=\,& 1 && \;| + \frac{3}{2} \\
\Leftrightarrow & v_{1} &\,=\,& \frac{5}{2} &&
\end{alignat*}
Der Vektor $u$ ist eine Linearkombination von $v_{1}$, $v_{2}$ und $v_{3}$.
$w$:\\
\begin{alignat*}{3}
\overset{IV}{\Rightarrow} & 0v_{3} &\,=\,& -1 &&
\end{alignat*}
Der Vektor $w$ ist keine Linearkombination von $v_{1}$, $v_{2}$ und $v_{3}$, da es kein $v_{3}$ gibt, für das die Gleichung $0v_{3} = -1$ gilt.
\section{} %4
\subsection{} %a
\subsubsection{} %i
\begin{alignat*}{2}
A^{-1} &=& \frac{1}{4 \cdot 2 - 1 \cdot 3} \begin{bmatrix} 2 & -1 \\ -3 & 4 \end{bmatrix} \\
&=& \frac{1}{5} \begin{bmatrix} 2 & -1 \\ -3 & 4 \end{bmatrix}
\end{alignat*}
\subsubsection{} %ii
\begin{alignat*}{2}
\begin{bmatrix}[c|c]
A & E_{2}
\end{bmatrix} &=&
\begin{bmatrix}[cc|cc]
4 & 1 & 1 & 0 \\
3 & 2 & 0 & 1
\end{bmatrix} \\
\intertext{$I = \frac{1}{4}I$}
&=&
\begin{bmatrix}[cc|cc]
1 & \frac{1}{4} & \frac{1}{4} & 0 \\
3 & 2 & 0 & 1
\end{bmatrix} \\
\intertext{$II = II - 3I$}
&=&
\begin{bmatrix}[cc|cc]
1 & \frac{1}{4} & \frac{1}{4} & 0 \\
0 & \frac{5}{4} & -\frac{3}{4} & 1
\end{bmatrix} \\
\intertext{$II = \frac{4}{5}II$}
&=&
\begin{bmatrix}[cc|cc]
1 & \frac{1}{4} & \frac{1}{4} & 0 \\
0 & 1 & -\frac{3}{5} & \frac{4}{5}
\end{bmatrix} \\
\intertext{$I = I - \frac{1}{4}II$}
&=&
\begin{bmatrix}[cc|cc]
1 & 0 & \frac{2}{5} & -\frac{1}{5} \\
0 & 1 & -\frac{3}{5} & \frac{4}{5}
\end{bmatrix} \\
\end{alignat*}
\subsection{} %b
\begin{alignat*}{2}
\begin{bmatrix}[c|c]
A & E_{3}
\end{bmatrix} &=&
\begin{bmatrix}[ccc|ccc]
2 & 4 & -2 & 1 & 0 & 0 \\
4 & 9 & -3 & 0 & 1 & 0 \\
-2 & -3 & 7 & 0 & 0 & 1
\end{bmatrix} \\
\intertext{$I = \frac{1}{2}I$}
&=&
\begin{bmatrix}[ccc|ccc]
1 & 2 & -1 & \frac{1}{2} & 0 & 0 \\
4 & 9 & -3 & 0 & 1 & 0 \\
-2 & -3 & 7 & 0 & 0 & 1
\end{bmatrix} \\
\intertext{$II = II - 4I, III = III + 2I$}
&=&
\begin{bmatrix}[ccc|ccc]
1 & 2 & -1 & \frac{1}{2} & 0 & 0 \\
0 & 1 & 1 & -2 & 1 & 0 \\
0 & 1 & 5 & 1 & 0 & 1
\end{bmatrix} \\
\intertext{$III = III - II$}
&=&
\begin{bmatrix}[ccc|ccc]
1 & 2 & -1 & \frac{1}{2} & 0 & 0 \\
0 & 1 & 1 & -2 & 1 & 0 \\
0 & 0 & 4 & 3 & -1 & 1
\end{bmatrix} \\
\intertext{$III = \frac{1}{4}III$}
&=&
\begin{bmatrix}[ccc|ccc]
1 & 2 & -1 & \frac{1}{2} & 0 & 0 \\
0 & 1 & 1 & -2 & 1 & 0 \\
0 & 0 & 1 & \frac{3}{4} & -\frac{1}{4} & \frac{1}{4}
\end{bmatrix} \\
\intertext{$II = II - III, I = I + III$}
&=&
\begin{bmatrix}[ccc|ccc]
1 & 2 & 0 & \frac{5}{4} & -\frac{1}{4} & \frac{1}{4} \\
0 & 1 & 0 & -\frac{11}{4} & \frac{5}{4} & -\frac{1}{4} \\
0 & 0 & 1 & \frac{3}{4} & -\frac{1}{4} & \frac{1}{4}
\end{bmatrix} \\
\intertext{$I = I - 2II$}
&=&
\begin{bmatrix}[ccc|ccc]
1 & 0 & 0 & \frac{27}{4} & -\frac{11}{4} & \frac{3}{4} \\
0 & 1 & 0 & -\frac{11}{4} & \frac{5}{4} & -\frac{1}{4} \\
0 & 0 & 1 & \frac{3}{4} & -\frac{1}{4} & \frac{1}{4}
\end{bmatrix}
\end{alignat*}
\begin{alignat*}{2}
AA^{-1} &=&
\begin{bmatrix}
2 & 4 & -2 \\
4 & 9 & -3 \\
-2 & -3 & 7
\end{bmatrix} \cdot
\begin{bmatrix}
\frac{27}{4} & -\frac{11}{4} & \frac{3}{4} \\
-\frac{11}{4} & \frac{5}{4} & -\frac{1}{4} \\
\frac{3}{4} & -\frac{1}{4} & \frac{1}{4}
\end{bmatrix} \\
&=& \begin{bmatrix}
1 & 0 & 0 \\
0 & 1 & 0 \\
0 & 0 & 1
\end{bmatrix}
\end{alignat*}
\begin{alignat*}{3}
Ax &=& b^{T} && \;| \cdot A^{-1} \\
(A^{-1}A)x &=& A^{-1} \cdot b && \\
x &=& A^{-1} \cdot b && \\
&=&
\begin{bmatrix}
\frac{27}{4} & -\frac{11}{4} & \frac{3}{4} \\
-\frac{11}{4} & \frac{5}{4} & -\frac{1}{4} \\
\frac{3}{4} & -\frac{1}{4} & \frac{1}{4}
\end{bmatrix} \cdot
\begin{bmatrix}
-2 \\
4 \\
2
\end{bmatrix} \\
&=&
\begin{bmatrix}
-23 \\
10 \\
-4
\end{bmatrix}
\end{alignat*}
\subsection{} %c
\begin{alignat*}{2}
\begin{bmatrix}[c|c]
B & E_{3}
\end{bmatrix} &=&
\begin{bmatrix}[ccc|ccc]
-1 & -2 & 3 & 1 & 0 & 0 \\
6 & 4 & -1 & 0 & 1 & 0 \\
-10 & -12 & 13 & 0 & 0 & 1
\end{bmatrix} \\
\intertext{$I = -I$}
&=&
\begin{bmatrix}[ccc|ccc]
1 & 2 & -3 & -1 & 0 & 0 \\
6 & 4 & -1 & 0 & 1 & 0 \\
-10 & -12 & 13 & 0 & 0 & 1
\end{bmatrix} \\
\intertext{$II = -6I, III = III + 10I$}
&=&
\begin{bmatrix}[ccc|ccc]
1 & 2 & -3 & -1 & 0 & 0 \\
0 & -8 & 17 & 0 & 1 & 0 \\
0 & 8 & -17 & 0 & 0 & 1
\end{bmatrix} \\
\intertext{$II = -\frac{1}{8}II$}
&=&
\begin{bmatrix}[ccc|ccc]
1 & 2 & -3 & -1 & 0 & 0 \\
0 & 1 & -\frac{17}{8} & 0 & -\frac{1}{8} & 0 \\
0 & 8 & -17 & 0 & 0 & 1
\end{bmatrix} \\
\intertext{$III = III - 8II$}
&=&
\begin{bmatrix}[ccc|ccc]
1 & 2 & -3 & -1 & 0 & 0 \\
0 & 1 & -\frac{17}{8} & 0 & -\frac{1}{8} & 0 \\
0 & 0 & 0 & 0 & 0 & 1
\end{bmatrix}
\end{alignat*}
Auf der linken Seite der Blockmatrix kann unmöglich die Einheitsmatrix erreicht werden. Daher hat die Matrix B keine inverse Matrix.
\end{document}

View File

@ -0,0 +1,429 @@
\documentclass[10pt,a4paper,oneside,ngerman,numbers=noenddot]{scrartcl}
\usepackage[T1]{fontenc}
\usepackage[utf8]{inputenc}
\usepackage[ngerman]{babel}
\usepackage{amsmath}
\usepackage{amsfonts}
\usepackage{amssymb}
\usepackage{paralist}
\usepackage{gauss}
\usepackage[locale=DE,exponent-product=\cdot,detect-all]{siunitx}
\usepackage{tikz}
\usetikzlibrary{matrix,fadings,calc,positioning,decorations.pathreplacing,arrows,decorations.markings}
\usepackage{polynom}
\polyset{style=C, div=:,vars=x}
\pagenumbering{arabic}
\def\thesection{\arabic{section})}
\def\thesubsection{\alph{subsection})}
\def\thesubsubsection{(\roman{subsubsection})}
\makeatletter
\renewcommand*\env@matrix[1][*\c@MaxMatrixCols c]{%
\hskip -\arraycolsep
\let\@ifnextchar\new@ifnextchar
\array{#1}}
\makeatother
\begin{document}
\author{Jim Martens}
\title{Hausaufgaben zum 24./25. Januar}
\maketitle
\section{} %1
\subsection{} %a
\begin{equation*}
x =
\begin{bmatrix}
2 \\
-1 \\
3
\end{bmatrix} + t \cdot
\begin{bmatrix}
5 \\
-2 \\
-1
\end{bmatrix}
\end{equation*}
\subsection{} %b
\subsubsection{} %(i)
\begin{equation*}
x =
\begin{bmatrix}
5 \\
1 \\
2
\end{bmatrix} + s \cdot
\begin{bmatrix}
-8 \\
0 \\
2
\end{bmatrix} + t \cdot
\begin{bmatrix}
-3 \\
-2 \\
1
\end{bmatrix}
\end{equation*}
\subsubsection{} %(ii)
\begin{alignat*}{2}
\underset{0D}{\rightarrow} &=&
\underset{0A}{\rightarrow} + \underset{AB}{\rightarrow} + \underset{AC}{\rightarrow} \\
&=&
\begin{bmatrix}
-6 \\
-3 \\
5
\end{bmatrix} \\
x &=&
\begin{bmatrix}
-6 \\
-3 \\
5
\end{bmatrix} + s \cdot
\begin{bmatrix}
3 \\
4 \\
-1
\end{bmatrix} + t \cdot
\begin{bmatrix}
8 \\
2 \\
-2
\end{bmatrix}
\end{alignat*}
\subsection{} %c
\begin{alignat*}{3}
&& \begin{bmatrix}
x_{1} \\
x_{2} \\
x_{3}
\end{bmatrix} &=&
\begin{bmatrix}
5 \\
1 \\
2
\end{bmatrix} + s \cdot
\begin{bmatrix}
-8 \\
0 \\
2
\end{bmatrix} + t \cdot
\begin{bmatrix}
-3 \\
-2 \\
1
\end{bmatrix} \\
\Rightarrow & I & x_{1} &=& 5 - 8s - 3t \\
& II & x_{2} &=& 1 - 2t \\
& III & x_{3} &=& 2 + 2s + t \\
\overset{II}{\Rightarrow} && x_{2} &=& 1 - 2t & \;| -1 \\
&& x_{2} - 1 &=& -2t & \;| \cdot -\frac{1}{2} \\
&& -\frac{1}{2}x_{2} + \frac{1}{2} &=& t & \\
\overset{III}{\Rightarrow} && x_{3} &=& 2 + 2s + t & \\
&& x_{3} &=& 2 + 2s + (-\frac{1}{2}x_{2} + \frac{1}{2}) & \;| \text{Kl. aufl. + Zus.} \\
&& x_{3} &=& 2s - \frac{1}{2}x_{2} + \frac{5}{2} & \;| + \frac{1}{2}x_{2}, - \frac{5}{2} \\
&& x_{3} + \frac{1}{2}x_{2} - \frac{5}{2} &=& 2s & \;| \cdot \frac{1}{2} \\
&& \frac{1}{2}x_{3} + \frac{1}{4}x_{2} - \frac{5}{4} &=& s & \\
\overset{I}{\Rightarrow} && x_{1} &=& 5 - 8s - 3t & \\
&& x_{1} &=& 5 - 8(\frac{1}{2}x_{3} + \frac{1}{4}x_{2} - \frac{5}{4}) - 3(-\frac{1}{2}x_{2} + \frac{1}{2}) & \;| \text{Kl. aufl.} \\
&& x_{1} &=& 5 - 4x_{3} - 2x_{2} + 10 + \frac{3}{2}x_{2} - \frac{3}{2} & \;| \text{Zus.} \\
&& x_{1} &=& - 4x_{3} - \frac{1}{2}x_{2} + \frac{27}{2} & \;| +4x_{3} + \frac{1}{2}x_{2} - \frac{27}{2} \\
&& x_{1} + \frac{1}{2}x_{2} + 4x_{3} - \frac{27}{2} &=& 0 &
\end{alignat*}
\\
Schnittpunkt mit der $x_{1}$-Achse:\\
\begin{alignat*}{2}
x_{1} + \frac{1}{2}x_{2} + 4x_{3} - \frac{27}{2} &=& 0 & \;| x_{2} = 0, x_{3} = 0 \\
x_{1} - \frac{27}{2} &=& 0 & \;| + \frac{27}{2} \\
x_{1} &=& \frac{27}{2} &
\end{alignat*}
\\
Schnittpunkt mit der $x_{2}$-Achse:\\
\begin{alignat*}{2}
x_{1} + \frac{1}{2}x_{2} + 4x_{3} - \frac{27}{2} &=& 0 & \;| x_{1} = 0, x_{3} = 0 \\
\frac{1}{2}x_{2} - \frac{27}{2} &=& 0 & \;| + \frac{27}{2} \\
\frac{1}{2}x_{2} &=& \frac{27}{2} & \;| \cdot 2 \\
x_{2} &=& 27 &
\end{alignat*}
\\
Schnittpunkt mit der $x_{3}$-Achse:\\
\begin{alignat*}{2}
x_{1} + \frac{1}{2}x_{2} + 4x_{3} - \frac{27}{2} &=& 0 & \;| x_{1} = 0, x_{2} = 0 \\
4x_{3} - \frac{27}{2} &=& 0 & \;| + \frac{27}{2} \\
4x_{3} &=& \frac{27}{2} & \;| \cdot \frac{1}{4} \\
x_{3} &=& \frac{27}{8} &
\end{alignat*}
\section{} %2
\begin{alignat*}{3}
3x_{1} - 2x_{2} + 8x_{3} - 10 &=& 0 & \;| + 2x_{2} \\
2x_{2} &=& -10 + 3x_{1} + 8x_{3} & \;| \cdot \frac{1}{2} \\
x_{2} &=& -5 + \frac{3}{2}x_{1} + 4x_{3} &
\end{alignat*}
\begin{alignat*}{5}
\Rightarrow & x_{1} &=&& 0 &\,+\,& 1x_{1} &\,+\,& 0x_{3} \\
& x_{2} &=&& -5 &\,+\,& \frac{3}{2}x_{1} &\,+\,& 4x_{3}\\
& x_{3} &=&& 0 &\,+\,& 0x_{1} &\,+\,& 1x_{3}
\end{alignat*}
Daraus ergibt sich:\\
\begin{alignat*}{2}
\begin{bmatrix}
x_{1} \\
x_{2} \\
x_{3}
\end{bmatrix} &=&
\begin{bmatrix}
0 \\
-5 \\
0
\end{bmatrix} + x_{1}
\begin{bmatrix}
1 \\
\frac{3}{2} \\
0
\end{bmatrix} + x_{3}
\begin{bmatrix}
0 \\
4 \\
1
\end{bmatrix} \\
\intertext{$x_{1} = s, x_{3} = t$}
x &=&
\begin{bmatrix}
0 \\
-5 \\
0
\end{bmatrix} + s
\begin{bmatrix}
1 \\
\frac{3}{2} \\
0
\end{bmatrix} + t
\begin{bmatrix}
0 \\
4 \\
1
\end{bmatrix}
\end{alignat*}
\section{} %3
\subsection{} %a
\begin{alignat*}{3}
& a \cdot b &=& (2 \cdot 3) + (-1 \cdot 2) + (5 \cdot (-3)) & \;|\text{Kl. aufl.} \\
& &=& 6 - 2 - 15 & \;|\text{Zus.} \\
& &=& -11 & \\
\intertext{Einsetzen von $z$}
& a \cdot b &=& (2 \cdot 3) + (-1 \cdot 2) + (5 \cdot z) & \;|\text{Kl. aufl.} \\
& &=& 6 - 2 + 5z & \;|\text{Zus.} \\
& &=& 4 + 5z & \\
\Rightarrow & 0 &=& 4 + 5z & \;| -4 \\
& 5z &=& -4 & \;| \cdot \frac{1}{5} \\
& z &=& -\frac{4}{5} &
\end{alignat*}
$z$ ist $-\frac{4}{5}$.
\subsection{} %b
\begin{alignat*}{2}
|a| &=& \sqrt{2^{2} + (-1)^{2} + 5^{2}} \\
&=& \sqrt{4 + 1 + 25} \\
&=& \sqrt{30} \\
&\approx & 5,48
\end{alignat*}
\\
Errechnung von d:\\
\begin{alignat*}{2}
& \begin{bmatrix}
d_{1} \\
d_{2} \\
d_{3}
\end{bmatrix} &=& \frac{1}{\sqrt{30}} \cdot
\begin{bmatrix}
2 \\
-1 \\
5
\end{bmatrix} \\
\Rightarrow I: & d_{1} &=& \frac{2}{\sqrt{30}} \\
II: & d_{2} &=& -\frac{1}{\sqrt{30}} \\
III: & d_{3} &=& \frac{5}{\sqrt{30}}
\end{alignat*}
\subsection{} %c
\begin{alignat*}{2}
|P_{1}P_{2}| &=& \sqrt{(0-4)^{2} + (3-2)^{2} + (1-1)^{2}} \\
&=& \sqrt{-4^{2} + {1}^{2}} \\
&=& \sqrt{16 + 1} \\
&=& \sqrt{17} \\
&\approx & 4,12
\end{alignat*}
\subsection{} %d
\begin{alignat*}{2}
cos \phi &=& \frac{u \cdot v}{|u| \cdot |v|} \\
&=& \frac{2}{4 \cdot \sqrt{5}} \\
&=& \frac{1}{2 \cdot \sqrt{5}} \\
&\approx & 0,22 \\
arccos(0,22) &\approx & 77,29^{\circ}
\end{alignat*}
\section{} %4
\subsection{} %a
Es gilt: \\
\begin{alignat*}{2}
x \cdot u &=& 0 \\
x \cdot v &=& 0 \\
\begin{bmatrix}
x_{1} \\
x_{2} \\
x_{3}
\end{bmatrix} \cdot
\begin{bmatrix}
1 \\
2 \\
3
\end{bmatrix} &=& 0 \\
\begin{bmatrix}
x_{1} \\
x_{2} \\
x_{3}
\end{bmatrix} \cdot
\begin{bmatrix}
-4 \\
-7 \\
5
\end{bmatrix} &=& 0 \\
\intertext{In LGS umformen}
I: 1x_{1} + 2x_{2} + 3x_{3} &=& 0 \\
II: -4x_{1} - 7x_{2} + 5x_{3} &=& 0 \\
\intertext{II = II + 4I}
1x_{1} + 2x_{2} + 3x_{3} &=& 0 \\
0x_{1} + 1x_{2} + 17x_{3} &=& 0 \\
\intertext{II = II - $17x_{3}$}
x_{2} &=& -17x_{3} \\
\intertext{In I einsetzen}
1x_{1} + 2 \cdot (-17x_{3}) + 3x_{3} &=& 0 \\
x_{1} - 31x_{3} &=& 0 \\
\intertext{I = I + $31x_{3}$}
x_{1} &=& 31x_{3} \\
\intertext{Sei $x_{3} = t, t \in \mathbb{R}$}
x_{1} &=& 31t \\
\intertext{Daraus ergibt sich für $x_{2}$}
x_{2} &=& -17t \\
\intertext{Daraus folgt für $x$}
x &=& \begin{bmatrix}
31t \\
-17t \\
t
\end{bmatrix}, \; t \in \mathbb{R} \\
&=& t \cdot \begin{bmatrix}
31 \\
-17 \\
1
\end{bmatrix}, \; t \in \mathbb{R}
\end{alignat*}
\subsection{} %b
Es gilt:\\
\begin{alignat*}{2}
x \cdot u &=& 0 \\
\begin{bmatrix}
x_{1} \\
x_{2} \\
x_{3}
\end{bmatrix} \cdot
\begin{bmatrix}
2 \\
4 \\
1
\end{bmatrix} &=& 0 \\
\intertext{In LGS umwandeln}
2x_{1} + 4x_{2} + x_{3} &=& 0 \\
\intertext{Umstellen nach $x_{3}$}
x_{3} &=& -2x_{1} - 4x_{2} \\
\intertext{Sei $x_{1} = s, s \in \mathbb{R}$ und $x_{2} = t, t \in \mathbb{R}$}
x_{3} &=& -2s - 4t \\
\intertext{Daraus ergibt sich für $x$}
x &=& \begin{bmatrix}
s \\
t \\
-2s - 4t
\end{bmatrix}, \; s,t \in \mathbb{R} \\
&=& s \cdot \begin{bmatrix}
1 \\
0 \\
-2
\end{bmatrix} + t \cdot
\begin{bmatrix}
0 \\
1 \\
-4
\end{bmatrix}, \; s,t \in \mathbb{R}
\end{alignat*}
\subsection{} %c
Es gilt: \\
\begin{alignat*}{2}
x \cdot u &=& 0 \\
x \cdot v &=& 0 \\
\begin{bmatrix}
x_{1} \\
x_{2} \\
x_{3} \\
x_{4}
\end{bmatrix} \cdot
\begin{bmatrix}
1 \\
2 \\
-1 \\
2
\end{bmatrix} &=& 0 \\
\begin{bmatrix}
x_{1} \\
x_{2} \\
x_{3} \\
x_{4}
\end{bmatrix} \cdot
\begin{bmatrix}
1 \\
1 \\
4 \\
2
\end{bmatrix} &=& 0 \\
\intertext{In LGS umformen}
I: 1x_{1} + 2x_{2} - 1x_{3} + 2x_{4} &=& 0 \\
II: 1x_{1} + 1x_{2} + 4x_{3} + 2x_{4} &=& 0 \\
\intertext{II = II - I}
1x_{1} + 2x_{2} - 1x_{3} + 2x_{4} &=& 0 \\
0x_{1} - 1x_{2} + 5x_{3} + 0x_{4} &=& 0 \\
\intertext{II = II + $x_{2}$}
x_{2} &=& 5x_{3} \\
\intertext{In I einsetzen}
x_{1} + 2 \cdot (5x_{3}) - x_{3} + 2x_{4} &=& 0 \\
x_{1} + 9x_{3} + 2x_{4} &=& 0 \\
\intertext{Nach $x_{1}$ umstellen}
x_{1} &=& -9x_{3} - 2x_{4} \\
\intertext{Sei $x_{3} = s, s \in \mathbb{R}$ und $x_{4} = t, t \in \mathbb{R}$}
x_{1} &=& -9s - 2t \\
\intertext{Daraus ergibt sich für $x_{2}$}
x_{2} &=& 5s \\
\intertext{Daraus folgt für $x$}
x &=& \begin{bmatrix}
-9s - 2t \\
5s \\
s \\
t
\end{bmatrix}, \; s,t \in \mathbb{R} \\
&=& s \cdot \begin{bmatrix}
-9 \\
5 \\
1 \\
0
\end{bmatrix} + t \cdot
\begin{bmatrix}
-2 \\
0 \\
0 \\
1
\end{bmatrix}, \; s,t \in \mathbb{R}
\end{alignat*}
\subsection{} %d
\begin{alignat*}{2}
|u| &=& \sqrt{1^{2} + 2^{2} + (-1)^{2} + 2^{2}} \\
&=& \sqrt{1 + 4 + 1 + 4} \\
&=& \sqrt{10} \\
&\approx & 3,16 \\
|v| &=& \sqrt{1^{2} + 1^{2} + 4^{2} + 2^{2}} \\
&=& \sqrt{1 + 1 + 16 + 4} \\
&=& \sqrt{22} \\
&\approx & 4,69
\end{alignat*}
\end{document}

View File

@ -0,0 +1,777 @@
\documentclass[10pt,a4paper,oneside,ngerman,numbers=noenddot]{scrartcl}
\usepackage[T1]{fontenc}
\usepackage[utf8]{inputenc}
\usepackage[ngerman]{babel}
\usepackage{amsmath}
\usepackage{amsfonts}
\usepackage{amssymb}
\usepackage{paralist}
\usepackage{gauss}
\usepackage[locale=DE,exponent-product=\cdot,detect-all]{siunitx}
\usepackage{tikz}
\usetikzlibrary{matrix,fadings,calc,positioning,decorations.pathreplacing,arrows,decorations.markings}
\usepackage{polynom}
\polyset{style=C, div=:,vars=x}
\pagenumbering{arabic}
\def\thesection{\arabic{section})}
\def\thesubsection{\alph{subsection})}
\def\thesubsubsection{(\roman{subsubsection})}
\makeatletter
\renewcommand*\env@matrix[1][*\c@MaxMatrixCols c]{%
\hskip -\arraycolsep
\let\@ifnextchar\new@ifnextchar
\array{#1}}
\makeatother
\begin{document}
\author{Jim Martens}
\title{Hausaufgaben zum 31. Januar/1. Februar}
\maketitle
\section{} %1
\subsection{} %a
In ein LGS umgeformt, ergibt sich dieses Bild:\\
\begin{alignat*}{3}
x_{1} &=& \frac{5}{3} &-& \frac{2}{3}t \\
x_{2} &=& - \frac{4}{3} &+& \frac{1}{3}t \\
x_{3} &=& 0 &+& 1t \\
\intertext{Daraus ergeben sich diese Vektoren}
\begin{bmatrix}
x_{1} \\
x_{2} \\
x_{3}
\end{bmatrix} &=&
\begin{bmatrix}
\frac{5}{3} \\
-\frac{4}{3} \\
0
\end{bmatrix} &+&
\begin{bmatrix}
-\frac{2}{3}t \\
\frac{1}{3}t \\
1t
\end{bmatrix} \\
\intertext{t vor die Matrix ziehen}
&=& \begin{bmatrix}
\frac{5}{3} \\
-\frac{4}{3} \\
0
\end{bmatrix} &+& t \cdot
\begin{bmatrix}
-\frac{2}{3} \\
\frac{1}{3} \\
1
\end{bmatrix}
\end{alignat*}
\subsection{} %b
In ein LGS umgeformt, ergibt sich dieses Bild:\\
\begin{alignat*}{4}
x_{1} &=& 2 &-& \frac{1}{2}s &-& \frac{1}{2}t \\
x_{2} &=& 0 &+& 1s &+& 0t \\
x_{3} &=& 0 &+& 0s &+& 1t \\
\intertext{Daraus ergeben sich diese Vektoren}
\begin{bmatrix}
x_{1} \\
x_{2} \\
x_{3}
\end{bmatrix} &=&
\begin{bmatrix}
2 \\
0 \\
0
\end{bmatrix} &+&
\begin{bmatrix}
\frac{1}{2}s \\
1s \\
0
\end{bmatrix} &+&
\begin{bmatrix}
-\frac{1}{2}t \\
0 \\
1t
\end{bmatrix} \\
\intertext{Koeffizienten vor die Matrix ziehen}
&=& \begin{bmatrix}
2 \\
0 \\
0
\end{bmatrix} &+& s \cdot
\begin{bmatrix}
\frac{1}{2} \\
1 \\
0
\end{bmatrix} &+& t \cdot
\begin{bmatrix}
-\frac{1}{2} \\
0 \\
1
\end{bmatrix}
\end{alignat*}
\subsection{} %c
In ein LGS umgeformt, ergibt sich dieses Bild:\\
\begin{alignat*}{5}
x_{1} &=& -3 &+& 5r &+& 3s &+& t \\
x_{2} &=& 1 &-& 2r &-& 3s &+& 0 \\
x_{3} &=& 0 &+& r &+& 0 &+& 0 \\
x_{4} &=& 0 &+& 0 &+& s &+& 0 \\
x_{5} &=& -2 &+& 0 &+& 0 &+& 3t \\
x_{6} &=& 0 &+& 0 &+& 0 &+& t \\
\intertext{Umgewandelt in Vektoren ergibt sich dies:}
\begin{bmatrix}
x_{1} \\
x_{2} \\
x_{3} \\
x_{4} \\
x_{5} \\
x_{6}
\end{bmatrix} &=&
\begin{bmatrix}
-3 \\
1 \\
0 \\
0 \\
-2 \\
0
\end{bmatrix} &+&
\begin{bmatrix}
5r \\
-2r \\
r \\
0 \\
0 \\
0
\end{bmatrix} &+&
\begin{bmatrix}
3s \\
-3s \\
0 \\
s \\
0 \\
0
\end{bmatrix} &+&
\begin{bmatrix}
t \\
0 \\
0 \\
0 \\
3t \\
t
\end{bmatrix} \\
\intertext{Koeffizienten vor Vektor ziehen}
&=&
\begin{bmatrix}
-3 \\
1 \\
0 \\
0 \\
-2 \\
0
\end{bmatrix} &+& r \cdot
\begin{bmatrix}
5 \\
-2 \\
1 \\
0 \\
0 \\
0
\end{bmatrix} &+& s \cdot
\begin{bmatrix}
3 \\
-3 \\
0 \\
1 \\
0 \\
0
\end{bmatrix} &+& t \cdot
\begin{bmatrix}
1 \\
0 \\
0 \\
0 \\
3 \\
1
\end{bmatrix}
\end{alignat*}
\section{} %2
\subsection{} %a
\begin{alignat*}{2}
u_{1} &=& \begin{bmatrix}
0 \\
0 \\
0 \\
0
\end{bmatrix} \in U \\
u_{2} &=& \begin{bmatrix}
1 \\
1 \\
1 \\
1
\end{bmatrix} \in U \\
u_{3} &=& \begin{bmatrix}
2 \\
2 \\
2 \\
2
\end{bmatrix} \not\in U \\
u_{4} &=& \begin{bmatrix}
1 \\
-1 \\
1 \\
1
\end{bmatrix} \not\in U
\end{alignat*}
U ist kein Unterraum von $V = \mathbb{R}^{4}$, da $u_{1} + u_{1} = u_{2} \not\in U$. Demnach ist $U$ nicht abgeschlossen bezüglich der Addition.
\subsection{} %b
\begin{alignat*}{2}
u_{1} &=& \begin{bmatrix}
0 \\
0 \\
0 \\
0
\end{bmatrix} \in U \\
u_{2} &=& \begin{bmatrix}
1 \\
1 \\
1 \\
1
\end{bmatrix} \in U \\
u_{3} &=& \begin{bmatrix}
2 \\
0 \\
0 \\
1
\end{bmatrix} \not\in U \\
u_{4} &=& \begin{bmatrix}
0 \\
0 \\
0 \\
-1
\end{bmatrix} \not\in U
\end{alignat*}
1. Der Nullvektor ist in $U$ enthalten. \\
2. $u,v \in U \; c,d \in \mathbb{R}^{\geq 0}$
\begin{alignat*}{2}
u &=& \begin{bmatrix}
u_{1} \\
u_{2} \\
u_{3} \\
u_{4}
\end{bmatrix} =
\begin{bmatrix}
u_{1} \\
u_{2} \\
u_{3} \\
u_{1} + c
\end{bmatrix} \\
v &=& \begin{bmatrix}
v_{1} \\
v_{2} \\
v_{3} \\
v_{4}
\end{bmatrix} =
\begin{bmatrix}
v_{1} \\
v_{2} \\
v_{3} \\
v_{1} + d
\end{bmatrix} \\
u + v &=& \begin{bmatrix}
u_{1} + v_{1} \\
u_{2} + v_{2} \\
u_{3} + v_{3} \\
(u_{1} + c) + (v_{1} + d)
\end{bmatrix} \\
&=& \begin{bmatrix}
u_{1} + v_{1} \\
u_{2} + v_{2} \\
u_{3} + v_{3} \\
u_{1} + v_{1} + c + d
\end{bmatrix}
\end{alignat*}
Wenn zwei Vektoren $u,v$ in $U$ enthalten sind, dann ist es auch ihre Summe.\\
3. $u \in U, t \in \mathbb{R}, c \in \mathbb{R}^{\geq 0}$
\begin{alignat*}{2}
u &=& \begin{bmatrix}
u_{1} \\
u_{2} \\
u_{3} \\
u_{4}
\end{bmatrix} =
\begin{bmatrix}
u_{1} \\
u_{2} \\
u_{3} \\
u_{1} + c
\end{bmatrix} \\
t \cdot u &=& t \cdot \begin{bmatrix}
u_{1} \\
u_{2} \\
u_{3} \\
u_{1} + c
\end{bmatrix} \\
&=& \begin{bmatrix}
tu_{1} \\
tu_{2} \\
tu_{3} \\
t(u_{1} + c)
\end{bmatrix} \\
&=& \begin{bmatrix}
tu_{1} \\
tu_{2} \\
tu_{3} \\
tu_{1} + tc
\end{bmatrix}
\end{alignat*}
Wenn ein Vektor in $U$ ist, dann ist es auch das Produkt mit einem beliebigen Skalar.
Daher ist $U$ ein Unterraum von $V = \mathbb{R}^{4}$.\hfill $\Box$
\subsection{} %c
\begin{alignat*}{2}
u_{1} &=& \begin{bmatrix}
0 \\
0 \\
0 \\
0
\end{bmatrix} \in U \\
u_{2} &=& \begin{bmatrix}
1 \\
1 \\
1 \\
4
\end{bmatrix} \in U \\
u_{3} &=& \begin{bmatrix}
2 \\
0 \\
0 \\
1
\end{bmatrix} \not\in U \\
u_{4} &=& \begin{bmatrix}
0 \\
0 \\
0 \\
-1
\end{bmatrix} \not\in U
\end{alignat*}
1. Der Nullvektor ist in $U$ enthalten. \\
2. $u,v \in U$
\begin{alignat*}{2}
u &=& \begin{bmatrix}
u_{1} \\
u_{2} \\
u_{3} \\
u_{4}
\end{bmatrix} =
\begin{bmatrix}
u_{1} \\
u_{2} \\
u_{3} \\
2u_{1} + u_{2} + u_{3}
\end{bmatrix} \\
v &=& \begin{bmatrix}
v_{1} \\
v_{2} \\
v_{3} \\
v_{4}
\end{bmatrix} =
\begin{bmatrix}
v_{1} \\
v_{2} \\
v_{3} \\
2v_{1} + v_{2} + v_{3}
\end{bmatrix} \\
u + v &=& \begin{bmatrix}
u_{1} + v_{1} \\
u_{2} + v_{2} \\
u_{3} + v_{3} \\
(2u_{1} + u_{2} + u_{3}) + (2v_{1} + v_{2} + v_{3})
\end{bmatrix} \\
&=& \begin{bmatrix}
u_{1} + v_{1} \\
u_{2} + v_{2} \\
u_{3} + v_{3} \\
2u_{1} + 2v_{1} + u_{2} + v_{2} + u_{3} + v_{3}
\end{bmatrix} \\
&=& \begin{bmatrix}
u_{1} + v_{1} \\
u_{2} + v_{2} \\
u_{3} + v_{3} \\
2(u_{1} + v_{1}) + (u_{2} + v_{2}) + (u_{3} + v_{3})
\end{bmatrix}
\end{alignat*}
Wenn zwei Vektoren in $U$ sind, dann ist es auch ihre Summe.\\
3. $u \in U, t \in \mathbb{R}$
\begin{alignat*}{2}
u &=& \begin{bmatrix}
u_{1} \\
u_{2} \\
u_{3} \\
u_{4}
\end{bmatrix} =
\begin{bmatrix}
u_{1} \\
u_{2} \\
u_{3} \\
2u_{1} + u_{2} + u_{3}
\end{bmatrix} \\
t \cdot u &=& t \cdot \begin{bmatrix}
u_{1} \\
u_{2} \\
u_{3} \\
2u_{1} + u_{2} + u_{3}
\end{bmatrix} \\
&=& \begin{bmatrix}
tu_{1} \\
tu_{2} \\
tu_{3} \\
t(2u_{1} + u_{2} + u_{3})
\end{bmatrix} \\
&=& \begin{bmatrix}
tu_{1} \\
tu_{2} \\
tu_{3} \\
2tu_{1} + tu_{2} + tu_{3}
\end{bmatrix}
\end{alignat*}
Wenn ein Vektor in $U$ ist, dann ist es auch das Produkt mit einem beliebigen Skalar.
Daher ist $U$ ein Unterraum von $V = \mathbb{R}^{4}$.\hfill $\Box$
\subsection{} %d
\begin{alignat*}{2}
u_{1} &=& \begin{bmatrix}
0 \\
-1 \\
0 \\
0
\end{bmatrix} \in U \\
u_{2} &=& \begin{bmatrix}
1 \\
-1 \\
-2 \\
0
\end{bmatrix} \in U \\
u_{3} &=& \begin{bmatrix}
0 \\
0 \\
0 \\
0
\end{bmatrix} \not\in U \\
u_{4} &=& \begin{bmatrix}
0 \\
0 \\
0 \\
-1
\end{bmatrix} \not\in U
\end{alignat*}
$U$ ist kein Unterraum, da der Nullvektor nicht enthalten ist.\hfill $\Box$
\section{} %3
\subsection{} %a
In einem LGS ergibt sich: \\
\begin{alignat*}{2}
c_{1} + 3c_{2} - c_{3} + 6c_{4} &=& 0 \\
c_{1} - c_{2} + 3c_{3} + 2c_{4} &=& 0 \\
2c_{1} + 0c_{2} + c_{3} + 0c_{4} &=& 0 \\
3c_{1} + c_{2} + 0c_{3} + 4c_{4} &=& 0 \\
\intertext{In Matrixform bringen}
\begin{bmatrix}[cccc|c]
1 & 3 & -1 & 6 & 0 \\
1 & -1 & 3 & 2 & 0 \\
2 & 0 & 1 & 0 & 0 \\
3 & 1 & 0 & 4 & 0
\end{bmatrix} \\
\intertext{II = II - I, III = III - 2I, IV = IV - 3I}
\begin{bmatrix}[cccc|c]
1 & 3 & -1 & 6 & 0 \\
0 & -4 & 4 & -4 & 0 \\
0 & -6 & 3 & -12 & 0 \\
0 & -8 & 3 & -14 & 0
\end{bmatrix} \\
\intertext{II = II$ \cdot -\frac{1}{4}$}
\begin{bmatrix}[cccc|c]
1 & 3 & -1 & 6 & 0 \\
0 & 1 & -1 & 1 & 0 \\
0 & -6 & 3 & -12 & 0 \\
0 & -8 & 3 & -14 & 0
\end{bmatrix} \\
\intertext{III = III + 6II, IV = IV + 8II}
\begin{bmatrix}[cccc|c]
1 & 3 & -1 & 6 & 0 \\
0 & 1 & -1 & 1 & 0 \\
0 & 0 & -3 & -6 & 0 \\
0 & 0 & -3 & -8 & 0
\end{bmatrix} \\
\intertext{III = III$ \cdot -\frac{1}{3}$}
\begin{bmatrix}[cccc|c]
1 & 3 & -1 & 6 & 0 \\
0 & 1 & -1 & 1 & 0 \\
0 & 0 & 1 & 2 & 0 \\
0 & 0 & -3 & -8 & 0
\end{bmatrix} \\
\intertext{IV = IV + 3III}
\begin{bmatrix}[cccc|c]
1 & 3 & -1 & 6 & 0 \\
0 & 1 & -1 & 1 & 0 \\
0 & 0 & 1 & 2 & 0 \\
0 & 0 & 0 & -2 & 0
\end{bmatrix} \\
\intertext{IV = IV$ \cdot -\frac{1}{2}$}
\begin{bmatrix}[cccc|c]
1 & 3 & -1 & 6 & 0 \\
0 & 1 & -1 & 1 & 0 \\
0 & 0 & 1 & 2 & 0 \\
0 & 0 & 0 & 1 & 0
\end{bmatrix} \\
\overset{IV}{\Rightarrow} c_{4} &=& 0 \\
\overset{III}{\Rightarrow} c_{3} + 2c_{4} &=& 0 \\
c_{3} + 2 \cdot 0 &=& 0 \\
c_{3} &=& 0 \\
\overset{II}{\Rightarrow} c_{2} - c_{3} + c_{4} &=& 0 \\
c_{2} - 0 + 0 &=& 0 \\
c_{2} &=& 0 \\
\overset{I}{\Rightarrow} c_{1} + 3_c{2} - c_{3} + 6c_{4} &=& 0 \\
c_{1} + 3 \cdot 0 - 0 + 6 \cdot 0 &=& 0 \\
c_{1} &=& 0
\end{alignat*}
Es gibt keine $c_{1}, c_{2}, c_{3}, c_{4}$ für die $c_{1}v_{1} + c_{2}v_{2} + c_{3}v_{3} + c_{4}v_{4} = 0$ gilt und mindestens ein $c_{i}$ ungleich Null ist.
\subsection{} %b
In einem LGS ergibt sich: \\
\begin{alignat*}{2}
c_{1} + 3c_{2} + c_{3} - 5c_{4} &=& 0 \\
c_{1} - c_{2} + 3c_{3} - c_{4} &=& 0 \\
2c_{1} + 0c_{2} - c_{3} + 8c_{4} &=& 0 \\
3c_{1} + c_{2} + 0c_{3} + 7c_{4} &=& 0 \\
\intertext{In Matrixform bringen}
\begin{bmatrix}[cccc|c]
1 & 3 & 1 & -5 & 0 \\
1 & -1 & 3 & -1 & 0 \\
2 & 0 & -1 & 8 & 0 \\
3 & 1 & 0 & 7 & 0
\end{bmatrix} \\
\intertext{II = II - I, III = III - 2I, IV = IV - 3I}
\begin{bmatrix}[cccc|c]
1 & 3 & 1 & -5 & 0 \\
0 & -4 & 2 & +4 & 0 \\
0 & -6 & -3 & 18 & 0 \\
0 & -8 & -3 & 22 & 0
\end{bmatrix} \\
\intertext{II = II$ \cdot -\frac{1}{4}$}
\begin{bmatrix}[cccc|c]
1 & 3 & 1 & -5 & 0 \\
0 & 1 & -\frac{1}{2} & -1 & 0 \\
0 & -6 & -3 & 18 & 0 \\
0 & -8 & -3 & 22 & 0
\end{bmatrix} \\
\intertext{III = III + 6II, IV = IV + 8II}
\begin{bmatrix}[cccc|c]
1 & 3 & 1 & -5 & 0 \\
0 & 1 & -\frac{1}{2} & -1 & 0 \\
0 & 0 & -6 & 12 & 0 \\
0 & 0 & -7 & 14 & 0
\end{bmatrix} \\
\intertext{III = III$ \cdot -\frac{1}{6}$}
\begin{bmatrix}[cccc|c]
1 & 3 & 1 & -5 & 0 \\
0 & 1 & -\frac{1}{2} & -1 & 0 \\
0 & 0 & 1 & -2 & 0 \\
0 & 0 & -7 & 14 & 0
\end{bmatrix} \\
\intertext{IV = IV + 7III}
\begin{bmatrix}[cccc|c]
1 & 3 & 1 & -5 & 0 \\
0 & 1 & -\frac{1}{2} & -1 & 0 \\
0 & 0 & 1 & -2 & 0 \\
0 & 0 & 0 & 0 & 0
\end{bmatrix} \\
\overset{IV}{\Rightarrow} 0c_{4} &=& 0 \\
c_{4} &=& t, t \in \mathbb{R} \\
\overset{III}{\Rightarrow} c_{3} - 2c_{4} &=& 0 \\
c_{3} - 2t &=& 0 \\
c_{3} &=& 2t \\
\overset{II}{\Rightarrow} c_{2} - \frac{1}{2}c_{3} - c_{4} &=& 0 \\
c_{2} - \frac{1}{2}(2t) - t &=& 0 \\
c_{2} - 2t &=& 0 \\
c_{2} &=& 2t \\
\overset{I}{\Rightarrow} c_{1} + 3_c{2} + c_{3} - 5c_{4} &=& 0 \\
c_{1} + 3 \cdot 2t + 2t - 5 \cdot t &=& 0 \\
c_{1} + 6t + 2t - 5t &=& 0 \\
c_{1} + 3t &=& 0 \\
c_{1} &=& -3t
\end{alignat*}
Es gibt unendlich viele $c_{1}, c_{2}, c_{3}, c_{4}$ für die $c_{1}v_{1} + c_{2}v_{2} + c_{3}v_{3} + c_{4}v_{4} = 0$ gilt und mindestens ein $c_{i}$ ungleich Null ist.
\subsection{} %c
Wenn es mehr als die triviale Lösung für das lineare Gleichungssystem $c_{1}v_{1} + c_{2}v_{2} + c_{3}v_{3} + c_{4}v_{4} = 0$ gibt, dann sind die Vektoren linear voneinander abhängig.
Gibt es nur die triviale Lösung, dann sind sie linear voneinander unabhängig.
\section{} %4
\subsection{} %a
Wenn die Vektoren $v_{1}, v_{2}, v_{3}$ linear unabhängig sind, dann gibt es nur die triviale Lösung für die Gleichung $c_{1}v_{1} + c_{2}v_{2} + c_{3}v_{3} = 0$.
In ein LGS überführt, ergibt sich dies:\\
\begin{alignat*}{2}
2c_{1} - 2c_{2} - 5c_{3} &=& 0 \\
4c_{1} + 3c_{2} + 18c_{3} &=& 0 \\
-2c_{1} + 3c_{2} + 9c_{3} &=& 0 \\
-4c_{1} + 3c_{2} + 6c_{3} &=& 0 \\
\intertext{In Matrixform ergibt sich dies}
\begin{bmatrix}[ccc|c]
2 & -2 & -5 & 0 \\
4 & 3 & 18 & 0 \\
-2 & 3 & 9 & 0 \\
-4 & 3 & 6 & 0
\end{bmatrix} \\
\intertext{I = I$ \cdot \frac{1}{2}$}
\begin{bmatrix}[ccc|c]
1 & -1 & -\frac{5}{2} & 0 \\
4 & 3 & 18 & 0 \\
-2 & 3 & 9 & 0 \\
-4 & 3 & 6 & 0
\end{bmatrix} \\
\intertext{II = II - 4I, III = III + 2I, IV = IV + 4I}
\begin{bmatrix}[ccc|c]
1 & -1 & -\frac{5}{2} & 0 \\
0 & 7 & 28 & 0 \\
0 & 1 & 4 & 0 \\
0 & -1 & -4 & 0
\end{bmatrix} \\
\intertext{II = II$ \cdot \frac{1}{7}$}
\begin{bmatrix}[ccc|c]
1 & -1 & -\frac{5}{2} & 0 \\
0 & 1 & 4 & 0 \\
0 & 1 & 4 & 0 \\
0 & -1 & -4 & 0
\end{bmatrix} \\
\intertext{III = III - II, IV = IV + II}
\begin{bmatrix}[ccc|c]
1 & -1 & -\frac{5}{2} & 0 \\
0 & 1 & 4 & 0 \\
0 & 0 & 0 & 0 \\
0 & 0 & 0 & 0
\end{bmatrix} \\
\overset{IV}{\Rightarrow} 0c_{3} &=& 0 \\
c_{3} &=& t, t \in \mathbb{R} \\
\overset{II}{\Rightarrow} c_{2} + 4c_{3} &=& 0 \\
c_{2} + 4t &=& 0 \\
c_{2} &=& -4t \\
\overset{I}{\Rightarrow} c_{1} - c_{2} - \frac{5}{2}c_{3} &=& 0 \\
c_{1} + 4t - \frac{5}{2}t &=& 0 \\
c_{1} + \frac{3}{2}t &=& 0 \\
c_{1} &=& -\frac{3}{2}t
\end{alignat*}
Es gibt unendlich viele Lösungen der Gleichung, daher sind die drei Vektoren voneinander linear abhängig.
\subsection{} %b
Wie in a).
In ein LGS überführt, ergibt sich dies:\\
\begin{alignat*}{2}
2c_{1} - 2c_{2} - 4c_{3} &=& 0 \\
4c_{1} + 3c_{2} - 1c_{3} &=& 0 \\
-2c_{1} + 3c_{2} + 6c_{3} &=& 0 \\
-4c_{1} + 3c_{2} + 7c_{3} &=& 0 \\
\intertext{In Matrixform ergibt sich dies}
\begin{bmatrix}[ccc|c]
2 & -2 & -4 & 0 \\
4 & 3 & -1 & 0 \\
-2 & 3 & 6 & 0 \\
-4 & 3 & 7 & 0
\end{bmatrix} \\
\intertext{I = I$ \cdot \frac{1}{2}$}
\begin{bmatrix}[ccc|c]
1 & -1 & -2 & 0 \\
4 & 3 & -1 & 0 \\
-2 & 3 & 6 & 0 \\
-4 & 3 & 7 & 0
\end{bmatrix} \\
\intertext{II = II - 4I, III = III + 2I, IV = IV + 4I}
\begin{bmatrix}[ccc|c]
1 & -1 & -2 & 0 \\
0 & 7 & 7 & 0 \\
0 & 1 & 2 & 0 \\
0 & -1 & -1 & 0
\end{bmatrix} \\
\intertext{II = II$ \cdot \frac{1}{7}$}
\begin{bmatrix}[ccc|c]
1 & -1 & -2 & 0 \\
0 & 1 & 1 & 0 \\
0 & 1 & 2 & 0 \\
0 & -1 & -1 & 0
\end{bmatrix} \\
\intertext{III = III - II, IV = IV + II}
\begin{bmatrix}[ccc|c]
1 & -1 & -2 & 0 \\
0 & 1 & 1 & 0 \\
0 & 0 & 1 & 0 \\
0 & 0 & 0 & 0
\end{bmatrix} \\
\overset{III}{\Rightarrow} c_{3} &=& 0 \\
\overset{II}{\Rightarrow} c_{2} + c_{3} &=& 0 \\
c_{2} + 0 &=& 0 \\
c_{2} &=& 0 \\
\overset{I}{\Rightarrow} c_{1} - c_{2} - 2c_{3} &=& 0 \\
c_{1} - 0 - 2 \cdot 0 &=& 0 \\
c_{1} &=& 0
\end{alignat*}
Es gibt nur die triviale Lösung, daher sind die drei Vektoren voneinander linear unabhängig.
\subsection{} %c
Wie in a).
In ein LGS überführt, ergibt sich dies:\\
\begin{alignat*}{2}
1c_{1} - 1c_{2} + 2c_{3} - 1c_{4} &=& 0 \\
1c_{1} + 0c_{2} + 2c_{3} + 5c_{4} &=& 0 \\
1c_{1} + 1c_{2} + 0c_{3} + 13c_{4} &=& 0 \\
1c_{1} + 1c_{2} - 1c_{3} + 14c_{4} &=& 0 \\
\intertext{In Matrixform ergibt sich dies}
\begin{bmatrix}[cccc|c]
1 & -1 & 2 & -1 & 0 \\
1 & 0 & 2 & 5 & 0 \\
1 & 1 & 0 & 13 & 0 \\
1 & 1 & -1 & 14 & 0
\end{bmatrix} \\
\intertext{II = II - I, III = III - I, IV = IV - I}
\begin{bmatrix}[cccc|c]
1 & -1 & 2 & -1 & 0 \\
0 & 1 & 0 & 6 & 0 \\
0 & 2 & 0 & 14 & 0 \\
0 & 2 & -3 & 15 & 0
\end{bmatrix} \\
\intertext{III = III - 2II, IV = IV - 2II}
\begin{bmatrix}[cccc|c]
1 & -1 & 2 & -1 & 0 \\
0 & 1 & 0 & 6 & 0 \\
0 & 0 & 0 & 2 & 0 \\
0 & 0 & -3 & 3 & 0
\end{bmatrix} \\
\intertext{III $\curvearrowright$ IV, IV $\curvearrowright$ III}
\begin{bmatrix}[cccc|c]
1 & -1 & 2 & -1 & 0 \\
0 & 1 & 0 & 6 & 0 \\
0 & 0 & -3 & 3 & 0 \\
0 & 0 & 0 & 2 & 0
\end{bmatrix} \\
\intertext{III = III$ \cdot -\frac{1}{3}$}
\begin{bmatrix}[cccc|c]
1 & -1 & 2 & -1 & 0 \\
0 & 1 & 0 & 6 & 0 \\
0 & 0 & 1 & -1 & 0 \\
0 & 0 & 0 & 2 & 0
\end{bmatrix} \\
\intertext{IV = IV$ \cdot \frac{1}{2}$}
\begin{bmatrix}[cccc|c]
1 & -1 & 2 & -1 & 0 \\
0 & 1 & 0 & 6 & 0 \\
0 & 0 & 1 & -1 & 0 \\
0 & 0 & 0 & 1 & 0
\end{bmatrix} \\
\overset{IV}{\Rightarrow} c_{4} &=& 0 \\
\overset{III}{\Rightarrow} c_{3} - c_{4} &=& 0 \\
c_{3} - 0 &=& 0 \\
c_{3} &=& 0 \\
\overset{II}{\Rightarrow} c_{2} + 0c_{3} + 6c_{4} &=& 0 \\
c_{2} + 0 + 6 \cdot 0 &=& 0 \\
c_{2} &=& 0 \\
\overset{I}{\Rightarrow} c_{1} - c_{2} + 2c_{3} - c_{4} &=& 0 \\
c_{1} - 0 + 2 \cdot 0 - 0 &=& 0 \\
c_{1} &=& 0
\end{alignat*}
Es gibt nur die triviale Lösung, daher sind die vier Vektoren voneinander linear unabhängig.
\subsection{} %d
Es gibt fünf Vektoren bei vier Dimensionen im Vektorraum. Daher sind diese Vektoren linear voneinander abhängig.
\end{document}

View File

@ -0,0 +1,204 @@
\documentclass[10pt,a4paper,oneside,ngerman,numbers=noenddot]{scrartcl}
\usepackage[T1]{fontenc}
\usepackage[utf8]{inputenc}
\usepackage[ngerman]{babel}
\usepackage{amsmath}
\usepackage{amsfonts}
\usepackage{amssymb}
\usepackage{paralist}
\usepackage[locale=DE,exponent-product=\cdot,detect-all]{siunitx}
\usepackage{tikz}
\usetikzlibrary{matrix,fadings,calc,positioning,decorations.pathreplacing,arrows}
\pagenumbering{arabic}
\def\thesection{\arabic{section})}
\def\thesubsection{\alph{subsection})}
\def\thesubsubsection{(\arabic{subsubsection})}
\begin{document}
\author{Jim Martens}
\title{Hausaufgaben zum 08./09. November}
\maketitle
\section{}
\subsection{}
\begin{enumerate}[(i)]
\item $177 \equiv 18$ (mod 5) \\
Falsch, da $177-18=169$ ergibt und $169\, \text{mod}\, 5 \neq 0$.
\item $177 \equiv -18$ (mod 5) \\
Wahr, da $177-(-18)=195$ ergibt und $195\, \text{mod}\, 5 = 0$.
\item $-89 \equiv -12$ (mod 6) \\
Falsch, da $-89-(-12)=-77$ ergibt und $-77\, \text{mod}\, 6 \neq 0$.
\item $-123 \equiv 33$ (mod 13) \\
Wahr, da $-123-33=-156$ ergibt und $-156\, \text{mod}\, 13 = 0$.
\item $39 \equiv -1$ (mod 40) \\
Wahr, da $39-(-1)=40$ ergibt und $40\, \text{mod}\, 40 = 0$.
\item $77 \equiv 0$ (mod 11) \\
Wahr, da $77-0=77$ ergibt und $77\, \text{mod}\, 11 = 0$.
\item $2^{51} \equiv 51$ (mod 2) \\
Falsch, da $2^{x}$ immer eine gerade Zahl ergibt und bei Addition/Subtraktion einer ungeraden Zahl immer eine ungerade Zahl herauskommt. $51$ ist eine ungerade Zahl, wodurch $2^{51}-51$ nicht durch $2$ restlos teilbar ist.
\end{enumerate}
\subsection{}
\begin{alignat}{4}
7293& =&\: 19 &\: \cdot &\: 378& \indent\text{Rest}&\indent 111\\
378& =&\: 3 &\: \cdot &\: 111& \indent\text{Rest}&\indent 45\\
111& =&\: 2 &\: \cdot &\: 45& \indent\text{Rest}&\indent 21\\
45& =&\: 2 &\: \cdot &\: 21& \indent\text{Rest}&\indent 3\\
21& =&\: 7 &\: \cdot &\: 3& \indent\text{Rest}&\indent 0
\end{alignat}
Der $ggt(7293,378)$ ist $3$.
\subsection{}
\begin{enumerate}
\item $\lceil \sqrt{7} \rceil$ \\
Da $2^{2}=4$ und $3^{2}=9$ gelten, liegt die Quadratwurzel von $7$ zwischen $2$ und $3$. Durch die oberen Gaußklammern wird auf die nächsthöhere ganze Zahl aufgerundet. Daher ergibt sich $\lceil \sqrt{7} \rceil=3$.
\item $\lfloor \sqrt{7} \rfloor$ \\
Bei den unteren Gaußklammern wird auf die nächstniedrigere ganze Zahl abgerundet. Durch das eben Festgestellte gilt hier $\lfloor \sqrt{7} \rfloor = 2$.
\item $\lceil 7.1 \rceil = 8$ \\
Gilt aufgrund der eben festgestellten Sachverhalte.
\item $\lfloor 7.1 \rfloor = 7$ \\
Gilt analog.
\item $\lceil -7.1 \rceil = -7$ \\
Gilt analog.
\item $\lfloor -7.1 \rfloor = -8$ \\
Gilt analog.
\item $\lceil -7 \rceil = -7$ \\
Gilt analog.
\item $\lfloor -7 \rfloor = -8$ \\
Gilt analog.
\end{enumerate}
\section{}
\addtocounter{subsubsection}{1}
\subsubsection{}
\textbf{Behauptung:} Aus $b_{1} \mid a_{1}$ und $b_{2} \mid a_{2}$ folgt $b_{1} \cdot b_{2} \mid a_{1} \cdot a_{2}$. \\
\textbf{Beweis:}\\
$b_{1} \mid a_{1}$ bedeutet, dass $a_{1}=c \cdot b_{1}$ für ein $c \in \mathbb{Z}$ gilt; $b_{2} \mid a_{2}$ bedeutet, dass $a_{2}=d \cdot b_{2}$ für ein $d \in \mathbb{Z}$ gilt. \\
Multipliziert man die beiden Gleichungen, so ergibt sich: $a_{1} \cdot a_{2} = c \cdot d \cdot b_{1} \cdot b_{2}$. Damit ist klar, dass $b_{1} \cdot b_{2} \mid a_{1} \cdot a_{2}$ gilt.\hfill $\Box$
\subsubsection{}
\textbf{Behauptung:} Aus $c \cdot b \mid c \cdot a$ (für $c \neq 0$) folgt $b \mid a$.\\
\textbf{Beweis:}\\
$c \cdot b \mid c \cdot a$ bedeutet, dass $c \cdot a = d\cdot c \cdot b$ für ein $d \in \mathbb{Z}$ gilt. Nach teilen durch $c$ ergibt sich $a = d \cdot b$. Damit ist klar, dass $b \mid a$ gilt.\hfill $\Box$
\subsubsection{}
\textbf{Behauptung:} Aus $b \mid a_{1}$ und $b \mid a_{2}$ folgt $b \mid c_{1} \cdot a_{1} + c_{2} \cdot a_{2}$ für beliebige ganze Zahlen $c_{1}$ und $c_{2}$.\\
\textbf{Beweis:}\\
$b \mid a_{1}$ bedeutet, dass $a_{1} = c \cdot b$ für ein $c \in \mathbb{Z}$ gilt; $b \mid a_{2}$ bedeutet, dass $a_{2} = d \cdot b$ für ein $d \in \mathbb{Z}$ gilt.\\
Durch Multiplikation mit $c_{1}$ bzw. $c_{2}$ ergibt sich: $a_{1} \cdot c_{1} = c_{1} \cdot c \cdot b$ bzw. $a_{2} \cdot c_{2} = c_{2} \cdot d \cdot b$.\\
Addiert man die beiden Gleichungen ergibt sich $a_{1} \cdot c_{1} + a_{2} \cdot c_{2} = c_{1} \cdot c \cdot b + c_{2} \cdot d \cdot b$.\\
Durch Ausklammern des $b$ ergibt sich: $a_{1} \cdot c_{1} + a_{2} \cdot c_{2} = b \cdot (c_{1} \cdot c + c_{2} \cdot d)$.\\
Daher gilt $b \mid c_{1} \cdot a_{1} + c_{2} \cdot a_{2}$.\hfill $\Box$
\section{}
\subsection{}
\textbf{Behauptung:} Die Aussage $3 \mid (n^{3} + 2n)$ gilt für alle $n \geq 0$ mit $n \in \mathbb{Z}$.\\
\textbf{Beweis:} Durch vollständige Induktion.\\
Mit $A(n)$ sei die Aussage $3 \mid (n^{3} + 2 \cdot n)$ bezeichnet.\\
\underline{Induktionsanfang:} $A(0)$ ist richtig, da $3 \mid (0^{3} + 2 \cdot 0)$ bzw. $3 \mid 0$ gilt.\\\\
\underline{Induktionsannahme:} Für ein beliebig fest gewähltes $n \in \mathbb{Z}$ mit $n \geq 0$ gilt $A(n)$, d. h. es gelte $3 \mid (n^{3} + 2 \cdot n)$.\\\\
\underline{Zu zeigen:} $A(n+1)$ gilt, d. h. $3 \mid ((n+1)^{3} + 2 \cdot (n+1))$ gilt.\\\\
\underline{Induktionsschluss:}\\
Sei $3 \mid (n^{3} + 2 \cdot n)$, d. h. es gibt $c \in \mathbb{Z}$, sodass $n^3 + 2 \cdot n = 3 \cdot c$.
\begin{alignat}{2}
&\: (n+1)^{3} + 2 \cdot (n+1) &=&\: n^{3} + 1^{3} + 2n + 2\\
\Leftrightarrow &\: (n+1)^{3} + 2 \cdot (n+1) &=&\: n^{3} + 2n + 1^{3} + 2\\
\intertext{Anwendung der Induktionsannahme}
\Leftrightarrow &\: (n+1)^{3} + 2 \cdot (n+1) &=&\: 3 \cdot c + 3\\
\Leftrightarrow &\: (n+1)^{3} + 2 \cdot (n+1) &=&\: 3 \cdot (c + 1)
\end{alignat}
Nach dem Induktionsprinzip folgt aus dem Induktionsanfang und dem Induktionsschluss die Behauptung.\hfill $\Box$
\subsection{}
\textbf{Behauptung:} Für alle $n \in \mathbb{N}$ gilt, dass ein Schachfeld der Größe $2^{n} \cdot 2^{n}$ überdeckungsfrei mit L-Stücken der Größe 3 so belegt werden kann, dass nur das Feld oben rechts frei bleibt.\\
\textbf{Beweis:} Mit vollständiger Induktion.\\
\underline{Induktionsanfang:} Für $n=1$ ist die Aussage wahr, da bei $4$ Feldern das L-Stück die Felder oben links, unten links und unten rechts bedeckt.
\begin{tikzpicture}
\draw (0,0) -- +(0,1); %linke Kante
\draw (0,1) -- +(1,0); %obere Kante
\draw (0,0) -- +(1,0); %untere Kante
\draw (1,0) -- +(0,1); %rechte Kante
\draw (0.5,0) -- +(0,1); %vertikaler mittlerer Strich
\draw (0,0.5) -- +(1,0); %horizontaler mittlerer Strich
\draw (0.25,0.25) -- +(0,0.5); %linker Teil des L
\draw (0.25,0.25) -- +(0.5,0); %rechter Teil des L
\end{tikzpicture}
\\\\
\underline{Induktionsannahme:} Für ein beliebig fest gewähltes $n \in \mathbb{N}$ gilt die Behauptung, d. h. es gelte, dass ein Schachfeld der Größe $2^{n} \cdot 2^{n}$ überdeckungsfrei mit L-Stücken belegt werden kann, sodass nur das Feld oben rechts frei bleibt.\\\\
\underline{Zu zeigen:} Die Behauptung gilt auch für $n+1$.\\\\
\underline{Induktionsschluss:}\\
Sei $2^{n} \cdot 2^{n}$ ein Schachfeld mit den Kantenlängen $2^{n+1}$. Dieses kann in vier gleichgroße Teile mit der Kantenlänge $2^{n}$ aufgeteilt werden. Auf diese wende ich die Induktionsannahme an. Nun habe ich vier Mal ein Feld oben rechts frei.
\begin{tikzpicture}
\draw (0,0) -- +(0,1); %linke Kante
\draw (0,1) -- +(1,0); %obere Kante
\draw (0,0) -- +(1,0); %untere Kante
\draw (1,0) -- +(0,1); %rechte Kante
\draw (0.5,0) -- +(0,1); %vertikaler mittlerer Strich
\draw (0,0.5) -- +(1,0); %horizontaler mittlerer Strich
\draw (0.4,0.9) -- +(0,0.1); %linke Kante des oberen linken freien Feldes
\draw (0.4,0.9) -- +(0.1,0); %untere Kante des oberen linken freien Feldes
\draw (0.4,0.4) -- +(0,0.1); %linke Kante des unteren linken freien Feldes
\draw (0.4,0.4) -- +(0.1,0); %untere Kante des unteren linken freien Feldes
\draw (0.9,0.9) -- +(0,0.1); %linke Kante des oberen rechten freien Feldes
\draw (0.9,0.9) -- +(0.1,0); %untere Kante des oberen rechten freien Feldes
\draw (0.9,0.4) -- +(0,0.1); %linke Kante des unteren rechten freien Feldes
\draw (0.9,0.4) -- +(0.1,0); %untere Kante des unteren rechten freien Feldes
\end{tikzpicture}
Durch Rotation des oberen linken Teils um $90$ Grad nach rechts und des unteren rechten Teiles um $90$ Grad nach links, entstehen drei freie Felder in der Mitte. Diese können mit einem weiteren L-Stück belegt werden, sodass nur das Feld oben rechts frei bleibt.
\begin{tikzpicture}
\draw (0,0) -- +(0,1); %linke Kante
\draw (0,1) -- +(1,0); %obere Kante
\draw (0,0) -- +(1,0); %untere Kante
\draw (1,0) -- +(0,1); %rechte Kante
\draw (0.5,0) -- +(0,1); %vertikaler mittlerer Strich
\draw (0,0.5) -- +(1,0); %horizontaler mittlerer Strich
\draw (0.4,0.6) -- +(0,-0.1); %linke Kante des oberen linken freien Feldes
\draw (0.4,0.6) -- +(0.1,0); %untere Kante des oberen linken freien Feldes
\draw (0.4,0.4) -- +(0,0.1); %linke Kante des unteren linken freien Feldes
\draw (0.4,0.4) -- +(0.1,0); %untere Kante des unteren linken freien Feldes
\draw (0.9,0.9) -- +(0,0.1); %linke Kante des oberen rechten freien Feldes
\draw (0.9,0.9) -- +(0.1,0); %untere Kante des oberen rechten freien Feldes
\draw (0.6,0.4) -- +(0,0.1); %linke Kante des unteren rechten freien Feldes
\draw (0.6,0.4) -- +(-0.1,0); %untere Kante des unteren rechten freien Feldes
\draw (0.45,0.45) -- +(0,0.1); %linker Strich des L
\draw (0.45,0.45) -- +(0.1,0); %rechter Strich des L
\end{tikzpicture}
Nach dem Induktionsprinzip ergibt sich aus dem Induktionsanfang und dem Induktionsschluss die Behauptung. \hfill $\Box$
\section{}
\subsection{}
\textbf{Behauptung:} $g(x,y)$ mit $(x,y) \in \mathbb{Q} \times \mathbb{Q}, \mathbb{Q} \times \mathbb{Q} \rightarrow \mathbb{Q} \times \mathbb{Q} \times \mathbb{Q}$ ist injektiv.\\
\textbf{Beweis:} Durch Widerspruch.\\
Angenommen $g(x,y)$ sei nicht injektiv, dann gilt $(x,y) \neq (a,b)$ mit $(x,y),(a,b) \in \mathbb{Q} \times \mathbb{Q}$ und $g(x,y) = g(a,b)$.\\
Daraus ergibt sich:
$g(x,y) = g(a,b) = (xy^{2},xy^{2}-3x,(x^{2}-2)y) = (ab^{2},ab^{2}-3a,(a^{2}-2)b)$.\\
Es ergeben sich drei Gleichungen:
\begin{alignat}{2}
\label{eq:1} xy^{2} &=& ab^{2} \\
\label{eq:2} xy^{2}-3x &=& ab^{2}-3a \\
(x^{2}-2)y &=& (a^{2}-2)b \\
\intertext{Subtrahiere \eqref{eq:2} von \eqref{eq:1}:}
-3x &=& -3a \\
\intertext{Geteilt durch (-3) ergibt sich:}
\label{eq:3} x &=& a\\
\intertext{Wegen \eqref{eq:3} ergibt sich:}
(a^{2}-2)y &=& (a^{2}-2)b \\
\intertext{Geteilt durch $(a^{2}-2)$ ($\neq 0$ wg. $\pm \sqrt{2} \not\in \mathbb{Q}$) ergibt sich:}
y &=& b
\end{alignat}
Damit würde $(x,y) = (a,b)$ gelten, was im Widerspruch zur Annahme steht. Also ist $g(x,y)$ injektiv. \hfill $\Box$
\subsection{}
\textbf{Behauptung:} $h(z)$ mit $z \in \mathbb{Z}, \mathbb{Z} \rightarrow \mathbb{Z} \times \mathbb{Z}$ ist nicht surjektiv.\\
\textbf{Beweis:}\\
Es sei $h(z) = (0,0)$. Daraus ergeben sich folgende Gleichungen:
\begin{alignat}{2}
\label{eq:4} 0 &=& z+2 \\
\label{eq:5} 0 &=& z-1 \\
\intertext{\eqref{eq:4} - \eqref{eq:5} ergibt:}
0 &=& 3
\end{alignat}
Also ist $h(z)$ nicht surjektiv. \hfill $\Box$
\end{document}

View File

@ -0,0 +1,185 @@
\documentclass[10pt,a4paper,oneside,ngerman,numbers=noenddot]{scrartcl}
\usepackage[T1]{fontenc}
\usepackage[utf8]{inputenc}
\usepackage[ngerman]{babel}
\usepackage{amsmath}
\usepackage{amsfonts}
\usepackage{amssymb}
\usepackage{paralist}
\usepackage[locale=DE,exponent-product=\cdot,detect-all]{siunitx}
\usepackage{tikz}
\usetikzlibrary{matrix,fadings,calc,positioning,decorations.pathreplacing,arrows}
\pagenumbering{arabic}
\def\thesection{\arabic{section})}
\def\thesubsection{\alph{subsection})}
\def\thesubsubsection{(\arabic{subsubsection})}
\begin{document}
\author{Jim Martens}
\title{Hausaufgaben zum 15./16. November}
\maketitle
\section{} %1
\subsection{} %a
Wie viele Abbildungen $g: X \rightarrow Y$ gibt es?\\
Es gibt $7^{5}$ mögliche Abbildungen. Da die Belegung egal ist und jedes $x \in X$ sieben $y \in Y$ zur Auswahl hat, ergibt sich $7 \cdot 7 \cdot 7 \cdot 7 \cdot 7 = 7^{5}=16807$.\\\\
Wie viele davon sind injektiv?\\
Es sind $7 \cdot 6 \cdot 5 \cdot 4 \cdot 3=\frac{7!}{(7-5)!}=\frac{7!}{2!}=2520$ Abbildungen, weil zwei unterschiedliche Elemente $x \in X$ auf zwei unterschiedliche $y \in Y$ abbilden müssen. Das erste $x$ kann allen sieben $y$ zugeordnet werden. Das zweite $x$ nur noch sechs. Das dritte $x$ kann fünf $y$ zugeordnet werden, das vierte $x$ kann vier $y$ zugeordnet werden und das fünfte $x$ kann nur noch drei $y$ zugeordnet werden.
\subsection{} %b
Lotto funktioniert nach dem Prinzip Ziehen ohne Zurücklegen, ohne Beachtung der Reihenfolge.
Das entspricht $\binom{n}{k}$. In diesem Fall gilt $n=49$ und $k=6$. Es ergibt sich also: $\binom{49}{6}$. Dies entspricht dem Ergebnis $13983816$.
Es gibt also $13983816$ verschiedene Tipps beim Lotto.
\subsection{} %c
Es sei die Menge $M$ mit $|M| = 1000$ und $T$ die Anzahl der Teilmengen mit mindestens $997$ Elementen. Wie viele Teilmengen mit mindestens $997$ Elementen besitzt $M$?\\
Zu den Teilmengen gehört auf jeden Fall die Menge $M$ selber, da diese mehr als $997$ Elemente hat. Die restliche Anzahl an Teilmengen lässt sich mithilfe der Binomialkoeffizienten $\binom{n}{k}$ lösen.
In diesem Fall gilt $n=1000$ und $k_{1}=997,k_{2}=998,k_{3}=999$. Daraus ergibt sich folgende Rechnung:
\begin{alignat}{2}
&\; T &=& \binom{1000}{997} + \binom{1000}{998} + \binom{1000}{999} + 1 \\
\Leftrightarrow &\; T &=& 166167000 + 499500 + 1000 + 1 \\
\Leftrightarrow &\; T &=& 166667501
\end{alignat}
$M$ besitzt $166667501$ Teilmengen mit mindestens $997$ Elementen.
\section{} %2
\subsection{} %a
Zur Lösung der Aufgabe wird der Multinomialsatz angewendet.
Wie lautet der Koeffizient von $x^{5}y^{11}$ in $(x+y)^{16}$?
\begin{alignat}{2}
&\; \binom{16}{5,11} &=& \frac{16!}{5! \cdot 11!} \\
\intertext{Teilen durch $5!$}
\Leftrightarrow &\; \binom{16}{5,11} &=& \frac{6 \cdot 7 \cdot 8 \cdot 9 \cdot 10 \cdot 11 \cdot 12 \cdot 13 \cdot 14 \cdot 15 \cdot 16}{11!} \\
\intertext{Teilen durch $11!$}
\Leftrightarrow &\; \binom{16}{5,11} &=& \frac{12 \cdot 13 \cdot 14 \cdot 15 \cdot 16}{1 \cdot 2 \cdot 3 \cdot 4 \cdot 5} \\
\intertext{Kürzen von 14 mit 2, 12 mit 3, 16 mit 4 und 15 mit 5}
\Leftrightarrow &\; \binom{16}{5,11} &=& 4 \cdot 13 \cdot 7 \cdot 3 \cdot 4 \\
\Leftrightarrow &\; \binom{16}{5,11} &=& 4368
\end{alignat}
Der Koeffizient von $x^{5}y^{11}$ in $(x+y)^{16}$ lautet $4368$.
Wie lautet der Koeffizient von $x^{3}y^{5}z^{2}$ in $(x+y+z)^{10}$?
\begin{alignat}{2}
&\; \binom{10}{3,5,2} &=& \frac{10!}{3! \cdot 5! \cdot 2!} \\
\intertext{Teilen durch $5!$}
\Leftrightarrow &\; \binom{10}{3,5,2} &=& \frac{6 \cdot 7 \cdot 8 \cdot 9 \cdot 10}{3! \cdot 2!} \\
\Leftrightarrow &\; \binom{10}{3,5,2} &=& \frac{6 \cdot 7 \cdot 8 \cdot 9 \cdot 10}{6 \cdot 2!} \\
\intertext{Teilen durch 6}
\Leftrightarrow &\; \binom{10}{3,5,2} &=& \frac{7 \cdot 8 \cdot 9 \cdot 10}{2} \\
\intertext{Kürzen der $8$ mit der $2$ auf $4$}
\Leftrightarrow &\; \binom{10}{3,5,2} &=& 7 \cdot 4 \cdot 9 \cdot 10 \\
\Leftrightarrow &\; \binom{10}{3,5,2} &=& 2520
\end{alignat}
Der Koeffizient von $x^{3}y^{5}z^{2}$ in $(x+y+z)^{10}$ lautet $2520$.
\subsection{} %b
Diese Aufgabe lässt sich mit der fünften Grundaufgabe der Kombinatorik lösen.
Das Wort CAPPUCINO besteht aus $9$ Buchstaben, wobei es das C und das P zweimal gibt. Es ergibt sich daher nach der fünften Grundaufgabe folgende Gleichung:
\begin{alignat}{2}
&\; \binom{8}{2,2,1,1,1,1,1} &=& \frac{8!}{2! \cdot 2!} \\
\intertext{Teilen durch $2!$}
\Leftrightarrow &\; \binom{8}{2,2,1,1,1,1,1} &=& \frac{3 \cdot 4 \cdot 5 \cdot 6 \cdot 7 \cdot 8}{2 \cdot 1} \\
\intertext{Kürzen der $4$ mit $2$ auf $2$}
\Leftrightarrow &\; \binom{8}{2,2,1,1,1,1,1} &=& 3 \cdot 2 \cdot 5 \cdot 6 \cdot 7 \cdot 8 \\
\Leftrightarrow &\; \binom{8}{2,2,1,1,1,1,1} &=& 10080
\end{alignat}
Es lassen sich also $10080$ verschiedene (sinnlose oder sinvolle) Wörter mit $8$ Buchstaben aus dem Wort CAPPUCINO bilden.
Bei dem Wort MANGOLASSI verhält es sich analog. Insgesamt gibt es hier $10$ Buchstaben, wobei das A und das S zweimal vorkommen. Aus der fünften Grundaufgabe ergibt sich folgende Gleichung:
\begin{alignat}{2}
&\; \binom{10}{2,2,1,1,1,1,1,1} &=& \frac{10!}{2! \cdot 2!} \\
\intertext{Teilen durch $2!$}
\Leftrightarrow &\; \binom{10}{2,2,1,1,1,1,1,1} &=& \frac{3 \cdot 4 \cdot 5 \cdot 6 \cdot 7 \cdot 8 \cdot 9 \cdot 10}{2 \cdot 1} \\
\intertext{Kürzen der $4$ mit $2$ auf $2$}
\Leftrightarrow &\; \binom{10}{2,2,1,1,1,1,1,1} &=& 3 \cdot 2 \cdot 5 \cdot 6 \cdot 7 \cdot 8 \cdot 9 \cdot 10 \\
\Leftrightarrow &\; \binom{10}{2,2,1,1,1,1,1,1} &=& 907200
\end{alignat}
Es lassen sich also $907200$ verschiedene (sinnlose oder sinnvolle) Wörter mit $10$ Buchstaben aus dem Wort MANGOLASSI bilden.
Bei dem Wort SELTERWASSER verhält es sich analog. Insgesamt gibt es hier $12$ Buchstaben, wobei E und S dreimal und das R zweimal vorkommen. Aus der fünften Grundaufgabe ergibt sich folgende Gleichung:
\begin{alignat}{2}
&\; \binom{12}{3,3,2,1,1,1,1} &=& \frac{12!}{3! \cdot 3! \cdot 2!} \\
\intertext{Teilen durch $3!$}
\Leftrightarrow &\; \binom{12}{3,3,2,1,1,1,1} &=& \frac{4 \cdot 5 \cdot 6 \cdot 7 \cdot 8 \cdot 9 \cdot 10 \cdot 11 \cdot 12}{3 \cdot 2 \cdot 1 \cdot 2 \cdot 1} \\
\Leftrightarrow &\; \binom{12}{3,3,2,1,1,1,1} &=& \frac{4 \cdot 5 \cdot 6 \cdot 7 \cdot 8 \cdot 9 \cdot 10 \cdot 11 \cdot 12}{6 \cdot 2} \\
\intertext{Teilen durch $6$}
\Leftrightarrow &\; \binom{12}{3,3,2,1,1,1,1} &=& \frac{4 \cdot 5 \cdot 7 \cdot 8 \cdot 9 \cdot 10 \cdot 11 \cdot 12}{2} \\
\intertext{Kürzen der $4$ mit $2$ auf $2$}
\Leftrightarrow &\; \binom{12}{3,3,2,1,1,1,1} &=& 2 \cdot 5 \cdot 7 \cdot 8 \cdot 9 \cdot 10 \cdot 11 \cdot 12 \\
\Leftrightarrow &\; \binom{12}{3,3,2,1,1,1,1} &=& 6652800
\end{alignat}
Es lassen sich also $6652800$ verschiedene (sinnlose oder sinnvolle) Wörter mit $12$ Buchstaben aus dem Wort SELTERWASSER bilden.
\subsection{} %c
Es gibt $10$ verschiedene Sorten zu je mindestens $6$ Flaschen. Wie viele Möglichkeiten gibt es einen Getränkekarton mit $6$ Flaschen zusammenzustellen?
Diese Aufgabe entspricht einem Ziehen mit Zurücklegen ohne Beachtung der Reihenfolge (Grundaufgabe 4). Es gibt $10$ Sorten, also gilt $n=10$. Aus diesen $10$ müssen nun $6$ gezogen werden. Es gilt demnach $k=6$. Mithilfe der vierten Grundaufgabe ergibt sich daraus folgendes:
\begin{alignat}{4}
&\; \binom{6+10-1}{6} &=& \binom{15}{6} &=& \binom{15}{15-6} &=& \binom{15}{9} \\
\Leftrightarrow &\; \binom{6+10-1}{6} &=& 5005
\end{alignat}
Es gibt also $5005$ Möglichkeiten einen Getränkekarton mit $6$ Flaschen zusammenzustellen.
\section{} %3
\textbf{Behauptung:} Die folgende Aussage gilt für alle $n \in \mathbb{N},n \geq 3$:
\begin{alignat}{2}
\sum_{i=3}^{n} \binom{i}{i-3} &=& \binom{n+1}{4} \label{eq:1}
\end{alignat}
\textbf{Beweis:} Durch vollständige Induktion.\\
Mit $A(n)$ sei die Aussage \eqref{eq:1} bezeichnet.\\\\
\underline{Induktionsanfang:} \\
$A(3)$ ist richtig, da folgendes gilt:
\begin{alignat}{2}
&\; \sum_{i=3}^{3} \binom{i}{i-3} &=& \binom{3+1}{4} \\
\Leftrightarrow &\; \sum_{i=3}^{3} \binom{i}{i-3} &=& \binom{4}{4} \\
\Leftrightarrow &\; \sum_{i=3}^{3} \binom{i}{i-3} &=& 1 = \binom{3}{0} = 1
\end{alignat}
\underline{Induktionsannahme:}\\
Die Aussage \eqref{eq:1} gilt für ein beliebig fest gewähltes $n \in \mathbb{N}, n \geq 3$.\\\\
\\
\underline{Zu zeigen:}\\
$A(n+1)$ gilt, d. h. folgende Gleichung gilt:
\begin{alignat}{2}
\sum_{i=3}^{n+1} \binom{i}{i-3} &=& \binom{(n+1)+1}{4} \label{eq:2}
\end{alignat}
\underline{Induktionsschluss:}\\
Aus \eqref{eq:2} folgt folgendes:
\begin{alignat}{2}
&\; \sum_{i=3}^{n} \binom{i}{i-3} + \binom{n+1}{(n+1)-3} &=& \binom{(n+1)+1}{4} \\
\intertext{Anwendung der Induktionsannahme:}
\Leftrightarrow &\; \sum_{i=3}^{n} \binom{i}{i-3} + \binom{n+1}{(n+1)-3} &=& \binom{n+1}{4} + \binom{n+1}{(n+1)-3} \\
\Leftrightarrow &\; \sum_{i=3}^{n} \binom{i}{i-3} + \binom{n+1}{(n+1)-3} &=& \binom{n+1}{4} + \binom{n+1}{n-2} \\
\intertext{Aufgrund der Symmetrie des Pascalschen Dreiecks gilt:}
\Leftrightarrow &\; \sum_{i=3}^{n} \binom{i}{i-3} + \binom{n+1}{(n+1)-3} &=& \binom{n+1}{n-3} + \binom{n+1}{n-2} \\
\intertext{Durch Anwendung der Rekursionsformel ergibt sich:}
\Leftrightarrow &\; \sum_{i=3}^{n} \binom{i}{i-3} + \binom{n+1}{(n+1)-3} &=& \binom{n+2}{n-2} \\
\intertext{Aufgrund der Symmetrie des Pascalschen Dreiecks gilt:}
\Leftrightarrow &\; \sum_{i=3}^{n} \binom{i}{i-3} + \binom{n+1}{(n+1)-3} &=& \binom{n+2}{4}
\end{alignat}
Nach dem Induktionsprinzip folgt aus dem Induktionsanfang und dem Induktionsschluss die Behauptung. \hfill $\Box$
\section{} %4
\subsection{} %a
Es sei $S = {k \in \mathbb{N} : 1 \leq k \leq 2000}$. Es soll die Anzahl derjenigen $k \in S$ bestimmt werden, die weder durch 3 noch durch 5 noch durch 7 teilbar sind.
Es sei $A_{1} = \{k \in S : 3 \mid k\}, A_{2} = \{k \in S : 5 \mid k\}, A_{3} = \{k \in S : 7 \mid k\}$. Es folgt $N = |S| = 2000$, sowie $|A_{1}| = \lfloor\frac{2000}{3}\rfloor = 666, |A_{2}| = \frac{2000}{5} = 400, |A_{3}| = \lfloor\frac{2000}{7}\rfloor = 285$.
Ferner gilt $A_{1} \cap A_{2} = \{k \in S : 15 \mid k\}, A_{1} \cap A_{3} = \{k \in S : 21 \mid k\}, A_{2} \cap A_{3} = \{k \in S : 35 \mid k\}$ und $A_{1} \cap A_{2} \cap A_{3} = \{k \in S : 105 \mid k\}$. Also gilt $|A_{1} \cap A_{2}| = \lfloor\frac{2000}{15}\rfloor = 133, |A_{1} \cap A_{3}| = \lfloor\frac{2000}{21}\rfloor = 95, |A_{2} \cap A_{3}| = \lfloor\frac{2000}{35}\rfloor = 57, |A_{1} \cap A_{2} \cap A_{3}| = \lfloor\frac{2000}{105}\rfloor = 19$.
Insgesamt erhält man $|S \setminus (A_{1} \cup A_{2} \cup A_{3})| = 2000 - (666 + 400 + 285) + 133 + 95 + 57 - 19 = \underline{\underline{915}}$.
\subsection{} %b
Es sei $S = {k \in \mathbb{N} : 1 \leq k \leq 1000}$. Es soll die Anzahl derjenigen $k \in S$ bestimmt werden, die weder durch 3 noch durch 5 noch durch 7 noch durch 11 teilbar sind.
Es sei $A_{1} = \{k \in S : 3 \mid k\}, A_{2} = \{k \in S : 5 \mid k\}, A_{3} = \{k \in S : 7 \mid k\}, A_{4} = \{k \in S : 11 \mid k\}$. Es folgt $N = |S| = 1000$, sowie $|A_{1}| = \lfloor\frac{1000}{3}\rfloor = 333, |A_{2}| = \frac{1000}{5} = 200, |A_{3}| = \lfloor\frac{1000}{7}\rfloor = 142, |A_{4}| = \lfloor\frac{2000}{11}\rfloor = 90$.
Ferner gilt $A_{1} \cap A_{2} = \{k \in S : 15 \mid k\}, A_{1} \cap A_{3} = \{k \in S : 21 \mid k\}, A_{1} \cap A_{4} = \{k \in S : 33 \mid k\}, A_{2} \cap A_{3} = \{k \in S : 35 \mid k\}, A_{2} \cap A_{4} = \{k \in S : 55 \mid k\}, A_{3} \cap A_{4} = \{k \in S : 77 \mid k\}$ und $A_{1} \cap A_{2} \cap A_{3} \cap A_{4} = \{k \in S : 1155 \mid k\}$. Also gilt $|A_{1} \cap A_{2}| = \lfloor\frac{1000}{15}\rfloor = 66, |A_{1} \cap A_{3}| = \lfloor\frac{1000}{21}\rfloor = 47, |A_{1} \cap A_{4}| = \lfloor\frac{1000}{33}\rfloor = 30, |A_{2} \cap A_{3}| = \lfloor\frac{1000}{35}\rfloor = 28, |A_{2} \cap A_{4}| = \lfloor\frac{1000}{55}\rfloor = 18, |A_{3} \cap A_{4}| = \lfloor\frac{1000}{77}\rfloor = 12, |A_{1} \cap A_{2} \cap A_{3} \cap A_{4}| = \lfloor\frac{1000}{1155}\rfloor = 0$.
Insgesamt erhält man $|S \setminus (A_{1} \cup A_{2} \cup A_{3} \cup A_{4})| = 1000 - (333 + 200 + 142 + 90) + 66 + 47 + 30 +28 + 18 + 12 - 0 = \underline{\underline{436}}$.
\end{document}

View File

@ -0,0 +1,286 @@
\documentclass[10pt,a4paper,oneside,ngerman,numbers=noenddot]{scrartcl}
\usepackage[T1]{fontenc}
\usepackage[utf8]{inputenc}
\usepackage[ngerman]{babel}
\usepackage{amsmath}
\usepackage{amsfonts}
\usepackage{amssymb}
\usepackage{paralist}
\usepackage[locale=DE,exponent-product=\cdot,detect-all]{siunitx}
\usepackage{tikz}
\usetikzlibrary{matrix,fadings,calc,positioning,decorations.pathreplacing,arrows,decorations.markings}
\pagenumbering{arabic}
\def\thesection{\arabic{section})}
\def\thesubsection{\alph{subsection})}
\def\thesubsubsection{(\arabic{subsubsection})}
\begin{document}
\author{Jim Martens}
\title{Hausaufgaben zum 22./23. November}
\maketitle
\section{} %1
\subsection{} %a
\begin{tikzpicture}
\draw[->] (3, 0) node [below] {a} -- +(-0.5,0.5) node [left] {b};
\draw[->] (2.5,0.6) -- +(0.5,0.5) node [above] {c};
\draw[->] (3.1,1.1) -- +(0.5,0) node [right] {d};
\draw[->] (2.6,0.55) -- +(2,0.35) node [right] {e};
\draw[->] (4.6,0.8) -- +(0,-0.5) node [below] {f};
\end{tikzpicture}
\\
\\
\\
\begin{tikzpicture}
\matrix (firstMatrix) [matrix of nodes]
{ \ & a & b & c & d & e & f \\
a & 0 & 1 & 0 & 0 & 0 & 0 \\
b & 0 & 0 & 1 & 0 & 1 & 0 \\
c & 0 & 0 & 0 & 1 & 0 & 0 \\
d & 0 & 0 & 0 & 0 & 0 & 0 \\
e & 0 & 0 & 0 & 0 & 0 & 1 \\
f & 0 & 0 & 0 & 0 & 0 & 0 \\
};
\draw (-1.1,-1.6) -- +(0, 3.2);
\draw (-1.5,1.2) -- +(3, 0);
\end{tikzpicture}
\subsection{} %b
Es müssen aufgrund der geforderten Reflexivität folgende Paare hinzugefügt werden:
(a,a), (b,b), (c,c), (d,d), (e,e), (f,f)
Wegen der geforderten Transitivität sind folgende Paare hinzuzufügen:
(a,c), (a,e), (a,f), (b,d), (b,f)
\subsection{} %c
$R^{+}$:\\
\begin{tikzpicture}
\node (a) {a};
\node (b) [above left=0.4cm of a] {b};
\node (c) [above right=0.5cm of b] {c};
\node (d) [above right=0.4cm of c] {d};
\node (e) [below right=0.3cm of d] {e};
\node (f) [above=0.5cm of e] {f};
\draw (a) -- (b);
\draw (b) -- (c);
\draw (c) -- (d);
\draw (b) -- (e);
\draw (e) -- (f);
\end{tikzpicture}
\subsection{} %d
$R$:\\
\begin{tikzpicture}
\node (a) {a};
\node (b) [above left=0.4cm of a] {b};
\node (c) [above right=0.5cm of b] {c};
\node (d) [right=of c] {d};
\node (e) [below right=0.4cm of d] {e};
\draw[->] (a) -- (b);
\draw[->] (b) -- (c);
\draw[->] (c) -- (d);
\draw[->] (b) -- (e);
\end{tikzpicture}\\
Um die Bedingungen einer Äquivalenzrelation zu erfüllen, müssen folgende Paare hinzugefügt werden:\\
reflexiv: (a,a), (b,b), (c,c), (d,d), (e,e)\\
symmetrisch und transitiv: (a,c), (a,d), (a,e), (b,a), (b,d), (c,a), (c,b), (d,a), (d,b), (d,c), (d,e), (e,a), (e,b), (e,c), (e,d)\\
Kurz geschrieben schreibt man $S = A \times A$.
\section{} %2
\subsubsection{} %(1)
\begin{tikzpicture}
\node (a) {a};
\node (b) [above left=0.5cm of a] {b};
\node (c) [above right=0.4cm of b] {c};
\node (d) [right=0.5cm of c] {d};
\node (e) [right=0.5cm of d] {e};
\node (f) [below right=0.4cm of e] {f};
\draw[->] (a) -- (b);
\draw[->] (b) -- (d);
\draw[->] (e) -- (f);
\end{tikzpicture}
\\
\\
\\
\begin{tikzpicture}
\matrix (secondMatrix) [matrix of nodes]
{ \ & a & b & c & d & e & f \\
a & 0 & 1 & 0 & 0 & 0 & 0 \\
b & 0 & 0 & 0 & 1 & 0 & 0 \\
c & 0 & 0 & 0 & 0 & 0 & 0 \\
d & 0 & 0 & 0 & 0 & 0 & 0 \\
e & 0 & 0 & 0 & 0 & 0 & 1 \\
f & 0 & 0 & 0 & 0 & 0 & 0 \\
};
\draw (-1.1,-1.6) -- +(0, 3.2);
\draw (-1.5,1.2) -- +(3, 0);
\end{tikzpicture}
\subsubsection{} %(2)
Um die Bedingungen zu erfüllen müssen folgende Paare hinzugefügt werden:\\
reflexiv: (a,a), (b,b), (c,c), (d,d), (e,e), (f,f)\\
transitiv: (a,d)
\subsubsection{} %(3)
\begin{tikzpicture}
\node (a) {a};
\node (b) [above left=0.5cm of a] {b};
\node (c) [above right=0.4cm of b] {c};
\node (d) [right=0.4cm of c] {d};
\node (e) [right=of d] {e};
\node (f) [above right=0.4cm of e] {f};
\draw (a) -- (b);
\draw (b) -- (d);
\draw (e) -- (f);
\end{tikzpicture}
\subsubsection{} %(4)
Um die Bedingungen zu erfüllen müssen folgende Paare hinzugefügt werden:\\
reflexiv: (a,a), (b,b), (c,c), (d,d), (e,e), (f,f)\\
symmetrisch: (b,a), (d,a), (d,b), (f,e)\\
transitiv: (a,d)\\
Verkürzt kann folgendes geschrieben werden: \\
\begin{equation*}
S = R \,\cup\, \{(a,a), (a,d), (b,a), (b,b), (c,c), (d,a), (d,b), (d,d), (e,e), (f,e), (f,f)\}
\end{equation*}
\section{} %3
\subsection{} %a
R = {(a,a), (a,b), (b,a), (b,b), (b,c), (c,b), (c,c), (d,d)}
\begin{tikzpicture}
\node (a) {a};
\node (b) [above left=0.4cm of a] {b};
\node (c) [above right=0.5cm of b] {c};
\node (d) [below right=0.4cm of c] {d};
\path (a) edge[loop below] (a);
\draw[->] (a) to[->,out=135,in=315] (b);
\draw[->] (b) to[->,out=270,in=180] (a);
\path (b) edge[loop left] (b);
\draw[->] (b) to[->,out=90,in=180] (c);
\draw[->] (c) to[->,out=225,in=45] (b);
\path (c) edge[loop above] (c);
\path (d) edge[loop right] (d);
\end{tikzpicture}
Es gibt eine Kante von a nach b und von b nach c, aber nicht von a nach c, also ist diese Relation nicht transitiv. Jedes Element der Grundmenge steht in Relation zu sich selbst, also ist die Relation reflexiv. Zu jeder Kante x nach y gibt es eine Rückkante y nach x, also ist R symmetrisch.
\subsection{} %b
R = {(a,a), (a,b), (b,b), (c,c), (d,d)}
\begin{tikzpicture}
\node (a) {a};
\node (b) [above left=0.4cm of a] {b};
\node (c) [above right=0.5cm of b] {c};
\node (d) [below right=0.4cm of c] {d};
\path (a) edge[loop below] (a);
\draw[->] (a) to[->,out=135,in=315] (b);
\path (b) edge[loop left] (b);
\path (c) edge[loop above] (c);
\path (d) edge[loop right] (d);
\end{tikzpicture}
Man kommt von a nach b, aber nicht von b nach a, also ist die Relation nicht symmetrisch. Jedes Element der Grundmenge steht in Relation zu sich selbst, also ist die Relation reflexiv. Es gibt keine Kanten x nach y und y nach z, für die keine Kante x nach z existiert, also ist die Relation transitiv.
\subsection{} %c
R = {(a,a), (a,b), (b,a), (b,b)}
\begin{tikzpicture}
\node (a) {a};
\node (b) [above left=0.4cm of a] {b};
\node (c) [above right=0.5cm of b] {c};
\node (d) [below right=0.4cm of c] {d};
\path (a) edge[loop below] (a);
\draw[->] (a) to[->,out=135,in=315] (b);
\draw[->] (b) to[->,out=270,in=180] (a);
\path (b) edge[loop left] (b);
\end{tikzpicture}
Nicht jedes Element der Grundmenge steht in Relation mit sich selbst, also ist die Relation nicht reflexiv. Zu jeder Kante x nach y gibt es eine Rückkante y nach x, also ist die Relation symmetrisch. Es gibt keine Kanten x nach y und y nach z, für die keine Kante x nach z existiert, also ist die Relation transitiv.
\section{} %4
\subsection{} %a
R = {(1,1), (1,2), (1,3), (1,4), (1,5), (1,6), (2,2), (2,4), (2,6), (3,3), (3,6), (4,4), (5,5), (6,6)}
Graph:\\
\begin{tikzpicture}
\node (1) {1};
\node (2) [above left=0.4cm of 1] {2};
\node (3) [above=0.4cm of 2] {3};
\node (4) [above right=0.4cm of 3] {4};
\node (5) [below right=0.4cm of 4] {5};
\node (6) [above right=0.4cm of 1] {6};
\path (1) edge[loop below] (1);
\draw[->] (1) -- (2);
\draw[->] (1) -- (3);
\draw[->] (1) -- (4);
\draw[->] (1) -- (5);
\draw[->] (1) -- (6);
\path (2) edge[loop left] (2);
\draw[->] (2) -- (4);
\draw[->] (2) -- (6);
\path (3) edge[loop left] (3);
\draw[->] (3) -- (6);
\path (4) edge[loop above] (4);
\path (5) edge[loop right] (5);
\path (6) edge[loop right] (6);
\end{tikzpicture}
Hasse-Diagramm:\\
\begin{tikzpicture}
\node (1) {1};
\node (2) [above left=0.4cm of 1] {2};
\node (3) [above right=0.4cm of 1] {3};
\node (4) [above=0.4cm of 2] {4};
\node (5) [right=0.5cm of 3] {5};
\node (6) [above right=0.4cm of 4] {6};
\draw (1) -- (2);
\draw (1) -- (3);
\draw (1) -- (5);
\draw (2) -- (4);
\draw (2) -- (6);
\draw (3) -- (6);
\end{tikzpicture}
\subsection{} %b
Graph:\\
\begin{tikzpicture}
\node (0) {$\emptyset$};
\node (1) [above left=2.5 of 0] {$\{1\}$};
\node (2) [above right=2.5 of 0] {$\{2\}$};
\node (3) [above=4 of 0] {$\{1,2\}$};
\path (0) edge[loop below] (0);
\draw[->] (0) -- (1);
\draw[->] (0) -- (2);
\draw[->] (0) -- (3);
\path (1) edge[loop left] (1);
\draw[->] (1) -- (3);
\path (2) edge[loop right] (2);
\draw[->] (2) -- (3);
\path (3) edge[loop above] (3);
\end{tikzpicture}
Hasse-Diagramm:\\
\begin{tikzpicture}
\node (0) {$\emptyset$};
\node (1) [above left=2.5 of 0] {$\{1\}$};
\node (2) [above right=2.5 of 0] {$\{2\}$};
\node (3) [above=4 of 0] {$\{1,2\}$};
\draw (0) -- (1);
\draw (0) -- (2);
\draw (1) -- (3);
\draw (2) -- (3);
\end{tikzpicture}
\subsection{} %c
$A = P(M) = \{\emptyset, \{1\}, \{2\}, \{3\}, \{1,2\}, \{1,3\}, \{2,3\}, \{1,2,3\}\}$
\begin{tikzpicture}
\node (0) {$\emptyset$};
\node (1) [above left=2.5 of 0] {$\{1\}$};
\node (2) [above right=2.5 of 0] {$\{2\}$};
\node (3) [above=1.5 of 2] {$\{3\}$};
\node (4) [above=1.5 of 1] {$\{1,2\}$};
\node (5) [above=1.5 of 3] {$\{1,3\}$};
\node (6) [above=1.5 of 4] {$\{2,3\}$};
\node (7) [above=6.5 of 0] {$\{1,2,3\}$};
\draw (0) -- (1);
\draw (0) -- (2);
\draw (0) -- (3);
\draw (1) -- (4);
\draw (1) -- (5);
\draw (2) -- (4);
\draw (2) -- (6);
\draw (3) -- (5);
\draw (3) -- (6);
\draw (4) -- (7);
\draw (5) -- (7);
\draw (6) -- (7);
\end{tikzpicture}
\end{document}

View File

@ -0,0 +1,165 @@
\documentclass[10pt,a4paper,oneside,ngerman,numbers=noenddot]{scrartcl}
\usepackage[T1]{fontenc}
\usepackage[utf8]{inputenc}
\usepackage[ngerman]{babel}
\usepackage{amsmath}
\usepackage{amsfonts}
\usepackage{amssymb}
\usepackage{paralist}
\usepackage[locale=DE,exponent-product=\cdot,detect-all]{siunitx}
\usepackage{tikz}
\usetikzlibrary{matrix,fadings,calc,positioning,decorations.pathreplacing,arrows,decorations.markings}
\pagenumbering{arabic}
\def\thesection{\arabic{section})}
\def\thesubsection{\alph{subsection})}
\def\thesubsubsection{(\arabic{subsubsection})}
\begin{document}
\author{Jim Martens}
\title{Hausaufgaben zum 29./30. November}
\maketitle
\section{} %1
\subsection{} %a
Das Produkt AB existiert, da A gleich viele Spalten (3) wie B Zeilen (3) hat.
\begin{alignat*}{2}
A \cdot B &=& \begin{pmatrix} 2 & 0 & 1 \\ 1 & 0 & -1 \\ 7 & 6 & 3 \\ -1 & 2 & 4 \end{pmatrix} \cdot \begin{pmatrix} 3 & 2 & -1 \\ 1 & 0 & 2 \\ 1 & 1 & 0 \end{pmatrix} \\
&=& \begin{pmatrix} 7 & 5 & -2 \\ 2 & 1 & -1 \\ 30 & 17 & 5 \\ 3 & 2 & 5 \end{pmatrix}
\end{alignat*}
BA existiert nicht, da B weniger Spalten (3) als A Zeilen (4) hat.
AC existiert nicht, da A mehr Spalten (3) als C Zeilen (1) hat.
AD existiert, da A gleich viele Spalten (3) wie D Zeilen (3) hat.
\begin{alignat*}{2}
A \cdot D &=& \begin{pmatrix} 2 & 0 & 1 \\ 1 & 0 & -1 \\ 7 & 6 & 3 \\ -1 & 2 & 4 \end{pmatrix} \cdot \begin{pmatrix} 2 \\ 3 \\ -2 \end{pmatrix} \\
&=& \begin{pmatrix} 2 \\ 0 \\ 26 \\ -4 \end{pmatrix}
\end{alignat*}
AA existiert nicht, da A weniger Spalten (3) hat als Zeilen (4).
BB existiert, da B gleich viele Spalten (3) und Zeilen (3) hat.
\begin{alignat*}{2}
A \cdot D &=& \begin{pmatrix} 3 & 2 & -1 \\ 1 & 0 & 2 \\ 1 & 1 & 0 \end{pmatrix} \cdot \begin{pmatrix} 3 & 2 & -1 \\ 1 & 0 & 2 \\ 1 & 1 & 0 \end{pmatrix} \\
&=& \begin{pmatrix} 10 & 5 & 1 \\ 5 & 4 & -1 \\ 4 & 2 & 1 \end{pmatrix}
\end{alignat*}
CD existiert, da C gleich viele Spalten (3) wie D Zeilen (3) hat.
\begin{alignat*}{2}
C \cdot D &=& \begin{pmatrix} 1 & 2 & -2 \end{pmatrix} \cdot \begin{pmatrix} 2 \\ 3 \\ -2 \end{pmatrix} \\
&=& \begin{pmatrix} 12 \end{pmatrix}
\end{alignat*}
DC existiert, da D gleich viele Spalten (1) wie C Zeilen (1) hat.
\begin{alignat*}{2}
D \cdot C &=& \begin{pmatrix} 2 \\ 3 \\ -2 \end{pmatrix} \cdot \begin{pmatrix} 1 & 2 & -2 \end{pmatrix} \\
&=& \begin{pmatrix} 2 & 4 & -4 \\ 3 & 6 & -6 \\ -2 & -4 & 4 \end{pmatrix}
\end{alignat*}
\subsection{} %b
Um das Element zu berechnen, das in AB in der dritten Zeile und zweiten Spalte steht, benötigen wir die dritte Zeile von A und die zweite Spalte von B. Diese packen wir in eigene Matrizen und multiplizieren diese.
\begin{alignat*}{2}
A_{3j} \cdot B_{i2} &=& \begin{pmatrix} 1 & 2 & 3 & 4 \end{pmatrix} \cdot \begin{pmatrix} -2 \\ 2 \\ 3 \\ 1 \end{pmatrix} \\
&=& \begin{pmatrix} 15 \end{pmatrix}
\end{alignat*}
Für die vierte Spalte von AB benötigen wir A und die vierte Spalte von B. Wir multiplizieren also A und die vierte Spalte von B, die in einer eigenen Matrix steht.
\begin{alignat*}{2}
A \cdot B_{i4} &=& \begin{pmatrix} 3 & 4 & 5 & 6 \\ 2 & 3 & 4 & 5 \\ 1 & 2 & 3 & 4 \\ 4 & 7 & 7 & 4 \end{pmatrix} \cdot \begin{pmatrix} 4 \\ 4 \\ -3 \\ 0 \end{pmatrix} \\
&=& \begin{pmatrix} 13 \\ 8 \\ 3 \\ 23 \end{pmatrix}
\end{alignat*}
\section{} %2
\subsection{} %a
Um das Distributivgesetz zu bestätigen, wird zunächst $A(B_{1} + B_{2})$ gerechnet und anschließend $AB_{1} + AB_{2}$. Das Ergebnis muss gleich sein, um das Gesetz zu bestätigen.
\begin{alignat*}{2}
A \cdot (B_{1} + B_{2}) &=& \begin{pmatrix} 5 & 7 \\ 9 & -1 \\ 8 & 2 \end{pmatrix} \cdot \left( \begin{pmatrix} 1 & 2 \\ 3 & 6 \end{pmatrix} + \begin{pmatrix} 1 & -2 \\ 3 & 2 \end{pmatrix} \right)\\
&=& \begin{pmatrix} 5 & 7 \\ 9 & -1 \\ 8 & 2 \end{pmatrix} \cdot \begin{pmatrix} 2 & 0 \\ 6 & 8 \end{pmatrix} \\
&=& \begin{pmatrix} 52 & 56 \\ 12 & -8 \\ 28 & 16 \end{pmatrix}
\end{alignat*}
Nun folgt die Berechnung des zweiten Teils.
\begin{alignat*}{2}
AB_{1} + AB_{2} &=& \left( \begin{pmatrix} 5 & 7 \\ 9 & -1 \\ 8 & 2 \end{pmatrix} \cdot \begin{pmatrix} 1 & 2 \\ 3 & 6 \end{pmatrix} \right) + \left(\begin{pmatrix} 5 & 7 \\ 9 & -1 \\ 8 & 2 \end{pmatrix} \cdot \begin{pmatrix} 1 & -2 \\ 3 & 2 \end{pmatrix} \right)\\
&=& \begin{pmatrix} 26 & 52 \\ 6 & 12 \\ 14 & 28 \end{pmatrix} + \begin{pmatrix} 26 & 4 \\ 6 & -20 \\ 14 & -12 \end{pmatrix} \\
&=& \begin{pmatrix} 52 & 56 \\ 12 & -8 \\ 28 & 16 \end{pmatrix}
\end{alignat*}
Beide Seiten der Gleichung ergeben dasselbe Ergebnis, somit ist das Distributivgesetz mit den angegebenen Matrizen bestätigt.
\subsection{} %b
Um die Gleichung $(AB)^{T} = B^{T}A^{T}$ zu bestätigen, wird zunächst $(AB)^{T}$ berechnet und anschließend $B^{T}A^{T}$. Ergeben beide Berechnungen das gleiche Ergebnis, so ist die Gleichung damit bestätigt.
\begin{alignat*}{2}
(AB)^{T} &=& \left( \begin{pmatrix} 1 & 3 \\ 2 & 6 \end{pmatrix} \cdot \begin{pmatrix} 2 & -1 & 5 \\ 3 & 2 & 4 \end{pmatrix} \right)^{T} \\
&=& \begin{pmatrix} 11 & 5 & 17 \\ 22 & 10 & 34 \end{pmatrix}^{T} \\
&=& \begin{pmatrix} 11 & 22 \\ 5 & 10 \\ 17 & 34 \end{pmatrix}
\end{alignat*}
Nun folgt die Berechnung des zweiten Teils.
\begin{alignat}{2}
B^{T}A^{T} &=& \begin{pmatrix} 2 & -1 & 5 \\ 3 & 2 & 4 \end{pmatrix}^{T} \cdot \begin{pmatrix} 1 & 3 \\ 2 & 6 \end{pmatrix}^{T} \notag\\
&=& \begin{pmatrix} 2 & 3 \\ -1 & 2 \\ 5 & 4 \end{pmatrix} \cdot \begin{pmatrix} 1 & 2 \\ 3 & 6 \end{pmatrix} \label{eq:transponiert} \\
&=& \begin{pmatrix} 11 & 22 \\ 5 & 10 \\ 17 & 34 \end{pmatrix} \notag
\end{alignat}
Beide Seiten der Gleichung ergeben das gleiche Ergebnis, somit ist die Gleichung $(AB)^{T} = B^{T}A^{T}$ bestätigt.
\subsection{} %c
Wie in \eqref{eq:transponiert} sichtbar, hat $A^{T}$ weniger Spalten als $B^{T}$ Zeilen hat. Somit kann $A^{T}B^{T}$ gar nicht berechnet werden.
\section{} %3
Beweis des Distributivgesetzes $A(B_{1} + B_{2}) = AB_{1} + AB_{2}$.
Es werden die beiden Seiten der Gleichung zunächst einzeln gerechnet und im Anschluss verglichen. Bei gleichem Ergebnis stimmt die Aussage.
Es seien $A,B_{1},B_{2}$ folgende Matrizen:
\begin{alignat*}{2}
A &=& \begin{pmatrix} a_{11} & a_{12} & a_{13} \\ a_{21} & a_{22} & a_{23} \\ a_{31} & a_{32} & a_{33} \end{pmatrix} \\
B_{1} = B &=& \begin{pmatrix} b_{11} & b_{12} & b_{13} \\ b_{21} & b_{22} & b_{23} \\ b_{31} & b_{32} & b_{33} \end{pmatrix} \\
B_{2} = C &=& \begin{pmatrix} c_{11} & c_{12} & c_{13} \\ c_{21} & c_{22} & c_{23} \\ c_{31} & c_{32} & c_{33} \end{pmatrix} \\
\intertext{Berechnung des linken Teils der Gleichung}
B + C &=& \begin{pmatrix} b_{11} + c_{11} & b_{12} + c_{12} & b_{13} + c_{13} \\ b_{21} + c_{21} & b_{22} + c_{22} & b_{23} + c_{23} \\ b_{31} + c_{31} & b_{32} + c_{32} & b_{33} + c_{33} \end{pmatrix} \\
% todo
A \cdot (B+C) &=& \begin{pmatrix} \sum\limits_{i=1}^{3} a_{1i} \cdot (b_{i1} + c_{i1}) & \sum\limits_{i=1}^{3} a_{1i} \cdot (b_{i2} + c_{i2}) & \sum\limits_{i=1}^{3} a_{1i} \cdot (b_{i3} + c_{i3}) \\ \sum\limits_{i=1}^{3} a_{2i} \cdot (b_{i1} + c_{i1}) & \sum\limits_{i=1}^{3} a_{2i} \cdot (b_{i2} + c_{i2}) & \sum\limits_{i=1}^{3} a_{2i} \cdot (b_{i3} + c_{i3}) \\ \sum\limits_{i=1}^{3} a_{3i} \cdot (b_{i1} + c_{i1}) & \sum\limits_{i=1}^{3} a_{3i} \cdot (b_{i2} + c_{i2}) & \sum\limits_{i=1}^{3} a_{3i} \cdot (b_{i3} + c_{i3}) \end{pmatrix} \\
\intertext{Berechnung des rechten Teils der Gleichung}
AB &=& \begin{pmatrix} \sum\limits_{i=1}^{3} a_{1i} \cdot b_{i1} & \sum\limits_{i=1}^{3} a_{1i} \cdot b_{i2} & \sum\limits_{i=1}^{3} a_{1i} \cdot b_{i3} \\ \sum\limits_{i=1}^{3} a_{2i} \cdot b_{i1} & \sum\limits_{i=1}^{3} a_{2i} \cdot b_{i2} & \sum\limits_{i=1}^{3} a_{2i} \cdot b_{i3} \\ \sum\limits_{i=1}^{3} a_{3i} \cdot b_{i1} & \sum\limits_{i=1}^{3} a_{3i} \cdot b_{i2} & \sum\limits_{i=1}^{3} a_{3i} \cdot b_{i3} \end{pmatrix} \\
AC &=& \begin{pmatrix} \sum\limits_{i=1}^{3} a_{1i} \cdot c_{i1} & \sum\limits_{i=1}^{3} a_{1i} \cdot c_{i2} & \sum\limits_{i=1}^{3} a_{1i} \cdot c_{i3} \\ \sum\limits_{i=1}^{3} a_{2i} \cdot c_{i1} & \sum\limits_{i=1}^{3} a_{2i} \cdot c_{i2} & \sum\limits_{i=1}^{3} a_{2i} \cdot c_{i3} \\ \sum\limits_{i=1}^{3} a_{3i} \cdot c_{i1} & \sum\limits_{i=1}^{3} a_{3i} \cdot c_{i2} & \sum\limits_{i=1}^{3} a_{3i} \cdot c_{i3} \end{pmatrix} \\
AB + AC &=& \begin{pmatrix} \sum\limits_{i=1}^{3} a_{1i} \cdot b_{i1} + a_{1i} \cdot c_{i1} & \sum\limits_{i=1}^{3} a_{1i} \cdot b_{i2} + a_{1i} \cdot c_{i2} & \sum\limits_{i=1}^{3} a_{1i} \cdot b_{i3} + a_{1i} \cdot c_{i3} \\ \sum\limits_{i=1}^{3} a_{2i} \cdot b_{i1} + a_{2i} \cdot c_{i1} & \sum\limits_{i=1}^{3} a_{2i} \cdot b_{i2} + a_{2i} \cdot c_{i2} & \sum\limits_{i=1}^{3} a_{2i} \cdot b_{i3} + a_{2i} \cdot c_{i3} \\ \sum\limits_{i=1}^{3} a_{3i} \cdot b_{i1} + a_{3i} \cdot c_{i1} & \sum\limits_{i=1}^{3} a_{3i} \cdot b_{i2} + a_{3i} \cdot c_{i2} & \sum\limits_{i=1}^{3} a_{3i} \cdot b_{i3} + a_{3i} \cdot c_{i3} \end{pmatrix} \\
\intertext{Durch Ausklammern des a ergibt sich:}
&=& \begin{pmatrix} \sum\limits_{i=1}^{3} a_{1i} \cdot (b_{i1} + c_{i1}) & \sum\limits_{i=1}^{3} a_{1i} \cdot (b_{i2} + c_{i2}) & \sum\limits_{i=1}^{3} a_{1i} \cdot (b_{i3} + c_{i3}) \\ \sum\limits_{i=1}^{3} a_{2i} \cdot (b_{i1} + c_{i1}) & \sum\limits_{i=1}^{3} a_{2i} \cdot (b_{i2} + c_{i2}) & \sum\limits_{i=1}^{3} a_{2i} \cdot (b_{i3} + c_{i3}) \\ \sum\limits_{i=1}^{3} a_{3i} \cdot (b_{i1} + c_{i1}) & \sum\limits_{i=1}^{3} a_{3i} \cdot (b_{i2} + c_{i2}) & \sum\limits_{i=1}^{3} a_{3i} \cdot (b_{i3} + c_{i3}) \end{pmatrix}
\end{alignat*}
Auf beiden Seiten kommt das gleiche Ergebnis heraus. Also ist das Distributivgesetz $A(B_{1} + B_{2}) = AB_{1} + AB_{2}$ damit bewiesen. \hfill $\Box$
\section{} %4
\subsection{} %a
Beweis der Aussage (6), Skript Seite 61. Es ist zu zeigen, dass für jede Abbildung $f: A \rightarrow B$ und jedes $B' \subseteq B$ Folgendes gilt:
\begin{equation*}
f(f^{-1}(B')) \subseteq B'
\end{equation*}
%todo, siehe Tutorium Di
Es gelte $b \in f(f^{-1}(B'))$. Daraus ergeben sich zwei mögliche Fälle:
\begin{alignat*}{1}
\intertext{Fall 1}
\exists a \in f^{-1}(B'): f(a)=b \\
\Rightarrow f(a)=b \in B'
\intertext{Fall 2}
\nexists a \in f^{-1}(B'): f(a)=b
\end{alignat*}
Anhand von Fall 1 ist klar, dass es für jedes Urbild von einem $b \in B'$ ein Bild $b' \in B'$ gibt. Fall 2 behandelt die $b \in B'$, die kein Urbild haben. Die Menge $f(f^{-1}(B'))$ enthält nur solche $b \in B'$, die ein Urbild haben und ist demzufolge eine Teilmenge von $B'$.
Also gilt die Aussage (6), Skript Seite 61. \hfill $\Box$
\subsection{} %b
Es gebe die Mengen $A$ und $B'$ mit $A = \{1\}$ und $B' = \{5, 6\}$. Es gebe die folgende Abbildungsvorschrift $f(1) = 5$.
Die Urbildmenge von $B'$ ist in diesem Fall $A$. Die Bildmenge von $A$ ist in diesem Fall $\{5\}$. Es gilt $\{5\} \neq B'$.
\end{document}

View File

@ -0,0 +1,84 @@
\documentclass[10pt,a4paper,oneside,ngerman,numbers=noenddot]{scrartcl}
\usepackage[T1]{fontenc}
\usepackage[utf8]{inputenc}
\usepackage[ngerman]{babel}
\usepackage{amsmath}
\usepackage{amsfonts}
\usepackage{amssymb}
\usepackage{paralist}
\usepackage[locale=DE,exponent-product=\cdot,detect-all]{siunitx}
\usepackage{tikz}
\usetikzlibrary{matrix,fadings,calc,positioning,decorations.pathreplacing,arrows,decorations.markings}
\pagenumbering{arabic}
\def\thesection{\arabic{section})}
\def\thesubsection{\alph{subsection})}
\def\thesubsubsection{(\arabic{subsubsection})}
\begin{document}
\author{Jim Martens}
\title{Hausaufgaben zum 06./07. Dezember}
\maketitle
\section{} %1
\subsection{} %a
Euklidischer Algorithmus:\\
\begin{alignat*}{4}
2413 &=& 5 &\cdot & 473 &+& 48 \\
473 &=& 9 &\cdot & 48 &+& 41 \\
48 &=& 1 &\cdot & 41 &+& 7 \\
41 &=& 5 &\cdot & 7 &+& 6 \\
7 &=& 1 &\cdot & 6 &+& \underline{1} \\
6 &=& 6 &\cdot & 1 &+& 0
\end{alignat*}
Es gilt also $d = \,\text{ggT}\,(a,b) = 1$. Ausgehend von der vorletzten Gleichung erhält man durch Rückwärtseinsetzen:
\begin{alignat*}{2}
d = 6& = &&7 - 1 \cdot 6 \\
&=&& 7 - 1 \cdot (41 - 5 \cdot 7) \\
&=&& (-1) \cdot 41 + 6 \cdot 7 \\
&=&& (-1) \cdot 41 + 6 \cdot (48 - 1 \cdot 41) \\
&=&& 6 \cdot 48 + (-7) \cdot 41 \\
&=&& 6 \cdot 48 + (-7) \cdot (473 - 9 \cdot 48) \\
&=&& (-7) \cdot 473 + 69 \cdot 48 \\
&=&& (-7) \cdot 473 + 69 \cdot (2413 - 5 \cdot 473) \\
&=&& 69 \cdot 2413 + (-352) \cdot 473 \\
&=&& \lambda \cdot 2413 + \mu \cdot 473 \;\text{für} \; \lambda = 69 \;\text{und}\; \mu = -352
\end{alignat*}
Daraus folgt $1 \equiv \lambda \cdot 2413 + \mu \cdot 473 \equiv \mu \cdot 473$ (mod $2413$). Also ist $\mu$ bzw. $2061$ das Inverse von $473$ in $\mathbb{Z}_{2413}$.
\subsection{} %b
Euklidischer Algorithmus:\\
\begin{alignat*}{4}
2413 &=& 1 &\cdot & 1672 &+& 741 \\
1672 &=& 2 &\cdot & 741 &+& 190 \\
741 &=& 3 &\cdot & 190 &+& 171 \\
190 &=& 1 &\cdot & 171 &+& 19 \\
171 &=& 9 &\cdot & 19 &+& 0
\end{alignat*}
Da ggT$(2413,1672) = 19$ gilt, ist $1672$ in $\mathbb{Z}_{2413}$ nicht invertierbar.
\subsection{} %c
Das Inverse von $2412$ in $\mathbb{Z}_{2413}$ ist $-1$. Dies gilt, da $2412$ durch eine beliebige Zahl in der gleichen Restklasse ausgetauscht werden kann. $-1$ liegt in derselben Restklasse, denn von $-2413$ bis $-1$ sind es genau $2412$ Rest. Und $-1$ mit sich selbst multipliziert ergibt $1$.
\section{} %2
Es gilt $1000 = 12$ (mod 19). Damit ergibt sich $3^{12} = 3^{1000}$ (mod 19). %todo
\section{} %3
\subsection{} %a
\begin{alignat*}{2}
\pi &=& (1,7,6) \circ (2,10,8,5,11,13) \circ (3,4) \circ (9,12)
\end{alignat*}
\subsection{} %b
\begin{alignat*}{2}
\pi &=& (1,6) \circ (1,7) \circ (2,13) \circ (2,11) \circ (2,5) \circ (2,8) \circ (2,10) \circ (3,4) \circ (9,12)
\end{alignat*}
\subsection{} %c
\begin{alignat*}{2}
\text{sign}\, \pi &=& -1
\end{alignat*}
\section{} %4
\subsection{} %a
Die Menge $A \times B \times C$ besitzt $30$ Elemente. Dies ergibt sich aus $3 \cdot 5 \cdot 2 = 30$.
\subsection{} %b
Es gibt $2^{30}$ verschiedene ternäre Relationen über A, B, C. Jede Teilmenge von $A \times B \times C$ ist eine ternäre Relation. Es gibt $30$ Elemente und demzufolge $2^{30}$ Teilmengen. Da jede Teilmenge eine ternäre Relation ist, ist $2^{30}$ richtig.
\end{document}

View File

@ -0,0 +1,266 @@
\documentclass[10pt,a4paper,oneside,ngerman,numbers=noenddot]{scrartcl}
\usepackage[T1]{fontenc}
\usepackage[utf8]{inputenc}
\usepackage[ngerman]{babel}
\usepackage{amsmath}
\usepackage{amsfonts}
\usepackage{amssymb}
\usepackage{paralist}
\usepackage[locale=DE,exponent-product=\cdot,detect-all]{siunitx}
\usepackage{tikz}
\usetikzlibrary{matrix,fadings,calc,positioning,decorations.pathreplacing,arrows,decorations.markings}
\pagenumbering{arabic}
\def\thesection{\arabic{section})}
\def\thesubsection{\alph{subsection})}
\def\thesubsubsection{(\arabic{subsubsection})}
\begin{document}
\author{Jim Martens}
\title{Hausaufgaben zum 13./14. Dezember}
\maketitle
\section{} %1
\subsection{} %a
Ja, die Graphen sind isomorph.
In Graph G gibt es vier Knoten mit je vier abgehenden Kanten und vier Knoten mit je drei abgehenden Kanten.
In Graph G' gibt es ebenfalls vier Knoten mit vier abgehenden Kanten und auch vier Knoten mit je drei abgehenden Kanten.
Die Knoten seien von oben links beginnend im Uhrzeigersinn aufsteigend nummeriert. Die äußeren vier Knoten im Graph G haben die Bezeichnungen 1,2,3 und 4. Die inneren vier Knoten (auch oben links beginnend) haben die Bezeichnungen 5,6,7 und 8.
Im Graph G' werden die Knoten in gleicher Weise mit Buchstaben bezeichnet. Somit ergeben sich für die äußeren Knoten a,b,c und d, sowie e,f,g, und h für die inneren Knoten (jeweils oben links beginnend).
Damit gebe es folgende Abbildung $f: G \rightarrow G'$:\\
\begin{alignat*}{2}
f(1) &=& b \\
f(2) &=& a \\
f(3) &=& c \\
f(4) &=& d \\
f(5) &=& f \\
f(6) &=& e \\
f(7) &=& g \\
f(8) &=& h
\end{alignat*}
\subsection{} %b
Alle drei Graphen sind isomorph zueinander.
\section{} %2
\subsection{} %a
G hat $\frac{10 \cdot 9}{2} = 45$ Kanten.
\subsection{} %b
Es gibt $\binom{10}{3} = 120$ Kreise mit der Länge $3$ in G.
\subsection{} %c
Es gibt $\binom{10}{4} = 210$ Kreise mit der Länge $4$ in G.
\subsection{} %d
Es gibt $\binom{10}{4}$ Möglichkeiten vier Knoten aus zehn zu nehmen. Für jede dieser vier Knoten gibt es zwei Möglichkeiten die Diagonale zu wählen. Es ergeben sich daher $\binom{10}{4} \cdot 2 = 210 \cdot 2 = 420$ Möglichkeiten einen solchen Teilgraphen aus G zu nehmen.
\section{} %3
\subsection{} %a
%\begin{figure}[h!]
$n=4$:\\
\begin{tikzpicture}[shorten >=1pt,node distance=1.1cm,on grid]
%h1
\node (v1) {$v_{1}$};
\node (v2) [below=of v1] {$v_{2}$};
\node (v3) [right=of v2] {$v_{3}$};
\node (v4) [right=of v1] {$v_{4}$};
\path[every node/.style={font=\scriptsize}]
(v1) edge (v2)
(v1) edge (v3)
(v1) edge (v4)
(v2) edge (v3)
(v2) edge (v4)
(v3) edge (v4);
%h2
\node (v5) [right=2 of v4] {$v_{5}$}
edge [bend right] (v1)
edge [bend left=50] (v2)
edge (v4)
edge [bend left] (v3);
\node (v6) [below=of v5] {$v_{6}$}
edge [bend right=50] (v1)
edge [bend right] (v4)
edge (v3)
edge [bend left] (v2);
\node (v7) [right=of v6] {$v_{7}$}
edge [bend left=60] (v1)
edge [bend left] (v4)
edge [bend left] (v3)
edge [bend left] (v2);
\node (v8) [right=of v5] {$v_{8}$}
edge [bend right] (v1)
edge [bend right=60] (v2)
edge [bend right] (v4)
edge [bend right] (v3);
\path[every node/.style={font=\scriptsize}]
(v5) edge (v6)
(v5) edge (v7)
(v5) edge (v8)
(v6) edge (v7)
(v6) edge (v8)
(v7) edge (v8);
\end{tikzpicture}
$n=6$:\\
\begin{tikzpicture}[shorten >=1pt,node distance=1.1cm,on grid]
%h1
\node (v1) {$v_{1}$};
\node (v2) [below left=of v1] {$v_{2}$};
\node (v3) [below right=of v2] {$v_{3}$};
\node (v4) [right=of v3] {$v_{4}$};
\node (v5) [above right=of v4] {$v_{5}$};
\node (v6) [above left=of v5] {$v_{6}$};
\path[every node/.style={font=\scriptsize}]
(v1) edge (v2)
(v1) edge (v6)
(v1) edge (v4)
(v2) edge (v3)
(v2) edge (v5)
(v3) edge (v4)
(v3) edge (v6)
(v4) edge (v5)
(v5) edge (v6);
%h2
\node (v7) [right=2.5 of v6] {$v_{7}$}
edge (v6)
edge [bend right=10] (v5)
edge (v4)
edge [bend right] (v1)
edge [bend right=70] (v2)
edge [bend right=30] (v3);
\node (v8) [below left=of v7] {$v_{8}$}
edge (v5)
edge [bend right=10] (v6)
edge [bend left=10] (v4)
edge [bend right=87] (v2)
edge [bend right=10] (v1)
edge [bend left=10] (v3);
\node (v9) [below right=of v8] {$v_{9}$}
edge (v4)
edge [bend left=10] (v5)
edge (v6)
edge [bend left] (v3)
edge [bend left=70] (v2)
edge [bend left=30] (v1);
\node (v10) [right=of v9] {$v_{10}$}
edge [bend left] (v3)
edge [bend left] (v4)
edge [bend left] (v5)
edge [bend left=60] (v2)
edge [bend right=15] (v6)
edge [bend left=40] (v1);
\node (v11) [above right=of v10] {$v_{11}$}
edge [bend right=15] (v5)
edge (v6)
edge (v4)
edge [bend right=85] (v2)
edge [bend right=45] (v1)
edge [bend left=45] (v3);
\node (v12) [above left=of v11] {$v_{12}$}
edge [bend right] (v1)
edge [bend right] (v6)
edge [bend right] (v5)
edge [bend right=60] (v2)
edge [bend left=15] (v4)
edge [bend right=40] (v3);
\path[every node/.style={font=\scriptsize}]
(v7) edge (v8)
(v7) edge (v9)
(v7) edge (v10)
(v7) edge (v11)
(v7) edge (v12)
(v8) edge (v9)
(v8) edge (v10)
(v8) edge (v11)
(v8) edge (v12)
(v9) edge (v10)
(v9) edge (v11)
(v9) edge (v12)
(v10) edge (v11)
(v10) edge (v12)
(v11) edge (v12);
\end{tikzpicture}
%\end{figure}
\subsection{} %b
Es ist zu zeigen, dass $|E(G)| = \frac{3}{2}n^{2} + n$ gilt.\\
In der Zusammenhangskomponente $H_{1}$ hat jeder Knoten den Grad $3$. Bei $n$ Knoten ergibt dies $\frac{3n}{2}$ Kanten in $H_{1}$. In der Zusammenhangskomponente $H_{2}$ gibt es auch $n$ Knoten und jeder Knoten ist mit jedem verbunden. Es ergeben sich also dort $\frac{n \cdot (n-1)}{2}$ Kanten. In $G$ gibt es nun zusätzlich zwischen jedem Knoten aus $H_{1}$ und $H_{2}$ eine Kante, wodurch sich $\frac{n \cdot n}{2}$ Kanten für die Verbindungen ergeben. Zusammengefasst ergibt sich:\\
\begin{alignat*}{2}
|E(G)| &=& \frac{3n + n \cdot (n-1) + n \cdot n}{2} \\
&=& \frac{3n + n^{2} -n + n^{2}}{2} \\
&=& \frac{2n + 2n^{2}}{2} \\
&=& n + n^{2}
\end{alignat*}
Die zu zeigende Formel gilt augenscheinlich nicht.
\subsection{} %c
Für $n=4$ startet man bei $v_{4}$, geht zu $v_{1}$, dann $v_{2}$, $v_{3}$, weiter zu $v_{6}$, $v_{7}$, $v_{8}$, $v_{5}$, dann wieder zurück zu $v_{4}$.
\subsection{} %d
$G$ besitzt keine Eulersche Linie, da der Grad jedes Knotens $2n-1$ beträgt. Für die Existenz einer Eulerschen Linie muss jeder Knoten einen geraden Grad besitzen.
\section{} %4
\subsection{} %a
$P(M)$ enthält $2^{n} = 2^{4} = 16$ Elemente. Es gilt:\\
\begin{alignat*}{2}
P(M) &=& \{\emptyset, \{a\}, \{b\}, \{c\}, \{d\}, \{a,b\}, \{a,c\}, \{a,d\}, \{b,c\}, \{b,d\}, \{c,d\}, \\
&& \{a,b,c\}, \{a,b,d\}, \{a,c,d\}, \{b,c,d\}, \{a,b,c,d\}\}
\end{alignat*}
\subsection{} %b
\begin{tikzpicture}[shorten >=1pt,node distance=1.1cm,on grid]
\node (empty) {$\emptyset$};
\node (a) [above left=1.5 and 2.0 of empty] {$\{a\}$};
\node (b) [right=1.25 of a] {$\{b\}$};
\node (c) [right=1.25 of b] {$\{c\}$};
\node (d) [right=1.25 of c] {$\{d\}$};
\node (ab) [above left=1.5 and 0.5 of a] {$\{a,b\}$};
\node (ac) [right=1.0 of ab] {$\{a,c\}$};
\node (ad) [right=2.0 of ab] {$\{a,d\}$};
\node (bc) [right=3.0 of ab] {$\{b,c\}$};
\node (bd) [right=4.0 of ab] {$\{b,d\}$};
\node (cd) [right=5.0 of ab] {$\{c,d\}$};
\node (abc) [above=3.0 of a] {$\{a,b,c\}$};
\node (abd) [above=3.0 of b] {$\{a,b,d\}$};
\node (acd) [above=3.0 of c] {$\{a,c,d\}$};
\node (bcd) [above=3.0 of d] {$\{b,c,d\}$};
\node (abcd) [above=6.0 of empty] {$\{a,b,c,d\}$};
\path[every node/.style={font=\scriptsize}]
(empty) edge (a)
(empty) edge (b)
(empty) edge (c)
(empty) edge (d)
(a) edge (ab)
(a) edge (ac)
(a) edge (ad)
(b) edge (ab)
(b) edge (bc)
(b) edge (bd)
(c) edge (ac)
(c) edge (bc)
(c) edge (cd)
(d) edge (ad)
(d) edge (bd)
(d) edge (cd)
(ab) edge (abc)
(ac) edge (abc)
(bc) edge (abc)
(ab) edge (abd)
(ad) edge (abd)
(bd) edge (abd)
(ac) edge (acd)
(ad) edge (acd)
(cd) edge (acd)
(bc) edge (bcd)
(bd) edge (bcd)
(cd) edge (bcd)
(abc) edge (abcd)
(abd) edge (abcd)
(acd) edge (abcd)
(bcd) edge (abcd);
\end{tikzpicture}
\subsection{} %c
Solch ein Graph kam auf dem Übungsblatt (Präsenz- und Hausaufgaben) nicht vor. Entweder hat keiner der Graphen $16$ Knoten und/oder die Knoten haben nicht alle den Grad $4$. Von daher kann die Eigenschaft der Isomorphie von vornherein verneint werden.
\end{document}

View File

@ -0,0 +1,129 @@
\documentclass[10pt,a4paper,oneside,ngerman,numbers=noenddot]{scrartcl}
\usepackage[T1]{fontenc}
\usepackage[utf8]{inputenc}
\usepackage[ngerman]{babel}
\usepackage{amsmath}
\usepackage{amsfonts}
\usepackage{amssymb}
\usepackage{paralist}
\usepackage[locale=DE,exponent-product=\cdot,detect-all]{siunitx}
\usepackage{tikz}
\usetikzlibrary{matrix,fadings,calc,positioning,decorations.pathreplacing,arrows,decorations.markings}
\pagenumbering{arabic}
\def\thesection{\arabic{section})}
\def\thesubsection{\alph{subsection})}
\def\thesubsubsection{(\roman{subsubsection})}
\begin{document}
\author{Jim Martens}
\title{Hausaufgaben zum 20./21. Dezember}
\maketitle
\section{} %1
\subsection{} %a
Für $a$ hat man sechs Möglichkeiten in $\mathbb{Z}_{7}$, da $a \neq 0$ gilt. Für $b$ hat man sieben Möglichkeiten in $\mathbb{Z}_{7}$, da es für $b$ keine Einschränkungen gibt. Daher gibt es $6 \cdot 7 = 42$ Matrizen der angegebenen Form in $M$. Somit ist die Ordnung von $M$ gleich $42$.
\subsection{} %b
Zur Bestimmung des Inversen zu A mache man sich $\mathbb{Z}_{7}$ zu nutze. Es gilt $2 \cdot 2 \cdot 2 = 2 \cdot 4 = 8 = 1$. Ferner gilt $4 \cdot 1 + 3 \cdot 1 = 7 = 0$.
Damit ergibt sich als inverse Matrix $\begin{pmatrix} 4 & 3 \\ 0 & 1 \end{pmatrix}$.
\subsection{} %c
Da nur Teiler der Ordnung von $M$ für Halbgruppen in Frage kommen, kann man alle Ordnungen der Elemente auslassen, die keine Teiler von der Ordnung von $M$ sind.
\begin{alignat*}{2}
B^{2} = B \cdot B &=& \begin{pmatrix} 4 & 1 \\ 0 & 1 \end{pmatrix} \cdot \begin{pmatrix} 4 & 1 \\ 0 & 1 \end{pmatrix} \\
&=& \begin{pmatrix} 2 & 5 \\ 0 & 1 \end{pmatrix} \\
B^{3} = B^{2} \cdot B &=& \begin{pmatrix} 2 & 5 \\ 0 & 1 \end{pmatrix} \cdot \begin{pmatrix} 4 & 1 \\ 0 & 1 \end{pmatrix} \\
&=& \begin{pmatrix} 1 & 0 \\ 0 & 1 \end{pmatrix} \\
\intertext{$B$ hat demzufolge die Ordnung $3$}
C^{2} = C \cdot C &=& \begin{pmatrix} 3 & 3 \\ 0 & 1 \end{pmatrix} \cdot \begin{pmatrix} 3 & 3 \\ 0 & 1 \end{pmatrix} \\
&=& \begin{pmatrix} 2 & 5 \\ 0 & 1 \end{pmatrix} \\
C^{3} = C^{2} \cdot C &=& \begin{pmatrix} 2 & 5 \\ 0 & 1 \end{pmatrix} \cdot \begin{pmatrix} 3 & 3 \\ 0 & 1 \end{pmatrix} \\
&=& \begin{pmatrix} 6 & 4 \\ 0 & 1 \end{pmatrix} \\
C^{6} = C^{3} \cdot C^{3} &=& \begin{pmatrix} 6 & 4 \\ 0 & 1 \end{pmatrix} \cdot \begin{pmatrix} 6 & 4 \\ 0 & 1 \end{pmatrix} \\
&=& \begin{pmatrix} 1 & 0 \\ 0 & 1 \end{pmatrix} \\
\intertext{$C$ hat demzufolge die Ordnung $6$}
D^{2} = D \cdot D &=& \begin{pmatrix} 1 & 4 \\ 0 & 1 \end{pmatrix} \cdot \begin{pmatrix} 1 & 4 \\ 0 & 1 \end{pmatrix} \\
&=& \begin{pmatrix} 1 & 1 \\ 0 & 1 \end{pmatrix} \\
D^{3} = D^{2} \cdot D &=& \begin{pmatrix} 1 & 1 \\ 0 & 1 \end{pmatrix} \cdot \begin{pmatrix} 1 & 4 \\ 0 & 1 \end{pmatrix} \\
&=& \begin{pmatrix} 1 & 5 \\ 0 & 1 \end{pmatrix} \\
D^{6} = D^{3} \cdot D^{3} &=& \begin{pmatrix} 1 & 5 \\ 0 & 1 \end{pmatrix} \cdot \begin{pmatrix} 1 & 5 \\ 0 & 1 \end{pmatrix} \\
&=& \begin{pmatrix} 1 & 3 \\ 0 & 1 \end{pmatrix} \\
D^{7} = D^{6} \cdot D &=& \begin{pmatrix} 1 & 3 \\ 0 & 1 \end{pmatrix} \cdot \begin{pmatrix} 1 & 4 \\ 0 & 1 \end{pmatrix} \\
&=& \begin{pmatrix} 1 & 0 \\ 0 & 1 \end{pmatrix} \\
\intertext{$D$ hat demzufolge die Ordnung $7$}
\end{alignat*}
\subsection{} %d
Ja und zwar $\begin{pmatrix} 6 & 6 \\ 0 & 1 \end{pmatrix}$.
\section{} %2
\subsection{} %a
\begin{alignat*}{2}
s \ast y &=& z \\
z^{-1} &=& z \\
x \ast r &=& y \\
y^{-1} &=& y \\
x \ast y &=& r \\
r^{-1} &=& t
\end{alignat*}
\subsection{} %b
$G$ ist nicht kommutativ, da nicht alle Operationen kommutativ ausführbar sind. Als Beispiel sei hier $w \ast r = z$ und $r \ast w = y$ genannt.
$G$ ist nicht zyklisch, da es kein Element gibt, mit dem alle Elemente erzeugt werden können.
Die Elemente von $G$ haben folgende Ordnungen:
\begin{alignat*}{2}
i &:& 1 \\
r &:& 4 \\
s &:& 2 \\
t &:& 4 \\
w &:& 2 \\
x &:& 2 \\
y &:& 2 \\
z &:& 2
\end{alignat*}
\subsection{} %c
Da es nur zwei Elemente von $G$ mit der Ordnung $4$ gibt ($r$ und $t$), fällt die Wahl beider Untergruppen leicht.
\section{} %3
\subsection{} %a
Es wird der folgende Ausdruck vereinfacht:
\begin{alignat*}{2}
a^{-1}(bd^{-1})^{-1}bc(b^{-1}cdc)^{-1}ab^{-1} \\
a^{-1}d(b^{-1}b)(cc^{-1})d^{-1}c^{-1}bab^{-1} \\
a^{-1}(dd^{-1})c^{-1}bab^{-1} \\
a^{-1}c^{-1}bab^{-1} \\
\intertext{Wenn $G$ abelsch ist, dann gilt weiter:}
(a^{-1}a)c^{-1}(bb^{-1}) \\
c^{-1}
\end{alignat*}
\subsection{} %b
\textbf{Behauptung:} Jede zyklische Gruppe ist abelsch.\\
\textbf{Beweis:}\\
Sei $a \in G$ der Erzeuger der Gruppe $G$. Dann gilt für ein beliebiges $b \in G$: $b=a^{k}$.
Es ist zu zeigen, dass $c \cdot d = d \cdot c$ für alle $c,d \in G$ gilt. Da $a$ das Erzeugerelement ist, ergibt sich $a^{m} \cdot a^{n} = a^{m+n} = a^{n} \cdot a^{m}$, womit gezeigt ist, dass jede zyklische Gruppe G abelsch ist. \hfill $\Box$
\subsection{} %c
\subsubsection{} %(i)
Wahr, da es für jedes $n \in \mathbb{N}$ die zyklische Gruppe $(\mathbb{Z}_{n},+)$ gibt.
\subsubsection{} %(ii)
Wahr, da die Elementordnungen gleich häufig vorkommen.
\section{} %4
\textbf{Behauptung:} Außer der zyklischen Gruppe $G = <a> = \{1,a,a^{2},a^{3}\}$ gibt es bis auf Isomorphie nur eine weitere Gruppe der Ordnung $4$.\\
\textbf{Beweis:}\\
Sei $G = \{1,a,b,c\}$ eine nicht zyklische Gruppe. $G$ hat die Ordnung $4$. Da $G$ keine zyklische Gruppe sein soll, bleiben die Ordnungen $1$ und $2$. Das neutrale Element $1$ hat die Ordnung $1$. Somit haben $a,b,c$ die Ordnung $2$. Dies gilt, da die Ordnung eines Elements einer Gruppe, die Gruppenordnung teilen muss.
Somit ergibt eine Verknüpfung eines Elements mit sich selbst immer das neutrale Element. Die erste Spalte und Zeile sind ohnehin klar. Bleiben $6$ Felder übrig. Je Feld sind aber bereits $3$ Zeichen belegt, sodass nur eins gewählt werden kann. Es ergibt sich diese Gruppentafel:
\begin{tabular}{c||c|c|c|c}
& 1 & a & b & c \\
\hline
\hline
1 & 1 & a & b & c \\
\hline
a & a & 1 & c & b \\
\hline
b & b & c & 1 & a \\
\hline
c & c & b & a & 1
\end{tabular}
Somit wurde die Behauptung bestätigt. \hfill $\Box$
Eine bereits bekannte nicht-zyklische Gruppe der Ordnung 4 wäre z. B.: $G = \{1,a,b,c\}$ konkret definiert als $\mathbb{Z}_{6} \setminus \{1,5\} = \{0,2,3,4\}$ definiert über die Operation $+$.
\end{document}

View File

@ -0,0 +1,134 @@
\documentclass[10pt,a4paper,oneside,ngerman,numbers=noenddot]{scrartcl}
\usepackage[T1]{fontenc}
\usepackage[utf8]{inputenc}
\usepackage[ngerman]{babel}
\usepackage{amsmath}
\usepackage{amsfonts}
\usepackage{amssymb}
\usepackage{paralist}
\usepackage{qtree}
\usepackage[locale=DE,exponent-product=\cdot ,detect-all]{siunitx}
\usepackage{tikz}
\usepackage[scaled=0.78]{luximono}
\usepackage{listings}
\usepackage{subfigure}
\usetikzlibrary{automata,matrix,fadings,calc,positioning,decorations.pathreplacing,decorations.text,arrows}
\pagenumbering{arabic}
\def\thesection{10.\arabic{section})}
\def\thesubsection{(\alph{subsection})}
\def\thesubsubsection{(\arabic{subsubsection})}
\renewcommand{\labelenumi}{(\roman{enumi})}
\hyphenation{Nach-komma-stel-len}
\lstnewenvironment{java}[1][]{%
\lstset{basicstyle=\ttfamily ,backgroundcolor=\color[gray]{.95},columns=flexible,fontadjust=true,language=Java,tabsize=4,numbers=none,#1}%
}{%
}
\graphicspath{{D:/Users/Jim-Studium/Pictures/Studium/RS/}}
\tikzstyle{huffmanNodes}=[matrix of nodes,
nodes={circle,thin,draw=black!20,minimum size=10mm,text height=1.5ex,text depth=.25ex,inner sep=-10pt}]
\tikzstyle{huffmanBase}=[matrix of nodes,
nodes={minimum size=10mm,text height=1.5ex,text depth=.25ex,inner sep=-10pt}]
\begin{document}
\author{Jim Martens (Matrikelnummer 6420323) \and Marlo Kornblum (Matrikelnummer 6427301)}
\title{Rechnerstrukturen Aufgabenblatt 10}
\maketitle
\section{} %10.1
\subsection{} %a
Der Ampelautomat gibt den Befehl zum Zurücksetzen des Zählers an den Zähler. Der Zähler wiederum gibt den Zählerstand an den Ampelautomat.
\subsection{} %b
Wenn sich der Hauptautomat im Zustand rot-gelb befindet, dann erfährt er bei der nächsten Taktflanke vom Zähler den aktuellen Stand. Auf diesen kann er aber erst bei der nächsten Vorderflanke reagieren und einen Reset schicken, falls der eingestellte Maximalwert erreicht wurde.
Gleichzeitig wird er also bei der zweiten Taktvorderflanke, falls dem so war, in den Zustand grün wechseln und dem Zähler einen neuen Zählauftrag geben. Dieser erfährt davon erst im darauffolgenden Takt und startet dann den Zähler.
In jedem Takt gibt der Zähler den Zählerstand an den Hauptautomaten. Dieser kann darauf erst im nachfolgenden Takt reagieren.
Sind die $30$ Sekunden vorbei, dann wird der Zähler erst im nachfolgenden Takt diese Info weitergeben können und der Hauptautomat erst im darauffolgenden Takt darauf reagieren können und somit grün verlassen.
\subsection{} %c
Der Zähler arbeitet mit der Vorderflanke und der Hauptautomat mit der Rückflanke.
Der Hauptautomat befindet sich in der Phase rot-gelb und wartet auf das Erreichen des Maximalwertes vom Zähler. In der Vorderflanke meldet der Zähler den Zählerstand. In der Rückflanke prüft der Hauptautomat diesen auf Erreichen des Maximalwertes. Falls dem so ist, schaltet er auf grün und setzt den Zähler mit einem neuen Maximalwert zurück. Diesen bekommt der Zähler bei der nächsten Vorderflanke.
Somit liegt der Zähler auch hier einen Takt hinten, da er die Lücke zwischen zwei Takten noch gar nicht mitzählen konnte. Der Hauptautomat erfährt nun, dass der Zähler bei $30$ angelangt ist und schaltet auf gelb. Gleichzeitig wird der Zähler erneut zurückgesetzt.
\subsection{} %d
\begin{tikzpicture}[shorten >=1pt,node distance=2.0cm,on grid,auto,/tikz/initial text=,>=stealth']
\tikzset{every state/.style={minimum size=5.0em}}
\node[state,initial] (Z0) {$Z_{0}$};
\node[state] (Z1) [below right=2.6 and 3 of Z0] {$Z_{1}$};
\node[state] (Z2) [below left=2.6 and 3 of Z1] {$Z_{2}$};
\node[state] (Z3) [above left=2.6 and 3 of Z2] {$Z_{3}$};
\node (z0) [below=0.2 of Z0] {\tiny{$00$}};
\node (z1) [below=0.2 of Z1] {\tiny{$01$}};
\node (z2) [below=0.2 of Z2] {\tiny{$10$}};
\node (z3) [below=0.2 of Z3] {\tiny{$11$}};
\path[every node/.style={font=\scriptsize}]
(Z0) edge[loop above,->] node [near start] {$i<30000$} (Z0)
(Z0) edge[->] node [near start] {$i=30000;$ reset i} (Z1)
(Z1) edge[loop right,->] node [near start] {$i<3000$} (Z1)
(Z1) edge[->] node [near start] {$i=3000;$ reset i} (Z2)
(Z2) edge[loop below,->] node [near start] {$i<35000$} (Z2)
(Z2) edge[->] node [near start] {$i=35000;$ reset i} (Z3)
(Z3) edge[loop left,->] node [near start] {$i<5000$} (Z3)
(Z3) edge[->] node [near start] {$i=5000;$ reset i} (Z0);
\draw (-0.8,-0.3) -- +(1.67,0);
\draw (2.2,-2.9) -- +(1.67,0);
\draw (-0.80,-5.5) -- +(1.67,0);
\draw (-3.83,-2.9) -- +(1.67,0);
\node (z00) [below=0.45 of Z0] {\small{$r;i++$}};
\node (z01) [below=0.45 of Z1] {\small{$r,ge;i++$}};
\node (z02) [below=0.45 of Z2] {\small{$gr;i++$}};
\node (z03) [below=0.45 of Z3] {\small{$ge;i++$}};
\end{tikzpicture}
\section{} %10.2
\subsection{} %a
\begin{tabular}{cccc||cc}
$z_{1}$ & $z_{0}$ & $x_{1}$ & $x_{0}$ & $z_{1}^{+}$ & $z_{0}^{+}$ \\
\hline
0 & 0 & * & 1 & 0 & 0 \\
0 & 0 & * & 0 & 0 & 1 \\
0 & 1 & * & 0 & 0 & 1 \\
0 & 1 & 1 & 1 & 1 & 0 \\
0 & 1 & 0 & 1 & 1 & 1 \\
1 & 0 & 1 & * & 1 & 0 \\
1 & 0 & 0 & 0 & 0 & 1 \\
1 & 0 & 0 & 1 & 0 & 0 \\
1 & 1 & * & 1 & 1 & 1 \\
1 & 1 & * & 0 & 1 & 0 \\
\end{tabular}
\begin{alignat*}{2}
\delta(Z_{0}, x_{0}) &=& Z_{0}\\
\delta(Z_{0}, \overline{x_{0}}) &=& Z_{1}\\
\delta(Z_{1}, \overline{x_{1}}, x_{0}) &=& Z_{3}\\
\delta(Z_{1}, x_{1}, x_{0}) &=& Z_{2}\\
\delta(Z_{1}, \overline{x_{0}}) &=& Z_{1}\\
\delta(Z_{2}, \overline{x_{1}}, x_{0}) &=& Z_{0}\\
\delta(Z_{2}, \overline{x_{1}}, \overline{x_{0}}) &=& Z_{1}\\
\delta(Z_{2}, x_{1}) &=& Z_{2}\\
\delta(Z_{3}, x_{0}) &=& Z_{3}\\
\delta(Z_{3}, \overline{x_{0}}) &=& Z_{2}
\end{alignat*}
\subsection{} %b
\begin{tabular}{cc||cc}
$z_{1}$ & $z_{0}$ & $y_{1}$ & $y_{0}$ \\
\hline
0 & 0 & 1 & 0 \\
0 & 1 & 0 & 1 \\
1 & 0 & 0 & 1 \\
1 & 1 & 1 & 1 \\
\end{tabular}
\begin{alignat*}{2}
\lambda(Z) &=& (y_{1}, y_{0}) \\
\lambda(Z_{0}) &=& (1, 0) \\
\lambda(Z_{1}) &=& (0, 1) \\
\lambda(Z_{2}) &=& (0, 1) \\
\lambda(Z_{3}) &=& (1, 1) \\
\end{alignat*}
\subsection{} %c
In allen Zuständen gibt es mindestens einen Übergang für eine Eingangsbelegung und ebenso maximal eine. Demzufolge gibt es genau einen Übergang pro Eingangsbelegung. Unabhängig davon ist es jedoch möglich, dass mehrere Belegungen den gleichen Ausgang benutzen.
\end{document}

View File

@ -0,0 +1,202 @@
\documentclass[10pt,a4paper,oneside,ngerman,numbers=noenddot]{scrartcl}
\usepackage[T1]{fontenc}
\usepackage[utf8]{inputenc}
\usepackage[ngerman]{babel}
\usepackage{amsmath}
\usepackage{amsfonts}
\usepackage{amssymb}
\usepackage{paralist}
\usepackage{qtree}
\usepackage[locale=DE,exponent-product=\cdot ,detect-all]{siunitx}
\usepackage{tikz}
\usepackage[scaled=0.78]{luximono}
\usepackage{listings}
\usepackage{subfigure}
\usetikzlibrary{automata,matrix,fadings,calc,positioning,decorations.pathreplacing,decorations.text,arrows}
\pagenumbering{arabic}
\def\thesection{11.\arabic{section})}
\def\thesubsection{(\alph{subsection})}
\def\thesubsubsection{(\arabic{subsubsection})}
\renewcommand{\labelenumi}{(\roman{enumi})}
\hyphenation{Nach-komma-stel-len}
\lstnewenvironment{java}[1][]{%
\lstset{basicstyle=\ttfamily ,backgroundcolor=\color[gray]{.95},columns=flexible,fontadjust=true,language=Java,tabsize=4,numbers=none,#1}%
}{%
}
\graphicspath{{D:/Users/Jim-Studium/Pictures/Studium/RS/}}
\tikzstyle{huffmanNodes}=[matrix of nodes,
nodes={circle,thin,draw=black!20,minimum size=10mm,text height=1.5ex,text depth=.25ex,inner sep=-10pt}]
\tikzstyle{huffmanBase}=[matrix of nodes,
nodes={minimum size=10mm,text height=1.5ex,text depth=.25ex,inner sep=-10pt}]
\begin{document}
\author{Jim Martens (Matrikelnummer 6420323) \and Marlo Kornblum (Matrikelnummer 6427301)}
\title{Rechnerstrukturen Aufgabenblatt 11}
\maketitle
\section{} %11.1
\subsection{} %a
30
\subsection{} %b
40
\subsection{} %c
50
\subsection{} %d
30
\subsection{} %e
40
\section{} %11.2
\begin{figure}[h]
7 Befehle mit einer 5-bit Registernummer und einer 24-bit Adresse:\\
\begin{tikzpicture}
\draw (0,0) -- +(12,0); %untere Kante
\draw (0,0) -- +(0,0.5); %linke Kante
\draw (0,0.5) -- +(12,0); %obere Kante
\draw (12,0) -- +(0,0.5); %rechte Kante
\draw (1.5,0) -- +(0,0.5); %rechte Kante von Opcode
\draw (4.0,0) -- +(0,0.5); %rechte Kante der Reg-nr.
\node at ++(0.7, 0.2) (opcode) {Opcode}; %Opcode
\node at ++(2.2, 0.2) (regNr) {Reg-Nr.}; %Reg-Nr.
\node at ++(6.2, 0.2) (address) {Adresse}; %Adresse
\node at ++(0.2,-0.4) (31) {$31$};
\node at ++(1.3,-0.4) (29) {$29$};
\node at ++(1.7,-0.4) (28) {$28$};
\node at ++(4.8,-0.4) (24) {$24$};
\node at ++(5.2,-0.4) (23) {$23$};
\node at ++(11.8,-0.4) (0) {$0$};
\end{tikzpicture}
\end{figure}
\begin{figure}[h]
100 Befehle mit zwei 5-bit Registernummern und einem Adressoffset:\\
\begin{tikzpicture}
\draw (0,0) -- +(12,0); %untere Kante
\draw (0,0) -- +(0,0.5); %linke Kante
\draw (0,0.5) -- +(12,0); %obere Kante
\draw (12,0) -- +(0,0.5); %rechte Kante
\draw (1.5,0) -- +(0,0.5); %rechte Kante von Opcode1
\draw (5.0,0) -- +(0,0.5); %rechte Kante von Opcode2
\draw (7.5,0) -- +(0,0.5); %rechte Kante von Reg-Nr.
\draw (10.0,0) -- +(0,0.5); %rechte Kante von Reg-Nr. 2
\node at ++(0.7,0.2) (opcode1) {$111$}; %Opcode1
\node at ++(2.2,0.2) (opcode2) {Opcode}; %Opcode2
\node at ++(5.7,0.2) (regNr) {Reg-Nr.}; %Reg-Nr.
\node at ++(8.4,0.2) (regNr2) {Reg-Nr. 2}; %Reg-Nr. 2
\node at ++(10.7,0.2) (offset) {Offset}; %Adressoffset
\node at ++(0.2,-0.4) (31) {$31$};
\node at ++(1.3,-0.4) (29) {$29$};
\node at ++(1.7,-0.4) (28) {$28$};
\node at ++(4.8,-0.4) (22) {$22$};
\node at ++(5.2,-0.4) (21) {$21$};
\node at ++(7.3,-0.4) (17) {$17$};
\node at ++(7.7,-0.4) (16) {$16$};
\node at ++(9.8,-0.4) (12) {$12$};
\node at ++(10.2,-0.4) (11) {$11$};
\node at ++(11.8,-0.4) (0) {$0$};
\end{tikzpicture}\\
Es stehen maximal $12$ Bit für den Adressoffset zur Verfügung.
\end{figure}
\begin{figure}[h]
30 Befehle ohne Adressen oder Registerangaben:\\
\begin{tikzpicture}
\draw (0,0) -- +(12,0); %untere Kante
\draw (0,0) -- +(0,0.5); %linke Kante
\draw (0,0.5) -- +(12,0); %obere Kante
\draw (12,0) -- +(0,0.5); %rechte Kante
\draw (1.5,0) -- +(0,0.5); %rechte Kante von Opcode1
\draw (5.0,0) -- +(0,0.5); %rechte Kante von Opcode2
\draw (7.5,0) -- +(0,0.5); %rechte Kante von Opcode3
\node at ++(0.7,0.2) (opcode1) {$111$}; %Opcode1
\node at ++(2.2,0.2) (opcode2) {$1111111$}; %Opcode2
\node at ++(5.7,0.2) (opcode3) {Opcode}; %Opcode3
\node at ++(8.5,0.2) (undefined) {unbenutzt}; %unbenutzt
\node at ++(0.2,-0.4) (31) {$31$};
\node at ++(1.3,-0.4) (29) {$29$};
\node at ++(1.7,-0.4) (28) {$28$};
\node at ++(4.8,-0.4) (22) {$22$};
\node at ++(5.2,-0.4) (21) {$21$};
\node at ++(7.3,-0.4) (17) {$17$};
\node at ++(7.7,-0.4) (16) {$16$};
\node at ++(11.8,-0.4) (0) {$0$};
\end{tikzpicture}
\end{figure}
Für die ersten sieben Befehle wird ein dreistelliger Opcode benötigt, um sie voneinander unterscheiden zu können. Da 8 Codes maximal möglich sind, aber nur 7 benutzt werden, kann die achte Kombination (111) als Differenzierung zur zweiten Befehlsanzahl fungieren.
Bei dieser wird ein siebenstelliger Opcode benötigt, um die 100 Befehle voneinander unterscheiden zu können. Ein sechsstelliger Opcode würde nur $64$ Befehle unterstützen.
Die 5-bit Registernummern sind in den beiden ersten Fällen vorhanden und verbrauchen beide Male 5-bit.
Der Adressoffset benutzt schließlich die übrigen unbenutzten Stellen.
Die 30 restlichen Befehle werden von den vorigen beiden Befehlstypen durch die drei Einsen der ersten Befehle und weiteren 7 Einsen der zweiten Befehle (beide Male sind diese Werte ohne Benutzung) unterschieden. Innerhalb dieser 30 Befehle wird mithilfe eines 5-bit Opcodes zwischen den einzelnen Befehlen unterschieden.
\section{} %11.3
Wir nehmen an, dass anstatt 12-bit tatsächlich die 8-bit Kodierung gemeint war. Schließlich kann der Immediate-Wert nur 8-bit darstellen. Daher werden wir im Folgenden die Werte in 8-bit Kodierung darstellen.
\subsection{} %a
$185_{10} = 10111001_{2}, rot = 16$
\subsection{} %b
$355_{10} = 101100011_{2}$
Kann nicht dargestellt werden, da der Immediate-Wert nur 8-bit nutzen kann. Hier werden jedoch 9-bit benutzt.
\subsection{} %c
$1576_{10} = 11000101_{2}, rot = n.e.$
Kann nicht dargestellt werden, da durch Rotierung des Immediate-Wertes nicht die nötige Position erreicht werden kann. Es würde eine ungerade Anzahl an Schiebeoperationen erfordern, was hier nicht möglich ist.
\subsection{} %d
$1584_{10} = 01100011_{2}, rot = 14$
\subsection{} %e
$2415919104_{10} = 00001001_{2}, rot = 2$
\section{} %11.4
\subsection{} %a
0-Adress-Maschine:\\
PUSH F\\
PUSH E\\
PUSH D\\
MUL\\
ADD\\
PUSH C\\
PUSH B\\
MUL\\
PUSH A\\
SUB\\
DIV\\
POP R\\
\\
1-Adress-Maschine:\\
LOAD D\\
MUL E\\
ADD F\\
STORE G\\
LOAD B\\
MUL C\\
STORE H\\
LOAD A\\
SUB H\\
DIV G\\
STORE R\\
\\
2-Adress-Maschine:\\
MOV G,D\\
MUL G,E\\
ADD G,F\\
MOV H,B\\
MUL H,C\\
SUB A,H\\
DIV A,G\\
MOV R,A\\
\\
3-Adress-Maschine:\\
LOAD X,D\\
LOAD Y,E\\
MUL X,X,Y\\
LOAD Y,F\\
ADD X,X,Y\\
LOAD Y,B\\
LOAD Z,C\\
MUL Y,Y,Z\\
LOAD Z,A\\
SUB Z,Z,Y\\
DIV Z,Z,X\\
STORE R,Z\\
\subsection{} %b
Die 0-Adress-Maschine verbraucht $212$ Bit. Die 1-Adress-Maschine verbraucht $236$ Bit. Die 2-Adress-Maschine verbraucht $208$ Bit. Die 3-Adress-Maschine verbraucht $220$ Bit.
Die 2-Adress-Maschine verbraucht am wenigsten Bit.
\end{document}

View File

@ -0,0 +1,153 @@
\documentclass[10pt,a4paper,oneside,ngerman,numbers=noenddot]{scrartcl}
\usepackage[T1]{fontenc}
\usepackage[utf8]{inputenc}
\usepackage[ngerman]{babel}
\usepackage{amsmath}
\usepackage{amsfonts}
\usepackage{amssymb}
\usepackage{paralist}
\usepackage{qtree}
\usepackage[locale=DE,exponent-product=\cdot ,detect-all]{siunitx}
\usepackage{tikz}
\usepackage[scaled=0.78]{luximono}
\usepackage{listings}
\usepackage{subfigure}
\usetikzlibrary{automata,matrix,fadings,calc,positioning,decorations.pathreplacing,decorations.text,arrows}
\pagenumbering{arabic}
\def\thesection{12.\arabic{section})}
\def\thesubsection{(\alph{subsection})}
\def\thesubsubsection{(\arabic{subsubsection})}
\renewcommand{\labelenumi}{(\roman{enumi})}
\hyphenation{Nach-komma-stel-len}
\lstnewenvironment{java}[1][]{%
\lstset{basicstyle=\ttfamily ,backgroundcolor=\color[gray]{.95},columns=flexible,fontadjust=true,language=Java,tabsize=4,numbers=none,#1}%
}{%
}
\graphicspath{{D:/Users/Jim-Studium/Pictures/Studium/RS/}}
\tikzstyle{huffmanNodes}=[matrix of nodes,
nodes={circle,thin,draw=black!20,minimum size=10mm,text height=1.5ex,text depth=.25ex,inner sep=-10pt}]
\tikzstyle{huffmanBase}=[matrix of nodes,
nodes={minimum size=10mm,text height=1.5ex,text depth=.25ex,inner sep=-10pt}]
\begin{document}
\author{Jim Martens (Matrikelnummer 6420323) \and Marlo Kornblum (Matrikelnummer 6427301)}
\title{Rechnerstrukturen Aufgabenblatt 12}
\maketitle
\section{} %12.1
\subsection{} %a
Ziel: $0x100$ \\
Resultat in Ziel: $0x0000CB03$
\subsection{} %b
Ziel: $0x104$ \\
Resultat in Ziel: $0x000000BF$
\subsection{} %c
Ziel: $0x108$ \\
Resultat in Ziel: $0x00246000$
\subsection{} %d
Ziel: $0x10C$ \\
Resultat in Ziel: $0x00000043$
\subsection{} %e
Ziel: $\%ecx$ \\
Resultat in Ziel: $0x0000000B$
\subsection{} %f
Ziel: $\%eax$ \\
Resultat in Ziel: $0x000000FC$
\section{} %12.2
xorl $<Rb>, <Rb>$
\section{} %12.3
movl: $\%eip, \%eax$
\section{} %12.4
pushl $\%ebp$\\
movl $\%esp, \%ebp$ \\
movl $8(\%ebp), \%eax$ \\
subl $\$32, \%eax$ \\
imull $\$142, \%eax$ \\
movl $\%ebp, \%esp$ \\
popl $\%ebp$ \\
ret
\section{} %12.5
\subsection{} %a
Der Reihenfolge nach speichert das Assemblerprogramm zunächst das Register $\%ebx$ auf den Stack. Anschließend wird vom Register $\%esp$ die dezimale Konstante $24$ abgezogen und der neue Wert in $\%esp$ gespeichert. Nun wird der Inhalt, der ausgehend vom Wert von $\%esp$ plus einem Offset von $32$ im Stack steht, in das Register $\%ebx$ geschrieben. Im nächsten Schritt wird der Inhalt, der ausgehend vom Wert von $\%esp$ plus einem Offset von $36$ im Stack liegt, in das Register $\%edx$ kopiert.
In das Register $\%eax$ wird schließlich die Konstante $1$ gespeichert. Nun wird das Register $\%edx$ mit sich selbst verglichen. Dabei werden nur die Flags gesetzt, das Ergebnis selber wird jedoch verworfen. Das hier interessante (wegen dem nachfolgenden Jumpbefehl) Zero Flag wird gesetzt, wenn $\%edx$ gleich $0$ ist, ansonsten wird es nicht gesetzt.
In der darauffolgenden Zeile folgt ein Jumpbefehl, der nur ausgeführt wird, wenn das Zero Flag gesetzt ist. Falls dem so ist, wird zu L2 gesprungen.
Dort wird zum Inhalt des Registers $\%esp$ die dezimale Konstante $24$ hinzugezählt. Anschließend wird das oberste Element des Stacks in das Register $\%ebx$ gespeichert und der aktuelle Aufruf beendet.
Wenn nicht gesprungen wird, dann wird dieses hier ausgeführt:
Nun wird $1$ vom Inhalt von $\%edx$ subtrahiert. Anschließend wird der Inhalt von $\%edx$ an die Stelle geschrieben, die in $\%esp$ plus dem Offset von $4$ referenziert ist. Dann wird der Inhalt von $\%ebx$ an die Stelle geschrieben, die in $\%esp$ referenziert ist.
Jetzt ruft sich myst rekursiv erneut auf. Sobald bei einem Aufruf von myst das Register $\%edx$ gleich $0$ ist, wird der Aufruf beendet und abschließend in $\%eax$ das Produkt der Werte von $\%eax$ und $\%ebx$ geschrieben.
Damit wird das Unterprogramm beendet.
\subsection{} %b
Das Unterprogramm berechnet $5^{2}=25$. Allgemeiner wird $a^{b}$ berechnet.
\subsection{} %c
\begin{tikzpicture}
\draw (0, 0) -- +(2, 0);
\draw (0, 0.5) -- +(2, 0);
\draw (0, 1) -- +(2, 0);
\draw (0, 1.5) -- +(2, 0);
\draw (0, 2) -- +(2, 0);
\draw (0, 2.5) -- +(2, 0);
\draw (0, 3) -- +(2, 0);
\draw (0, 3.5) -- +(2, 0);
\draw (0, 4) -- +(2, 0);
\draw (0, 4.5) -- +(2, 0);
\draw (0, 5) -- +(2, 0);
\draw (0, 5.5) -- +(2, 0);
\draw (0, 6) -- +(2, 0);
\draw (0, 6.5) -- +(2, 0);
\draw (0, 7) -- +(2, 0);
\draw (0, 7.5) -- +(2, 0);
\draw (0, 8) -- +(2, 0);
\draw (0, 8.5) -- +(2, 0);
\draw (0, 9) -- +(2, 0);
\draw (0, 9.5) -- +(2, 0);
\draw (0, 10) -- +(2, 0);
\draw (0, 10.5) -- +(2, 0);
\draw (0, 11) -- +(2, 0);
\draw (0, 11.5) -- +(2, 0);
\draw (0, 12) -- +(2, 0);
\draw (0, 12.5) -- +(2, 0);
\draw (0, 13) -- +(2, 0);
%\draw (0, 13.5) -- +(2, 0);
\draw (0,0) -- +(0, 13.5);
\draw (2,0) -- +(0, 13.5);
\node (other) at (1, 13.25) {...};
\node (b1) at (1, 12.75) {b = 2};
\node (a1) at (1, 12.25) {a = 5};
\node (main) at (1, 11.75) {$\%eip$ main()};
\node (ebx1) at (1, 11.25) {$\%ebx \rightarrow ?$};
\node (other2) at (1, 10.75) {...};
\node (other3) at (1, 10.25) {...};
\node (other4) at (1, 9.75) {...};
\node (other5) at (1, 9.25) {...};
\node (b2) at (1, 8.75) {b = 1};
\node (a2) at (1, 8.25) {a = 5};
\node (myst1) at (1, 7.75) {$\%eip$ myst()};
\node (ebx2) at (1, 7.25) {$\%ebx \rightarrow 5$};
\node (other6) at (1, 6.75) {...};
\node (other7) at (1, 6.25) {...};
\node (other8) at (1, 5.75) {...};
\node (other9) at (1, 5.25) {...};
\node (b3) at (1, 4.75) {b = 0};
\node (a3) at (1, 4.25) {a = 5};
\node (myst2) at (1, 3.75) {$\%eip$ myst()};
\node (ebx3) at (1, 3.25) {$\%ebx \rightarrow 5$};
\node (other10) at (1, 2.75) {...};
\node (other11) at (1, 2.25) {...};
\node (other12) at (1, 1.75) {...};
\node (other13) at (1, 1.25) {...};
\node (other14) at (1, 0.75) {...};
\node (other15) at (1, 0.25) {...};
\node (esp) at (-0.75, 0.25) {$\%esp \rightarrow$};
\end{tikzpicture}
\subsection{} %d
Es würde eine Endlosschleife entstehen, da der zweite Parameter nie $0$ wird und damit nie die Sprungmarke L2 erreicht wird, wodurch nie ein Aufruf beendet würde und unendlich viele Rekursionsschritte durchgeführt würden.
\end{document}

View File

@ -0,0 +1,144 @@
\documentclass[10pt,a4paper,oneside,ngerman,numbers=noenddot]{scrartcl}
\usepackage[T1]{fontenc}
\usepackage[utf8]{inputenc}
\usepackage[ngerman]{babel}
\usepackage{amsmath}
\usepackage{amsfonts}
\usepackage{amssymb}
\usepackage{paralist}
\usepackage[locale=DE,exponent-product=\cdot,detect-all]{siunitx}
\usepackage{tikz}
\usetikzlibrary{matrix,fadings,calc,positioning,decorations.pathreplacing,arrows}
\pagenumbering{arabic}
\def\thesection{3.\arabic{section})}
\def\thesubsection{(\alph{subsection})}
\def\thesubsubsection{(\arabic{subsubsection})}
\renewcommand{\labelenumi}{(\roman{enumi})}
\hyphenation{Nach-komma-stel-len}
\begin{document}
\author{Jim Martens (Matrikelnummer 6420323) \and Marlo Kornblum (Matrikelnummer 6427301)}
\title{Rechnerstrukturen Aufgabenblatt 3}
\maketitle
\section{}%3.1
\subsection{}
\begin{alignat}{3}
&\; 1385_{10} &-&\: 532_{10} &=&\: x\\
\intertext{Bildung des 10-Komplements des Subtrahenden:}
\Leftrightarrow &\; 1385_{10} &+&\: K_{10}(532)_{10} &=&\: x\\
\Leftrightarrow &\; 1385_{10} &+&\: 9467_{10} &=&\: 10852_{10}\\
\intertext{Streichen der führenden Eins}
\Leftrightarrow &\; 1385_{10} &-&\: 532_{10} &=&\: \underline{\underline{0852_{10}}}
\end{alignat}
\subsection{}
\begin{alignat}{3}
&\; 372_{10} &-&\: 687_{10} &=&\: x\\
\intertext{Bildung des 10-Komplements des Subtrahenden:}
\Leftrightarrow &\; 372_{10} &+&\: K_{10}(0687)_{10} &=&\: x\\
\Leftrightarrow &\; 372_{10} &+&\: 9313_{10} &=&\: 9685_{10}\\
\intertext{Bildung des 10-Komplements des Ergebnisses}
\Leftrightarrow &\; K_{10}(9685)_{10} &=&\: \underline{\underline{-0315_{10}}}
\end{alignat}
\subsection{}
\begin{alignat}{3}
&\; 1385_{10} &-&\: 532_{10} &=&\: x\\
\intertext{Umwandlung in Dualzahlen:}
&\; 10101101001_{2} &-&\: 1000010100_{2} &=&\: x\\
\intertext{Bildung des 2-Komplements des Subtrahenden:}
\Leftrightarrow &\; 10101101001_{2} &+&\: K_{2}(001000010100)_{2} &=&\: x\\
\Leftrightarrow &\; 10101101001_{2} &+&\: 110111101100_{2} &=&\: 101101010101_{2}\\
\intertext{Streichen der führenden Eins}
\Leftrightarrow &\; 10101101001_{2} &-&\: 001000010100_{2} &=&\: \underline{\underline{001101010101_{2}}}
\end{alignat}
\subsection{}
\begin{alignat}{3}
&\; 372_{10} &-&\: 687_{10} &=&\: x\\
\intertext{Umwandlung in Dualzahlen:}
&\; 101110100_{2} &-&\: 1010101111_{2} &=&\: x\\
\intertext{Bildung des 2-Komplements:}
\Leftrightarrow &\; 101110100_{2} &+&\: K_{2}(001010101111)_{2} &=&\: x\\
\Leftrightarrow &\; 000101110100_{2} &+&\: 110101010001_{2} &=&\: 111011000101_{2}\\
\intertext{Bildung des 2-Komplements des Ergebnisses}
\Leftrightarrow &\; K_{2}(111011000101)_{2} &=&\: \underline{\underline{000100111011_{2}}}
\end{alignat}
\section{}%3.2 _todo
\subsection{}
$(6,9242 \mid 4)_{10}$
\subsection{}
$(-1,100101 \mid -10)_{2}$
\subsection{}
$(-2,D4A \mid B)_{16}$
\section{}%3.3
\subsection{}
\begin{tikzpicture}
\draw (0,0) -- +(8,0); %untere Kante
\draw (0,0) -- +(0,0.5); %linke Kante
\draw (0,0.5) -- +(8,0); %obere Kante
\draw (8,0) -- +(0,0.5); %rechte Kante
\draw (0.25,0) -- +(0,0.5); %rechte Kante von Vorzeichen
\draw (2.25,0) -- +(0,0.5); %rechte Kante des Exponenten
\node at ++(0.15,0.2) (sign) {$0$}; %Vorzeichen
\node at ++(1.25,0.2) (exp) {$0000\, 0110$}; %Exponent
\node at ++(5.125,0.2) (mantisse) {$011\, 0110\, 0000\, 0000\, 0000\, 0000$}; %Mantisse
\end{tikzpicture}
\subsection{}
\begin{tikzpicture}
\draw (0,0) -- +(8,0); %untere Kante
\draw (0,0) -- +(0,0.5); %linke Kante
\draw (0,0.5) -- +(8,0); %obere Kante
\draw (8,0) -- +(0,0.5); %rechte Kante
\draw (0.25,0) -- +(0,0.5); %rechte Kante von Vorzeichen
\draw (2.25,0) -- +(0,0.5); %rechte Kante des Exponenten
\node at ++(0.15,0.2) (sign) {$1$}; %Vorzeichen
\node at ++(1.25,0.2) (exp) {$0000\, 0111$}; %Exponent
\node at ++(5.125,0.2) (mantisse) {$010\, 1000\, 1010\, 0000\, 0000\, 0000$}; %Mantisse
\end{tikzpicture}
\section{}%3.4
$8,626 \cdot 10^{5} + 9,9442 \cdot 10^{7}$\\
\subsection{}
Skalierung des kleineren Summanden, bis beide Exponenten gleich sind:
$0,08626 \cdot 10^{7} + 9,9442 \cdot 10^{7}$.\\
Daraus folgt:
\begin{alignat}{2}
&\; 0,08626 \cdot 10^{7} + 9,9442 \cdot 10^{7} &=&\: x\\
\Leftrightarrow &\; 0,08626 \cdot 10^{7} + 9,94420 \cdot 10^{7} &=&\: 10,03046 \cdot 10^{7}
\end{alignat}
Daraus ergibt sich dieses normalisierte Ergebnis:\\
$1,003046 \cdot 10^{8}$\\
Gerundet ergibt sich:\\
$1,0030 \cdot 10^{8}$\\
\subsection{}
Skalierung des kleineren Summanden, bis beide Exponenten gleich sind:
$0,0863 \cdot 10^{7} + 9,9442 \cdot 10^{7}$.\\
Daraus folgt:
\begin{alignat}{2}
&\; 0,0863 \cdot 10^{7} + 9,9442 \cdot 10^{7} &=&\: x\\
\Leftrightarrow &\; 0,0863 \cdot 10^{7} + 9,9442 \cdot 10^{7} &=&\: 10,0305 \cdot 10^{7}
\end{alignat}
Daraus ergibt sich dieses normalisierte Ergebnis:\\
$1,00305 \cdot 10^{8}$\\
Gerundet ergibt sich:\\
$1,0031 \cdot 10^{8}$\\
\subsection{}
Bei Zahlen mit vielen Nachkommastellen ist eine Rundung nach jedem Schritt vorteilhafter, weil dadurch kompletter Präzisionsverlust vermieden werden kann (mehr Nachkommastellen als sinnvoll darstellbar).
Bei Zahlen mit wenigen Nachkommastellen eignet sich das Verfahren mit einmaliger Rundung am Ende besser, da hierbei ohne Genauigkeitsverlust (zu viele Nachkommastellen) ein relativ gutes Ergebnis ermittelt werde kann.
Anhand des Beispiels wird sichtbar, dass die Entscheidung Auswirkungen auf das Ergebnis haben können (in diesem Fall im Wert von $10000$), welche je nach Kontext trivial sind oder bereits katastrophale Ausmaße annehmen können.
Von daher hängt die Wahl des "`perfekten"' Verfahrens davon ab, was man machen muss.
\section{}%3.5
$5,6538 \cdot 10^{7} * 3,1415 \cdot 10^{4}$\\
Daraus ergibt sich:\\
\begin{alignat}{2}
&\; 5,6538 \cdot 10^{7} * 3,1415 \cdot 10^{4} &=&\: x\\
\Leftrightarrow &\; 5,6538 \cdot 10^{7} * 3,1415 \cdot 10^{4} &=& (5,6538 \cdot 3,1415) \cdot 10^{7+4}\\
\Leftrightarrow &\; 5,6538 \cdot 10^{7} * 3,1415 \cdot 10^{4} &=& 17,7614127 \cdot 10^{11}
\end{alignat}
Daraus ergibt sich dieses normalisierte Ergebnis:\\
$1,77614127 \cdot 10^{12}$\\
Gerundet ergibt sich:\\
$1,7761 \cdot 10^{12}$
\end{document}

View File

@ -0,0 +1,116 @@
\documentclass[10pt,a4paper,oneside,ngerman,numbers=noenddot]{scrartcl}
\usepackage[T1]{fontenc}
\usepackage[utf8]{inputenc}
\usepackage[ngerman]{babel}
\usepackage{amsmath}
\usepackage{amsfonts}
\usepackage{amssymb}
\usepackage{paralist}
\usepackage[locale=DE,exponent-product=\cdot,detect-all]{siunitx}
\usepackage{tikz}
\usetikzlibrary{matrix,fadings,calc,positioning,decorations.pathreplacing,arrows}
\pagenumbering{arabic}
\def\thesection{4.\arabic{section})}
\def\thesubsection{(\alph{subsection})}
\def\thesubsubsection{(\arabic{subsubsection})}
\renewcommand{\labelenumi}{(\roman{enumi})}
\hyphenation{Nach-komma-stel-len}
\begin{document}
\author{Jim Martens (Matrikelnummer 6420323) \and Marlo Kornblum (Matrikelnummer 6427301)}
\title{Rechnerstrukturen Aufgabenblatt 4}
\maketitle
\section{}%4.1
\subsection{} %a
Der Nachteil dieser Methode liegt offensichtlich darin, dass in Zahlenbereichen mit vielen darstellbaren Zahlen zwei ganz unterschiedliche Zahlen als gleich angesehen werden.
Außerdem hängt die Feststellung der Gleichheit vom jeweiligen Nutzer ab, wodurch auf zwei unterschiedlichen Rechnern mit unterschiedlichen festgelegten Toleranzgrenzen die gleichen Zahlen unterschiedliche Vergleichsergebnisse liefern.
Ein Vergleich ist demzufolge also nicht eindeutig, sondern hängt von anderen Faktoren ab.
\subsection{} %b
Die relative Abweichung lässt sich wie folgt berechnen:
\begin{alignat*}{2}
y &=& |\frac{x - rd(x)}{x}|
\end{alignat*}
Bei diesem Beispiel sei $x$ die echte Gleitkommazahl und $rd(x)$ die gerundete Gleitkommazahl.
Als gleich können beide Werte angesehen werden, wenn folgendes gilt:
\begin{alignat*}{2}
y &\leq & \varepsilon = 0.5 \cdot b^{-t-1}
\end{alignat*}
$t$ sei die Anzahl der Bits für die Mantisse (bei Float $23$ und bei Double $52$) und $b$ die Basis der Gleitkommadarstellung.
\subsection{} %c
Dieses Verfahren hat den großen Nachteil, dass die Anzahl an Nachkommastellen bei der Berechnung der relativen Abweichung sehr groß wird.
%todo, beheben
\section{}%4.2
\subsection{} %a
Die CRLF SP Lösung CRLF SP SP der CRLF SP SP SP Übungsaufgabe CRLF SP SP SP SP liegt CRLF SP SP SP SP SP vor CRLF SP SP SP SP SP SP Ihnen!
\subsection{} %b
Der Rechner wurde auf Windows erstellt, da Unix-basierte Rechner nur LF nutzen, wohingegen Windows CRLF nutzt. Ab Mac OS X wird dort auch LF genutzt, vorher (bis Mac OS 9) wurde CR genutzt.
\section{}%4.3
\subsection{} %a
Bei Kodierung nach ISO-8559-1 belegt dieser Text $1 \cdot 800000 = 800000$ Byte, da jedes Zeichen mit einem Byte kodiert wird. In direkter Unicode-Kodierung belegt der Text $2 \cdot 800000 = 1600000$ Byte, da Unicode jedes Zeichen mit zwei Byte kodiert.
In UTF-8 werden Zeichen mit Vielfachen von $8$ Bit kodiert. Dabei werden alle Zeichen, die auch im ISO-8559-1 vorhanden sind mit einem Byte kodiert ($8$ Bits) und alle deutschen Sonderzeichen (Umlaute, ß) mit zwei Byte.
Demnach ergeben sich $814240$ Byte.
\subsection{} %b
Der Bereich für CJK-Symbole umfasst $3400_{16}$ bis $4DBF_{16}$ und $4E00_{16}$ bis $9FCF_{16}$. Das sind $27536$ Symbole.
\subsection{} %c
Bei direkter Unicode-Kodierung belegt der chinesische Text $2 \cdot 800000 = 1600000$ Bytes, da hier jedes Zeichen mit zwei Byte kodiert ist.
Bei UTF-8 belegt ein chinesisches Zeichen 3 Byte. Demzufolge belegt der Text in UTF-8-Kodierung $3 \cdot 800000 = 2400000$ Byte.
\section{}%4.4
\subsection{} %a
$y = (x \ll 3) + (x \ll 1)$
\subsection{} %b
$y = (x \ll 4) + (x \ll 3) + (x \ll 2) + (x \ll 1)$
\subsection{} %c
$y = (x \ll 4) - (x \ll 6)$
\subsection{} %d
$a = (x+6)$\\
$y = 60 \cdot a$\\
$y = (a \ll 5) + (a \ll 4) + (a \ll 3) + (a \ll 2)$\\
%$y = ((x+6) \ll 5) + ((x+6) \ll 4) + ((x+6) \ll 3) + ((x+6) \ll 2)$
\section{}%4.5
\subsection{} %a
$\text{bitNor}(x,y) := \sim x \, \& \sim y$
\subsection{} %b
$\text{bitXor}(x,y) := \sim(\sim(\sim x \,\&\, y) \,\&\, \sim(\sim y \,\&\, x))$
\subsection{} %c
$\text{rotateRight}(x,n) := (x\ggg n)_{2} \,|\, (x\ll (0111111_{2} \& (100000_{2} + (\sim n + 1)_{2})))$
\subsection{} %d
$\text{abs}(x) := (x \: \hat{} \: (x \gg 31)) + \sim (x \gg 31) + 1$ %todo
Die Funktion liefert für den Eingabewert $-2^{31}$ wieder denselben Wert. Das liegt daran, dass die größte positive darstellbare Zahl bei 32-bit und 2-Komplement $2^{31}-1$ ist. Bei der angegebenen Funktion wird zunächst der Wert mithilfe des Arithmetic Shift Right um 31 bit nach rechts verschoben. Ist die Zahl positiv ist die entstehende Bitmaske 0...0 und andernfalls 1...1. Diese Bitmaske wird nun xor mit dem Ausgangswert genommen. Dadurch wird der Ausgangswert wenn er negativ ist negiert und andernfalls so belassen, wie er ist.
Anschließend wird das 2-Komplement der vorher erzeugten Bitmaske addiert. Ist der Ausgangswert positiv, dann wird $0$ addiert, wodurch der Ausgangswert bestehen bleibt. Ist der Ausgangswert negativ, dann bekommt man dadurch den positiven Wert. Dies funktioniert für alle negativen Zahlen größer oder gleich $-2^{31}+1$. Noch einmal an den Beispielen $-2^{31}$(1) und $-1$(2) veranschaulicht:
Beispiel (1)
\begin{alignat*}{2}
&\; x &=& 10...0 \\
&\; a &=& x \gg 31 \\
\Leftrightarrow &\; a &=& 11...1 \\
&\; b &=& a \, \hat{}\, x \\
\Leftrightarrow &\; b &=& 01...1 \\
&\; c &=& b + K_{2}(a)_{2} \\
\Leftrightarrow &\; c &=& 01...1 + 00...01 \\
\Leftrightarrow &\; c &=& 10...0
\end{alignat*}
Beispiel (2)
\begin{alignat*}{2}
&\; x &=& 11...1 \\
&\; a &=& x \gg 31 \\
\Leftrightarrow &\; a &=& 11...1 \\
&\; b &=& a \, \hat{}\, x \\
\Leftrightarrow &\; b &=& 00...0 \\
&\; c &=& b + K_{2}(a)_{2} \\
\Leftrightarrow &\; c &=& 00...0 + 00...01 \\
\Leftrightarrow &\; c &=& 00...01
\end{alignat*}
\end{document}

View File

@ -0,0 +1,823 @@
\documentclass[10pt,a4paper,oneside,ngerman,numbers=noenddot]{scrartcl}
\usepackage[T1]{fontenc}
\usepackage[utf8]{inputenc}
\usepackage[ngerman]{babel}
\usepackage{amsmath}
\usepackage{amsfonts}
\usepackage{amssymb}
\usepackage{paralist}
\usepackage[locale=DE,exponent-product=\cdot ,detect-all]{siunitx}
\usepackage{tikz}
\usepackage[scaled=0.78]{luximono}
\usepackage{listings}
\usetikzlibrary{automata,matrix,fadings,calc,positioning,decorations.pathreplacing,decorations.text,arrows}
\pagenumbering{arabic}
\def\thesection{5.\arabic{section})}
\def\thesubsection{(\alph{subsection})}
\def\thesubsubsection{(\arabic{subsubsection})}
\renewcommand{\labelenumi}{(\roman{enumi})}
\hyphenation{Nach-komma-stel-len}
\lstnewenvironment{java}[1][]{%
\lstset{basicstyle=\ttfamily ,backgroundcolor=\color[gray]{.95},columns=flexible,fontadjust=true,language=Java,tabsize=4,numbers=none,#1}%
}{%
}
\tikzstyle{huffmanNodes}=[matrix of nodes,
nodes={circle,thin,draw=black!20,minimum size=10mm,text height=1.5ex,text depth=.25ex,inner sep=-10pt}]
\tikzstyle{huffmanBase}=[matrix of nodes,
nodes={minimum size=10mm,text height=1.5ex,text depth=.25ex,inner sep=-10pt}]
\begin{document}
\author{Jim Martens (Matrikelnummer 6420323) \and Marlo Kornblum (Matrikelnummer 6427301)}
\title{Rechnerstrukturen Aufgabenblatt 5}
\maketitle
\section{}%5.1
\begin{java}
int a1, a2, a3, a4;
int b1 = (a1 << 2) | (a2 >>> 4);
int b2 = (a2 << 4) | (a3 >>> 2);
int b3 = (a3 << 6) | a4;
...
\end{java}
\section{}%5.2
\subsection{} %a
Der Code hat $\frac{360^{\circ}}{15^{\circ}}=24$ Codewörter.
\subsection{} %b
Wir starten mit den beiden Symbolen $0$ und $1$. Daraus ergibt sich in weiteren Schritten folgendes:
\begin{alignat*}{2}
0, & 1 \\
00, 01, & 11, 10 \\
000, 001, 011, 010, & 110, 111, 101, 100 \\
0000, 0001, 0011, 0010, 0110, 0111, 0101, 0100, & 1100, 1101, 1111, 1110, 1010, 1011, 1001, 1000
\end{alignat*}
\begin{alignat*}{2}
\intertext{Nachfolgend der erste Teil der fünften Zeile}
00000, 000001, 00011, 00010, 00110, 00111, 00101, 00100,\\
\intertext{Der zweite Teil}
01100, 01101, 01111, 01110, 01010, 01011, 01001, 01000,\\
\intertext{Und nun der dritte Teil} 11000, 11001, 11011, 11010, 11110, 11111, 11101, 11100,\\
\intertext{Abschließend der vierte Teil}
10100, 10101, 10111, 10110, 10010, 10011, 10001, 10000
\end{alignat*}
Die zuletzt dargestellte Zeile enthält $32$ Codewörter. Da wir nur $24$ benötigen, werden beidseitig $4$ weggestrichen. Damit bleibt die Fano-Bedingung erfüllt. Das Ergebnis ist dieser Code (auf zwei Zeileneinträge aufgeteilt):
\begin{alignat*}{2}
00110, 00111, 00101, 00100, 01100, 01101, 01111, 01110, 01010, 01011, 01001, 01000, \\
11000, 11001, 11011, 11010, 11110, 11111, 11101, 11100, 10100, 10101, 10111, 10110
\end{alignat*}
\section{}%5.3
\subsection{} %a
Es ergeben sich die folgenden Codes:
\begin{tikzpicture}
\matrix[matrix of nodes]
{a & b & c & d & e & f & g & h & i & j & k & l \\
$100$ & $01011$ & $01000$ & $00$ & $010010$ & $1011$ & $111$ & $010011$ & $01010$ & $110$ & $011$ & $1010$ \\};
\end{tikzpicture}
\begin{figure}[hbp]
Erster Schritt:
\begin{tikzpicture}[shorten >=1pt,node distance=1.1cm,on grid,auto,/tikz/initial text=]
%level 1
\node[state] (e) {e};
\node[state] (h) [right=of e] {h};
\node[state] (b) [right=of h] {b};
\node[state] (i) [right=of b] {i};
\node[state] (c) [right=of i] {c};
\node[state] (f) [right=of c] {f};
\node[state] (l) [right=of f] {l};
\node[state] (g) [right=of l] {g};
\node[state] (j) [right=of g] {j};
\node[state] (a) [right=of j] {a};
\node[state] (k) [right=of a] {k};
\node[state] (d) [right=of k] {d};
%level 2
\node[state] (eh) [above right=1.3 and 0.6 of e] {$0.04$};
\path[every node/.style={font=\scriptsize}]
(e) edge node [near start] {$1$} (eh)
(h) edge node [near end] {$0$} (eh);
%\draw[decorate,decoration={text along path,text=$0.02$] (e) -- (eh);
\node (pE) [below=0.7 of e] {0.02};
\node (pH) [below=0.7 of h] {0.02};
\node (pB) [below=0.7 of b] {0.03};
\node (pI) [below=0.7 of i] {0.03};
\node (pC) [below=0.7 of c] {0.05};
\node (pF) [below=0.7 of f] {0.05};
\node (pL) [below=0.7 of l] {0.06};
\node (pG) [below=0.7 of g] {0.1};
\node (pJ) [below=0.7 of j] {0.1};
\node (pA) [below=0.7 of a] {0.12};
\node (pK) [below=0.7 of k] {0.12};
\node (pD) [below=0.7 of d] {0.3};
\end{tikzpicture}
\end{figure}
\begin{figure}[hbp]
Zweiter Schritt:
\begin{tikzpicture}[shorten >=1pt,node distance=1.1cm,on grid,auto,/tikz/initial text=, nodes={decoration={}} ]
%level 1
\node[state] (e2) {e};
\node[state] (h2) [right=of e2] {h};
%level 2
\node[state] (eh2) [above right=1.3 and 0.6 of e2] {$0.04$};
\node[state] (i2) [left=of eh2] {i};
\node[state] (b2) [left=of i2] {b};
\node[state] (c2) [right=of eh] {c};
\node[state] (f2) [right=of c2] {f};
\node[state] (l2) [right=of f2] {l};
\node[state] (g2) [right=of l2] {g};
\node[state] (j2) [right=of g2] {j};
\node[state] (a2) [right=of j2] {a};
\node[state] (k2) [right=of a2] {k};
\node[state] (d2) [right=of k2] {d};
%level 3
\node[state] (bi2) [above right=1.3 and 0.6 of b2] {$0.06$};
\path[every node/.style={font=\scriptsize}]
(e2) edge node [near start] {$1$} (eh2)
(h2) edge node [near end] {$0$} (eh2)
(b2) edge node [near start] {$1$} (bi2)
(i2) edge node [near end] {$0$} (bi2);
\node (pE2) [below=0.7 of e2] {0.02};
\node (pH2) [below=0.7 of h2] {0.02};
\node (pB2) [below=0.7 of b2] {0.03};
\node (pI2) [below=0.7 of i2] {0.03};
\node (pC2) [below=0.7 of c2] {0.05};
\node (pF2) [below=0.7 of f2] {0.05};
\node (pL2) [below=0.7 of l2] {0.06};
\node (pG2) [below=0.7 of g2] {0.1};
\node (pJ2) [below=0.7 of j2] {0.1};
\node (pA2) [below=0.7 of a2] {0.12};
\node (pK2) [below=0.7 of k2] {0.12};
\node (pD2) [below=0.7 of d2] {0.3};
\end{tikzpicture}
\end{figure}
\begin{figure}[hbp]
Dritter Schritt:
\begin{tikzpicture}[shorten >=1pt,node distance=1.1cm,on grid,auto,/tikz/initial text=]
%level 2
\node[state] (e3) {e};
\node[state] (h3) [right=of e3] {h};
\node[state] (b3) [right=2.2 of h3] {b};
\node[state] (i3) [right=of b3] {i};
%level 3
\node[state] (eh3) [above right=1.3 and 0.6 of e3] {$0.04$};
\node[state] (c3) [right=of eh3] {c};
\node[state] (f3) [right=of c3] {f};
\node[state] (bi3) [right=of f3] {$0.06$};
\node[state] (l3) [right=of bi3] {l};
\node[state] (g3) [right=of l3] {g};
\node[state] (j3) [right=of g3] {j};
\node[state] (a3) [right=of j3] {a};
\node[state] (k3) [right=of a3] {k};
\node[state] (d3) [right=of k3] {d};
%level 4
\node[state] (ehc3) [above right=1.3 and 0.6 of eh3] {$0.09$};
\path[every node/.style={font=\scriptsize}]
(e3) edge node [near start] {$1$} (eh3)
(h3) edge node [near end] {$0$} (eh3)
(b3) edge node [near start] {$1$} (bi3)
(i3) edge node [near end] {$0$} (bi3)
(eh3) edge node [near start] {$1$} (ehc3)
(c3) edge node [near end] {$0$} (ehc3);
\node (pE3) [below=0.7 of e3] {0.02};
\node (pH3) [below=0.7 of h3] {0.02};
\node (pB3) [below=0.7 of b3] {0.03};
\node (pI3) [below=0.7 of i3] {0.03};
\node (pC3) [below=0.7 of c3] {0.05};
\node (pF3) [below=0.7 of f3] {0.05};
\node (pL3) [below=0.7 of l3] {0.06};
\node (pG3) [below=0.7 of g3] {0.1};
\node (pJ3) [below=0.7 of j3] {0.1};
\node (pA3) [below=0.7 of a3] {0.12};
\node (pK3) [below=0.7 of k3] {0.12};
\node (pD3) [below=0.7 of d3] {0.3};
\end{tikzpicture}
\end{figure}
\begin{figure}[hbp]
Vierter Schritt:
\begin{tikzpicture}[shorten >=1pt,node distance=1.1cm,on grid,auto,/tikz/initial text=]
%level 2
\node[state] (e4) {e};
\node[state] (h4) [right=of e4] {h};
%level 3
\node[state] (eh4) [above right=1.3 and 0.6 of e4] {$0.04$};
\node[state] (i4) [left=of eh4] {i};
\node[state] (b4) [left=of i4] {b};
\node[state] (c4) [right=of eh4] {c};
%level 4
\node[state] (bi4) [above right=1.3 and 0.6 of b4] {$0.06$};
\node[state] (l4) [left=of bi4] {l};
\node[state] (f4) [left=of l4] {f};
\node[state] (ehc4) [right=2.2 of bi4] {$0.09$};
\node[state] (g4) [right=of ehc4] {g};
\node[state] (j4) [right=of g4] {j};
\node[state] (a4) [right=of j4] {a};
\node[state] (k4) [right=of a4] {k};
\node[state] (d4) [right=of k4] {d};
%level 5
\node[state] (fl4) [above right=1.3 and 0.6 of f4] {$0.11$};
\path[every node/.style={font=\scriptsize}]
(e4) edge node [near start] {$1$} (eh4)
(h4) edge node [near end] {$0$} (eh4)
(b4) edge node [near start] {$1$} (bi4)
(i4) edge node [near end] {$0$} (bi4)
(eh4) edge node [near start] {$1$} (ehc4)
(c4) edge node [near end] {$0$} (ehc4)
(f4) edge node [near start] {$1$} (fl4)
(l4) edge node [near end] {$0$} (fl4);
\node (pE4) [below=0.7 of e4] {0.02};
\node (pH4) [below=0.7 of h4] {0.02};
\node (pB4) [below=0.7 of b4] {0.03};
\node (pI4) [below=0.7 of i4] {0.03};
\node (pC4) [below=0.7 of c4] {0.05};
\node (pF4) [below=0.7 of f4] {0.05};
\node (pL4) [below=0.7 of l4] {0.06};
\node (pG4) [below=0.7 of g4] {0.1};
\node (pJ4) [below=0.7 of j4] {0.1};
\node (pA4) [below=0.7 of a4] {0.12};
\node (pK4) [below=0.7 of k4] {0.12};
\node (pD4) [below=0.7 of d4] {0.3};
\end{tikzpicture}
\end{figure}
\begin{figure}[hbp]
Fünfter Schritt:
\begin{tikzpicture}[shorten >=1pt,node distance=1.1cm,on grid,auto,/tikz/initial text=]
%level 3
\node[state] (e6) {e};
\node[state] (h6) [right=of e6] {h};
%level 4
\node[state] (eh6) [above right=1.3 and 0.6 of e6] {$0.04$};
\node[state] (i6) [left=of eh6] {i};
\node[state] (b6) [left=of i6] {b};
\node[state] (c6) [right=of eh6] {c};
\node[state] (f6) [right=2.2 of c6] {f};
\node[state] (l6) [right=of f6] {l};
%level 5
\node[state] (bi6) [above right=1.3 and 0.6 of b6] {$0.06$};
\node[state] (ehc6) [right=2.2 of bi6] {$0.09$};
\node[state] (g6) [right=of ehc6] {j};
\node[state] (j6) [right=of g6] {g};
\node[state] (fl6) [right=of j6] {$0.11$};
\node[state] (a6) [right=of fl6] {a};
\node[state] (k6) [right=of a6] {k};
\node[state] (d6) [right=of k6] {d};
%level 6
\node[state] (ehcbi6) [above right=1.3 and 1.1 of bi6] {$0.15$};
\path[every node/.style={font=\scriptsize}]
(e6) edge node [near start] {$1$} (eh6)
(h6) edge node [near end] {$0$} (eh6)
(b6) edge node [near start] {$1$} (bi6)
(i6) edge node [near end] {$0$} (bi6)
(eh6) edge node [near start] {$1$} (ehc6)
(c6) edge node [near end] {$0$} (ehc6)
(f6) edge node [near start] {$1$} (fl6)
(l6) edge node [near end] {$0$} (fl6)
(bi6) edge node [near start] {$1$} (ehcbi6)
(ehc6) edge node [near end] {$0$} (ehcbi6);
\node (pE6) [below=0.7 of e6] {0.02};
\node (pH6) [below=0.7 of h6] {0.02};
\node (pB6) [below=0.7 of b6] {0.03};
\node (pI6) [below=0.7 of i6] {0.03};
\node (pC6) [below=0.7 of c6] {0.05};
\node (pF6) [below=0.7 of f6] {0.05};
\node (pL6) [below=0.7 of l6] {0.06};
\node (pG6) [below=0.7 of g6] {0.1};
\node (pJ6) [below=0.7 of j6] {0.1};
\node (pA6) [below=0.7 of a6] {0.12};
\node (pK6) [below=0.7 of k6] {0.12};
\node (pD6) [below=0.7 of d6] {0.3};
\end{tikzpicture}
\end{figure}
\begin{figure}[hbp]
Sechster Schritt:
\begin{tikzpicture}[shorten >=1pt,node distance=1.1cm,on grid,auto,/tikz/initial text=]
%level 3
\node[state] (e7) {e};
\node[state] (h7) [right=of e7] {h};
%level 4
\node[state] (eh7) [above right=1.3 and 0.6 of e7] {$0.04$};
\node[state] (i7) [left=of eh7] {i};
\node[state] (b7) [left=of i7] {b};
\node[state] (c7) [right=of eh7] {c};
%level 5
\node[state] (bi7) [above right=1.3 and 0.6 of b7] {$0.06$};
\node[state] (l7) [left=1.65 of bi7] {l};
\node[state] (f7) [left=of l7] {f};
\node[state] (ehc7) [right=2.2 of bi7] {$0.09$};
%level 6
\node[state] (ehcbi7) [above right=1.3 and 1.1 of bi7] {$0.15$};
\node[state] (k7) [left=of ehcbi7] {k};
\node[state] (a7) [left=of k7] {a};
\node[state] (fl7) [left=of a7] {$0.11$};
\node[state] (j7) [left=of fl7] {j};
\node[state] (g7) [left=of j7] {g};
\node[state] (d7) [right=of ehcbi7] {d};
%level 7
\node[state] (gj7) [above right=1.3 and 0.6 of g7] {$0.2$};
\path[every node/.style={font=\scriptsize}]
(e7) edge node [near start] {$1$} (eh7)
(h7) edge node [near end] {$0$} (eh7)
(b7) edge node [near start] {$1$} (bi7)
(i7) edge node [near end] {$0$} (bi7)
(eh7) edge node [near start] {$1$} (ehc7)
(c7) edge node [near end] {$0$} (ehc7)
(f7) edge node [near start] {$1$} (fl7)
(l7) edge node [near end] {$0$} (fl7)
(bi7) edge node [near start] {$1$} (ehcbi7)
(ehc7) edge node [near end] {$0$} (ehcbi7)
(g7) edge node [near start] {$1$} (gj7)
(j7) edge node [near end] {$0$} (gj7);
\node (pE7) [below=0.7 of e7] {0.02};
\node (pH7) [below=0.7 of h7] {0.02};
\node (pB7) [below=0.7 of b7] {0.03};
\node (pI7) [below=0.7 of i7] {0.03};
\node (pC7) [below=0.7 of c7] {0.05};
\node (pF7) [below=0.7 of f7] {0.05};
\node (pL7) [below=0.7 of l7] {0.06};
\node (pG7) [below=0.7 of g7] {0.1};
\node (pJ7) [below=0.7 of j7] {0.1};
\node (pA7) [below=0.7 of a7] {0.12};
\node (pK7) [below=0.7 of k7] {0.12};
\node (pD7) [below=0.7 of d7] {0.3};
\end{tikzpicture}
\end{figure}
\begin{figure}[hbp]
Siebter Schritt:
\begin{tikzpicture}[shorten >=1pt,node distance=1.1cm,on grid,auto,/tikz/initial text=]
%level 4
\node[state] (e8) {e};
\node[state] (h8) [right=of e8] {h};
%level 5
\node[state] (eh8) [above right=1.3 and 0.6 of e8] {$0.04$};
\node[state] (i8) [left=of eh8] {i};
\node[state] (b8) [left=of i8] {b};
\node[state] (c8) [right=of eh8] {c};
%level 6
\node[state] (bi8) [above right=1.3 and 0.6 of b8] {$0.06$};
\node[state] (l8) [left=1.65 of bi8] {l};
\node[state] (f8) [left=of l8] {f};
\node[state] (ehc8) [right=2.2 of bi8] {$0.09$};
\node[state] (g8) [right=1.1 of ehc8] {g};
\node[state] (j8) [right=of g8] {j};
%level 7
\node[state] (gj8) [above right=1.3 and 0.6 of g8] {$0.2$};
\node[state] (ehcbi8) [left=2.75 of gj8] {$0.15$};
\node[state] (k8) [left=of ehcbi8] {k};
\node[state] (a8) [left=of k8] {a};
\node[state] (fl8) [left=1.1 of a8] {$0.11$};
\node[state] (d8) [right=of gj8] {d};
%level 8
\node[state] (fla8) [above right=1.3 and 0.6 of fl8] {$0.23$};
\path[every node/.style={font=\scriptsize}]
(e8) edge node [near start] {$1$} (eh8)
(h8) edge node [near end] {$0$} (eh8)
(b8) edge node [near start] {$1$} (bi8)
(i8) edge node [near end] {$0$} (bi8)
(eh8) edge node [near start] {$1$} (ehc8)
(c8) edge node [near end] {$0$} (ehc8)
(f8) edge node [near start] {$1$} (fl8)
(l8) edge node [near end] {$0$} (fl8)
(bi8) edge node [near start] {$1$} (ehcbi8)
(ehc8) edge node [near end] {$0$} (ehcbi8)
(g8) edge node [near start] {$1$} (gj8)
(j8) edge node [near end] {$0$} (gj8)
(fl8) edge node [near start] {$1$} (fla8)
(a8) edge node [near end] {$0$} (fla8);
\node (pE8) [below=0.7 of e8] {0.02};
\node (pH8) [below=0.7 of h8] {0.02};
\node (pB8) [below=0.7 of b8] {0.03};
\node (pI8) [below=0.7 of i8] {0.03};
\node (pC8) [below=0.7 of c8] {0.05};
\node (pF8) [below=0.7 of f8] {0.05};
\node (pL8) [below=0.7 of l8] {0.06};
\node (pG8) [below=0.7 of g8] {0.1};
\node (pJ8) [below=0.7 of j8] {0.1};
\node (pA8) [below=0.7 of a8] {0.12};
\node (pK8) [below=0.7 of k8] {0.12};
\node (pD8) [below=0.7 of d8] {0.3};
\end{tikzpicture}
\end{figure}
\begin{figure}[hbp]
Achter Schritt:
\begin{tikzpicture}[shorten >=1pt,node distance=1.1cm,on grid,auto,/tikz/initial text=]
%level 4
\node[state] (e9) {e};
\node[state] (h9) [right=of e9] {h};
%level 5
\node[state] (eh9) [above right=1.3 and 0.6 of e9] {$0.04$};
\node[state] (i9) [left=of eh9] {i};
\node[state] (b9) [left=of i9] {b};
\node[state] (c9) [right=of eh9] {c};
\node[state] (f9) [right=2.2 of c9] {f};
\node[state] (l9) [right=of f9] {l};
%level 6
\node[state] (bi9) [above right=1.3 and 0.6 of b9] {$0.06$};
\node[state] (ehc9) [right=2.2 of bi9] {$0.09$};
\node[state] (g9) [right=1.1 of ehc9] {g};
\node[state] (j9) [right=of g9] {j};
\node[state] (fl9) [right=of j9] {$0.11$};
\node[state] (a9) [right=of fl9] {a};
%level 7
\node[state] (gj9) [above right=1.3 and 0.6 of g9] {$0.2$};
\node[state] (ehcbi9) [left=2.75 of gj9] {$0.15$};
\node[state] (k9) [left=of ehcbi9] {k};
\node[state] (fla9) [right=2.2 of gj9] {$0.23$};
\node[state] (d9) [right=of fla9] {d};
%level 8
\node[state] (kehcbi9) [above right=1.3 and 0.6 of k9] {$0.27$};
\path[every node/.style={font=\scriptsize}]
(e9) edge node [near start] {$1$} (eh9)
(h9) edge node [near end] {$0$} (eh9)
(b9) edge node [near start] {$1$} (bi9)
(i9) edge node [near end] {$0$} (bi9)
(eh9) edge node [near start] {$1$} (ehc9)
(c9) edge node [near end] {$0$} (ehc9)
(f9) edge node [near start] {$1$} (fl9)
(l9) edge node [near end] {$0$} (fl9)
(bi9) edge node [near start] {$1$} (ehcbi9)
(ehc9) edge node [near end] {$0$} (ehcbi9)
(g9) edge node [near start] {$1$} (gj9)
(j9) edge node [near end] {$0$} (gj9)
(fl9) edge node [near start] {$1$} (fla9)
(a9) edge node [near end] {$0$} (fla9)
(k9) edge node [near start] {$1$} (kehcbi9)
(ehcbi9) edge node [near end] {$0$} (kehcbi9);
\node (pE9) [below=0.7 of e9] {0.02};
\node (pH9) [below=0.7 of h9] {0.02};
\node (pB9) [below=0.7 of b9] {0.03};
\node (pI9) [below=0.7 of i9] {0.03};
\node (pC9) [below=0.7 of c9] {0.05};
\node (pF9) [below=0.7 of f9] {0.05};
\node (pL9) [below=0.7 of l9] {0.06};
\node (pG9) [below=0.7 of g9] {0.1};
\node (pJ9) [below=0.7 of j9] {0.1};
\node (pA9) [below=0.7 of a9] {0.12};
\node (pK9) [below=0.7 of k9] {0.12};
\node (pD9) [below=0.7 of d9] {0.3};
\end{tikzpicture}
\end{figure}
\begin{figure}[hbp]
Neunter Schritt:
\begin{tikzpicture}[shorten >=1pt,node distance=1.1cm,on grid,auto,/tikz/initial text=]
%level 8
\node[state] [above right=5.2 and 3.6] (kehcbi10) {$0.27$};
\node[state] (fla10) [left=2.2 of kehcbi10] {$0.23$};
\node[state] (gj10) [left=2.2 of fla10] {$0.2$};
\node[state] (d10) [right=of kehcbi10] {d};
%level 7
\node[state] (ehcbi10) [below right=1.3 and 0.6 of kehcbi10] {$0.15$};
\node[state] (k10) [left=of ehcbi10] {k};
\node[state] (a10) [left=of k10] {a};
\node[state] (fl10) [left=of a10] {$0.11$};
\node[state] (j10) [left=of fl10] {j};
\node[state] (g10) [left=of j10] {g};
\node[state] (bi10) [below left=1.3 and 1.2 of ehcbi10] {$0.06$};
\node[state] (l10) [left=1.65 of bi10] {l};
\node[state] (f10) [left=of l10] {f};
\node[state] (ehc10) [right=2.2of bi10] {$0.09$};
\node[state] (eh10) [below left=1.3 and 0.6 of ehc10] {$0.04$};
\node[state] (i10) [left=of eh10] {i};
\node[state] (b10) [left=of i10] {b};
\node[state] (c10) [right=of eh10] {c};
\node[state] (e10) [below left=1.3 and 0.6 of eh10] {e};
\node[state] (h10) [right=of e10] {h};
%level 9
\node[state] (gjfla10) [above right=1.3 and 1.1 of gj10] {$0.43$};
\path[every node/.style={font=\scriptsize}]
(e10) edge node [near start] {$1$} (eh10)
(h10) edge node [near end] {$0$} (eh10)
(b10) edge node [near start] {$1$} (bi10)
(i10) edge node [near end] {$0$} (bi10)
(eh10) edge node [near start] {$1$} (ehc10)
(c10) edge node [near end] {$0$} (ehc10)
(f10) edge node [near start] {$1$} (fl10)
(l10) edge node [near end] {$0$} (fl10)
(bi10) edge node [near start] {$1$} (ehcbi10)
(ehc10) edge node [near end] {$0$} (ehcbi10)
(g10) edge node [near start] {$1$} (gj10)
(j10) edge node [near end] {$0$} (gj10)
(fl10) edge node [near start] {$1$} (fla10)
(a10) edge node [near end] {$0$} (fla10)
(k10) edge node [near start] {$1$} (kehcbi10)
(ehcbi10) edge node [near end] {$0$} (kehcbi10)
(gj10) edge node [near start] {$1$} (gjfla10)
(fla10) edge node [near end] {$0$} (gjfla10);
\node (pE10) [below=0.7 of e10] {0.02};
\node (pH10) [below=0.7 of h10] {0.02};
\node (pB10) [below=0.7 of b10] {0.03};
\node (pI10) [below=0.7 of i10] {0.03};
\node (pC10) [below=0.7 of c10] {0.05};
\node (pF10) [below=0.7 of f10] {0.05};
\node (pL10) [below=0.7 of l10] {0.06};
\node (pG10) [below=0.7 of g10] {0.1};
\node (pJ10) [below=0.7 of j10] {0.1};
\node (pA10) [below=0.7 of a10] {0.12};
\node (pK10) [below=0.7 of k10] {0.12};
\node (pD10) [below=0.7 of d10] {0.3};
\end{tikzpicture}
\end{figure}
\begin{figure}[hbp]
Zehnter Schritt:
\begin{tikzpicture}[shorten >=1pt,node distance=1.1cm,on grid,auto,/tikz/initial text=]
%level 4
\node[state] (e11) {e};
\node[state] (h11) [right=of e11] {h};
%level 5
\node[state] (eh11) [above right=1.3 and 0.6 of e11] {$0.04$};
\node[state] (i11) [left=of eh11] {i};
\node[state] (b11) [left=of i11] {b};
\node[state] (c11) [right=of eh11] {c};
\node[state] (f11) [right=2.2 of c11] {f};
\node[state] (l11) [right=of f11] {l};
%level 6
\node[state] (bi11) [above right=1.3 and 0.6 of b11] {$0.06$};
\node[state] (ehc11) [right=2.2 of bi11] {$0.09$};
\node[state] (g11) [right=of ehc11] {g};
\node[state] (j11) [right=of g11] {j};
\node[state] (fl11) [right=of j11] {$0.11$};
\node[state] (a11) [right=of fl11] {a};
%level 7
\node[state] (ehcbi11) [above right=1.3 and 1.2 of bi11] {$0.15$};
\node[state] (k11) [left=of ehcbi11] {k};
\node[state] (gj11) [right=2.75 of ehcbi11] {$0.2$};
\node[state] (fla11) [right=2.2 of gj11] {$0.23$};
%level 8
\node[state] (kehcbi11) [above right=1.3 and 0.6 of k11] {$0.27$};
\node[state] (d11) [right=of kehcbi11] {d};
\node[state] (gjfla11) [right=3.3 of d11] {$0.43$};
%level 9
\node[state] (kehcbid11) [above right=1.3 and 0.6 of kehcbi11] {$0.57$};
\path[every node/.style={font=\scriptsize}]
(e11) edge node [near start] {$1$} (eh11)
(h11) edge node [near end] {$0$} (eh11)
(b11) edge node [near start] {$1$} (bi11)
(i11) edge node [near end] {$0$} (bi11)
(eh11) edge node [near start] {$1$} (ehc11)
(c11) edge node [near end] {$0$} (ehc11)
(f11) edge node [near start] {$1$} (fl11)
(l11) edge node [near end] {$0$} (fl11)
(bi11) edge node [near start] {$1$} (ehcbi11)
(ehc11) edge node [near end] {$0$} (ehcbi11)
(g11) edge node [near start] {$1$} (gj11)
(j11) edge node [near end] {$0$} (gj11)
(fl11) edge node [near start] {$1$} (fla11)
(a11) edge node [near end] {$0$} (fla11)
(k11) edge node [near start] {$1$} (kehcbi11)
(ehcbi11) edge node [near end] {$0$} (kehcbi11)
(gj11) edge node [near start] {$1$} (gjfla11)
(fla11) edge node [near end] {$0$} (gjfla11)
(kehcbi11) edge node [near start] {$1$} (kehcbid11)
(d11) edge node [near end] {$0$} (kehcbid11);
\node (pE11) [below=0.7 of e11] {0.02};
\node (pH11) [below=0.7 of h11] {0.02};
\node (pB11) [below=0.7 of b11] {0.03};
\node (pI11) [below=0.7 of i11] {0.03};
\node (pC11) [below=0.7 of c11] {0.05};
\node (pF11) [below=0.7 of f11] {0.05};
\node (pL11) [below=0.7 of l11] {0.06};
\node (pG11) [below=0.7 of g11] {0.1};
\node (pJ11) [below=0.7 of j11] {0.1};
\node (pA11) [below=0.7 of a11] {0.12};
\node (pK11) [below=0.7 of k11] {0.12};
\node (pD11) [below=0.7 of d11] {0.3};
\end{tikzpicture}
\end{figure}
\begin{figure}[h!]
Elfter Schritt:
\begin{tikzpicture}[shorten >=1pt,node distance=1.1cm,on grid,auto,/tikz/initial text=]
%level 4
\node[state] (e12) {e};
\node[state] (h12) [right=of e12] {h};
%level 5
\node[state] (eh12) [above right=1.3 and 0.6 of e12] {$0.04$};
\node[state] (i12) [left=of eh12] {i};
\node[state] (b12) [left=of i12] {b};
\node[state] (c12) [right=of eh12] {c};
%level 6
\node[state] (bi12) [above right=1.3 and 0.6 of b12] {$0.06$};
\node[state] (l12) [left=1.65 of bi12] {l};
\node[state] (f12) [left=of l12] {f};
\node[state] (ehc12) [right=2.2 of bi12] {$0.09$};
%level 7
\node[state] (ehcbi12) [above right=1.3 and 1.2 of bi12] {$0.15$};
\node[state] (k12) [left=of ehcbi12] {k};
\node[state] (a12) [left=of k12] {a};
\node[state] (fl12) [left=of a12] {$0.11$};
\node[state] (j12) [left=of fl12] {j};
\node[state] (g12) [left=of j12] {g};
%level 8
\node[state] (kehcbi12) [above right=1.3 and 0.6 of k12] {$0.27$};
\node[state] (fla12) [left=2.2 of kehcbi12] {$0.23$};
\node[state] (gj12) [left=2.2 of fla12] {$0.2$};
\node[state] (d12) [right=of kehcbi12] {d};
%level 9
\node[state] (kehcbid12) [above right=1.3 and 0.6 of kehcbi12] {$0.57$};
\node[state] (gjfla12) [left=3.95 of kehcbid12] {$0.43$};
%level 10
\node[state] (gjflakehcbid12) [above right=1.3 and 2.0 of gjfla12] {$1$};
\path[every node/.style={font=\scriptsize}]
(e12) edge node [near start] {$1$} (eh12)
(h12) edge node [near end] {$0$} (eh12)
(b12) edge node [near start] {$1$} (bi12)
(i12) edge node [near end] {$0$} (bi12)
(eh12) edge node [near start] {$1$} (ehc12)
(c12) edge node [near end] {$0$} (ehc12)
(f12) edge node [near start] {$1$} (fl12)
(l12) edge node [near end] {$0$} (fl12)
(bi12) edge node [near start] {$1$} (ehcbi12)
(ehc12) edge node [near end] {$0$} (ehcbi12)
(g12) edge node [near start] {$1$} (gj12)
(j12) edge node [near end] {$0$} (gj12)
(fl12) edge node [near start] {$1$} (fla12)
(a12) edge node [near end] {$0$} (fla12)
(k12) edge node [near start] {$1$} (kehcbi12)
(ehcbi12) edge node [near end] {$0$} (kehcbi12)
(gj12) edge node [near start] {$1$} (gjfla12)
(fla12) edge node [near end] {$0$} (gjfla12)
(kehcbi12) edge node [near start] {$1$} (kehcbid12)
(d12) edge node [near end] {$0$} (kehcbid12)
(gjfla12) edge node [near start] {$1$} (gjflakehcbid12)
(kehcbid12) edge node [near end] {$0$} (gjflakehcbid12);
\node (pE12) [below=0.7 of e12] {0.02};
\node (pH12) [below=0.7 of h12] {0.02};
\node (pB12) [below=0.7 of b12] {0.03};
\node (pI12) [below=0.7 of i12] {0.03};
\node (pC12) [below=0.7 of c12] {0.05};
\node (pF12) [below=0.7 of f12] {0.05};
\node (pL12) [below=0.7 of l12] {0.06};
\node (pG12) [below=0.7 of g12] {0.1};
\node (pJ12) [below=0.7 of j12] {0.1};
\node (pA12) [below=0.7 of a12] {0.12};
\node (pK12) [below=0.7 of k12] {0.12};
\node (pD12) [below=0.7 of d12] {0.3};
\end{tikzpicture}
\end{figure}
\subsection{} %b
Mittlerer Informationsgehalt(Entropie):
\begin{alignat*}{2}
H &=& \sum\limits_{i} p_{i} \cdot \log_{2}{(\frac{1}{p_{i}})} \\
H &=& 2 \cdot (0.02 \cdot \log_{2}{(\frac{1}{0.02})}) + 2 \cdot (0.03 \cdot \log_{2}{(\frac{1}{0.03})}) \\
& & + 2 \cdot (0.05 \cdot \log_{2}{(\frac{1}{0.05})}) + (0.06 \cdot \log_{2}{(\frac{1}{0.06})})\\
& & + 2 \cdot ( 0.1 \cdot \log_{2}{(\frac{1}{0.1})}) + 2 \cdot (0.12 \cdot \log_{2}{(\frac{1}{0.12})})\\
& & + (0.3 \cdot \log_{2}{(\frac{1}{0.3})}) \\
H &\approx & 3.125 Bit
\end{alignat*}
\section{}%5.4
\subsection{} %a
Codieren von Dezimalziffern auf 4-Bit Binärwörter:
\begin{tikzpicture}
\matrix[matrix of math nodes,nodes={rectangle,thin,draw=black!20,minimum size=10mm}]
{0 & 1 & 2 & 3 & 4 & 5 & 6 & 7 & 8 & 9 \\
0000 & 0001 & 0010 & 0011 & 0100 & 0101 & 0110 & 0111 & 1000 & 1001 \\};
\end{tikzpicture}
4 Binärstellen für 10 Codewörter, da maximal mögliche Anzahl von Codewörtern gleich $2^{4} = 16$.
\begin{alignat*}{2}
H_{0} &=& \log_{2}{(2^{4})} \\
H_{0} &=& 4 \\
\intertext{Also 4 Bit möglicher Informationsgehalt}
H &=& 10 \cdot \frac{1}{10} \cdot \log_{2}{(\frac{1}{\frac{1}{10}})} \\
H &\approx & 3.32 \\
\intertext{Also rund 3.32 Bit mittlerer Informationsgehalt}
R &=& H_{0} - H \\
R &=& 0.68
\intertext{Also rund 0.68 Bit Redundanz}
\end{alignat*}
\subsection{} %b
Es werden $7$ bits verbraucht, da $2^{7}=128$ und $2^{6}=64$.
Code für die Dezimalziffern 0-9:
\begin{tikzpicture}
\matrix[matrix of math nodes,nodes={rectangle,thin,draw=black!20,minimum size=10mm}]
{0 & 1 & 2 & 3 & 4 & 5 & 6 & 7 & 8 & 9 \\
00 & 01 & 02 & 03 & 04 & 05 & 06 & 07 & 08 & 09 \\};
\end{tikzpicture}
Aufgrund der Formel $H_{0}=\log_{2}{2^{7}}=7$ gilt, dass der mögliche Informationsgehalt $7$ beträgt.
Aufgrund der Formel $H = 100 \cdot \frac{1}{100} \cdot \log_{2}{(\frac{1}{\frac{1}{100}})} \approx 6.43$ beträgt der mittlere Informationsgehalt rund $6.43$ Bit.
Daraus ergibt sich sich die Redundanz $R = H_{0} - H \approx 0.36$ Bit.
Die Redundanz bezogen auf eine einzelne Dezimalziffer beträgt $\frac{R}{2} \approx 0.18$ Bit.
\subsection{} %c
Bei Gruppen von drei Ziffern werden $10$ Bits belegt. Dies ergibt sich aus $2^{10} = 1024$.
Demzufolge ist der mögliche Informationsgehalt $H_{0}$ für alle Codewörter gleich $10$ Bit. Bezogen auf eine einzelne Ziffer sind es $\frac{10}{3}$ Bit.
Bei Gruppen von vier Ziffern werden $14$ Bits belegt. Dies ergibt sich aus $2^{14}=16384$.
Demzufolge ist der mögliche Informationsgehalt gleich $14$ Bit. Bezogen auf eine einzelne Ziffer sind es $\frac{14}{4} = \frac{7}{2} = 3.5$ Bit.
Die Unterschiede ergeben sich durch unterschiedlich viele Kombinationsmöglichkeiten. Bei 3-er Gruppen gibt es $3^{10}$ und bei 4-er Gruppen $4^{10}$ mögliche Kombinationen. Demzufolge ist der mögliche Informationsgehalt pro Ziffer bei letzterem minimal höher.
\subsection{} %d
Code mit variabler Länge (Fano-Code):
\begin{tikzpicture}
\matrix[matrix of nodes]
{0 & 1 & 2 & 3 & 4 & 5 & 6 & 7 & 8 & 9 \\
$000$ & $001$ & $010$ & $0110$ & $0111$ & $100$ & $101$ & $110$ & $1110$ & $1111$ \\};
\end{tikzpicture}
\begin{alignat*}{2}
P(a_{i}) &=& \frac{1}{10} \\
H_{0} &=& (\frac{1}{10} \cdot 4 \cdot \log_{2}{2^{4}}) + (\frac{1}{10} \cdot 6 \cdot \log_{2}{2^{3}}) \\
H_{0} &\approx & 1.6 + 1.8 = 3.4\, \text{Bit} \\
R &=& H_{0} - H \\
R &=& 0.08\,\text{Bit}
\end{alignat*}
Der Code wird noch effizienter, wenn x-beliebige Codewörter zu einem Codewort zusammengefasst werden.
\end{document}

View File

@ -0,0 +1,112 @@
\documentclass[10pt,a4paper,oneside,ngerman,numbers=noenddot]{scrartcl}
\usepackage[T1]{fontenc}
\usepackage[utf8]{inputenc}
\usepackage[ngerman]{babel}
\usepackage{amsmath}
\usepackage{amsfonts}
\usepackage{amssymb}
\usepackage{paralist}
\usepackage[locale=DE,exponent-product=\cdot ,detect-all]{siunitx}
\usepackage{tikz}
\usepackage[scaled=0.78]{luximono}
\usepackage{listings}
\usetikzlibrary{automata,matrix,fadings,calc,positioning,decorations.pathreplacing,decorations.text,arrows}
\pagenumbering{arabic}
\def\thesection{6.\arabic{section})}
\def\thesubsection{(\alph{subsection})}
\def\thesubsubsection{(\arabic{subsubsection})}
\renewcommand{\labelenumi}{(\roman{enumi})}
\hyphenation{Nach-komma-stel-len}
\lstnewenvironment{java}[1][]{%
\lstset{basicstyle=\ttfamily ,backgroundcolor=\color[gray]{.95},columns=flexible,fontadjust=true,language=Java,tabsize=4,numbers=none,#1}%
}{%
}
\tikzstyle{huffmanNodes}=[matrix of nodes,
nodes={circle,thin,draw=black!20,minimum size=10mm,text height=1.5ex,text depth=.25ex,inner sep=-10pt}]
\tikzstyle{huffmanBase}=[matrix of nodes,
nodes={minimum size=10mm,text height=1.5ex,text depth=.25ex,inner sep=-10pt}]
\begin{document}
\author{Jim Martens (Matrikelnummer 6420323) \and Marlo Kornblum (Matrikelnummer 6427301)}
\title{Rechnerstrukturen Aufgabenblatt 6}
\maketitle
\section{}%6.1
Ein zyklisch-einschrittiger Code hat die Voraussetzung, dass pro Schritt nur ein Bit verändert wird. Zusätzlich dürfen das erste und letzte Codewort ebenfalls nur ein Bit auseinanderliegen.
Bei einem Binärcode ergibt sich die Bedingung, dass die Parität pro Schritt wechselt. Startet man mit 0 und damit gerader Parität, muss als nächstes eine ungerade Parität folgen. Darauf wieder eine gerade und so weiter.
Die Eigenschaft des zyklischen Codes besagt, dass man an jeder beliebigen Stelle anfangen kann und immer die Bedingung erfüllt ist. Hätte man einen Code mit einer ungeraden Anzahl von Codewörtern, dann würde entweder eine gerade Parität auf eine gerade Parität folgen oder eine ungerade Parität auf eine ungerade Parität, womit der Abstand an mindestens einer Stelle von eins verschieden sein.
\section{}%6.2
\subsection{} %a
Die Minimaldistanz $d$ dieses Codes ist $4$, da $4$ Bits geändert werden müssen, um ein neues gültiges Codewort zu erzeugen. Man stelle sich einen Code vor, bei dem überall eine $0$ steht. Setzt man nun ein Datenbit auf $1$, so muss das zugehörige Zeilen- und Spaltenparitätsbit auf $1$ gesetzt werden und das Paritätsbit über die Spaltenparitätsbits muss auch auf $1$ gesetzt werden. Es werden also $4$ Bits verändert.
\subsection{} %b
Es können nur 1-bit Fehler erkannt und korrigiert werden. Dies liegt daran, dass nur solche Fehler eindeutig lokalisierbar sind. Zweibit- und 3-bit Fehler können noch erkannt werden, aber nicht mehr korrigiert. Dies liegt daran, dass schon bei 2-bit Fehlern mehrere Fehlerkonstellationen zum selben Ergebnis führen.
4-bit Fehler können nur noch teilweise erkannt werden. Rechteckige 4-bit Fehler können gar nicht mehr erkannt werden.
\subsection{} %c
Ein Beispiel eines nicht erkannten 4-bit Fehlers ist ein Fehler in den Bits $d_{0,0},d_{0,1},d_{1,0},d_{1,1}$.
\subsection{} %d
Es gibt $81 \cdot 16 \cdot 16 \cdot 1 = 20736$ Möglichkeiten 4-bit Fehler zu erzeugen, die nicht erkannt werden.
Das liegt daran, dass das erste Fehlerbit komplett frei in $81$ Feldern gesetzt werden kann. Das zweite Bit hat nun von jeweils $9$ Zeilen und $9$ Spalten noch $8$ Zeilen bzw. $8$ Spalten zur Verfügung. Demzufolge bleiben also $16$ Möglichkeiten. Das dritte Bit hat nun noch je nach Wahl des zweiten Bits $8$ Zeilen und $2$ Spalten zur Verfügung oder $8$ Spalten und $2$ Zeilen. Dies sind also $16$ Möglichkeiten. Das letzte Bit hat nur noch eine einzige mögliche Position.
Insgesamt gibt es $\binom{81}{4} = 1663740$ Möglichkeiten einen 4-bit Fehler zu erzeugen. In Relation zur Gesamtzahl der möglichen 4-bit Fehler machen die nicht erkennbaren Fehler $\frac{20736}{1663740} \cdot 100 \approx 1.2$ Prozent aus.
\section{}%6.3
\subsection{} %a
Anhand eines Beispiels werden wir zeigen, wie ein auftretender 1-bit Fehler lokalisiert und damit korrigiert werden kann. Dazu werden die Codewortstelle $c_{7}$ des 9. Codewortes verfälscht und anschließend die Prüfbits gebildet.\\\\
Legende:\\
R bedeutet richtig. FoP bedeutet Fehler ohne aktualisierte Prüfbits und FmP bedeutet Fehler mit aktualisierten Prüfbits.
\begin{tabular}{cccccccc}
& $c_{1}$ & $c_{2}$ & $c_{3}$ & $c_{4}$ & $c_{5}$ & $c_{6}$ & $c_{7}$ \\
Nr. & $p_{1}$ & $p_{2}$ & $d_{1}$ & $p_{3}$ & $d_{2}$ & $d_{3}$ & $d_{4}$ \\
\hline
8R & 1 & 1 & 1 & 0 & 0 & 0 & 0 \\
8FoP & 1 & 1 & 1 & 0 & 0 & 0 & 1 \\
8FmP & 0 & 0 & 1 & 1 & 0 & 0 & 1
\end{tabular}
Das Prüfwort gebildet von 8FoP: $x_{a} = 1, x_{b} = 1, x_{c} = 1$.
Anhand des Prüfwortes lässt sich die Stelle des Fehlers bestimmen. Nimmt man $x_{a}$ als $2^{0}$-Stelle, $x_{b}$ als $2^{1}$ und $x_{c}$ als $2^{2}$ Stelle an, so ergibt sich immer die binäre Ganzzahl der betroffenen Stelle.
Wichtig hierbei ist, dass das Prüfwort von dem fehlerhaften Code gebildet wird, bei dem die Prüfbits nicht angepasst wurden.
\subsection{} %b
Mithilfe des angegebenen Schemas für Hamming-Codes kann sowohl die Generatormatrix als auch die Prüfmatrix abgelesen werden.
Dazu muss man wissen, dass die Prüfbits so gebildet werden:
Das erste Prüfbit $p_{1}$ enthält auf diesem Schema alle Datenbits, die mit einem roten Punkt markiert sind.
Das zweite Prüfbit enthält auf diesem Schema alle Datenbits, die mit einem grünen Punkt markiert sind.
Das dritte Prüfbit enthält auf diesem Schema alle Datenbits, die mit einem dunkelblauen Punkt markiert sind.
Die Generatormatrix bildet sich zeilenweise über die angegebenen Prüf- und Datenbits. Die erste Zeile entspricht dem ersten Prüfbit, die zweite Zeile dem zweiten Prüfbit, die dritte Zeile dem ersten Datenbit, und so weiter.
Die Einsen in jeder Zeile markieren welche Datenbits enthalten sind. In den Datenbitzeilen gibt es logischerweise nur eine Eins und zwar bei der Zeile des jeweiligen Datenbits, welches sich selbst enthält. In den Prüfbitzeilen gibt es dort Einsen, wo Datenbits in die Berechnung einfließen.
Als Generatormatrix ergibt sich bei einem ($7,4$)-Hamming-Code demnach also folgende Matrix:
\begin{equation*}
G := \begin{pmatrix} 1 & 1 & 0 & 1 \\ 1 & 0 & 1 & 1 \\ 1 & 0 & 0 & 0 \\ 0 & 1 & 1 & 1 \\ 0 & 1 & 0 & 0 \\ 0 & 0 & 1 & 0 \\ 0 & 0 & 0 & 1 \end{pmatrix}
\end{equation*}
Die Kontrollmatrix lässt sich noch einfacher ablesen. In der ersten Zeile steht überall dort eine Eins, wo auf dem Schema ein roter Punkt ist. Der Rest der Zeile wir mit 0 aufgefüllt.
In der zweiten Zeile steht überall dort eine Eins, wo auf dem Schema ein grüner Punkt ist. Der Rest der Zeile wir mit 0 aufgefüllt.
In der dritten Zeile steht überall dort eine Eins, wo auf dem Schema ein dunkelblauer Punkt ist. Der Rest der Zeile wir mit 0 aufgefüllt.
Daraus ergibt sich für ein ($7,4$)-Hamming-Code Folgendes:
\begin{equation*}
H := \begin{pmatrix} 1 & 0 & 1 & 0 & 1 & 0 & 1 \\ 0 & 1 & 1 & 0 & 0 & 1 & 1 \\ 0 & 0 & 0 & 1 & 1 & 1 & 1 \end{pmatrix}
\end{equation*}
Die Kontrollmatrix hat dabei so viele Spalten, wie die Generatormatrix Zeilen hat.
\end{document}

View File

@ -0,0 +1,402 @@
\documentclass[10pt,a4paper,oneside,ngerman,numbers=noenddot]{scrartcl}
\usepackage[T1]{fontenc}
\usepackage[utf8]{inputenc}
\usepackage[ngerman]{babel}
\usepackage{amsmath}
\usepackage{amsfonts}
\usepackage{amssymb}
\usepackage{paralist}
\usepackage{qtree}
\usepackage[locale=DE,exponent-product=\cdot ,detect-all]{siunitx}
\usepackage{tikz}
\usepackage[scaled=0.78]{luximono}
\usepackage{listings}
\usetikzlibrary{automata,matrix,fadings,calc,positioning,decorations.pathreplacing,decorations.text,arrows}
\pagenumbering{arabic}
\def\thesection{7.\arabic{section})}
\def\thesubsection{(\alph{subsection})}
\def\thesubsubsection{(\arabic{subsubsection})}
\renewcommand{\labelenumi}{(\roman{enumi})}
\hyphenation{Nach-komma-stel-len}
\lstnewenvironment{java}[1][]{%
\lstset{basicstyle=\ttfamily ,backgroundcolor=\color[gray]{.95},columns=flexible,fontadjust=true,language=Java,tabsize=4,numbers=none,#1}%
}{%
}
\graphicspath{{D:/Users/Jim-Studium/Pictures/Studium/RS/}}
\tikzstyle{huffmanNodes}=[matrix of nodes,
nodes={circle,thin,draw=black!20,minimum size=10mm,text height=1.5ex,text depth=.25ex,inner sep=-10pt}]
\tikzstyle{huffmanBase}=[matrix of nodes,
nodes={minimum size=10mm,text height=1.5ex,text depth=.25ex,inner sep=-10pt}]
\begin{document}
\author{Jim Martens (Matrikelnummer 6420323) \and Marlo Kornblum (Matrikelnummer 6427301)}
\title{Rechnerstrukturen Aufgabenblatt 6}
\maketitle
\section{} %7.1
\subsection{} %a
DNF:\\
\begin{alignat*}{2}
f(x) &=& ((\overline{x_{3}} \vee \overline{x_{2}}) \wedge (\overline{x_{2}} \vee \overline{x_{1}}))\vee ((x_{3} \vee \overline{x_{2}}) \wedge (\overline{x_{2}} \vee \overline{x_{1}})) \\
&& \vee ((x_{3} \vee x_{2}) \wedge (x_{2} \vee \overline{x_{1}})) \vee ((x_{3} \vee x_{2}) \wedge (x_{2} \vee x_{1}))
\end{alignat*}\\
KNF:\\
\begin{alignat*}{2}
f(x) &=& ((x_{3} \vee x_{2}) \wedge (x_{2} \vee x_{1})) \wedge ((x_{3} \vee x_{2}) \wedge (x_{2} \vee \overline{x_{1}})) \wedge ((x_{3} \vee \overline{x_{2}}) \\
&& \wedge (x_{2} \vee x_{1})) \wedge ((x_{3} \vee \overline{x_{2}}) \wedge (x_{2} \vee \overline{x_{1}})) \wedge ((\overline{x_{3}} \vee x_{2}) \wedge (x_{2} \vee \overline{x_{1}}))
\end{alignat*}\\
Reed-Muller Form:\\
\begin{alignat*}{2}
f(x) &=& (x_{3} \vee \overline{x_{2}}) \wedge (x_{2} \vee \overline{x_{1}}) \\
&=& (x_{3} \oplus \overline{x_{2}} \oplus x_{3}\overline{x_{2}}) \wedge (x_{2} \oplus \overline{x_{1}} \oplus x_{2}\overline{x_{1}}) \\
&=& (x_{3} \oplus (x_{2} \oplus 1) \oplus x_{3}(x_{2} \oplus 1)) \wedge (x_{2} \oplus (x_{1} \oplus 1) \oplus x_{2}(x_{1} \oplus 1)) \\
&=& (x_{3} \oplus x_{2} \oplus 1 \oplus x_{3}x_{2} \oplus x_{3}) \wedge (x_{2} \oplus x_{1} \oplus 1 \oplus x_{2}x_{1} \oplus x_{2}) \\
&=& (x_{2} \oplus 1 \oplus x_{3}x_{2}) \wedge (x_{1} \oplus 1 \oplus x_{2}x_{1})
\end{alignat*}
\subsection{} %b
DNF:\\
\begin{alignat*}{2}
g(x) &=& (\overline{x_{3}} \oplus x_{1}) \vee (x_{3} \oplus \overline{x_{1}})
\end{alignat*}\\
KNF:\\
\begin{alignat*}{2}
g(x) &=& (x_{3} \oplus x_{1}) \wedge (\overline{x_{3}} \oplus \overline{x_{1}})
\end{alignat*}\\
Reed-Muller Form:\\
\begin{alignat*}{2}
g(x) &=& \overline{x_{3}} \oplus \overline{x_{1}} \\
&=& (x_{3} \oplus 1) \oplus (x_{1} \oplus 1) \\
&=& x_{3} \oplus 1 \oplus x_{1} \oplus 1 \\
&=& x_{3} \oplus x_{1}
\end{alignat*}
\section{} %7.2
\subsection{} %a
AND: $a \wedge b = \overline{(\overline{a \wedge b}) \wedge (\overline{a \wedge b})}$\\
ODER: $a \vee b = \overline{\overline{a} \wedge \overline{b}} = \overline{(\overline{a \wedge a}) \wedge (\overline{b \wedge b})}$\\
Negation: $\overline{a} = \overline{a \wedge a}$\\
\subsection{} %b
\begin{alignat*}{2}
f(x_{3}, x_{2}, x_{1}) &=&& (\overline{x_{3}}(\overline{x_{2}} \vee x_{1})) \vee (x_{1}(\overline{x_{2}} \vee x_{1})) \\
&=&& ((\overline{x_{3}} \wedge \overline{x_{2}}) \vee \overline{x_{3}}x_{1}) \vee (x_{1}\overline{x_{2}} \vee x_{1}x_{1})
%&=&& ( \overline{( \overline{( \overline{x_{3}} \wedge \overline{x_{2}} ) \wedge ( \overline{x_{3}} \wedge \overline{x_{2}} ) } ) \wedge ( \overline{( \overline{x_{3}}x_{1} ) \wedge ( \overline{x_{3}}x_{1} )} )} ) \vee \\
%&&&( \overline{ (\overline{ (x_{1} \wedge \overline{x_{2}}) \wedge (x_{1} \wedge \overline{x_{2}})}) \wedge (\overline{ (x_{1} \wedge x_{1}) \wedge (x_{1} \wedge x_{1})}) } ) \\
%
%&=&& (\overline{(\overline{( (\overline{x_{3} \wedge x_{3}}) \wedge (\overline{x_{2} \wedge x_{2}}))} \wedge ((\overline{x_{3} \wedge x_{3}}) \wedge (\overline{x_{2} \wedge x_{2}) )}) \wedge (\overline{( (\overline{x_{3} \wedge x_{3}}) \wedge x_{1}) \wedge ( (\overline{x_{3} \wedge x_{3}}) \wedge x_{1})})}) \vee \\
%&&&(\overline{(\overline{(x_{1} \wedge (\overline{x_{2} \wedge x_{2}})) \wedge (x_{1} \wedge (\overline{x_{2} \wedge x_{2}}) )}) \wedge (\overline{ (x_{1} \wedge x_{1}) \wedge (x_{1} \wedge x_{1})})}) \\
%
%&=&& \overline{(\overline{(\overline{( (\overline{x_{3} \wedge x_{3}}) \wedge (\overline{x_{2} \wedge x_{2}}))} \wedge ((\overline{x_{3} \wedge x_{3}}) \wedge (\overline{x_{2} \wedge x_{2}) )}) \wedge (\overline{( (\overline{x_{3} \wedge x_{3}}) \wedge x_{1}) \wedge ( (\overline{x_{3} \wedge x_{3}}) \wedge x_{1})})}) \wedge (\overline{(\overline{( (\overline{x_{3} \wedge x_{3}}) \wedge (\overline{x_{2} \wedge x_{2}}))} \wedge ((\overline{x_{3} \wedge x_{3}}) \wedge (\overline{x_{2} \wedge x_{2}) )}) \wedge (\overline{( (\overline{x_{3} \wedge x_{3}}) \wedge x_{1}) \wedge ( (\overline{x_{3} \wedge x_{3}}) \wedge x_{1})})})} \wedge \\
%&&&\overline{(\overline{(\overline{(x_{1} \wedge (\overline{x_{2} \wedge x_{2}})) \wedge (x_{1} \wedge (\overline{x_{2} \wedge x_{2}}) )}) \wedge (\overline{ (x_{1} \wedge x_{1}) \wedge (x_{1} \wedge x_{1})})}) \wedge (\overline{(\overline{(x_{1} \wedge (\overline{x_{2} \wedge x_{2}})) \wedge (x_{1} \wedge (\overline{x_{2} \wedge x_{2}}) )}) \wedge (\overline{ (x_{1} \wedge x_{1}) \wedge (x_{1} \wedge x_{1})})})}
%wie soll das bitte gehen? (Trick17?)
\end{alignat*}
\section{} %7.3
\subsection{} %a
\begin{tabular}{r|ccccc}
dez.& $x_{3}$ & $x_{2}$ & $x_{1}$ & $x_{0}$ & A \\
\hline
0 & 0 & 0 & 0 & 0 & 1 \\
1 & 0 & 0 & 0 & 1 & 0 \\
2 & 0 & 0 & 1 & 0 & 1 \\
3 & 0 & 0 & 1 & 1 & 1 \\
4 & 0 & 1 & 0 & 0 & 0 \\
5 & 0 & 1 & 0 & 1 & 1 \\
6 & 0 & 1 & 1 & 0 & 1 \\
7 & 0 & 1 & 1 & 1 & 1 \\
8 & 1 & 0 & 0 & 0 & 1 \\
9 & 1 & 0 & 0 & 1 & 1 \\
10 & 1 & 0 & 1 & 0 & * \\
11 & 1 & 0 & 1 & 1 & * \\
12 & 1 & 1 & 0 & 0 & * \\
13 & 1 & 1 & 0 & 1 & * \\
14 & 1 & 1 & 1 & 0 & * \\
15 & 1 & 1 & 1 & 1 & *
\end{tabular}
\begin{tikzpicture}
\draw (0,0) -- +(2,0); %obere Kante
\draw (0,0) -- +(0,-2); %linke Kante
\draw (1,0) -- +(0,-2); %mittlere vertikale Kante
\draw (0,-1) -- +(2,0); %mittlere horizontale Kante
\draw (0,-2) -- +(2,0); %untere Kante
\draw (2,0) -- +(0,-2); %rechte Kante
\draw (0.5,0) -- +(0,-2); %linke mittlere Kante
\draw (1.5,0) -- +(0,-2); %rechte mittlere Kante
\draw (0,-0.5) -- +(2,0); %obere mittlere Kante
\draw (0,-1.5) -- +(2,0); %untere mittlere Kante
\draw (0,0) -- +(-0.5,0.5); %schräger Strich
\node at ++(0.2,0.25) (start11) {$00$}; %Variablenzeile 0
\node at ++(0.7,0.25) (start12) {$01$};
\node at ++(1.2,0.25) (start13) {$11$};
\node at ++(1.7,0.25) (start14) {$10$};
\node at ++(-0.25,-0.2) (start10) {$00$}; %Variablenspalte 0
\node at ++(-0.25,-0.7) (start20) {$01$};
\node at ++(-0.25,-1.2) (start30) {$11$};
\node at ++(-0.25,-1.7) (start40) {$10$};
\node at ++(-0.8,0.2) (var1) {$x3\,x2$};
\node at ++(0.0,0.6) (var1) {$x1\,x0$};
\node at ++(0.25, -0.25) (row11) {$1$}; %erste Zeile
\node at ++(0.75, -0.25) (row12) {$0$};
\node at ++(1.25, -0.25) (row13) {$1$};
\node at ++(1.75, -0.25) (row14) {$1$};
\node at ++(0.25, -0.75) (row21) {$0$}; %zweite Zeile
\node at ++(0.75, -0.75) (row22) {$1$};
\node at ++(1.25, -0.75) (row23) {$1$};
\node at ++(1.75, -0.75) (row24) {$1$};
\node at ++(0.25, -1.25) (row31) {$*$}; %dritte Zeile
\node at ++(0.75, -1.25) (row32) {$*$};
\node at ++(1.25, -1.25) (row33) {$*$};
\node at ++(1.75, -1.25) (row34) {$*$};
\node at ++(0.25, -1.75) (row41) {$1$}; %vierte Zeile
\node at ++(0.75, -1.75) (row42) {$1$};
\node at ++(1.25, -1.75) (row43) {$*$};
\node at ++(1.75, -1.75) (row44) {$*$};
\end{tikzpicture}
\begin{figure}[h]
\begin{tabular}{r|ccccc}
dez.& $x_{3}$ & $x_{2}$ & $x_{1}$ & $x_{0}$ & B \\
\hline
0 & 0 & 0 & 0 & 0 & 1 \\
1 & 0 & 0 & 0 & 1 & 1 \\
2 & 0 & 0 & 1 & 0 & 1 \\
3 & 0 & 0 & 1 & 1 & 1 \\
4 & 0 & 1 & 0 & 0 & 1 \\
5 & 0 & 1 & 0 & 1 & 0 \\
6 & 0 & 1 & 1 & 0 & 0 \\
7 & 0 & 1 & 1 & 1 & 1 \\
8 & 1 & 0 & 0 & 0 & 1 \\
9 & 1 & 0 & 0 & 1 & 1 \\
10 & 1 & 0 & 1 & 0 & * \\
11 & 1 & 0 & 1 & 1 & * \\
12 & 1 & 1 & 0 & 0 & * \\
13 & 1 & 1 & 0 & 1 & * \\
14 & 1 & 1 & 1 & 0 & * \\
15 & 1 & 1 & 1 & 1 & *
\end{tabular}
\begin{tikzpicture}
\draw (0,0) -- +(2,0); %obere Kante
\draw (0,0) -- +(0,-2); %linke Kante
\draw (1,0) -- +(0,-2); %mittlere vertikale Kante
\draw (0,-1) -- +(2,0); %mittlere horizontale Kante
\draw (0,-2) -- +(2,0); %untere Kante
\draw (2,0) -- +(0,-2); %rechte Kante
\draw (0.5,0) -- +(0,-2); %linke mittlere Kante
\draw (1.5,0) -- +(0,-2); %rechte mittlere Kante
\draw (0,-0.5) -- +(2,0); %obere mittlere Kante
\draw (0,-1.5) -- +(2,0); %untere mittlere Kante
\draw (0,0) -- +(-0.5,0.5); %schräger Strich
\node at ++(0.2,0.25) (start11) {$00$}; %Variablenzeile 0
\node at ++(0.7,0.25) (start12) {$01$};
\node at ++(1.2,0.25) (start13) {$11$};
\node at ++(1.7,0.25) (start14) {$10$};
\node at ++(-0.25,-0.2) (start10) {$00$}; %Variablenspalte 0
\node at ++(-0.25,-0.7) (start20) {$01$};
\node at ++(-0.25,-1.2) (start30) {$11$};
\node at ++(-0.25,-1.7) (start40) {$10$};
\node at ++(-0.8,0.2) (var1) {$x3\,x2$};
\node at ++(0.0,0.6) (var1) {$x1\,x0$};
\node at ++(0.25, -0.25) (row11) {$1$}; %erste Zeile
\node at ++(0.75, -0.25) (row12) {$1$};
\node at ++(1.25, -0.25) (row13) {$1$};
\node at ++(1.75, -0.25) (row14) {$1$};
\node at ++(0.25, -0.75) (row21) {$1$}; %zweite Zeile
\node at ++(0.75, -0.75) (row22) {$0$};
\node at ++(1.25, -0.75) (row23) {$1$};
\node at ++(1.75, -0.75) (row24) {$0$};
\node at ++(0.25, -1.25) (row31) {$*$}; %dritte Zeile
\node at ++(0.75, -1.25) (row32) {$*$};
\node at ++(1.25, -1.25) (row33) {$*$};
\node at ++(1.75, -1.25) (row34) {$*$};
\node at ++(0.25, -1.75) (row41) {$1$}; %vierte Zeile
\node at ++(0.75, -1.75) (row42) {$1$};
\node at ++(1.25, -1.75) (row43) {$*$};
\node at ++(1.75, -1.75) (row44) {$*$};
\end{tikzpicture}
\end{figure}
\subsection{} %b
A:\\
Es lassen sich vier Schleifen bilden. Man kann die dritte und vierte Spalte und dritte und vierte Zeile jeweils komplett in eine Schleife packen. Zudem kann die 1 ganz oben links mit der 1 ganz unten links verbunden werden. Außerdem lassen sich die 1 in der zweiten Zeile und Spalte mit dem Don't-Care-Term in der zweiten Spalte und dritten Zeile verbinden.
Daraus ergibt sich dieser Term für A:
\begin{equation*}
A(x_{3},x_{2},x_{1},x_{0}) = (x_{3}) \vee (x_{1}) \vee (x_{2}\overline{x_{1}}x_{0}) \vee (\overline{x_{2}} \wedge \overline{x_{1}} \wedge \overline{x_{0}})
\end{equation*}
\\
B:\\
Es lassen sich auch hier wieder vier Schleifen bilden. Man kann die dritte und vierte Zeile komplett in eine Schleife üacken. Zudem kann man die erste Zeile, erste Spalte und dritte Spalte jeweils in eine Schleife packen.
Daraus ergibt sich dieser Term für B:
\begin{equation*}
B(x_{3},x_{2},x_{1},x_{0}) = (x_{3}) \vee (x_{1}x_{0}) \vee (\overline{x_{3}} \wedge \overline{x_{2}}) \vee (\overline{x_{1}} \wedge \overline{x_{0}})
\end{equation*}
\section{} %7.4
\subsection{} %a
\begin{tabular}{r|ccccc}
dez.& $x_{3}$ & $x_{2}$ & $x_{1}$ & $x_{0}$ & y \\
\hline
0 & 0 & 0 & 0 & 0 & 0 \\
1 & 0 & 0 & 0 & 1 & 0 \\
2 & 0 & 0 & 1 & 0 & 0 \\
3 & 0 & 0 & 1 & 1 & 0 \\
4 & 0 & 1 & 0 & 0 & 0 \\
5 & 0 & 1 & 0 & 1 & 1 \\
6 & 0 & 1 & 1 & 0 & 0 \\
7 & 0 & 1 & 1 & 1 & 1 \\
8 & 1 & 0 & 0 & 0 & 0 \\
9 & 1 & 0 & 0 & 1 & 0 \\
10 & 1 & 0 & 1 & 0 & 0 \\
11 & 1 & 0 & 1 & 1 & 0 \\
12 & 1 & 1 & 0 & 0 & 1 \\
13 & 1 & 1 & 0 & 1 & 1 \\
14 & 1 & 1 & 1 & 0 & 1 \\
15 & 1 & 1 & 1 & 1 & 1
\end{tabular}
\subsection{} %b
\begin{tikzpicture}
\draw (0,0) -- +(2,0); %obere Kante
\draw (0,0) -- +(0,-2); %linke Kante
\draw (1,0) -- +(0,-2); %mittlere vertikale Kante
\draw (0,-1) -- +(2,0); %mittlere horizontale Kante
\draw (0,-2) -- +(2,0); %untere Kante
\draw (2,0) -- +(0,-2); %rechte Kante
\draw (0.5,0) -- +(0,-2); %linke mittlere Kante
\draw (1.5,0) -- +(0,-2); %rechte mittlere Kante
\draw (0,-0.5) -- +(2,0); %obere mittlere Kante
\draw (0,-1.5) -- +(2,0); %untere mittlere Kante
\draw (0,0) -- +(-0.5,0.5); %schräger Strich
\node at ++(0.2,0.25) (start11) {$00$}; %Variablenzeile 0
\node at ++(0.7,0.25) (start12) {$01$};
\node at ++(1.2,0.25) (start13) {$11$};
\node at ++(1.7,0.25) (start14) {$10$};
\node at ++(-0.25,-0.2) (start10) {$00$}; %Variablenspalte 0
\node at ++(-0.25,-0.7) (start20) {$01$};
\node at ++(-0.25,-1.2) (start30) {$11$};
\node at ++(-0.25,-1.7) (start40) {$10$};
\node at ++(-0.8,0.2) (var1) {$x3\,x2$};
\node at ++(0.0,0.6) (var1) {$x1\,x0$};
\node at ++(0.25, -0.25) (row11) {$0$}; %erste Zeile
\node at ++(0.75, -0.25) (row12) {$0$};
\node at ++(1.25, -0.25) (row13) {$0$};
\node at ++(1.75, -0.25) (row14) {$0$};
\node at ++(0.25, -0.75) (row21) {$0$}; %zweite Zeile
\node at ++(0.75, -0.75) (row22) {$1$};
\node at ++(1.25, -0.75) (row23) {$1$};
\node at ++(1.75, -0.75) (row24) {$0$};
\node at ++(0.25, -1.25) (row31) {$1$}; %dritte Zeile
\node at ++(0.75, -1.25) (row32) {$1$};
\node at ++(1.25, -1.25) (row33) {$1$};
\node at ++(1.75, -1.25) (row34) {$1$};
\node at ++(0.25, -1.75) (row41) {$0$}; %vierte Zeile
\node at ++(0.75, -1.75) (row42) {$0$};
\node at ++(1.25, -1.75) (row43) {$0$};
\node at ++(1.75, -1.75) (row44) {$0$};
\end{tikzpicture}
\subsection{} %c
Es können zwei Schleifen gebildet werden. Zunächst kann die dritte Zeile komplett als Schleife genommen werden. Außerdem können die beiden Einsen in der zweiten Zeile und die mittleren Einsen in der dritten Zeile in eine Schleife gepackt werden.
Daraus ergibt sich folgender Term für y:
\begin{equation*}
y(x_{3},x_{2},x_{1},x_{0}) = (x_{3}x_{2}) \vee (x_{2}x_{0})
\end{equation*}
\subsection{} %d
\includegraphics[scale=.5]{Uebung7-Schaltplan.png}
\\\\\\\\\\\\\\\\\\\\\\\\
\subsection{} %e
\begin{figure}[h!bp]
\begin{tikzpicture}[shorten >=1pt,node distance=1.1cm,on grid,auto,/tikz/initial text=]
%level 4
\node[state] (f1) {$1$};
%\node[state] (f2) [right=1 of f1] {$1$};
\node[state] (f3) [right=2 of f1] {$0$};
%\node[state] (f4) [right=3 of f1] {$0$};
%\node[state] (f5) [right=4 of f1] {$1$};
%\node[state] (f6) [right=5 of f1] {$1$};
%\node[state] (f7) [right=6 of f1] {$0$};
%\node[state] (f8) [right=7 of f1] {$0$};
\node[state] (f9) [right=4 of f1] {$1$};
\node[state] (f10) [right=5 of f1] {$0$};
\node[state] (f11) [right=6 of f1] {$0$};
%\node[state] (f12) [right=11 of f1] {$0$};
\node[state] (f13) [right=8 of f1] {$1$};
\node[state] (f14) [right=9 of f1] {$0$};
%\node[state] (f15) [right=14 of f1] {$0$};
\node[state] (f16) [right=11 of f1] {$0$};
%level 3
%\node[state] (x31) [above right=1.3 and 0.5 of f1] {$x_{3}$};
%\node[state] (x32) [right=2 of x31] {$x_{3}$};
%\node[state] (x33) [above right=1.3 and 0.5 of f5] {$x_{3}$};
%\node[state] (x34) [right=2 of x33] {$x_{3}$};
\node[state] (x35) [above right=1.3 and 0.5 of f9] {$x_{3}$};
%\node[state] (x36) [right=2 of x35] {$x_{3}$};
\node[state] (x37) [right=4 of x35] {$x_{3}$};
%\node[state] (x38) [right=6 of x35] {$x_{3}$};
%level 2
\node[state] (x23) [above right=1.3 and 1 of x35] {$x_{2}$};
%\node[state] (x22) [left=4 of x23] {$x_{2}$};
\node[state] (x21) [left=4 of x23] {$x_{2}$};
\node[state] (x24) [right=4 of x23] {$x_{2}$};
%level 1
%\node[state] (x11) [above right=1.3 and 2 of x21] {$x_{1}$};
\node[state] (x12) [above right=1.3 and 2 of x23] {$x_{1}$};
%level 0
\node[state] (x0) [above left=1.3 and 4 of x12] {$x_{0}$};
\path[every node/.style={font=\scriptsize}]
(f1) edge node [near start] {$1$} (x21)
%(f2) edge node [near end] {$0$} (x31)
(f3) edge node [near end] {$0$} (x21)
%(f4) edge node [near end] {$0$} (x32)
%(f5) edge node [near start] {$1$} (x22)
%(f6) edge node [near end] {$0$} (x33)
%(f7) edge node [near end] {$0$} (x22)
%(f8) edge node [near end] {$0$} (x34)
(f9) edge node [near start] {$1$} (x35)
(f10) edge node [near end] {$0$} (x35)
(f11) edge node [near end] {$0$} (x23)
%(f12) edge node [near end] {$0$} (x36)
(f13) edge node [near start] {$1$} (x37)
(f14) edge node [near end] {$0$} (x37)
%(f15) edge node [near start] {$1$} (x38)
(f16) edge node [near end] {$0$} (x24)
%(x31) edge node [near start] {$1$} (x21)
%(x32) edge node [near end] {$0$} (x21)
%(x33) edge node [near start] {$1$} (x22)
%(x34) edge node [near end] {$0$} (x22)
(x35) edge node [near start] {$1$} (x23)
%(x36) edge node [near end] {$0$} (x23)
(x37) edge node [near start] {$1$} (x24)
%(x38) edge node [near end] {$0$} (x24)
(x21) edge node [near start] {$1$} (x0)
%(x22) edge node [near end] {$0$} (x11)
(x23) edge node [near start] {$1$} (x12)
(x24) edge node [near end] {$0$} (x12)
%(x11) edge node [near start] {$1$} (x0)
(x12) edge node [near end] {$0$} (x0);
\end{tikzpicture}
\end{figure}
\end{document}

View File

@ -0,0 +1,154 @@
\documentclass[10pt,a4paper,oneside,ngerman,numbers=noenddot]{scrartcl}
\usepackage[T1]{fontenc}
\usepackage[utf8]{inputenc}
\usepackage[ngerman]{babel}
\usepackage{amsmath}
\usepackage{amsfonts}
\usepackage{amssymb}
\usepackage{paralist}
\usepackage{qtree}
\usepackage[locale=DE,exponent-product=\cdot ,detect-all]{siunitx}
\usepackage{tikz}
\usepackage[scaled=0.78]{luximono}
\usepackage{listings}
\usetikzlibrary{automata,matrix,fadings,calc,positioning,decorations.pathreplacing,decorations.text,arrows}
\pagenumbering{arabic}
\def\thesection{8.\arabic{section})}
\def\thesubsection{(\alph{subsection})}
\def\thesubsubsection{(\arabic{subsubsection})}
\renewcommand{\labelenumi}{(\roman{enumi})}
\hyphenation{Nach-komma-stel-len}
\lstnewenvironment{java}[1][]{%
\lstset{basicstyle=\ttfamily ,backgroundcolor=\color[gray]{.95},columns=flexible,fontadjust=true,language=Java,tabsize=4,numbers=none,#1}%
}{%
}
\graphicspath{{D:/Users/Jim-Studium/Pictures/Studium/RS/}}
\tikzstyle{huffmanNodes}=[matrix of nodes,
nodes={circle,thin,draw=black!20,minimum size=10mm,text height=1.5ex,text depth=.25ex,inner sep=-10pt}]
\tikzstyle{huffmanBase}=[matrix of nodes,
nodes={minimum size=10mm,text height=1.5ex,text depth=.25ex,inner sep=-10pt}]
\begin{document}
\author{Jim Martens (Matrikelnummer 6420323) \and Marlo Kornblum (Matrikelnummer 6427301)}
\title{Rechnerstrukturen Aufgabenblatt 8}
\maketitle
\section{} %8.1
1-bit Volladdierer nur mit 2:1 Multiplexern:\\
\begin{alignat*}{2}
s &=& a \oplus b \oplus c_{i} \\
&=& ((a \vee b) \wedge (\overline{a \wedge b})) \oplus c_{i} \\
&=& (((a \vee b) \wedge (\overline{a \wedge b})) \vee c_{i}) \wedge (\overline{((a \vee b) \wedge (\overline{a \wedge b})) \wedge c_{i}}) \\
c_{0} &=& (ab) \vee (a \vee b)c_{i}
\end{alignat*}
\includegraphics[scale=.5]{Uebung8-Schaltplan.png}
\section{} %8.2
\subsection{} %a
\begin{tabular}{r|ccccc}
dez.& $x_{3}$ & $x_{2}$ & $x_{1}$ & $x_{0}$ & A \\
\hline
0 & 0 & 0 & 0 & 0 & 1 \\
1 & 0 & 0 & 0 & 1 & 1 \\
2 & 0 & 0 & 1 & 0 & 1 \\
3 & 0 & 0 & 1 & 1 & 1 \\
4 & 0 & 1 & 0 & 0 & 1 \\
5 & 0 & 1 & 0 & 1 & 0 \\
6 & 0 & 1 & 1 & 0 & 0 \\
7 & 0 & 1 & 1 & 1 & 0 \\
8 & 1 & 0 & 0 & 0 & 1 \\
9 & 1 & 0 & 0 & 1 & 1 \\
10 & 1 & 0 & 1 & 0 & 1 \\
11 & 1 & 0 & 1 & 1 & 1 \\
12 & 1 & 1 & 0 & 0 & 1 \\
13 & 1 & 1 & 0 & 1 & 1 \\
14 & 1 & 1 & 1 & 0 & 1 \\
15 & 1 & 1 & 1 & 1 & 1
\end{tabular}
\begin{tikzpicture}
\draw (0,0) -- +(2,0); %obere Kante
\draw (0,0) -- +(0,-2); %linke Kante
\draw (1,0) -- +(0,-2); %mittlere vertikale Kante
\draw (0,-1) -- +(2,0); %mittlere horizontale Kante
\draw (0,-2) -- +(2,0); %untere Kante
\draw (2,0) -- +(0,-2); %rechte Kante
\draw (0.5,0) -- +(0,-2); %linke mittlere Kante
\draw (1.5,0) -- +(0,-2); %rechte mittlere Kante
\draw (0,-0.5) -- +(2,0); %obere mittlere Kante
\draw (0,-1.5) -- +(2,0); %untere mittlere Kante
\draw (0,0) -- +(-0.5,0.5); %schräger Strich
\node at ++(0.2,0.25) (start11) {$00$}; %Variablenzeile 0
\node at ++(0.7,0.25) (start12) {$01$};
\node at ++(1.2,0.25) (start13) {$11$};
\node at ++(1.7,0.25) (start14) {$10$};
\node at ++(-0.25,-0.2) (start10) {$00$}; %Variablenspalte 0
\node at ++(-0.25,-0.7) (start20) {$01$};
\node at ++(-0.25,-1.2) (start30) {$11$};
\node at ++(-0.25,-1.7) (start40) {$10$};
\node at ++(-0.8,0.2) (var1) {$x3\,x2$};
\node at ++(0.0,0.6) (var1) {$x1\,x0$};
\node at ++(0.25, -0.25) (row11) {$1$}; %erste Zeile
\node at ++(0.75, -0.25) (row12) {$1$};
\node at ++(1.25, -0.25) (row13) {$1$};
\node at ++(1.75, -0.25) (row14) {$1$};
\node at ++(0.25, -0.75) (row21) {$1$}; %zweite Zeile
\node at ++(0.75, -0.75) (row22) {$0$};
\node at ++(1.25, -0.75) (row23) {$0$};
\node at ++(1.75, -0.75) (row24) {$0$};
\node at ++(0.25, -1.25) (row31) {$1$}; %dritte Zeile
\node at ++(0.75, -1.25) (row32) {$1$};
\node at ++(1.25, -1.25) (row33) {$1$};
\node at ++(1.75, -1.25) (row34) {$1$};
\node at ++(0.25, -1.75) (row41) {$1$}; %vierte Zeile
\node at ++(0.75, -1.75) (row42) {$1$};
\node at ++(1.25, -1.75) (row43) {$1$};
\node at ++(1.75, -1.75) (row44) {$1$};
\end{tikzpicture}
\subsection{} %b
disjunktiv:\\
Es lassen sich die erste und vierte Zeile als eine Schleife einzeichnen. Außerdem kann die erste Spalte als eine Schleife genommen werden. Zusätzlich können die die dritte und vierte Zeile komplett als eine Schleife realisiert werden. Es ergibt sich:
\begin{equation*}
f(x_{3},x_{2},x_{1},x_{0}) = \overline{x_{2}} \vee (\overline{x_{1}}\overline{x_{0}}) \vee x_{3}
\end{equation*}
konjunktiv:\\
Die zwei rechten Nullen und die zwei linken Nullen können jeweils in eine Schleife gepackt werden. Es ergibt sich:
\begin{equation*}
f(x_{3},x_{2},x_{1},x_{0}) = (x_{3} \vee \overline{x_{2}} \vee \overline{x_{0}}) \wedge (x_{3} \vee \overline{x_{2}} \vee \overline{x_{1}})
\end{equation*}
\subsection{} %c
disjunktiv:\\
\includegraphics[scale=.5]{Uebung8_2_c1-Schaltplan.png}
konjunktiv:\\
\includegraphics[scale=.5]{Uebung8_2_c2-Schaltplan.png}
\section{} %8.3
\subsection{} %a
\includegraphics[scale=1.0]{Uebung8-Impulsdiagramm.png}
Es tritt ein statischer 1-Hazard beim Ausgang x auf. Beim Ausgang y tritt kein Hazard auf.
\subsection{} %b
\includegraphics[scale=1.0]{Uebung8-Impulsdiagramm2.png}
Es treten keine Hazards auf.
\section{} %8.4
\subsection{} %a
Für $n$ Bits ergibt sich folgende Formel für die Zeitschritte T: $T(n) = 2 \cdot \log_{2} (n) + 4$.
\subsection{} %b
Dieser Addierer benötigt $\frac{n}{m} + 1$ Zeitschritte für die Blöcke und daher insgesamt $m \cdot (2 \cdot \frac{n}{m} + 1)$ Zeitschritte.
Um die Verzögerung zu minimieren muss $m = \lfloor \sqrt{n} \rfloor$ gelten.
\subsection{} %c
Für $n=64$ ergeben sich für den Ripple-Carry-Addierer $64$ Zeiteinheiten und damit $64 \cdot 35ps=2240ps$. Der Carry-Lookahead-Addierer benötigt $2 \cdot \log_{2} (64) + 4 = 2 \cdot 6 + 4 = 12 + 4 = 16$ Zeiteinheiten und damit $16 \cdot 35ps=560ps$.
Mit dem Carry-Select-Addierer erreicht man $8 \cdot (2\cdot 8 + 1) = 8 \cdot (16 + 1) = 8 \cdot 17 = 136$ Zeiteinheiten. Somit ergeben sich $136 \cdot 35ps=4760ps$.
Für $n=256$ ergeben sich für den Ripple-Carry-Addierer $256$ Zeiteinheiten und damit $256 \cdot 35ps=8960ps$. Der Carry-Lookahead-Addierer benötigt $2 \cdot \log_{2} (256) + 4 = 2 \cdot 8 + 4 = 16 + 4 = 20$ Zeiteinheiten und damit $20 \cdot 35ps=700ps$.
Mit dem Carry-Select-Addierer erreicht man $16 \cdot (2 \cdot 16 + 1) = 16 \cdot (32 + 1) = 16 \cdot 33 = 528$ Zeiteinheiten. Somit ergeben sich $528 \cdot 35ps=18480ps$.
\end{document}

View File

@ -0,0 +1,607 @@
\documentclass[10pt,a4paper,oneside,ngerman,numbers=noenddot]{scrartcl}
\usepackage[T1]{fontenc}
\usepackage[utf8]{inputenc}
\usepackage[ngerman]{babel}
\usepackage{amsmath}
\usepackage{amsfonts}
\usepackage{amssymb}
\usepackage{paralist}
\usepackage{qtree}
\usepackage[locale=DE,exponent-product=\cdot ,detect-all]{siunitx}
\usepackage{tikz}
\usepackage[scaled=0.78]{luximono}
\usepackage{listings}
\usepackage{subfigure}
\usetikzlibrary{automata,matrix,fadings,calc,positioning,decorations.pathreplacing,decorations.text,arrows}
\pagenumbering{arabic}
\def\thesection{9.\arabic{section})}
\def\thesubsection{(\alph{subsection})}
\def\thesubsubsection{(\arabic{subsubsection})}
\renewcommand{\labelenumi}{(\roman{enumi})}
\hyphenation{Nach-komma-stel-len}
\lstnewenvironment{java}[1][]{%
\lstset{basicstyle=\ttfamily ,backgroundcolor=\color[gray]{.95},columns=flexible,fontadjust=true,language=Java,tabsize=4,numbers=none,#1}%
}{%
}
\graphicspath{{D:/Users/Jim-Studium/Pictures/Studium/RS/}}
\tikzstyle{huffmanNodes}=[matrix of nodes,
nodes={circle,thin,draw=black!20,minimum size=10mm,text height=1.5ex,text depth=.25ex,inner sep=-10pt}]
\tikzstyle{huffmanBase}=[matrix of nodes,
nodes={minimum size=10mm,text height=1.5ex,text depth=.25ex,inner sep=-10pt}]
\begin{document}
\author{Jim Martens (Matrikelnummer 6420323) \and Marlo Kornblum (Matrikelnummer 6427301)}
\title{Rechnerstrukturen Aufgabenblatt 9}
\maketitle
\section{} %9.1
\includegraphics[scale=1.0]{Uebung9-1.png}
\section{} %9.2
\subsection{} %a
\begin{figure}[h!]
Erste Schaltung:
\hfill
ZweiteSchaltung:
\subfigure{
\begin{tabular}{ccc||c}
D & E & CLK & $Q^{+}$ \\
\hline
* & * & 0 & Q \\
* & * & 1 & Q \\
* & 0 & $\uparrow$ & Q \\
0 & 1 & $\uparrow$ & 1 \\
1 & 1 & $\uparrow$ & 1
\end{tabular}
}\hfill
\subfigure{
\begin{tabular}{ccc||c}
D & E & CLK & $Q^{+}$ \\
\hline
* & 0 & * & Q \\
* & 1 & 0 & Q \\
* & 1 & 1 & Q \\
0 & $\uparrow$ & 1 & 0 \\
1 & $\uparrow$ & 1 & 1 \\
0 & 1 & $\uparrow$ & 0 \\
1 & 1 & $\uparrow$ & 1 \\
0 & $\uparrow$ & $\uparrow$ & 0 \\
1 & $\uparrow$ & $\uparrow$ & 1
\end{tabular}
}
\end{figure}
\subsection{} %b
Bei beiden Schaltungen werden nur bestimmte Dateneingänge gespeichert. Dies wird für Schaltungen eingesetzt, bei denen sich der Zustand nur in festgelegten Intervallen (Takten) ändern soll.
\subsection{} %c
Der Vorteil der zweiten Lösung ist die Nichtverwendung eines Multiplexers. Das macht die Schaltung weitaus simpler und reduziert womöglich die Gatterlaufzeiten.
Auf der anderen Seite ist ein großer Nachteil der zweiten Lösung, dass auch mit E "geclockt" werden kann. Somit ist nicht immer eindeutig, ob immer ein gewollter Takt anliegt.
Durch Hazards kann an ungewollten Stellen geschaltet werden.
Die Vor- und Nachteile der zweiten Lösung, sind die Nach- und Vorteile der ersten Lösung. Negativ ist, dass ein Multiplexer verwendet wird. Positiv ist hingegen, dass der Takt an sich immer eindeutig ist und auch immer eindeutige Daten am Flipflop ankommen. Entweder D (E ist auf 1) oder Q. Ist E auf 0 kann sich der Folgezustand demzufolge gar nicht ändern.
\section{} %9.3
\subsection{} %a
\begin{tikzpicture}[shorten >=1pt,node distance=2.0cm,on grid,auto,/tikz/initial text=,>=stealth']
\tikzset{every state/.style={minimum size=3.6em}}
\node[state,initial] (Z0) {$Z_{0}$};
\node[state] (Z1) [below right=1.3 and 1.5 of Z0] {$Z_{1}$};
\node[state] (Z2) [below right=1.3 and 1.5 of Z1] {$Z_{2}$};
\node[state] (Z3) [below left=1.3 and 1.5 of Z2] {$Z_{3}$};
\node[state] (Z4) [below left=1.3 and 1.5 of Z3] {$Z_{4}$};
\node[state] (Z5) [above left=1.3 and 1.5 of Z4] {$Z_{5}$};
\node[state] (Z6) [above left=1.3 and 1.5 of Z5] {$Z_{6}$};
\node[state] (Z7) [above right=1.3 and 1.5 of Z6] {$Z_{7}$};
\node (z0) [below=0.2 of Z0] {\tiny{$000$}};
\node (z1) [below=0.2 of Z1] {\tiny{$001$}};
\node (z2) [below=0.2 of Z2] {\tiny{$010$}};
\node (z3) [below=0.2 of Z3] {\tiny{$011$}};
\node (z4) [below=0.2 of Z4] {\tiny{$100$}};
\node (z5) [below=0.2 of Z5] {\tiny{$101$}};
\node (z6) [below=0.2 of Z6] {\tiny{$110$}};
\node (z7) [below=0.2 of Z7] {\tiny{$111$}};
\path[every node/.style={font=\scriptsize}]
(Z0) edge[->] (Z1)
(Z1) edge[->] (Z2)
(Z2) edge[loop right,->] node [near start] {$i=0$} (Z2)
(Z2) edge[->] node [near start] {$i=1$} (Z3)
(Z3) edge[->] (Z4)
(Z4) edge[->] (Z5)
(Z5) edge[->] (Z6)
(Z6) edge[->] (Z7)
(Z7) edge[->] (Z0);
\draw (-0.55,-0.3) -- +(1.15,0);
\draw (0.95,-1.6) -- +(1.15,0);
\draw (2.45,-2.9) -- +(1.15,0);
\draw (0.95,-4.2) -- +(1.15,0);
\draw (-0.55,-5.5) -- +(1.15,0);
\draw (-2.05,-4.2) -- +(1.15,0);
\draw (-3.55,-2.9) -- +(1.15,0);
\draw (-2.05,-1.6) -- +(1.15,0);
\node (z00) [below=0.45 of Z0] {\small{$r;r$}};
\node (z01) [below=0.45 of Z1] {\small{$r,ge;r$}};
\node (z02) [below=0.45 of Z2] {\small{$gr;r$}};
\node (z03) [below=0.45 of Z3] {\small{$ge;r$}};
\node (z04) [below=0.45 of Z4] {\small{$r;r$}};
\node (z05) [below=0.45 of Z5] {\small{$r;r,ge$}};
\node (z06) [below=0.45 of Z6] {\small{$r;gr$}};
\node (z07) [below=0.45 of Z7] {\small{$r;ge$}};
\end{tikzpicture}
\subsection{} %b
\begin{tabular}{c|ccc||ccc|ccc|ccc}
i & $z_{2}$ & $z_{1}$ & $z_{0}$ & $z_{2}^{+}$ & $z_{1}^{+}$ & $z_{0}^{+}$ & $rt_{H}$ & $ge_{H}$ & $gr_{H}$ & $rt_{N}$ & $ge_{N}$ & $gr_{N}$\\
\hline
* & 0 & 0 & 0 & 0 & 0 & 1 & 1 & 0 & 0 & 1 & 0 & 0\\
* & 0 & 0 & 1 & 0 & 1 & 0 & 1 & 1 & 0 & 1 & 0 & 0\\
0 & 0 & 1 & 0 & 0 & 1 & 0 & 0 & 0 & 1 & 1 & 0 & 0\\
1 & 0 & 1 & 0 & 0 & 1 & 1 & 0 & 0 & 1 & 1 & 0 & 0\\
* & 0 & 1 & 1 & 1 & 0 & 0 & 0 & 1 & 0 & 1 & 0 & 0\\
* & 1 & 0 & 0 & 1 & 0 & 1 & 1 & 0 & 0 & 1 & 0 & 0\\
* & 1 & 0 & 1 & 1 & 1 & 0 & 1 & 0 & 0 & 1 & 1 & 0\\
* & 1 & 1 & 0 & 1 & 1 & 1 & 1 & 0 & 0 & 0 & 0 & 1\\
* & 1 & 1 & 1 & 0 & 0 & 0 & 1 & 0 & 0 & 0 & 1 & 0
\end{tabular}
\subsection{} %c
\begin{figure}[hbp]
\subfigure{
\begin{tikzpicture}
\draw (0,0) -- +(2,0); %obere Kante
\draw (0,0) -- +(0,-2); %linke Kante
\draw (1,0) -- +(0,-2); %mittlere vertikale Kante
\draw (0,-1) -- +(2,0); %mittlere horizontale Kante
\draw (0,-2) -- +(2,0); %untere Kante
\draw (2,0) -- +(0,-2); %rechte Kante
\draw (0.5,0) -- +(0,-2); %linke mittlere Kante
\draw (1.5,0) -- +(0,-2); %rechte mittlere Kante
\draw (0,-0.5) -- +(2,0); %obere mittlere Kante
\draw (0,-1.5) -- +(2,0); %untere mittlere Kante
\draw (0,0) -- +(-0.5,0.5); %schräger Strich
\node at ++(0.2,0.25) (start11) {$00$}; %Variablenzeile 0
\node at ++(0.7,0.25) (start12) {$01$};
\node at ++(1.2,0.25) (start13) {$11$};
\node at ++(1.7,0.25) (start14) {$10$};
\node at ++(-0.25,-0.2) (start10) {$00$}; %Variablenspalte 0
\node at ++(-0.25,-0.7) (start20) {$01$};
\node at ++(-0.25,-1.2) (start30) {$11$};
\node at ++(-0.25,-1.7) (start40) {$10$};
\node at ++(-0.8,0.2) (var1) {$i, z_{2}$};
\node at ++(0.0,0.6) (var1) {$z_{1}\,z_{0}$};
\node at ++(0.25, -0.25) (row11) {$0$}; %erste Zeile
\node at ++(0.75, -0.25) (row12) {$0$};
\node at ++(1.25, -0.25) (row13) {$0$};
\node at ++(1.75, -0.25) (row14) {$1$};
\node at ++(0.25, -0.75) (row21) {$1$}; %zweite Zeile
\node at ++(0.75, -0.75) (row22) {$1$};
\node at ++(1.25, -0.75) (row23) {$1$};
\node at ++(1.75, -0.75) (row24) {$0$};
\node at ++(0.25, -1.25) (row31) {$1$}; %dritte Zeile
\node at ++(0.75, -1.25) (row32) {$1$};
\node at ++(1.25, -1.25) (row33) {$1$};
\node at ++(1.75, -1.25) (row34) {$0$};
\node at ++(0.25, -1.75) (row41) {$0$}; %vierte Zeile
\node at ++(0.75, -1.75) (row42) {$0$};
\node at ++(1.25, -1.75) (row43) {$0$};
\node at ++(1.75, -1.75) (row44) {$1$};
\end{tikzpicture}
}\hfill
\subfigure{
\begin{tikzpicture}
\draw (0,0) -- +(2,0); %obere Kante
\draw (0,0) -- +(0,-2); %linke Kante
\draw (1,0) -- +(0,-2); %mittlere vertikale Kante
\draw (0,-1) -- +(2,0); %mittlere horizontale Kante
\draw (0,-2) -- +(2,0); %untere Kante
\draw (2,0) -- +(0,-2); %rechte Kante
\draw (0.5,0) -- +(0,-2); %linke mittlere Kante
\draw (1.5,0) -- +(0,-2); %rechte mittlere Kante
\draw (0,-0.5) -- +(2,0); %obere mittlere Kante
\draw (0,-1.5) -- +(2,0); %untere mittlere Kante
\draw (0,0) -- +(-0.5,0.5); %schräger Strich
\node at ++(0.2,0.25) (start11) {$00$}; %Variablenzeile 0
\node at ++(0.7,0.25) (start12) {$01$};
\node at ++(1.2,0.25) (start13) {$11$};
\node at ++(1.7,0.25) (start14) {$10$};
\node at ++(-0.25,-0.2) (start10) {$00$}; %Variablenspalte 0
\node at ++(-0.25,-0.7) (start20) {$01$};
\node at ++(-0.25,-1.2) (start30) {$11$};
\node at ++(-0.25,-1.7) (start40) {$10$};
\node at ++(-0.8,0.2) (var1) {$i, z_{2}$};
\node at ++(0.0,0.6) (var1) {$z_{1}\,z_{0}$};
\node at ++(0.25, -0.25) (row11) {$0$}; %erste Zeile
\node at ++(0.75, -0.25) (row12) {$1$};
\node at ++(1.25, -0.25) (row13) {$0$};
\node at ++(1.75, -0.25) (row14) {$1$};
\node at ++(0.25, -0.75) (row21) {$0$}; %zweite Zeile
\node at ++(0.75, -0.75) (row22) {$1$};
\node at ++(1.25, -0.75) (row23) {$0$};
\node at ++(1.75, -0.75) (row24) {$1$};
\node at ++(0.25, -1.25) (row31) {$0$}; %dritte Zeile
\node at ++(0.75, -1.25) (row32) {$1$};
\node at ++(1.25, -1.25) (row33) {$0$};
\node at ++(1.75, -1.25) (row34) {$1$};
\node at ++(0.25, -1.75) (row41) {$0$}; %vierte Zeile
\node at ++(0.75, -1.75) (row42) {$1$};
\node at ++(1.25, -1.75) (row43) {$0$};
\node at ++(1.75, -1.75) (row44) {$1$};
\end{tikzpicture}
}
KV-Diagramm für $z_{2}^{+}$
\hfill
KV-Diagramm für $z_{1}^{+}$
\end{figure}
\begin{figure}[hbp]
\subfigure{
\begin{tikzpicture}
\draw (0,0) -- +(2,0); %obere Kante
\draw (0,0) -- +(0,-2); %linke Kante
\draw (1,0) -- +(0,-2); %mittlere vertikale Kante
\draw (0,-1) -- +(2,0); %mittlere horizontale Kante
\draw (0,-2) -- +(2,0); %untere Kante
\draw (2,0) -- +(0,-2); %rechte Kante
\draw (0.5,0) -- +(0,-2); %linke mittlere Kante
\draw (1.5,0) -- +(0,-2); %rechte mittlere Kante
\draw (0,-0.5) -- +(2,0); %obere mittlere Kante
\draw (0,-1.5) -- +(2,0); %untere mittlere Kante
\draw (0,0) -- +(-0.5,0.5); %schräger Strich
\node at ++(0.2,0.25) (start11) {$00$}; %Variablenzeile 0
\node at ++(0.7,0.25) (start12) {$01$};
\node at ++(1.2,0.25) (start13) {$11$};
\node at ++(1.7,0.25) (start14) {$10$};
\node at ++(-0.25,-0.2) (start10) {$00$}; %Variablenspalte 0
\node at ++(-0.25,-0.7) (start20) {$01$};
\node at ++(-0.25,-1.2) (start30) {$11$};
\node at ++(-0.25,-1.7) (start40) {$10$};
\node at ++(-0.8,0.2) (var1) {$i, z_{2}$};
\node at ++(0.0,0.6) (var1) {$z_{1}\,z_{0}$};
\node at ++(0.25, -0.25) (row11) {$1$}; %erste Zeile
\node at ++(0.75, -0.25) (row12) {$0$};
\node at ++(1.25, -0.25) (row13) {$0$};
\node at ++(1.75, -0.25) (row14) {$0$};
\node at ++(0.25, -0.75) (row21) {$1$}; %zweite Zeile
\node at ++(0.75, -0.75) (row22) {$0$};
\node at ++(1.25, -0.75) (row23) {$0$};
\node at ++(1.75, -0.75) (row24) {$1$};
\node at ++(0.25, -1.25) (row31) {$1$}; %dritte Zeile
\node at ++(0.75, -1.25) (row32) {$0$};
\node at ++(1.25, -1.25) (row33) {$0$};
\node at ++(1.75, -1.25) (row34) {$1$};
\node at ++(0.25, -1.75) (row41) {$1$}; %vierte Zeile
\node at ++(0.75, -1.75) (row42) {$0$};
\node at ++(1.25, -1.75) (row43) {$0$};
\node at ++(1.75, -1.75) (row44) {$1$};
\end{tikzpicture}
}\hfill
\subfigure{
\begin{tikzpicture}
\draw (0,0) -- +(2,0); %obere Kante
\draw (0,0) -- +(0,-2); %linke Kante
\draw (1,0) -- +(0,-2); %mittlere vertikale Kante
\draw (0,-1) -- +(2,0); %mittlere horizontale Kante
\draw (0,-2) -- +(2,0); %untere Kante
\draw (2,0) -- +(0,-2); %rechte Kante
\draw (0.5,0) -- +(0,-2); %linke mittlere Kante
\draw (1.5,0) -- +(0,-2); %rechte mittlere Kante
\draw (0,-0.5) -- +(2,0); %obere mittlere Kante
\draw (0,-1.5) -- +(2,0); %untere mittlere Kante
\draw (0,0) -- +(-0.5,0.5); %schräger Strich
\node at ++(0.2,0.25) (start11) {$00$}; %Variablenzeile 0
\node at ++(0.7,0.25) (start12) {$01$};
\node at ++(1.2,0.25) (start13) {$11$};
\node at ++(1.7,0.25) (start14) {$10$};
\node at ++(-0.25,-0.2) (start10) {$00$}; %Variablenspalte 0
\node at ++(-0.25,-0.7) (start20) {$01$};
\node at ++(-0.25,-1.2) (start30) {$11$};
\node at ++(-0.25,-1.7) (start40) {$10$};
\node at ++(-0.8,0.2) (var1) {$i, z_{2}$};
\node at ++(0.0,0.6) (var1) {$z_{1}\,z_{0}$};
\node at ++(0.25, -0.25) (row11) {$1$}; %erste Zeile
\node at ++(0.75, -0.25) (row12) {$1$};
\node at ++(1.25, -0.25) (row13) {$0$};
\node at ++(1.75, -0.25) (row14) {$0$};
\node at ++(0.25, -0.75) (row21) {$1$}; %zweite Zeile
\node at ++(0.75, -0.75) (row22) {$1$};
\node at ++(1.25, -0.75) (row23) {$1$};
\node at ++(1.75, -0.75) (row24) {$1$};
\node at ++(0.25, -1.25) (row31) {$1$}; %dritte Zeile
\node at ++(0.75, -1.25) (row32) {$1$};
\node at ++(1.25, -1.25) (row33) {$1$};
\node at ++(1.75, -1.25) (row34) {$1$};
\node at ++(0.25, -1.75) (row41) {$1$}; %vierte Zeile
\node at ++(0.75, -1.75) (row42) {$1$};
\node at ++(1.25, -1.75) (row43) {$0$};
\node at ++(1.75, -1.75) (row44) {$0$};
\end{tikzpicture}
}
KV-Diagramm für $z_{0}^{+}$
\hfill
KV-Diagramm für $rt_{H}$
\end{figure}
\begin{figure}[hbp]
\subfigure{
\begin{tikzpicture}
\draw (0,0) -- +(2,0); %obere Kante
\draw (0,0) -- +(0,-2); %linke Kante
\draw (1,0) -- +(0,-2); %mittlere vertikale Kante
\draw (0,-1) -- +(2,0); %mittlere horizontale Kante
\draw (0,-2) -- +(2,0); %untere Kante
\draw (2,0) -- +(0,-2); %rechte Kante
\draw (0.5,0) -- +(0,-2); %linke mittlere Kante
\draw (1.5,0) -- +(0,-2); %rechte mittlere Kante
\draw (0,-0.5) -- +(2,0); %obere mittlere Kante
\draw (0,-1.5) -- +(2,0); %untere mittlere Kante
\draw (0,0) -- +(-0.5,0.5); %schräger Strich
\node at ++(0.2,0.25) (start11) {$00$}; %Variablenzeile 0
\node at ++(0.7,0.25) (start12) {$01$};
\node at ++(1.2,0.25) (start13) {$11$};
\node at ++(1.7,0.25) (start14) {$10$};
\node at ++(-0.25,-0.2) (start10) {$00$}; %Variablenspalte 0
\node at ++(-0.25,-0.7) (start20) {$01$};
\node at ++(-0.25,-1.2) (start30) {$11$};
\node at ++(-0.25,-1.7) (start40) {$10$};
\node at ++(-0.8,0.2) (var1) {$i, z_{2}$};
\node at ++(0.0,0.6) (var1) {$z_{1}\,z_{0}$};
\node at ++(0.25, -0.25) (row11) {$0$}; %erste Zeile
\node at ++(0.75, -0.25) (row12) {$1$};
\node at ++(1.25, -0.25) (row13) {$1$};
\node at ++(1.75, -0.25) (row14) {$0$};
\node at ++(0.25, -0.75) (row21) {$0$}; %zweite Zeile
\node at ++(0.75, -0.75) (row22) {$0$};
\node at ++(1.25, -0.75) (row23) {$0$};
\node at ++(1.75, -0.75) (row24) {$0$};
\node at ++(0.25, -1.25) (row31) {$0$}; %dritte Zeile
\node at ++(0.75, -1.25) (row32) {$0$};
\node at ++(1.25, -1.25) (row33) {$0$};
\node at ++(1.75, -1.25) (row34) {$0$};
\node at ++(0.25, -1.75) (row41) {$0$}; %vierte Zeile
\node at ++(0.75, -1.75) (row42) {$1$};
\node at ++(1.25, -1.75) (row43) {$1$};
\node at ++(1.75, -1.75) (row44) {$0$};
\end{tikzpicture}
}\hfill
\subfigure{
\begin{tikzpicture}
\draw (0,0) -- +(2,0); %obere Kante
\draw (0,0) -- +(0,-2); %linke Kante
\draw (1,0) -- +(0,-2); %mittlere vertikale Kante
\draw (0,-1) -- +(2,0); %mittlere horizontale Kante
\draw (0,-2) -- +(2,0); %untere Kante
\draw (2,0) -- +(0,-2); %rechte Kante
\draw (0.5,0) -- +(0,-2); %linke mittlere Kante
\draw (1.5,0) -- +(0,-2); %rechte mittlere Kante
\draw (0,-0.5) -- +(2,0); %obere mittlere Kante
\draw (0,-1.5) -- +(2,0); %untere mittlere Kante
\draw (0,0) -- +(-0.5,0.5); %schräger Strich
\node at ++(0.2,0.25) (start11) {$00$}; %Variablenzeile 0
\node at ++(0.7,0.25) (start12) {$01$};
\node at ++(1.2,0.25) (start13) {$11$};
\node at ++(1.7,0.25) (start14) {$10$};
\node at ++(-0.25,-0.2) (start10) {$00$}; %Variablenspalte 0
\node at ++(-0.25,-0.7) (start20) {$01$};
\node at ++(-0.25,-1.2) (start30) {$11$};
\node at ++(-0.25,-1.7) (start40) {$10$};
\node at ++(-0.8,0.2) (var1) {$i, z_{2}$};
\node at ++(0.0,0.6) (var1) {$z_{1}\,z_{0}$};
\node at ++(0.25, -0.25) (row11) {$0$}; %erste Zeile
\node at ++(0.75, -0.25) (row12) {$0$};
\node at ++(1.25, -0.25) (row13) {$0$};
\node at ++(1.75, -0.25) (row14) {$1$};
\node at ++(0.25, -0.75) (row21) {$0$}; %zweite Zeile
\node at ++(0.75, -0.75) (row22) {$0$};
\node at ++(1.25, -0.75) (row23) {$0$};
\node at ++(1.75, -0.75) (row24) {$0$};
\node at ++(0.25, -1.25) (row31) {$0$}; %dritte Zeile
\node at ++(0.75, -1.25) (row32) {$0$};
\node at ++(1.25, -1.25) (row33) {$0$};
\node at ++(1.75, -1.25) (row34) {$0$};
\node at ++(0.25, -1.75) (row41) {$0$}; %vierte Zeile
\node at ++(0.75, -1.75) (row42) {$0$};
\node at ++(1.25, -1.75) (row43) {$0$};
\node at ++(1.75, -1.75) (row44) {$1$};
\end{tikzpicture}
}
KV-Diagramm für $ge_{H}$
\hfill
KV-Diagramm für $gr_{H}$
\end{figure}
\begin{figure}[hbp]
\subfigure{
\begin{tikzpicture}
\draw (0,0) -- +(2,0); %obere Kante
\draw (0,0) -- +(0,-2); %linke Kante
\draw (1,0) -- +(0,-2); %mittlere vertikale Kante
\draw (0,-1) -- +(2,0); %mittlere horizontale Kante
\draw (0,-2) -- +(2,0); %untere Kante
\draw (2,0) -- +(0,-2); %rechte Kante
\draw (0.5,0) -- +(0,-2); %linke mittlere Kante
\draw (1.5,0) -- +(0,-2); %rechte mittlere Kante
\draw (0,-0.5) -- +(2,0); %obere mittlere Kante
\draw (0,-1.5) -- +(2,0); %untere mittlere Kante
\draw (0,0) -- +(-0.5,0.5); %schräger Strich
\node at ++(0.2,0.25) (start11) {$00$}; %Variablenzeile 0
\node at ++(0.7,0.25) (start12) {$01$};
\node at ++(1.2,0.25) (start13) {$11$};
\node at ++(1.7,0.25) (start14) {$10$};
\node at ++(-0.25,-0.2) (start10) {$00$}; %Variablenspalte 0
\node at ++(-0.25,-0.7) (start20) {$01$};
\node at ++(-0.25,-1.2) (start30) {$11$};
\node at ++(-0.25,-1.7) (start40) {$10$};
\node at ++(-0.8,0.2) (var1) {$i, z_{2}$};
\node at ++(0.0,0.6) (var1) {$z_{1}\,z_{0}$};
\node at ++(0.25, -0.25) (row11) {$1$}; %erste Zeile
\node at ++(0.75, -0.25) (row12) {$1$};
\node at ++(1.25, -0.25) (row13) {$1$};
\node at ++(1.75, -0.25) (row14) {$1$};
\node at ++(0.25, -0.75) (row21) {$1$}; %zweite Zeile
\node at ++(0.75, -0.75) (row22) {$1$};
\node at ++(1.25, -0.75) (row23) {$0$};
\node at ++(1.75, -0.75) (row24) {$0$};
\node at ++(0.25, -1.25) (row31) {$1$}; %dritte Zeile
\node at ++(0.75, -1.25) (row32) {$1$};
\node at ++(1.25, -1.25) (row33) {$0$};
\node at ++(1.75, -1.25) (row34) {$0$};
\node at ++(0.25, -1.75) (row41) {$1$}; %vierte Zeile
\node at ++(0.75, -1.75) (row42) {$1$};
\node at ++(1.25, -1.75) (row43) {$1$};
\node at ++(1.75, -1.75) (row44) {$1$};
\end{tikzpicture}
}\hfill
\subfigure{
\begin{tikzpicture}
\draw (0,0) -- +(2,0); %obere Kante
\draw (0,0) -- +(0,-2); %linke Kante
\draw (1,0) -- +(0,-2); %mittlere vertikale Kante
\draw (0,-1) -- +(2,0); %mittlere horizontale Kante
\draw (0,-2) -- +(2,0); %untere Kante
\draw (2,0) -- +(0,-2); %rechte Kante
\draw (0.5,0) -- +(0,-2); %linke mittlere Kante
\draw (1.5,0) -- +(0,-2); %rechte mittlere Kante
\draw (0,-0.5) -- +(2,0); %obere mittlere Kante
\draw (0,-1.5) -- +(2,0); %untere mittlere Kante
\draw (0,0) -- +(-0.5,0.5); %schräger Strich
\node at ++(0.2,0.25) (start11) {$00$}; %Variablenzeile 0
\node at ++(0.7,0.25) (start12) {$01$};
\node at ++(1.2,0.25) (start13) {$11$};
\node at ++(1.7,0.25) (start14) {$10$};
\node at ++(-0.25,-0.2) (start10) {$00$}; %Variablenspalte 0
\node at ++(-0.25,-0.7) (start20) {$01$};
\node at ++(-0.25,-1.2) (start30) {$11$};
\node at ++(-0.25,-1.7) (start40) {$10$};
\node at ++(-0.8,0.2) (var1) {$i, z_{2}$};
\node at ++(0.0,0.6) (var1) {$z_{1}\,z_{0}$};
\node at ++(0.25, -0.25) (row11) {$0$}; %erste Zeile
\node at ++(0.75, -0.25) (row12) {$0$};
\node at ++(1.25, -0.25) (row13) {$0$};
\node at ++(1.75, -0.25) (row14) {$0$};
\node at ++(0.25, -0.75) (row21) {$0$}; %zweite Zeile
\node at ++(0.75, -0.75) (row22) {$1$};
\node at ++(1.25, -0.75) (row23) {$1$};
\node at ++(1.75, -0.75) (row24) {$0$};
\node at ++(0.25, -1.25) (row31) {$0$}; %dritte Zeile
\node at ++(0.75, -1.25) (row32) {$1$};
\node at ++(1.25, -1.25) (row33) {$1$};
\node at ++(1.75, -1.25) (row34) {$0$};
\node at ++(0.25, -1.75) (row41) {$0$}; %vierte Zeile
\node at ++(0.75, -1.75) (row42) {$0$};
\node at ++(1.25, -1.75) (row43) {$0$};
\node at ++(1.75, -1.75) (row44) {$0$};
\end{tikzpicture}
}
KV-Diagramm für $rt_{N}$
\hfill
KV-Diagramm für $ge_{N}$
\end{figure}
\begin{figure}[hb]
\subfigure{
\begin{tikzpicture}
\draw (0,0) -- +(2,0); %obere Kante
\draw (0,0) -- +(0,-2); %linke Kante
\draw (1,0) -- +(0,-2); %mittlere vertikale Kante
\draw (0,-1) -- +(2,0); %mittlere horizontale Kante
\draw (0,-2) -- +(2,0); %untere Kante
\draw (2,0) -- +(0,-2); %rechte Kante
\draw (0.5,0) -- +(0,-2); %linke mittlere Kante
\draw (1.5,0) -- +(0,-2); %rechte mittlere Kante
\draw (0,-0.5) -- +(2,0); %obere mittlere Kante
\draw (0,-1.5) -- +(2,0); %untere mittlere Kante
\draw (0,0) -- +(-0.5,0.5); %schräger Strich
\node at ++(0.2,0.25) (start11) {$00$}; %Variablenzeile 0
\node at ++(0.7,0.25) (start12) {$01$};
\node at ++(1.2,0.25) (start13) {$11$};
\node at ++(1.7,0.25) (start14) {$10$};
\node at ++(-0.25,-0.2) (start10) {$00$}; %Variablenspalte 0
\node at ++(-0.25,-0.7) (start20) {$01$};
\node at ++(-0.25,-1.2) (start30) {$11$};
\node at ++(-0.25,-1.7) (start40) {$10$};
\node at ++(-0.8,0.2) (var1) {$i, z_{2}$};
\node at ++(0.0,0.6) (var1) {$z_{1}\,z_{0}$};
\node at ++(0.25, -0.25) (row11) {$0$}; %erste Zeile
\node at ++(0.75, -0.25) (row12) {$0$};
\node at ++(1.25, -0.25) (row13) {$0$};
\node at ++(1.75, -0.25) (row14) {$0$};
\node at ++(0.25, -0.75) (row21) {$0$}; %zweite Zeile
\node at ++(0.75, -0.75) (row22) {$0$};
\node at ++(1.25, -0.75) (row23) {$0$};
\node at ++(1.75, -0.75) (row24) {$1$};
\node at ++(0.25, -1.25) (row31) {$0$}; %dritte Zeile
\node at ++(0.75, -1.25) (row32) {$0$};
\node at ++(1.25, -1.25) (row33) {$0$};
\node at ++(1.75, -1.25) (row34) {$1$};
\node at ++(0.25, -1.75) (row41) {$0$}; %vierte Zeile
\node at ++(0.75, -1.75) (row42) {$0$};
\node at ++(1.25, -1.75) (row43) {$0$};
\node at ++(1.75, -1.75) (row44) {$0$};
\end{tikzpicture}
}
KV-Diagramm für $gr_{N}$
\end{figure}
\end{document}

View File

@ -1,11 +1,10 @@
%!TEX encoding = UTF-8 Unicode
\documentclass[12pt]{scrartcl}
%\usepackage[applemac]{inputenc} % Mac-Umlaute direkt verwenden öäüß
%\usepackage[isolatin]{inputenc} % PC-Umlaute direkt verwenden
\usepackage[utf8]{inputenc} % Unicode funktioniert unter Windows, Linux und Mac
\usepackage[T1]{fontenc}
\usepackage{times}
\usepackage[ngerman]{babel}
\usepackage{fancyref}
\usepackage{csquotes}
\usepackage[
backend=biber,
@ -19,7 +18,6 @@ citestyle=ieee
\usepackage[scaled]{helvet}
\usepackage{a4wide}
\usepackage{rotating}
\usepackage[german]{fancyref}
\usepackage{listings}\lstset{breaklines=true,breakatwhitespace=true,frame=leftline,captionpos=b,xleftmargin=6ex,tabsize=4,numbers=left,numberstyle=\ttfamily\footnotesize,basicstyle=\ttfamily\footnotesize}
\sloppy
\setlength{\parindent}{0em}
@ -28,6 +26,7 @@ citestyle=ieee
\addbibresource{sem.bib}
\begin{document}
\hyphenation{in-te-res-sant in-te-res-san-te}
\newpage
\thispagestyle{empty}
@ -56,31 +55,27 @@ Betreuer: Hannes Federrath \par
\newpage
\section{Vorbemerkung}
Problem: VPN (Virtual Private Network) aufsetzen
Relevanz: sichere Kommunikation zwischen zwei privaten Netzwerken
Mit den Enthüllungen von Edward Snowden hat sichere Kommunikation eine ganz neue Bedeutung bekommen. War es vorher hauptsächlich für Informatiker und Unternehmen von Interesse, so ist sichere Kommunikation mittlerweile im Bewusstsein der meisten Menschen angekommen. Doch wie sieht sichere Kommunikation überhaupt aus? Eine Möglichkeit ist die Verschlüsselung von E-Mails zur Sicherung der darin enthaltenen Korrespondenz. Dieses zugegeben sehr interessante Themengebiet wird uns aber in diesem Paper nicht beschäftigen. Stattdessen schauen wir uns die sichere Kommunikation zwischen zwei Rechnern an. Diese ist nämlich im Gegensatz zu E-Mail-Verschlüsselung nicht so offensichtlich und kann auf unterschiedliche Weise realisiert werden. Bei räumlich nicht weit entfernten Rechnern kann die Kommunikation über ein LAN-Kabel und nicht über das Internet erfolgen. Damit ist die Kommunikation solange sicher, wie die beiden Rechner und die Verbindung physisch sicher sind. Doch in den meisten Fällen stehen die beiden Rechner so weit voneinander entfernt, dass die Kommunikation über das Internet läuft. Doch spätestens am Router haben wir keine Kontrolle mehr über die Verbindungsstrecke und die Gegenstelle hat erst wieder am dem Router eine Kontrolle über die Verbindung. Wie also die unkontrollierbare Zwischenstrecke nutzen und gleichzeitig sicher kommunizieren? Geht das überhaupt? Die Lösung ist, wie bei E-Mail-Verschlüsselung, eine Ende-zu-Ende-Verschlüsselung, wobei das Ende hier nicht zwei Mailprogramme sind, sondern zwei Rechner.
Eine solche durch Verschlüsselung gesicherte Kommunikation zwischen zwei Rechnern über ein unsicheres Zwischennetz wird auch Virtual Private Network oder kurz VPN genannt. In diesem Paper werden wir uns mit zwei VPN-Lösungen beschäftigen: IPSec und OpenVPN. Im nächsten Abschnitt werden zunächst ein paar Grundbegriffe geklärt, um dann genauer in IPSec und anschließend in OpenVPN einzusteigen. Diesen beiden Vorstellungen folgt ein Vergleich, um dann mit den Schlussbemerkungen zu schließen.
\section{Grundlagen}
Was sind VPNs? Warum braucht man sie? Wozu werden sie verwendet?
Was ist das OSI-Referenzmodell? Wie ist es aufgebaut?
%%%%%%%%%%%%%%%%%%%%%%%%%%%%%%%%%%%%%%%%%%%%%%%%%%%%%%%%%%%%%%%55
\subsection{Begrifferklärung: VPN}
VPN ist eine Abkürzung für Virtual Private Network. Was ein private network, also ein privates Netwerk ist, sollte klar sein. Man kann kontrollieren wer auf dieses Netzwerk einen Zugriff haben soll und die Anzahl der Menschen die Zugriff haben ist außerdem örtlich begrenzt. Das heißt also der Vorteil eines privaten Netzwerkes ist, dass man einfach kontrollieren kann wer Zugriff haben soll, und dass der Datenaustausch nicht nach außen dringt.
Ein VPN ist eine gesicherte Kommunikationsverbindung zwischen zwei privaten Netzwerken. Das in der Vorbemerkung genannte Beispiel mit den zwei Rechnern ist eine Verkürzung, wenngleich sie korrekt ist. Der Internetzugang wird in den allermeisten Fällen durch Router sichergestellt. Diese sind selber kleine Computer und stehen im Internet repräsentativ für das Netzwerk, dem sie angehören. Daher passt dann auch wieder das in der Vorbemerkung genannte Beispiel mit den zwei Rechnern.
Nun gibt es aber Fälle in denen man Zugriff auf ein privates Netzwerk und dessen Ressourcen haben möchte, obwohl man nicht vor Ort ist. Ein gutes Beispiel ist hier der Zugriff auf das Netzwerk vom Arbeitsplatz, da dies häufig benötigt wird, wenn man von zu Hause aus arbeiten möchte.
Die VPN-Verbindung zwischen zwei privaten Netzwerken funktioniert in einer Weise, dass beide Netzwerke so miteinander verbunden werden, dass sie für Rechner in einer der beiden Netze wie ein großes privates Netz wirken. Daher ist auch der Zugriff auf die Rechner des anderen Netzes mit ihren privaten IP-Adressen (Adressen aus dem nicht-öffentlichen Bereich, meistens mit 192.168 beginnend) möglich. Dabei ist der Weg, den die IP-Pakete von einem Netz zum anderen nehmen, für die Rechner auf beiden Seiten irrelevant.
Ein VPN kann nun genau dafür verwendet werden. Um das ganze noch einmal zu verbildlichen, kann man sich das nun als ein sehr langes LAN-Kabel vorstellen. Es wird also eine Verbindung über das Internet mit dem privaten Netzwerk hergestellt. Verschiedene Aspekte eines privaten Netzwerkes können dabei unterschiedlich erreicht werden. Die Kontrolle wer auf das Netzwerk Zugriff hat kann z.B. erreicht werden, indem man die Identität beim Verbinden überprüft und unerwünschten Menschen den Zutritt verwehrt. So kann der Nutzer identifiziert werden und Fremde werden ausgeschlossen. Da die Daten nun über das Internet ausgetauscht werden, ist die Vertraulichkeit der Daten natürlich bedroht, was aber durch die Verschlüsselung dieser verhindert werden kann.
VPNs werden für verschiedenste Dinge eingesetzt. Bei Unternehmen kommen sie häufig zum Einsatz, um Inhalte aus dem Intranet auch von zu Hause oder unterwegs aus abrufen zu können. Diese Verwendung macht sich zunutze, dass die IP-Pakete mit der IP-Adresse der Endstelle des VPNs unterwegs sind (in diesem Fall der Router im Netzwerk des Unternehmens. Für z.B. den Webserver scheint es so, als ob man sich innerhalb des Netzwerks befindet, obgleich man physisch einige hundert Kilometer entfernt sein kann.
Durch dieses verschiedenen Maßnahmen hat man also die Vorteile eines privaten Netzwerkes Orts-unabhängig.
Diese gleiche Eigenschaft wird auch genutzt, um IP-Sperren zu umgehen. Viele Streamingdienste bieten unterschiedliche Produkte je geographischer Region an. Die Separierung geschieht durch die Kontrolle der IP-Adressräume, da jede geographische Region gewisse IP-Adressräume zur Verfügung hat. Mithilfe des VPNs kann nun bspw. von Europa aus eine Verbindung in die USA aufgebaut werden, sodass auf die USA beschränkte Angebote plötzlich sichtbar werden. Eine verwandte Anwendung ist die Umgehung von staatlichen Zensurmaßnahmen, in dem durch eine VPN-Verbindung aus dem abgeschotteten Netz "`ausgebrochen"' werden kann. Allerdings hängt der Erfolg davon stark von den staatlichen Gegenmaßnahmen ab. Ist VPN-Verkehr als solcher zu erkennen, dann kann dieser gezielt geblockt werden, auch wenn der Inhalt nicht erkannt wird.
Es gibt aber noch viel mehr Einsatzmöglichkeiten.
VPN werden oft benutzt, um den eigenen Traffic im Internet zu verschleiern. Ist man mit einem VPN verbunden, so können die Daten, die man aus dem Internet bezieht zunächst zum privaten Netzwerk geleitet werden und dann erst zum eigentlichen Ziel. So erreicht man, dass für die Außenwelt nur IP-Adresse des privaten Netzwerks sichtbar ist, nicht aber die eigene.
Diesen Umstand kann man beispielsweise auch für Services nutzen, die nur in bestimmten Ländern verfügbar sind, z.B. verschiedene Musik- und Video-Streamingdienste. Eine VPN-Verbindung in das entsprechende Land, kann diese Sperre umgehen.
IPSec und OpenVPN sind zwei Lösungen, um ein solches Netzwerk zu errichten. Im Folgenden werden wir näher darauf eingehen, wie dies jeweils technisch umgesetzt ist.
Es ist erkennbar, dass es eine Reihe von Anwendungsmöglichkeiten gibt. Die konkrete Umsetzung der VPN-Verbindung hängt jedoch stark von der verwendeten Technologie, in unserem Fall IPSec und OpenVPN ab. Im Folgenden daher eine Vorstellung von IPSec und OpenVPN.
\section{IPSec}
%TODO Referenzen
\subsection{Entstehungsgeschichte}
IPSec ist unter der Aufsicht eines Komitees entstanden. Diese Vorgehensweise wurde dabei gezwungenermaßen auferlegt und erschwerte nach Ansicht vieler Mitwirkender die Arbeit erheblich. \footnote{Ferguson, Niels, and Bruce Schneier. ``A cryptographic evaluation of IPsec."' Counterpane Internet Security, Inc 3031 (2000).}
Die Entstehungsweise führte dazu, dass IPSec unnötig kompliziert wurde. Die einen Mitglieder wollten die eine Lösung implementieren und andere plädierten für eine andere Lösung, was dazu führte, dass beide Lösungen implementiert wurden. Dies geht jedoch zu Lasten der Nutzbarkeit. Zu viele Optionen führen nicht nur zur Verwirrung und Komplizierung der Implementation von IPSec, es kann vor allem auch zu Sicherheitslücken führen. Experten bemängelten diese Komplexität.
Die Entstehungsweise führte dazu, dass IPSec unnötig kompliziert wurde. Die einen Mitglieder wollten die eine Lösung implementieren und andere plädierten für eine andere Lösung, was dazu führte, dass beide Lösungen implementiert wurden. Dies geht jedoch zu Lasten der Nutzbarkeit. Zu viele Optionen führen nicht nur zur Verwirrung und Komplizierung der Implementation von IPSec bei den Nutzern, es kann vor allem auch zu Sicherheitslücken führen. Experten bemängelten diese Komplexität.
Die erste Version von IPSec ist 1995\cite{RFC1825} spezifiziert worden. Doch über die Jahre ist der Standard immer weiter entwickelt worden und so gibt es eine zweite Version aus 1998\cite{RFC2401} und eine dritte aus 2005\cite{RFC4301}. Diese gelten zwar allgemein als V2 und V3, jedoch sind verschiedene Aspekte von IPSec in verschiedenen sogenannten ``Request for Comments"', kurz RFC, spezifiziert. So gibt es beispielsweise ein RFC, dass sich allgemein auf IPSec bezieht, aktuell RFC 4301 aus 2005, und es gibt beispielsweise ein RFC, dass sich auf die zu verwendenden Verschlüsselungsalgorithmen bezieht, aktuell RFC 7321 bereits aus 2014. So ist gewährleistet, dass IPSec weiterhin auf dem neuesten Stand gehalten werden kann, ohne dass man jedes mal die komplette Spezifikation überarbeiten muss.
@ -93,7 +88,7 @@ Die folgenden RFCs sind für IPSec relevant:
\item RFC 5996: Beschreibt das Internet Key Exchange Protocol Version 2 (IKEv2)\cite{RFC5996}
\item RFC 4307: Gibt vor welche Algorithmen für IKEv2 verwendet werden sollten\cite{RFC4307}
\end{itemize}
Es gibt noch weitere RFCs, die man IPSec zuordnen kann, doch dies sind die Wichtigsten. Im folgenden Abschnitt werden wir näher auf die technischen Aspekte von IPSec eingehen und die Themen der einzelnen RFCs näher betrachten.
Es gibt noch viel mehr RFCs die man IPSec zuordnen kann, doch dies sind die wichtigsten. Im folgenden Abschnitt werden wir näher auf die technischen Aspekte von IPSec eingehen und die Themen der einzelnen RFCs näher betrachten.
\subsection{Technischer Aufbau}
\subsubsection{Tunnel Modus und Transport Modus}
@ -152,22 +147,25 @@ Nachfolgende Kommunikation bezüglich IKE wird CREATE\_CHILD\_SA genannt und kan
%%%%%%%%%%%%%%%%%%%%%%%%%%%%%%%%%%%%%%%%%%%%%%%%%%%%%%%%%%%%%%%%%%%%%
\section{OpenVPN}
%\input{VPN}
%TODO insgesamt gesehen: Referenzen
Was ist OpenVPN? Wer ist dafür verantwortlich? Wie funktioniert es?
\subsection{Was ist ein VPN?}
Man stellt sich unter einem Netzwerk immer Verbindungen zwischen einzelnen Computern in Form von Verkabelungen vor. Ein VPN verbindet hingegen Rechner oder ganze Netzwerke über bereits vorhandene Netzwerke hinweg.
Virtuelle Private Netzwerke (VPN) erlauben eine sichere, stabile und preisgünstige Kommunikation über das Internet. Mit ihrer Hilfe können verteilte Unternehmensnetze verbunden werden oder Außendienstmitarbeiter auf Ressourcen und Daten in dem Unternehmensnetz zugreifen. Sie bieten eine kostengünstige und sichere Anbindung von Filialen an eine Zentrale und erlauben den Einsatz von Telearbeitsplätzen, bei denen die Angestellten ihre Arbeit zu Hause erledigen.
\subsection{Was ist OpenVPN?}
%TODO hier müsste eher stehen was OpenVPN technisch gesehen ist
Beispielsweise:
Open VPN ist eine VPN-Lösung, die TLS-basiert ist und im OSI-Referenzmodell in der Anwendungsschicht angesiedelt ist. Unter den TLS-basierten VPN-Implementationen ist Open VPN eine sehr weit verbreitete Implementation, die plattformübergreifend verfügbar ist.
\subsection{Was ist ein OpenVPN?}
%TODO auch interessant wäre (gerade im Gegensatz zu IPSec), wer für Open VPN verantwortlich zeichnet (Komitee, Firma, Entwicklercommunity)
Open VPN ist eine VPN-Lösung, die unter Linux, Windows und vielen weiteren Betriebssystemen eingesetzt werden kann.
Open VPN ist eine freie Software und steht unter der GPL ( GNU General Public License ).
\subsection{Verschlüsselung bei OpenVPN}
%TODO auf TLS eingehen in technischer Sicht -> Wie funktioniert die Verschlüsselung mit TLS (z.B. Handshake)
OpenVPN benutzt zur Authentifizierung und Verschlüsselung der Daten kryptografische Verfahren auf Basis von TLS (Transport Layer Security). TLS ist ein Verschlüsselungsprotokoll zur Datenübertragung und der Nachfolger vom bekannteren SSL (Secure Sockets Layer).Die meisten Internetbrowser unterstützen TLS-Verbindungen. Dieses Protokoll wird im Internet u.a. zum Aufbau von sicheren Verbindungen (HTTPS-Verbindungen) bei Banken oder Shopsystemen eingesetzt. Daran erkennt man , dass dies allgemein anerkannte und vor allem auch ausgereifte Standardverfahren sind.
Da OpenVPN diese Verfahren benutzt, kann kein IPSec eingesetzt werden! Dies bedeutet, dass man die Windows-internen Funktionen zum Herstellen einer Verbindung nicht verwenden kann. Das können wir als Nachteil, aber auch als Vorteil sehen. Der Nachteil ist, dass wir auf jedem Rechner OpenVPN instalieren müssen und mit Betriebssystemen, für die es nicht zur Verfügung steht, keine Verbindung herstellen können.Der Vorteil ist, dass wir sich nicht wirklich mit dem Einrichten von VPNs auf "fremden" Systemen auskennen müssen. Es wird ja immer OpenVPN benutzt. Weiterhin ist man auch nicht von eventuell auftretenden Problemen bei Updates des Systems betroffen.
Da OpenVPN diese Verfahren benutzt, kann kein IPSec eingesetzt werden! Dies bedeutet, dass man die Windows-internen Funktionen zum Herstellen einer Verbindung nicht verwenden kann. Das können wir als Nachteil, aber auch als Vorteil sehen. Der Nachteil ist, dass wir auf jedem Rechner OpenVPN instalieren müssen und mit Betriebssystemen, für die es nicht zur Verfügung steht, keine Verbindung herstellen können.Der Vorteil ist, dass wir sich nicht wirklich mit dem Einrichten von VPNs auf fremden Systemen auskennen müssen. Es wird ja immer OpenVPN benutzt. Weiterhin ist man auch nicht von eventuell auftretenden Problemen bei Updates des Systems betroffen.
Zur Authentifizierung können statische Schlüssel (Preshared Keys) oder Zertifikate benutzt werden. Statische Schlüssel gelten im Vergleich mit Zertifikaten als weniger sicher, sind aber einfacher zu erstellen und zu verwalten. Zertifikate kann man sich selbst ausstellen oder bei so genannten Zertifizierungsstellen beantragen. Für Firmen ist dies allerdings meist mit Kosten verbunden. Der Vorteil von Zertifizierungsstellen ist, dass die Zertifikate weltweit einsetzbar sind und ihre Echtheit von diesen Stellen bestätigt wird. Zum Aufbau von firmeninternen Verbindungen sind diese Echtheitsbestätigungen aber nicht zwingend erforderlich.
@ -189,9 +187,6 @@ Im Routing-Modus wird eine Verbindung zwischen zwei Gegenstellen über das IP-Pr
Beim Bridging-Modus werden zwei Netzwerke miteinander verbunden. Hierbei spielt das benutzte Protokoll der Netzwerke keine Rolle. Allerdings wird auch der komplette Netzwerkverkehr übermittelt. Daher hat man im Gegensatz zum Routing-Modus einen höheren Datendurchsatz.
\section{Vergleich von IPSec mit OpenVPN}
In den vorangegangenen Abschnitten wurden IPSec und OpenVPN vorgestellt. In diesem Abschnitt widmen wir uns dem Vergleich der beiden. Damit ein solcher Vergleich jedoch sinnvoll stattfinden kann, muss im Voraus klar sein, was verglichen wird. Wir benötigen daher Kriterien. Im Folgenden werden die Kriterien erarbeitet und erläutert, warum wir das jeweilige Kriterium für wichtig befinden.
@ -226,7 +221,10 @@ Ein weiterer Punkt bei der Vertraulichkeit ist die Authentifizierungsmethode. IP
Bei den Authentifizierungsalgorithmen hat jedoch IPSec einen Vorteil, denn es wird sowohl die Nutzung einer digitalen Signatur als auch die Verwendung eines Algorithmus, der auf einem geheimen Schlüssel basiert, ermöglicht. TLS benötigt hingegen zwangsweise eine digitale Signatur. Sollten alle Algorithmen mit digitaler Signatur ausfallen, kann IPSec weiterhin implementiert werden, TLS jedoch nicht.\cite{Alshamsi2005}
Zusammenfassend kann für den Bereich der Vertraulichkeit gesagt werden, dass sowohl IPSec als auch TLS hinreichende Vertraulichkeit bieten.
Abseits der Verschlüsselung selber ist jedoch auch die Reichweite der Verschlüsselung von Interesse. Hier vergleichen wir IPSec mit OpenVPN. Bei IPSec werden die Pakete verschlüsselt, nicht jedoch ihr Inhalt. Werden zwei Netze verbunden, so ist nur der VPN-Tunnel selber vor einem Angriff geschützt. Aber der Weg von der Anwendung zum VPN-Tunnel und vom VPN-Tunnel zur Anwendung ist nicht geschützt. Sobald sich also ein Angreifer innerhalb eines der beiden Netzwerke befindet, ist IPSec ohne Schutzwirkung.
Im Gegensatz dazu verschlüsselt OpenVPN von Anwendung zu Anwendung und damit wirklich Ende-zu-Ende. Da ist es dann auch nicht mehr wichtig, ob sich der Angreifer im Netzwerk befindet, da er nur verschlüsselte Daten sehen kann.\cite{Sun2011}
Zusammenfassend kann für den Bereich der Vertraulichkeit gesagt werden, dass sowohl IPSec als auch TLS hinreichende Vertraulichkeit bieten. Allerdings unterliegt IPSec bei der Effizienz der Vertraulichkeit ganz klar OpenVPN.
\subsection{Integrität}
Neben der Vertraulichkeit ist auch die Integrität wichtig. Daher werden wir im folgenden Abschnitt vergleichen, welche Methoden von IPSec und OpenVPN zur Wahrung der Integrität verwendet werden.
@ -240,21 +238,15 @@ Als Fazit für die Integrität kann gelten, dass es keine wirklichen Unterschied
\subsection{Verfügbarkeit}
Das dritte Schutzziel ist Verfügbarkeit. Traditionell fallen Dinge wie DDOS-Angriffe\footnote{Distributed-Denial-of-Service-Angriffe} unter dieses Schutzziel bzw. beeinträchtigen es. In unserem Zusammenhang ist die Durchlässigkeit bei Firewalls von Interesse, also ob ein VPN durch eine der beiden Technologien gezielt geblockt werden kann. Auch die Verbreitung spielt eine Rolle, denn eine weit verbreitete Technologie macht es intuitiv gesehen erst einmal "`einfacher"' eine Verbindung zwischen zwei Netzen aufzubauen.
Beginnen wir bei Firewalls. Dort gilt, dass IPSec in geringem Maße im Nachteil ist, da es feste Protokolle und Ports benötigt. Ein gezieltes Blocken von IPSec-basierten VPNs ist daher praktikabel. Im Vergleich dazu kann OpenVPN auf jedem Port mit UDP oder TCP laufen. Um restriktive Firewalls zu überwinden, kann der Port 443 und TCP benutzt werden. Dadurch sieht der Traffic von OpenVPN wie normaler HTTPS-Traffic aus und kann nicht gezielt gefiltert werden.\cite{ivpn.net}
Beginnen wir bei Firewalls. Dort gilt, dass IPSec in geringem Maße im Nachteil ist, da es feste Protokolle und Ports benötigt. Ein gezieltes Blocken von IPSec-basierten VPNs ist daher praktikabel. Im Vergleich dazu kann OpenVPN auf jedem Port mit UDP oder TCP laufen. Um restriktive Firewalls zu überwinden, kann der Port 443 und TCP benutzt werden. Dadurch sieht der Traffic von OpenVPN wie normaler HTTPS-Traffic aus und kann nicht gezielt gefiltert werden.\cite{Sun2011}
Bei der Verbreitung hat IPSec wiederum einen leichten Vorsprung, denn die meisten Betriebssysteme unterstützen es nativ. Für OpenVPN muss hingegen jeweils ein Client installiert werden. Die Form des Client unterscheidet sich je nach Betriebssystem und ist in den meisten Fällen innerhalb von 5 Minuten erledigt.\cite{ivpn.net}
Bei der Verfügbarkeit auf unterschiedlichen Betriebssystemen und Implementationen ist zu beachten, wofür das VPN eingesetzt wird. Wenn es für eine Remoteverbindung genutzt wird, dann liegt IPSec hinten, da es auf beiden Geräten der Verbindung einen entsprechenden Client benötigt. OpenVPN bedarf nur der Einrichtung in dem privaten Netz, auf das man zugreifen möchte. Für den Remotezugriff muss keine extra Software installiert werden. Wird das VPN stattdessen für eine feste Verbindung zwischen zwei Endpunkten mit statischen IP-Adressen verwendet (z.B. zwei Router), dann ist IPSec aufgrund der Standardisierung und vielen Möglichkeiten im Vorteil.\cite{Sun2011}
Für die Verwendung hinter hochrestriktiven Firewalls ist OpenVPN im Vorteil, da es die zu 99\% immer offenen Ports 80 und 443 mitbenutzen kann und somit nicht von normalem Browsertraffic zu unterscheiden ist. Bei allen Nutzungsszenarien spielt es jedoch keine Rolle, ob IPSec oder OpenVPN eingesetzt werden.
Vor diesem Hintergrund gewinnte OpenVPN eindeutig das Rennen, wenn es um Remoteverbindungen geht. Bei Verbindungen zwischen zwei festen Endpunkten hat IPSec jedoch die Nase vorn, da es beidseitig hohe Standards voraussetzt.
\subsection{Performance}
Nachdem wir IPSec und OpenVPN bezüglich der Schutzziele verglichen haben, widmen wir uns jetzt der Performance.
Dabei beziehen wir uns auf den bereits genannten Vergleich von Kotuliak\cite{Kotuliak2011}. Zum Vergleich wurden zwei Rechner genommen, die über ein 1 GBit Ethernet verbunden waren. Auf beiden Rechnern lief Windows Vista. Zusätzlich lief auf beiden eine virtuelle Maschine von Debian mit der VM-Software VMware. Die beiden virtuellen Maschinen waren mit Ausnahme der IP und des Rechnernamens identisch.
Aufgrund des höheren Rechenaufwands durch eine virtuelle Maschine können minimale Unterschiede besser erkannt werden. Diese Unterschiede sind vor allem technischer Natur: IPSec benötigt einen low-level Zugriff auf das Netzwerkinterface, um die Pakete zu manipulieren. Dafür operiert IPSec im Ring0 des Kernel-Space.\cite{Kotuliak2011}
Der Kernel-Space ist in Ringe unterteilt, wobei Ring0 für den Kernel und andere essentielle Prozesse reserviert ist. Ring1 ist für andere Systemprozesse reserviert, die low-level Zugriff auf Hardware benötigen. Je weiter man sich von Ring0 entfernt, desto weniger Privilegien haben die Prozesse. In Ring3 findet man schließlich die meisten Userprozesse, u.a. auch TLS-Implementationen. Prozesse in Ringen mit höheren Nummern können Prozesse aus Ringen mit niedrigeren Ringen nicht stören.\cite{Kotuliak2011}
Mit diesem technischen Hintergrundwissen ausgestattet, können wir die Ergebnisse des Vergleichs betrachten. Der durchschnittliche Durchsatz kann in \fref{tab:tp} gesehen werden. Es fällt auf, dass der Durchsatz ohne Verschlüsselung weit über dem der Verschlüsselung liegt. Allerdings ist das auch nicht anders zu erwarten. Zum Verständnis ist jedoch noch wichtig, dass die verwendete Ethernetverbindung nicht für die Verwendung von Jumbo-Frames eingerichtet wurde. Ohne diese Frames ist solch eine Gigabit-Verbindung jedoch erheblich langsamer als der theoretische Wert (1 GBit/s).\cite{Kotuliak2011}
Eine weitere Auffälligkeit ist die im Vergleich zu den übrigen Algorithmen schlechte Performance von 3DES. Dies liegt schlichtweg daran, dass 3DES ein veraltetes Verfahren ist und nicht weiter benutzt werden sollte. Bei diesem konkreten Verfahren ist IPSec (45 Mbps) langsamer als OpenVPN (60,98 Mbps), allerdings hat das kaum eine Relevanz, da 3DES wie gesagt veraltet ist.
@ -278,6 +270,7 @@ IPSec AES & 142 \\
IPSec Blowfish & 121,76
\end{tabular}
\end{table}
\subsection{Kompatibilität}
Abschließend gehen wir auf die Kompatibilität der Implementationen verschiedener Hersteller untereinander ein. Uns war es nicht möglich eigene Vergleiche hinsichtlich der Kompatibilität durchzuführen, daher werden wir nur oberflächlich darauf eingehen.
@ -288,7 +281,11 @@ Bei IPSec sieht das anders aus. Zwar ist auch IPSec standardisiert, ebenso wie T
Bezüglich der Kompatibilität kommen wir zu dem Fazit, dass IPSec noch im Vergleich zu TLS aufholen muss, allerdings bereits viele der anfänglichen Kompatibilitätsprobleme überwunden hat.
\section{Schlussbemerkungen}
Ausblick: (weitere) Vereinfachung von IPSec?, unterschiedliche Ansätze, Vor- und Nachteile bei beiden
Aus dem Vergleich geht hervor, dass IPSec und OpenVPN abhängig von dem Verwendungsszenario des VPN unterschiedlich gut abschneiden. Für Remoteverbindungen zwischen einem einzelnen Rechner und einem Netzwerk ist OpenVPN die bessere Wahl, da Ende-zu-Ende verschlüsselt wird, obgleich die Verschlüsselung selber bei IPSec und OpenVPN sich nicht viel gibt. Im Bereich der Performance hat IPSec aufgrund der Kernelnähe die Nase vorn. Im Bereich der Kompatibilität bestätigt IPSec nur zum Teil das Vorurteil der Inkompatibilität. Gleichwohl gibt es noch Aufholpotential.
Für die Zukunft können wir uns vorstellen, dass es eine weitere Überarbeitung von IPSec geben wird, um es leichter verständlich zu machen. Außerdem sind wir der Meinung, dass sich die teilweise noch bestehenden Kompatibilitätsprobleme auflösen werden, sodass es für eine immer größere Menge von Leuten möglich sein wird sichere Verbindungen zwischen zwei Netzwerken aufzubauen.
Aus wissenschaftlicher Sicht wäre ein umfassender Vergleich zwischen allen existierenden VPN-Lösungen wünschenswert, um für ein komplettes Bild nicht auf teilweise bereits veraltete Daten zurückgreifen zu müssen. Darüber hinaus wäre eine zentrale Anlaufstelle hilfreich, über die entsprechende Forschung koordiniert wird, um doppelte Arbeit zu vermeiden.
%%%%%%%%%%%%%%%%%%%%%%%%%%%%%%%%%%%%%%%%%%%%%%%%%%%%%%%%%%%%%%%%%%%%%%
\newpage

View File

@ -5,10 +5,11 @@
@Inproceedings{Alshamsi2005,
Title = {A {T}echnical {C}omparison of {IPSec} and {SSL}},
Author = {Alshamsi, AbdelNasir and Saito, Takamichi},
Booktitle = {Proceedings. 19th International Conference on Advanced Information Networking and Applications},
Year = {2005},
Month = {Mar},
Booktitle = {19th International Conference on Advanced Information Networking and Applications},
Date = {2005-03-28/2005-03-30},
Organization = {IEEE},
Pages = {395--398},
Volume = {2},
Owner = {jim},
Timestamp = {2014.10.24}
@ -445,14 +446,8 @@
Pages = {217--221},
Abstract = {IPSec is faster than OpenVPN with the same setup and cipher.
Details:
IPSec AES > OpenVPN AES
IPSec Blowfish > OpenVPN Blowfish
IPSec 3DES < OpenVPN 3DES
Details: IPSec AES > OpenVPN AES IPSec Blowfish > OpenVPN Blowfish IPSec 3DES < OpenVPN 3DES
3DES is a cipher of the past and should not be used anymore. AES and Blowfish have similar results under IPSec and OpenVPN. AES is standardized and has more support than Blowfish.
IPSec however is far more complex and difficult to set up while setting up OpenVPN is child's play.},
Owner = {jim},
Timestamp = {2014.11.28}
@ -535,7 +530,7 @@ IPSec however is far more complex and difficult to set up while setting up OpenV
Title = {Artificial intelligence: A Modern Approach},
Author = {Russel, Stuart J. and Norvig, Peter},
Booktitle = {Artificial intelligence: A Modern Approach},
Date = {December 11},
Date = {2009-12-11},
Year = {2009},
Bookauthor = {Russel, Stuart J. and Norvig, Peter},
Chapter = {23},
@ -587,7 +582,7 @@ IPSec however is far more complex and difficult to set up while setting up OpenV
Title = {Dependency Parsing by Belief Propagation},
Author = {Smith, David A. and Eisner, Jason},
Booktitle = {Conference on Empirical Methods in Natural Language Processing},
Date = {October 25 - October 27},
Date = {2008-10-25/2008-10-27},
Year = {2008},
Pages = {145-156},
@ -596,6 +591,22 @@ IPSec however is far more complex and difficult to set up while setting up OpenV
Timestamp = {2013.10.29}
}
@Inproceedings{Sun2011,
Title = {The advantages and the implementation of {SSL} {VPN}},
Author = {Sun, Su Hua},
Booktitle = {2nd {I}nternational {C}onference on Software {E}ngineering and {S}ervice {S}cience (ICSESS)},
Date = {2011-07-15/2011-07-17},
Location = {Beijing},
Organization = {IEEE},
Pages = {548--551},
Abstract = {SSL/TLS VPN is the better choice for remote access to a private network, while IPSec is better for the connection between two fixed endpoints.
With IPSec the data is transported unencrypted between the application and the VPN start point and from the VPN end point to the application. In a remote access example, all the data would be readable from inside the target network. Furthermore an attacker only needs access to the client computer and can then use the connection, without being authenticated. With SSL VPN the encryption is application to application and the authentication and authorization happens for each connection. Access to a connection is unequally more difficult than access to a computer.},
Owner = {jim},
Timestamp = {2014.12.13}
}
@Article{Venkateswaran2001,
Title = {Virtual Private Networks},
Author = {Venkateswaran, R.},

View File

@ -52,18 +52,38 @@ Studiengang BSc. Informatik
\newpage
\section{Vorbemerkung}
Aus den Haager Friedenskonferenzen ging die Haager Landkriegsordnung hervor. Die Konferenzen gelten als Meilenstein in der Kodifikation von geltendem Kriegsvölkerrechts. Doch sie sind keineswegs eine plötzliche Erscheinung gewesen, sondern stehen am Ende eines Weges, der sich über mehrere Jahrhunderte erstreckt. In diesem Paper wird die Haager Landkriegsordnung hauptsächlich unter dem Punkt des Kombattantenstatus betrachtet. Denn die Unterscheidung zwischen rechtmäßig Kämpfenden und der unschuldigen Zivilbevölkerung ist Grundlage unseres heutigen westlichen Werteverständnisses.
Im Folgenden wird daher der historische Weg mit den wichtigsten Meilensteinen erläutert, um anschließend beide Konferenzen vorzustellen, wobei natürlich auch die übrigen Regelungen, wie z.B. zu erlaubten Waffengattungen, Erwähnung finden werden. Beide Konferenzen werden schließlich unter dem Aspekt des Kombattantenstatus ausgewertet, um mit Schlussbemerkungen zu schließen.
\section{Weg zur Friedenskonferenz}
Der Weg zu den Konferenzen orientiert sich an \cite{Buss1992}.
\section{Weg zu den Friedenskonferenzen}
Der Weg zu den Haager Friedenskonferenzen beginnt nicht erst im 19. Jahrhundert, sondern bereits weit früher im Mittelalter. Eine Kodifikation des Kriegsrechts setzt voraus, dass die Einhaltung dieses Rechts auch sichergestellt werden kann. Dazu ist Disziplin in der kämpfenden Truppe notwendig, was wiederum andere Voraussetzungen hatte.
Die erste Möglichkeit einer solchen Kontrolle auf Einhaltung hatten die Römer mit ihren hochdisziplinierten Truppen. Allerdings wurde die Möglichkeit nicht genutzt. Die gegnerische Zivilbevölkerung war häufig genauso Ziel militärischer Handlungen wie die gegnerischen Soldaten. Ein paar hundert Jahre später gab es mit dem Rittertum ausgebildete Kämpfer, die jedoch Einzelkämpfer waren und keineswegs die Zivilbevölkerung schonten.
Erst mit den sogenannten Schweizer Haufen gelang eine Disziplinierung einer Armee, sodass Anweisungen von Vorgesetzten durchgesetzt werden konnten. So war die Plünderung und Schändung der Zivilbevölkerung (hauptsächlich Frauen und Kinder) untersagt.
Francisco de Vitoria hat im Jahre 1532 anhand der Kolonialisierung Amerikas durch die Spanier, die er für legal hielt, erklärt, dass ein Krieg für beide Parteien rechtmäßig sein kann. Zudem hält er fest, dass ein rechtmäßiger Krieg den Einsatz jeglicher Mittel erlaubt, um ihn schnellstmöglich siegreich zu beenden. Ist der Krieg jedoch beiderseitig gerechtfertigt, würde dies in einer Eskalation der Mittel enden, wodurch eine beiderseitige Mäßigung notwendig ist. Seine Ausführungen bilden zudem die Grundlage für die spätere Separierung zwischen Kämpfenden und der Zivilbevölkerung.
Der nächste große Schritt ereignete sich mit dem Unabhängigkeitskampf der späteren Niederlande gegen Spanien. Wilhelm von Oranien leitete dabei den Widerstand und schuf ein stehendes Söldnerheer, dass diszipliniert war, regelmäßig entlohnt wurde und als erstes Heer nach noch heute gültigen Grundsätzen aufgebaut war.
Einen Einschnitt gab es mit dem Dreißigjährigen Krieg, in dem überwiegend undisziplinierte Söldnerheere aufeinander trafen. Außerdem wurde überwiegend eine Ermattungsstrategie verfolgt, bei der offene Feldschlachten vermieden und eher Belagerungen und kleinere Gefechte ausgetragen wurden. Im Rahmen dieser Strategie dauerten Kriege auch länger und die Versorgung der Soldaten erfolgte meist aus dem Kampfgebiet. Durch diese kontinuierlichen Plünderungen zur Ernährung der Truppe war ein effektiver Schutz der Zivilbevölkerung, wie er zuvor teilweise in Kriegsartikeln bereits gefasst war, nicht möglich.
Während dieser Zeit hat Hugo Grotius ein dreibändiges Werk (De iure bella ac pacis) verfasst, in welchem er das "`freiwillige"' Völkerrecht in einer realistischen Weise darstellt und im dritten Buch auf ethische, moralische und religiöse Grundsätze hinweist. Obgleich dieses Rechtsbuch einen wichtigen Beitrag zum späteren Völkerrecht darstellt, so entspricht es doch in keinem Maße der Kodifikation in den Haager Konferenzen.
Eine Zäsur stellte der Westfälische Frieden von 1648 dar. Von da an wurden stehende Heere die Normalität und die Souveränität der einzelnen Staaten garantiert. Durch diese Entwicklung, die mit dem stehenden Heer der Oranier (spätere Utrechter Union und Niederlande) begann, wurde eine effektive Unterscheidung zwischen Kämpfenden und Zivilbevölkerung erstmals flächendeckend möglich. Dieser Frieden bildet daher in vielerlei Hinsicht die Grundlage dafür, dass 250 Jahre später die Haager Friedenskonferenzen stattfinden konnten.
In der Folgezeit gab es noch weitere Ereignisse, doch für den geschichtlichen Hintergrund sollte dies ausreichen. Im Folgenden wird eine zeitlich nähere Einordnung vorgenommen, die in der zweiten Hälfte 19. Jahrhundert verbleibt.
Der Lieber Code ist im Rahmen des Sezessionskriegs zwischen der USA und der Konföderation entstanden. Dieser ist in vielen Punkten speziell auf den amerikanischen Bürgerkrieg ausgerichtet, enthält aber eine detaillierte Auflistung dessen, was zu der Zeit als geltendes Kriegsrecht angenommen wurde. Er diente daher als Grundlage für die weiteren Verhandlungen in den kommenden Jahrzehnten.
Die Brüsseler Deklaration von 1874 schließlich beinhaltet die hauptsächliche Vorarbeit zu den Haager Friedenskonferenzen und stellt auch den letzten Schritt vor der ersten Konferenz dar, auf die im nächsten Abschnitt näher eingegangen wird.
\section{Erste Friedenskonferenz}
\begin{itemize}
\item Gründe für Zustandekommen\cite{Buss1992}
\item Begriff des Kombattant essentiell für westliches Werteverständnis
\item Entwicklung seit dem Mittelalter
\item zunehmende Kodifizierung des geltenden (Kriegs-)rechts seit dem 18. Jhd.
\item Brüsseler Konferenz 1870
\item Oxford Manual
\end{itemize}
\section{Erste Friedenskonferenz}
\begin{itemize}
\item Begriff des Kombattanten\cite{Buss1992}
\item Levée en masse\cite{Scott1920}
@ -84,7 +104,7 @@ Studiengang BSc. Informatik
\cite{Buss1992}
\cite{Fraenkel1968}
\cite{Heffter1951}
\cite{Reich2010}
\cite{DeutschesReich2010}
\cite{Scott1920}
\cite{Scott1921}

View File

@ -31,43 +31,20 @@
Title = {Der {K}ombattantenstatus},
Author = {Buß, Regina},
Year = {1992},
Chapter = {Die Haager Friedenskonferenzen},
Pages = {168--180},
Publisher = {Brockmeyer},
Series = {Bochumer Schriften zur Friedenssicherung und zum Humanitären Völkerrecht},
Abstract = {Oxford Manual:
Auf dem Weg zu den Haager Friedenskonferenzen nach der gescheiterten Brüsseler Deklaration von 1870 kam das sog. Oxford Manual heraus. Es wurde von einem Institut bestehend aus unabhängigen Wissenschaftlern entworfen und entsprach größtenteils den Vorstellungen der Großmächte. Es orientierte sich an der Praxis der europäischen Kriegführung und gestaltete die Bestimmungen der Brüsseler Deklaration dahingehend um. Das kontroverse Thema der Levée en masse wurde nahezu vollständig zugunsten der Großmächte ausgelegt. Nur in noch nicht besetzten Gebieten war es demnach gestattet, dass sich unorganisierte Menschenmengen erheben und gegen die Angreifer vorgehen. Laut dem Oxford Manual hatten die Bewohner der besetzten Gebiete eine Gehorsamspflicht, was notwendigerweise das Recht zur bewaffneten Gegenwehr ausschließt.
Vorarbeiten zur ersten Haager Friedenskonferenz:
Der russische Außenminister Graf Mouravieff lud im Auftrag von Zar Nikolaus II. zusätzlich zu den in Petersburg diplomatisch vertretenen Nationen auch Luxemburg, Montenegro und Siam ein. Alle Länder nahmen die Einladung ein. Der Kreis der Teilnehmer war mit 26 Teilnehmern damit doppelt so groß, wie bei der letzten Konferenz in Brüssel.
Teilnehmer:
- Deutschland, England, Frankreich, Italien, Österreich-Ungarn, Russland
- Belgien, Bulgarien, Dänemark, Griechenland, Luxemburg, Montenegro, die Niederlande, Portugal, Rumänien, Schweden-Norwegen, die Schweiz, Serbien, Spanien und die Türkei
- China, Japan, Persien, Siam
- USA und Mexiko
Kombattant in der Ersten Haager Landkriegsordnung:
Hauptziel der Konferenz war die Entwicklung von Instrumenten zur friedlichen Streitbeilegung und Verhinderung von Kriegen. Rüstungsbeschränkung war daher auch ein Ziel ("Abrüstungskonferenz"). Konferenz begann 18. Mai 1899 in Haag.
Levée en masse:
Dieser strittige Punkt war der Hauptgrund für das Scheitern von der Brüsseler Konferenz und auch die Haager Friedenskonferenz drohte daran zu scheitern. Der Punkt wurde in der 2. Kommission und dort in der 2. Unterkommission behandelt. Am 20. Juni 1899 hat der deutsche Vertreter, von Schwarzhoff, eine mittlerweile berühmte Rede gehalten. Seine Kernpunkte waren, dass Artikel 9 und 10 (beschäftigen sich mit der Levée en masse) keineswegs die Möglichkeit des Patriotismus und der Abwehr angreifender Truppen ausschließen. Es müsse lediglich sichergestellt werden, dass sich die Erhebenden klar als Kämpfer zu erkennen geben, einen Kommandeur haben, ihre Waffen offen tragen, sich an die Rechte des Kriegs halten und ein gemeinsames Zeichen haben. Ferner erläuterte Schwarzhoff, dass Soldaten auch Menschen seien und daher Humanität erwarten dürften. Wenn Soldaten, erschöpft von einem langen Marsch, in einem Dorf Rast machen, dann müssten sie sicher sein können, dass sich die friedlichen Passanten nicht unerwarteter Weise zu Angreifern wandelten. Der russische und niederländische Vertreter schlossen sich dieser Ausführung an.
Abstract = {Oxford Manual: Auf dem Weg zu den Haager Friedenskonferenzen nach der gescheiterten Brüsseler Deklaration von 1870 kam das sog. Oxford Manual heraus. Es wurde von einem Institut bestehend aus unabhängigen Wissenschaftlern entworfen und entsprach größtenteils den Vorstellungen der Großmächte. Es orientierte sich an der Praxis der europäischen Kriegführung und gestaltete die Bestimmungen der Brüsseler Deklaration dahingehend um. Das kontroverse Thema der Levée en masse wurde nahezu vollständig zugunsten der Großmächte ausgelegt. Nur in noch nicht besetzten Gebieten war es demnach gestattet, dass sich unorganisierte Menschenmengen erheben und gegen die Angreifer vorgehen. Laut dem Oxford Manual hatten die Bewohner der besetzten Gebiete eine Gehorsamspflicht, was notwendigerweise das Recht zur bewaffneten Gegenwehr ausschließt.
Vorarbeiten zur ersten Haager Friedenskonferenz: Der russische Außenminister Graf Mouravieff lud im Auftrag von Zar Nikolaus II. zusätzlich zu den in Petersburg diplomatisch vertretenen Nationen auch Luxemburg, Montenegro und Siam ein. Alle Länder nahmen die Einladung ein. Der Kreis der Teilnehmer war mit 26 Teilnehmern damit doppelt so groß, wie bei der letzten Konferenz in Brüssel.
Teilnehmer: - Deutschland, England, Frankreich, Italien, Österreich-Ungarn, Russland - Belgien, Bulgarien, Dänemark, Griechenland, Luxemburg, Montenegro, die Niederlande, Portugal, Rumänien, Schweden-Norwegen, die Schweiz, Serbien, Spanien und die Türkei - China, Japan, Persien, Siam - USA und Mexiko
Kombattant in der Ersten Haager Landkriegsordnung: Hauptziel der Konferenz war die Entwicklung von Instrumenten zur friedlichen Streitbeilegung und Verhinderung von Kriegen. Rüstungsbeschränkung war daher auch ein Ziel ("Abrüstungskonferenz"). Konferenz begann 18. Mai 1899 in Haag.
Levée en masse: Dieser strittige Punkt war der Hauptgrund für das Scheitern von der Brüsseler Konferenz und auch die Haager Friedenskonferenz drohte daran zu scheitern. Der Punkt wurde in der 2. Kommission und dort in der 2. Unterkommission behandelt. Am 20. Juni 1899 hat der deutsche Vertreter, von Schwarzhoff, eine mittlerweile berühmte Rede gehalten. Seine Kernpunkte waren, dass Artikel 9 und 10 (beschäftigen sich mit der Levée en masse) keineswegs die Möglichkeit des Patriotismus und der Abwehr angreifender Truppen ausschließen. Es müsse lediglich sichergestellt werden, dass sich die Erhebenden klar als Kämpfer zu erkennen geben, einen Kommandeur haben, ihre Waffen offen tragen, sich an die Rechte des Kriegs halten und ein gemeinsames Zeichen haben. Ferner erläuterte Schwarzhoff, dass Soldaten auch Menschen seien und daher Humanität erwarten dürften. Wenn Soldaten, erschöpft von einem langen Marsch, in einem Dorf Rast machen, dann müssten sie sicher sein können, dass sich die friedlichen Passanten nicht unerwarteter Weise zu Angreifern wandelten. Der russische und niederländische Vertreter schlossen sich dieser Ausführung an.
Martens'sche Klausel: In seiner Eingangsrede zu der obigen Sitzung verlas Martens seine Erklärung, die später in die Präambel der HLKO aufgenommen wurde: "[...]Until a perfectly complete code of the laws of war is issued, the Conference thinks it right to declare that in cases not included in the present arrangement, populations and belligerents remain under the protection and empire of the principles of international law, as they result from the usages established between civilized nations, from the laws of humanity, and the requirements of the public conscience."
Artikel 9 und 10: Beide Artikel wurden ohne Änderungen übernommen.
Artikel 11: Der Status von Nichtkombattanten in einer Armee wurde durch diesen Artikel gewürdigt. Sie stehen unter dem gleichen Schutz, wie Kombattanten.
Vorbereitung zweite Konferenz:
Der Impuls für die zweite Konferenz ging von den USA aus, obwohl die formelle Einladung erneut von dem russischen Zaren ausging. Diesmal gab es 44 Teilnehmerstaaten. Zusätzlich zu den Signatarstaaten der ersten Konferenz waren diesmal auch folgende Staaten dabei: Argentinien, Bolivien, Brasilien, Chile, Dominikanische Republik, Equador, Guatemala, Haiti, Kolumbien, Nicaragua, Panama, Paraguay, Peru, El Salvador, Uruguay, Venezuela und Norwegen (diesmal als selbstständiger Staat).
Levée en masse:
Deutschland möchte, dass Gruppen aus der Bevölkerung, die sich erheben, im Voraus ihr Emblem dem Feind bekannt geben. Frankreich antwortet darauf, dass es in den Wirren des Krieges oftmals nicht möglich ist, diese Information dem Feind zu übermitteln, ohne die eigene Position zu kompromittieren. Stattdessen sei es im Aufgabenbereich einer jeden Armee eine gute Aufklärung zu haben, um zu wissen, wo ihre eigenen Truppen sind und wo sich die des Feindes befinden. Der Vorschlag Deutschlands wurde mehrheitlich abgelehnt. Der zweite Änderungsantrag Deutschlands beschäftigte sich mit der Pflicht von den sich Erhebenden, ihre Waffen offen zu tragen. Dieser Vorschlag wurde mehrheitlich angenommen, da er nur eine Klarstellung war, denn die sich Erhebenden mussten auch vorher schon die Gebräuche des Landkriegs beachten, was den offenen Kampf einschließt.
Auswerung der Verhandlungen:
Die Haager Friedenskonferenzen können als gelungene Kodifikation in Bezug auf den Kombattantenstatus angesehen werden.},
Vorbereitung zweite Konferenz: Der Impuls für die zweite Konferenz ging von den USA aus, obwohl die formelle Einladung erneut von dem russischen Zaren ausging. Diesmal gab es 44 Teilnehmerstaaten. Zusätzlich zu den Signatarstaaten der ersten Konferenz waren diesmal auch folgende Staaten dabei: Argentinien, Bolivien, Brasilien, Chile, Dominikanische Republik, Equador, Guatemala, Haiti, Kolumbien, Nicaragua, Panama, Paraguay, Peru, El Salvador, Uruguay, Venezuela und Norwegen (diesmal als selbstständiger Staat).
Levée en masse: Deutschland möchte, dass Gruppen aus der Bevölkerung, die sich erheben, im Voraus ihr Emblem dem Feind bekannt geben. Frankreich antwortet darauf, dass es in den Wirren des Krieges oftmals nicht möglich ist, diese Information dem Feind zu übermitteln, ohne die eigene Position zu kompromittieren. Stattdessen sei es im Aufgabenbereich einer jeden Armee eine gute Aufklärung zu haben, um zu wissen, wo ihre eigenen Truppen sind und wo sich die des Feindes befinden. Der Vorschlag Deutschlands wurde mehrheitlich abgelehnt. Der zweite Änderungsantrag Deutschlands beschäftigte sich mit der Pflicht von den sich Erhebenden, ihre Waffen offen zu tragen. Dieser Vorschlag wurde mehrheitlich angenommen, da er nur eine Klarstellung war, denn die sich Erhebenden mussten auch vorher schon die Gebräuche des Landkriegs beachten, was den offenen Kampf einschließt.
Auswerung der Verhandlungen: Die Haager Friedenskonferenzen können als gelungene Kodifikation in Bezug auf den Kombattantenstatus angesehen werden.},
Bookauthor = {Buß, Regina},
Booktitle = {Der {K}ombattantenstatus},
Owner = {jim},
@ -102,6 +79,17 @@ Die Haager Friedenskonferenzen können als gelungene Kodifikation in Bezug auf d
Timestamp = {2013.10.29}
}
@Book{DeutschesReich2010,
Title = {Die {H}aager {L}andkriegsordnung},
Author = {{Deutsches Reich}},
Year = {2010},
Publisher = {Europ. Hochsch.-Verl.},
Abstract = {Erster Teil: Kombattantenstatus Zweiter Teil: Kriegshandlungen Dritter Teil: Regelung der Besetzung},
Owner = {jim},
Timestamp = {2014.10.30}
}
@Report{Dierks2008,
Title = {The Transport Layer Security (TLS) Protocol},
Author = {Dierks, T. and Rescorla, E.},
@ -397,17 +385,6 @@ Die Haager Friedenskonferenzen können als gelungene Kodifikation in Bezug auf d
Timestamp = {2014.01.12}
}
@Book{Reich2010,
Title = {Die {H}aager {L}andkriegsordnung},
Author = {Deutsches Reich},
Year = {2010},
Publisher = {Europ. Hochsch.-Verl.},
Abstract = {Erster Teil: Kombattantenstatus Zweiter Teil: Kriegshandlungen Dritter Teil: Regelung der Besetzung},
Owner = {jim},
Timestamp = {2014.10.30}
}
@Inbook{Russel2010,
Title = {Artificial intelligence: A Modern Approach},
Author = {Russel, Stuart J. and Norvig, Peter},